texts
sequence
meta
dict
scores
sequence
avg_score
float64
0
0.22
num_sents
int64
5
5
[ "Latest News\n\nNew app to keep drones safe\n\nA new Smartphone app has been released to make flying drones safer. ", "The app clearly shows crucial drone no-fly zones and drone fly with caution zones for drones operated in the under two kilogram commercial category. ", "This information can also be used as guide for recreational drone flyers and certified remotely piloted aircraft operators. ", "The app uses a drone flyers location to display no-fly zones around major airports, the flight paths of smaller airports and helicopter landing areas. ", "Users will also see restricted and military airspace where drones must not be flown. ", "The drone no-fly zones are shaded in red on the map. ", "Orange shading is used to show fly with caution zones around areas where aircraft are known to operate at low altitudes. ", "It is the first time an official app has been released in Australia to help drone flyers stay safe and abide by the safety regulations. ", "Information is also shown for uncontrolled aerodromes and aircraft landing areas, with written advice about what to do when flying a drone in those locations. ", "The app has been produced with specialist drone software company Drone Complier and will be available in Android, iOS and web-based HTML5." ]
{ "pile_set_name": "Pile-CC" }
[ 0.00909090909090909, 0, 0, 0, 0, 0, 0, 0, 0, 0.014492753623188406 ]
0.002358
5
[ "Q:\n\nUsing the REST api to retrieve multiple objects with a single request\n\nI'm trying to figure out how to best retrieve all of the ids returned by calling getUpdated. ", " I'm using the REST API and I don't see anyway of retrieving a list of ids with a single request.", "\nhttp://www.salesforce.com/us/developer/docs/api_rest/index_Left.htm#CSHID=intro_usage_rest_resources.htm|StartTopic=Content%2Fintro_usage_rest_resources.htm|SkinName=webhelp\nI've considered execution a SOQL query with a condition like Id in ids[] but the ids array is often to large and the request fails.", "\nWhat is the most efficient way using the REST API to retrieve all updated ids?", "\n\nA:\n\nI am reasonably certain the REST API allows for only one object per retrieve (?)", "\nThe SOAP API however allows you to retrieve up to 2000 IDs per API call!", "\nIf you're doing a lot of data loading, the Bulk API may be a better fit for your app. ", "For most other reasonably complex client apps, there may still be enough API differences that you'll benefit from using both the REST and SOAP APIs.", "\n\n" ]
{ "pile_set_name": "StackExchange" }
[ 0, 0, 0.006535947712418301, 0, 0, 0, 0, 0.006756756756756757, 0 ]
0.001477
5
[ "Q:\n\nInstall all packages from a folder with Ansible 2.0\n\nI have a folder where I place unmaintained python packages, so I installed them from the zip rather than from their repository.", "\nI am using Ansible 2.0, so the find command seems to be the way to do it\nSo far I was doing the following:\n- name: Install unmaintained dependencies\n pip:\n name: \"{{ my_project_app }}/requirements/{{ item }}\"\n virtualenv: \"{{ my_project_venv }}\"\n with_items:\n - django-hijack-2.0.0.zip\n - django-image-cropping-django-19.zip\n - pisa-3.0.33.zip\n\nNow I'm playing with the find command\n- name: Loading unmaintained dependencies\n find:\n paths: \"{{ my_project_app }}/requirements\"\n patterns: \"*.zip\"\n register: unmaintained_dependencies\n\n- debug:\n var: unmaintained_dependencies\n\nIf I run the playbook above, I get the following output\nok: [192.168.1.212] => {\n \"files_found\": {\n \"changed\": false,\n \"examined\": 3,\n \"files\": [\n {\n ...\n \"path\": \"/data/my_project/requirements/pisa-3.0.33.zip\",\n ...\n },\n ...\n ],\n \"matched\": 3,\n \"msg\": \"\"\n }\n}\n\nI guess that there must be a way to put everything together but here is where I'm stuck.", "\n\nA:\n\nI still don't get what are you trying to achieve here yet.. Are you using the find module just because pip module doesn't install packages from zip files?", "\nFor your find workaround, you can create a task that iterates over the results of the find task, using with_items: files_found.files, and using {{ item.path }} whenever you need the path:\n- name: Install unmaintained dependencies\n pip:\n name: \"{{ item.path }}\"\n virtualenv: \"{{ my_project_venv }}\"\n with_items: \"{{files_found.files}}\"\n\nAlso, instead of using file, you can try to make a loop using with_fileglob:\n- pip:\n name: {{ item }}\n virtualenv: \"{{ my_project_venv }}\"\n with_fileglob:\n - \"{{ my_project_app }}\"/requirements/*.zip\n\nNote, I didn't have time to test any of these solutions, or to ask more about what were you trying to achieve, but I hope they help with your problem.", "\n\n" ]
{ "pile_set_name": "StackExchange" }
[ 0, 0.0009337068160597573, 0, 0, 0 ]
0.000187
5
[ "Aji Molido\n\nAbout\n\nA dried, ground mild chili that is popular in Argentina. ", "Similar to red pepper flakes that can be found in the US, aji molido is softer in texture and less dried out. ", "The level of spice is quite mild yet has a bit of bite, similar to ground black pepper or even green peppercorns." ]
{ "pile_set_name": "Pile-CC" }
[ 0.013157894736842105, 0, 0 ]
0.004386
5
[ "Get set for ‘The Powerpuff Girls’ reboot in 2016\n\nNew Delhi, June 17: A reboot of the beloved animated series “The Powerpuff Girls” will hit the small screens worldwide in 2016, Cartoon Networkhas announced.", "\n\nCreated by Craig McCracken, the series centres on three sugar-coated superheroes whose mission in life alternates between going to school, fighting crime and winning at hopscotch. ", "The reboot will include an all-new television series produced by Cartoon Network Studios and a licensing program that will roll out across all regions.", "\n\n“The original ‘Powerpuff Girls’ was a franchise phenomenon for Cartoon Network, paving the way for a new generation of innovative storytellers and enthusiastic fans worldwide. ", "We’re calling these girls back into action based upon an overwhelming demand for sugar, spice and Chemical X,” Rob Sorcher, chief content officer, Cartoon Network, said in a statement.", "\n\n“The Powerpuff Girls”, which debuted in 1998, has earned two Emmy Awards along with five nominations, as well as countless animation honours throughout its 78-episode run. ", "The series also inspired a feature film in 2003.", "\n\nIt is also among the top grossing Cartoon Network brands of all time, with more than US$2.5 billion in retail sales generated since its debut, and has endured as a global fashion brand with a range of apparel partners and fashion collections around the world.", "\n\n“We can’t wait to unleash this unique brand of cute and chic into the Australia and New Zealand markets. ", "While in Asia, this will further expand and complement our existing Powerpuff business, a licensing program which already enjoys great success across our markets,” Melissa Tinker, vice president of CNE in Asia Pacific." ]
{ "pile_set_name": "Pile-CC" }
[ 0.00966183574879227, 0.01098901098901099, 0.006622516556291391, 0.0056179775280898875, 0.016304347826086956, 0.005747126436781609, 0, 0.0038314176245210726, 0, 0.013761467889908258 ]
0.007254
5
[ "In one well-known type of gas turbine engine (GTE), at least one high pressure turbine (HPT) nozzle is mounted within an engine casing between a combustor and a high pressure (HP) air turbine. ", "In single nozzle GTE platforms, the HPT nozzle typically includes an annular nozzle flowbody having an inner nozzle endwall and an outer nozzle endwall, which circumscribes the inner nozzle endwall. ", "A plurality of circumferentially spaced stator vanes extends between the outer and inner nozzle endwalls and cooperates therewith to define a number of flow passages through the nozzle flowbody. ", "The HPT nozzle further includes one or more radial mounting flanges, which extend radially outward from the HPT nozzle flowbody. ", "The radial mounting flanges are each rigidly joined to a different end portion of the nozzle flowbody and may be integrally formed therewith as a unitary machined piece. ", "When the GTE is assembled, the radial mounting flanges are each attached (e.g., bolted) to corresponding GTE-nozzle mounting interfaces (e.g., inner walls) provided within the GTE to secure the HPT nozzle within the engine casing.", "\nDuring GTE operation, the HPT nozzle conducts combustive gas flow from the combustor into the HP air turbine. ", "The combustive gas flow convectively heats the inner surfaces of the combustor and the HPT nozzle flowbody to highly elevated temperatures. ", "At the same time, the HPT nozzle's radial mounting flanges and the GTE-nozzle mounting interfaces are cooled by bypass air flowing over and around the combustor. ", "Significant temperature gradients thus occur within the GTE during operation, which result in relative thermal movement (also referred to as “thermal distortion”) between the HPT nozzle, the GTE-nozzle mounting interfaces, and the trailing end of the combustor. ", "Due to their inherent rigidity, conventional HPT nozzles of the type described above are often unable to adequately accommodate such thermal distortion and, as a result, can experience relatively rapid thermomechanical fatigue and reduced operational lifespan. ", "In addition, thermal distortion between the HPT nozzle, the combustor end, and the GTE-nozzle mounting interfaces can result in the formation of leakage paths, even if such mating components fit closely in a non-distorted, pre-combustion state. ", "Compression seals may be disposed between the nozzle mounting flanges and the GTE-nozzle mounting interfaces to minimize the formation of leakage paths. ", "However, the sealing characteristics of the compression seals can be compromised when the nozzle mounting flanges, and specifically when the mounting flange sealing surfaces contacting the compression seals, are heated to elevated temperatures by combustive gas flow through the turbine nozzle flowbody. ", "Although the radial height of the mounting flanges can be increased to further thermally isolate the flange sealing surfaces from the combustive gas flow, increasing the height of the radial mounting flanges undesirably increases the overall envelope of the HPT nozzle and consumes a greater volume of the limited space available within the engine casing.", "\nThere thus exists an ongoing need to provide a turbine nozzle or turbine nozzle assembly capable of accommodating the relative thermal movement between the turbine nozzle and the GTE-turbine nozzle mounting interface during GTE operation. ", "Preferably, embodiments of such a turbine nozzle assembly would be relatively compact while providing a mounting flange sealing surface sufficiently thermally isolated from the combustive gas flow to prevent overheating of any compression seals disposed between the mounting flange and the GTE-turbine nozzle mounting interface. ", "Other desirable features and characteristics of the present invention will become apparent from the subsequent Detailed Description and the appended Claims, taken in conjunction with the accompanying Drawings and this Background." ]
{ "pile_set_name": "USPTO Backgrounds" }
[ 0.0051813471502590676, 0.010050251256281407, 0, 0.015503875968992248, 0, 0.013043478260869565, 0.02702702702702703, 0.007142857142857143, 0.012345679012345678, 0.011450381679389313, 0, 0.00816326530612245, 0.006535947712418301, 0, 0, 0.008333333333333333, 0.00303951367781155, 0.004366812227074236 ]
0.007344
5
[ "Thousands of spectators have descended on Sydney for this year's Gay and Lesbian Mardi Gras Parade.", "\n\nThere were about 150 floats spanning five kilometres in the parade on Saturday night, with themes covering a range of topics including anti-homophobia in sport and marriage equality.", "\n\nThe event is in its 37th year with more than 10,000 people - and half a tonne of glitter - involved in this year's parade.", "\n\nSeveral high profile athletes, including former Olympians and football players, took part in the parade.", "\n\nThe first three floats in this year's parade centred on the success of elite gay athletes, the commitment by Australian sporting organisations to a more diverse and inclusive sporting environment, and the success of the Sydney Convicts, who last year won the Bingham Cup - the world cup of gay rugby.", "\n\n\"This fight against discrimination in sport is part of the great civil rights struggle that Sydney's Mardi Gras has led in Australia for 37 years,\" Ignatius Jones, Mardi Gras' senior parade creative consultant, said.", "\n\n\"As long as even one gay or lesbian kid is afraid to play sport, one of Australia's defining activities, for fear of being denounced, Mardi Gras has a purpose.\"", "\n\nThis year's parade also saw the largest number of Defence Force members ever to take part in the event.", "\n\nThe ADF officer in charge of the parade, Squadron Leader Vince Chong, said the growing participation showed that Defence culture was changing for the better.", "\n\nParade chief Michael Rolik said while the recognition of equality for the gay community had greatly improved, there were still many prejudices prevalent in society.", "\n\n\"There are a lot of battles. ", "We're one law reform short from equality - for marriage equality,\" he said.", "\n\n\"But that doesn't mean that attitudes are in sync with equality and diversity and I think our biggest challenge is really shaping attitudes, homophobia in schools, and the whole coming out process for vulnerable people ... they're the issues.\"", "\n\nAssistant Commissioner Alan Clarke said police were pleased with the behaviour of the crowd.", "\n\n\"We had an extremely busy night, but overall a very successful night,\" he said.", "\n\n\"We made around 30 arrests and they were for a range of offences, but there was no seriously violent issues.\"", "\n\nLoading\n\nLoading\n\nLoading\n\nLoading\n\nLoading\n\nLoading..." ]
{ "pile_set_name": "OpenWebText2" }
[ 0.010101010101010102, 0, 0, 0, 0.0033112582781456954, 0.013761467889908258, 0.006172839506172839, 0.009523809523809525, 0.025157232704402517, 0.006024096385542169, 0, 0, 0, 0.010638297872340425, 0, 0, 0 ]
0.004982
5
[ "For fractionating and purifying synthesis gases, it is known to use production plants in which one generally can only separate one part of the synthesis gas that has been produced by the partial oxidation of natural gas. ", "In these known methods, carbon monoxide is separated by means of refrigeration engineering in one plant, which is known as a device for partial gas condensation (1). ", "Means of refrigeration engineering are also employed to obtain the synthesis gas required for ammonia production, where a plant uses a method that is known as “nitrogen washing” (1). ", "Both plants have a completely different construction type. ", "The only common feature of these two techniques of refrigeration engineering is to be found in the use of a multiway plate heat exchanger for cooling off and reheating the gas streams. ", "Each of these methods requires a cooling device, which in a partial condensation unit in general can utilizes the expansion of carbon monoxide or hydrogen, in which case a unit for nitrogen washing requires liquid nitrogen." ]
{ "pile_set_name": "USPTO Backgrounds" }
[ 0, 0, 0, 0, 0, 0 ]
0
5
[ "Enjoyed that video! ", "Looks like a fun road to drive, but makes me happy I have DCT when I see how you have to take your hand of the wheel to shift going around those corners. ", "Other than for that reason, I miss having a manual!", "\n\n19\" advan RS?!?! ", "Ive been looking to see some silver ones on an e90 M3!!!! ", "I have a set in the mail on their way to me and may I say, they look amazing!", "\n\nThanks!", "\n\nQuote:\n\nOriginally Posted by Endless619\n\nKauai is very laid back and one of the smaller islands. ", "Depends on what you want to do. ", "Maui and Oahu are the party islands btw. ", "lol\n\nI lived in Hawaii for 6 years and did the island hoping all the time.", "\n\nGreat people, beautiful scenery, awesome food, you name it.", "\n\nyea for sure, kauai is great for hiking, beaches, and just overall relaxing. ", "More of a small town vibe. ", "Oahu is much more fun if you want to be able to go out to clubs at night, but then you usually sleep away the next day, so might as well just go to vegas for that lol.", "\n\nQuote:\n\nOriginally Posted by mamadub\n\nEnjoyed that video! ", "Looks like a fun road to drive, but makes me happy I have DCT when I see how you have to take your hand of the wheel to shift going around those corners. ", "Other than for that reason, I miss having a manual!", "\n\nyour car looks amazing! ", "ill be at kauai in august for my friends wedding, but thats just one day, imma be staying there for a whole week, is there anything fun to do there??", "\n\nWife and I were just there in March. ", "Anini beach is by far my favorite beach. ", "Super calm water, great for snorkeling. ", "Kilauea Fish market had my favorite poke. ", "We tried at least 10 different places. ", "Chicken in a barrel in Kaapa was awesome as well. ", "Have fun and enjoy!", "\n\nWife and I were just there in March. ", "Anini beach is by far my favorite beach. ", "Super calm water, great for snorkeling. ", "Kilauea Fish market had my favorite poke. ", "We tried at least 10 different places. ", "Chicken in a barrel in Kaapa was awesome as well. ", "Have fun and enjoy!", "\n\nThank you! ", "haha i thought it was gonna be a straight forest like jurassic park..." ]
{ "pile_set_name": "Pile-CC" }
[ 0, 0, 0, 0, 0, 0, 0, 0.010101010101010102, 0, 0.024390243902439025, 0, 0, 0, 0, 0.005988023952095809, 0, 0, 0, 0, 0, 0, 0.024390243902439025, 0, 0.023809523809523808, 0, 0.02, 0, 0, 0.024390243902439025, 0, 0.023809523809523808, 0, 0.02, 0, 0, 0 ]
0.004913
5
[ "Vehicle lamp components such as housings, reflectors, and extensions, which compose vehicle lamps such as head lamps and turn signal lamps of automobile, are exposed to a temperature of 160 to 180° C. by heat generated from light sources. ", "Consequently, when the vehicle lamp components are used for a long time, it sometimes happens that thermal decomposition products sublimate from the vehicle lamp components, adhere to lenses of the vehicle lamps, and thereby cause fogging of the lenses.", "\nIn addition, a direct- or pre-treatment, or a further treatment such as coating is applied on the vehicle lamp components, depending on a purpose. ", "In view of design, high gloss is required for the vehicle lamp components, in particular for extensions (decorative boards around reflectors), on which the foregoing treatment has been applied. ", "Therefore, high degree surface appearance is also required for the vehicle lamp components before the foregoing treatment is performed (base materials).", "\nAs the thermoplastic resin composition from which good vehicle lamp components being suppressed in fogging and having good surface appearance can be obtained, the following has been proposed:\na polyester resin composition comprising 100 parts by mass of a thermoplastic polyester resin (A), 5 to 50 parts by mass of an epoxy group-containing material (B), and 0.1 to 50 parts by mass of a reinforcement material (Patent Document 1).", "\nThe thermoplastic resin composition described in Patent Document 1, however, needs further improvement in the use where the shaped article is subjected to direct vapor deposition, although mold-staining hardly occurs at the time of shaping and surface appearance of the shaped article (a vehicle lamp component) is improved. ", " Patent Document 1: Japanese Patent Application Laid-Open No. ", "2001-316,573" ]
{ "pile_set_name": "USPTO Backgrounds" }
[ 0, 0, 0, 0, 0, 0, 0, 0.015625, 0 ]
0.001736
5
[ "Q:\n\nTaylor expansion of the Error function\n\nThe error function $\\operatorname{erf}(z)$ is defined by the integral \n$$\n\\operatorname{erf}(z)=\\frac{2}{\\sqrt{\\pi}} \\int_0^z e^{-t^2}\\,dt,\\quad t\\in\\mathbb R$$\n\nFind the Taylor expansion of $\\operatorname{erf}(z)$ around $z_0=0$.\n\nHow to expand a function, when it is defined by the integral sign? ", "Can somebody explain the steps to compute this? ", "\n\nA:\n\nAs\n$$e^t=\\sum_{n=0}^\\infty\\frac{t^n}{n!},$$\nwe have\n$$e^{-t^2}=\\sum_{n=0}^\\infty\\frac{(-1)^nt^{2n}}{n!}.$$\nIntegrating term to term (why is possible?),", "\n$$\\frac{\\sqrt\\pi}{2}\\text{erf}(z)=\\sum_{n=0}^\\infty\\frac{(-1)^nt^{2n+1}}{(2n+1)n!}.$$\n\n" ]
{ "pile_set_name": "StackExchange" }
[ 0.0058309037900874635, 0, 0, 0.011363636363636364 ]
0.004299
5
[ "Q:\n\nAccessing Large files stored in AWS s3 using AWS Lambda functions\n\nI have more than 30GB file stored in s3,and I want to write an Lambda function which will access that file, parse it and then run some algorithm on the same.", "\nI am not sure if my lambda function can take that big file and work on it as Max execution time for Lambda function is 300 sec(5 min).", "\nI found AWS S3 feature regarding faster acceleration, but will it help?", "\nConsidering the scenario other than lambda function can any one suggest any other service to host my code as micro service and parse the file?", "\nThanks in Advance\n\nA:\n\nIt is totally based on the processing requirements and frequency of processing. ", "\nYou can use Amazon EMR for parsing the file and run the algorithm, and based on the requirement you can terminate the cluster or keep it alive for frequent processing. ", "https://aws.amazon.com/emr/getting-started/\nYou can try using Amazon Athena (Recently launched) service, that will help you for parsing and processing files stored in S3. ", "The infrastructure need will be taken care by Amazon. ", "http://docs.aws.amazon.com/athena/latest/ug/getting-started.html\nFor Complex Processing flow requirements, you can use combinations of AWS services like AWS DataPipeline - for managing the flow and AWS EMR or EC2 - to run the processing task.https://aws.amazon.com/datapipeline/\nHope this helps, thanks\n\n" ]
{ "pile_set_name": "StackExchange" }
[ 0.0043859649122807015, 0.007407407407407408, 0, 0, 0, 0, 0.017543859649122806, 0.018518518518518517, 0.013157894736842105 ]
0.006779
5
[ "0\n\nFrom the sometimes disappointing but often great reality of Siri to the complicated potential of Samantha, the idea of virtual personal assistants helping us with our digital-fuelled lives has captured the public imagination.", "\n\nToday, another not-quite real female is entering the fray: Amy, a personal assistant that has been built to help you schedule meetings — summoned not through an app or an on-device client, but simply by copying her in on email correspondence between you and the people you are proposing to meet.", "\n\nGetting copied in triggers Amy (full name: Amy Ingram, apparently) to read the email, look for date and time and place suggestions and then continue to the conversation directly with the other person to find a time and place that works for everyone — and then put the detail into your diary.", "\n\nThe idea here is that rather than taking up your time, or other people’s time, scheduling these meetings, you can use Amy to do it all for you. “", "Anything which a human PA could add to a traditional meeting invite she can do,” co-founder and CEO Dennis Mortensen tells me.", "\n\nThe New York-based startup that makes Amy, X.ai (“Ex-dot-A I”), is today announcing $2.1 million in funding to develop the product, with the seed round coming from a top-shelf list of investors: IA Ventures, Softbank and Lerer Ventures.", "\n\nMeanwhile, Amy is still be in a closed beta. ", "You have to register your interest for now, and Mortensen says that a new intake will be made on August 1.", "\n\n“We are promising 100% service delivery,” he says, meaning that every meeting request is taken care of, and correctly applied, “and that requires a lot of AI training before we open up the floodgates.” ", "Those who have been given early access are already using the service to set up meetings.", "\n\nAmy currently integrates with basic email and the more popular calendar services like Google’s. ", "The plan down the line, Mortensen says, is to “quickly add new channels; so that you can text Amy, IM Amy or speak to Amy.” ", "He says that the latter is likely to come in two steps, the first being to Amy a message about, say, being late for a meeting. “", "Asynchronous like the former [features],” he says.", "\n\nAnd there are other plans for growth coming up. ", "From what I understand — although Mortensen would not confirm this to me — X.ai has already also made an agreement with Google to promote the service to its business users. ", "Google itself is not an investor in X.ai.", "\n\nMaking Amy as “light” an unintrusive as possible, Mortensen believes, is the key both to his company’s differentiation and potential success where others have done less well.", "\n\n“There are plenty who tried to solve this in the past and it almost resembles the Battle of Thermopylae. ", "Lots of glory, but 300 smart, well funded entrepreneurs before us, died trying,” he says.", "\n\n“We are different because we’ve accepted that this pain is not solved with YET another app. ", "We are middleware and do not exist outside of conceptually being a human-like creature who interacts with my friends, colleagues and business contacts. ", "Either we completely remove the pain and have something which could only be done by humans yesterday OR we die! ", "It requires no sign-in, no password, no download.”", "\n\nBut light and easy are deceptive concepts for what is happening under the hood.", "\n\nAs Mortensen describes it, “The two primary concepts that break that utopia of it just being a fairly simple engineering task of transposing one calendar on top of another is the personal preferences of each participant and the rather complex social dynamics that are built into meetings,” he says. “", "As I emailed Jordy from Softbank about my new venture, at no point in the conversation did we talk about where to meet for a casual presentation, it was understood that that would be his office. ", "Amy needs to know that.”", "\n\nCrucially, he notes that Amy itself is not an app; nor does it compete with calendar apps. “", "We do not compete with the new calendars like Tempo and Sunrise,” he says. “", "We would simply insert into those like we would to the standard Apple Cal./ Google Calendar etc.”", "\n\nThe list of VCs backing X.ai at this early stage is impressive, and so you may be a little unsurprised to hear that this is not the founder’s first rodeo.", "\n\nMortensen has been involved in a string of exits, with a lot of his experience specifically in areas like big data analytics, which come into play at X.ai, too. ", "His track record includes selling Visual Revenue to Outbrain (he was the CEO and founder); Indextools to Yahoo (he was a shareholder and COO); and Canvas Interactive to TJ Group (he was founder and CEO).", "\n\nIt’s not all wins, though: an early startup he founded called evonax — “think OpenTable/Seamless for Europe, with a twist” — went bust. “", "A rather expensive MBA,” he writes in LinkedIn.", "\n\nBut it seems, at least in part, that its the idea and solution that have attracted investors here, rather than Mortensen’s track record itself.", "\n\n“x.ai is taking on a well-accepted pain, using artificial intelligence and machine learning to a degree rarely seen,” said Brad Gillespie, Partner at IA Ventures, in a statement. “", "Dennis and the x.ai team is executing on a very ambitious plan, which, if successful, could change the paradigm for how we schedule meetings in the future, perhaps even how we choose to spend our time at work.”", "\n\nDown the line there are plans to extend Amy’s reach into more than just scheduling a meeting. ", "Think here of not just organising a meeting but ordering the Uber to take you there, or booking a hotel if it’s in another city, or buying the tickets if the meeting is with a friend at a gig. “", "But I want to be the best in the world at scheduling meetings before we do anything else,” Mortensen says. “", "My mom used to say, do one thing, but do it so well that is becomes obvious you are the best at it. ", "I am sticking to that!”", "\n\n0\n\nCrunchBase\n\nOverviewSiri is now a product of Apple Corp.\nSiri is a virtual personal assistant incorporated as a feature of Apple iPhones beginning with the 4S generation.", "\nSiri was originally developed by SRI International. ", "SRI spun off Siri, Inc., in 2007, and this company launched a personal assistant app in February 2010. ", "Siri, Inc., was acquired by Apple in 2010, and in October 2011 Apple announced that the iPhone …" ]
{ "pile_set_name": "Pile-CC" }
[ 0.0043859649122807015, 0, 0.006825938566552901, 0, 0.007936507936507936, 0.01680672268907563, 0, 0.009433962264150943, 0, 0, 0.01020408163265306, 0.008064516129032258, 0, 0, 0, 0.011560693641618497, 0, 0.005681818181818182, 0, 0, 0, 0, 0, 0, 0, 0.0033112582781456954, 0.010256410256410256, 0, 0, 0.02631578947368421, 0, 0.00641025641025641, 0.012269938650306749, 0.024630541871921183, 0, 0, 0.006896551724137931, 0.016483516483516484, 0.004761904761904762, 0, 0.005154639175257732, 0.009259259259259259, 0.01, 0, 0.017142857142857144, 0.03773584905660377, 0.009708737864077669, 0.041666666666666664 ]
0.006727
5
[ "Kotaku East East is your slice of Asian internet culture, bringing you the latest talking points from Japan, Korea, China and beyond. ", "Tune in every morning from 4am to 8am. ", "Prev Next View All\n\nReleased this past December, the NeoGeo X apparently won't be produced anymore. ", "According to Japanese site Gamer, production for the system is ending as of this month.", "\n\n\nEven though the machine carried the SNK and NeoGeo brands, the console was not designed and manufactured by SNK. ", "Instead, it was the work of tech company Tommo.", "\n\nGamer does not give a specific reason as to why production is ending so quickly, but, as the box says, it was a limited edition console. ", "That could be why. ", "Back in December, it was announced on Game Watch Impress that production for the machine would be ending, with a TBA for the next time the system would be imported into Japan.", "\n\n\nNo word on how this impacts regions outside Japan. ", "Kotaku did reach out to SNK for clarification, but did not hear back before this story was published.", "\n\nIt sounds like the NeoGeo X will continue to be supported as there are still plans to release more SNK games for it, as well as a screen protector for the NeoGeo X handheld and an AC adapter.", "\n\nUpdate: Siliconera reports that only the \"limited edition\" NeoGeo X units are out of production. ", "The original report from Japanese site Gamer did not make such a clarification.", "\n\n\n「NEOGEO X GOLD SYSTEM」1月22日の製造分をもって生産終了―追加のゲームカードは予定通り発売 [Gamer via Siliconera]\n\nKotaku East is your slice of Asian internet culture, bringing you the latest talking points from Japan, Korea, China and beyond. ", "Tune in every morning from 4am to 8am." ]
{ "pile_set_name": "OpenWebText2" }
[ 0, 0, 0.01, 0, 0.02586206896551724, 0.02127659574468085, 0, 0, 0.005714285714285714, 0, 0.009900990099009901, 0.010362694300518135, 0.010101010101010102, 0.012658227848101266, 0.004694835680751174, 0 ]
0.006911
5
[ "\r\n\r\n\r\n\r\n\r\nOpinion filed February 7, 2008\r\n\r\n\r\n\r\n\r\n\r\n\r\n\n\n\n\n\n \n\n\n\n\n\n\n\n\n \n\n\n\n\nOpinion filed February 7,\r\n2008\n \n \n \n \n \n \n                                                                        In The\n                                                                              \n    Eleventh\r\nCourt of Appeals\n                                                                   __________\n \n                                                          No. ", "11-06-00185-CR \n                                                    __________\n \n                                        RICKY MARTIN, Appellant\n \n                                                             V.\n \n                                        STATE\r\nOF TEXAS, Appellee\n \n\n \n                                          On\r\nAppeal from the 70th District Court\n \n                                                           Ector\r\nCounty, Texas\n \n                                                 Trial\r\nCourt Cause No. ", "A-27,714\n \n\n \n                                                                   O\r\nP I N I O N\n \nRicky\r\nMartin pleaded guilty to two counts of possession of a controlled substance and\r\nelected to have the jury determine his punishment.", "  The jury assessed his\r\npunishment at twenty years confinement and a $10,000 fine on count one and ten\r\nyears confinement on count two.", "  Martin appeals, contending that his counsel\r\nwas constitutionally ineffective.", "  Because Martin was not harmed by his trial\r\ncounsel=s actions, we\r\naffirm.", "\n                                                   Background\r\nFacts\n\n\n\n\nMartin\r\nwas indicted for two counts of possession of cocaine.", "  The first count alleged\r\nthat on May 5, 1999, Martin possessed four grams or more but less than two\r\nhundred grams.", "  The second count alleged that on May 22, 1999, he possessed one\r\ngram or more but less than four grams.", "  Martin pleaded guilty to both counts.", " \r\nHe asked for a jury trial on punishment, and he filed an application for felony\r\ncommunity supervision.", "  The State advised the trial court that it would\r\nintroduce evidence of four unadjudicated offenses and that, pursuant to Section\r\n12.45,[1] it would\r\ndismiss them after trial.", "  Although Martin was not required to admit that he\r\nwas guilty of the four unadjudicated offenses as required by Section 12.45,[2]\r\nhis counsel successfully requested an instruction that the jury could consider\r\nthem only if the jury was convinced of Martin=s\r\nguilt beyond a reasonable doubt.", "\nAt\r\ntrial, the State presented evidence that on May 5, 1999, Martin was stopped by\r\nan Odessa police officer who recognized him and knew that his license had been\r\nsuspended.", "  The officer found 6.12 grams of crack cocaine during this stop.", "  On\r\nMay 22, 1999, while conducting a drug-related investigation at the Royal Inn,\r\nOdessa police officers found marihuana and 2.93 grams of crack cocaine in a\r\nroom occupied by Martin and his girlfriend.", "\n\n\n\n\nThe\r\nfour unadjudicated offenses involved subsequent drug arrests.", "  On June 1, 2000,\r\nOdessa police officers observed Martin deliver .19 grams of crack cocaine at\r\nthe Sahara Motel.", "  Two weeks later, on June 15, Odessa police received a tip\r\nconcerning drug trafficking at the Travel Inn, and they went to the motel to\r\ninvestigate.", "  The tipster told police that Derek Pride was holding narcotics in\r\nRoom No. ", "108.", "  One officer went to the room and knocked on the door.", "  The\r\nofficer could hear movement inside the room, could smell marihuana, and could\r\nhear a cabinet drawer or window open.", "  The officers ultimately gained access to\r\nthe room.", "  Martin and Pride were inside.", "  The officers searched the room and\r\nnoticed a large baggie with what appeared to be crack cocaine on the ground\r\noutside the window.", "  They did not say anything about the baggie to Pride or\r\nMartin but went outside and established surveillance.", "  Approximately ten\r\nminutes later, they saw Martin stick his head out the window and point to the\r\nbag.", "  Martin was arrested, and the police discovered 13.8 grams of crack\r\ncocaine in two baggies outside the window.", "  They also found $1,243 in cash on\r\nMartin.", "  Most of this was in twenties.", "  The significance of this, according to\r\nthe police, was that crack cocaine was selling for $20 a rock.", "\nOn\r\nJuly 18, Martin was observed at the Desert Inn engaged in suspected drug\r\ntrafficking.", "  A police officer approached Martin and received consent to search\r\nhis motel room.", "  During this search, officers found a baggie with 6.45 grams of\r\ncrack cocaine.", "  Finally, on August 8, shortly before the start of his trial on\r\nthe 24th, Martin was arrested for selling 1.08 grams of crack cocaine to an\r\nundercover officer. ", "\nIn\r\naddition to the six drug arrests, the State also presented evidence that Martin\r\nwas arrested for possession of 77.3 grams of crack cocaine while on community\r\nsupervision as a juvenile, that he had been arrested several times for driving\r\nwithout a license, and that he resisted arrest during the August 8 incident.", "\n                                                             Issues\nMartin\r\nraises two ineffective-assistance-of-counsel issues.", "  First, he contends that\r\nhis trial counsel was ineffective for not objecting to a probation officer=s testimony that Martin\r\nwould not be a good candidate for community supervision and that he would not\r\nwant Martin on his caseload.", "  Second, he contends that trial counsel=s closing argument\r\nalienated the jury and cast further disdain upon him.", "\n                                                              Standard\r\nof Review\n\n\n\n\nTo\r\nprevail on a claim of ineffective assistance of counsel, an appellant must\r\nestablish that his lawyer=s\r\nperformance fell below an objective standard of reasonableness and that there\r\nis a Areasonable\r\nprobability@ the\r\nresult of the proceeding would have been different but for counsel=s deficient performance.", "  Strickland\r\nv. Washington, 466 U.S. 668, 693‑94 (1984); Mallett v. State,\r\n65 S.W.3d 59, 62‑63 (Tex. ", "Crim. ", "App. ", "2001).", "  A reasonable probability is\r\na probability sufficient to undermine confidence in the outcome. ", "Hernandez\r\nv. State, 726 S.W.2d 53, 55 (Tex. ", "Crim. ", "App. ", "1986).", "  The purpose of this\r\ntwo‑pronged test is to judge whether counsel=s conduct so compromised the proper\r\nfunctioning of the adversarial process that the trial cannot be said to have\r\nproduced a reliable result.", "  Thompson v. State, 9 S.W.3d 808, 812‑13\r\n(Tex. ", "Crim. ", "App. ", "1999) (citing McFarland v. State, 845 S.W.2d 824,\r\n843 (Tex. ", "Crim. ", "App. ", "1992)). ", "The review of defense counsel=s representation is highly\r\ndeferential and presumes that counsel=s\r\nactions fell within a wide range of reasonable professional assistance. ", "Tong\r\nv. State, 25 S.W.3d 707, 712 (Tex. ", "Crim. ", "App. ", "2000). ", "Appellant must\r\novercome the presumption that, under the circumstances, the challenged action\r\nmight be considered sound trial strategy. ", "Jackson v. State, 877 S.W.2d\r\n768 (Tex. ", "Crim. ", "App. ", "1994); Hayden v. State, 155 S.W.3d 640, 648 (Tex.", "\r\nApp.", "CEastland 2005,\r\npet. ", "ref=d).", "\n                                                                        Analysis\nThe\r\nProbation Officer=s\r\nOpinion Testimony.", "  \nMartin=s trial counsel called Chad\r\nHetrick, an Ector County Felony Probation Officer, as a witness.", "  Hetrick\r\ntestified about the rules and conditions applicable to probationers, and he\r\ntestified that there are people on community supervision in Ector County who\r\nwere convicted of drug possession, drug delivery, and sex crimes.", "  During the\r\nState=s\r\ncross-examination, Hetrick testified that he had not seen anyone placed on\r\ncommunity supervision with three pending cases.", "  The State then solicited the\r\nfollowing opinion testimony:\nQ: \r\nAnd if someone has six pending drug cases, would you recommend that person for\r\nprobation?", "\n \nA: \r\nWith six pending?", "  No, sir.", "  I couldn=t\r\ndo that just from my opinion.", "  No, sir.", "\n \nQ: \r\nAnd if someone was indicted on two cases and then commits other ones, does that\r\nindicateBwhat does\r\nthat indicate to you about their likelihood of success?", "  Or how promising they\r\nare?", "\n \nA: \r\nNot very promising.", "  Low probability.", "\n \nQ: \r\nAnd would you feel comfortable taking someone that had been indicted and then\r\nwent out and committed the same offense?", "\n \nA: \r\nI don=t know if\r\ncomfortable is the word.", "  I would, my expectation level would be very low.", "\n \nQ: \r\nAnd if someone has been on probation before and messed up, does that also\r\neffect the likelihood of their success on probation?", "\n \nA: \r\nYes, sir, it can.", "  It shows an effort, that their effort wasn=t very good before.", "  It=s kind of to be expected.", " \r\nOr performance is not very good.", "\n \n.", "\r\n. . .", "\n \n            Q: \r\nWhat about if you were asked to put somebody on probation that had committed a\r\nsimilar crime for the one that you were putting them on probation for two weeks\r\nprior to that, would that give                        you concern?", "\n \n            A: \r\nOh, yes, sir.", "\n \n            Q: \r\nWhat if they had done two of those in the past month, month and a half, would\r\nthat concern you even more?", "\n \n            A: \r\nYes, sir.", "\n \n            Q: \r\nWhat about if they had done four in the past six months?", "  Past three months?", "\n \n            A: \r\nYes, sir.", "  I would like to know all of that.", "     \n \n            . .", "\r\n. .", "\n            \n            Q: \r\nIt doesn=t sound like\r\nsomebody that you would want on your caseload?", "\n \n            A: \r\nNo, sir, not at all.", "  It kind of keeps probation from being a catchall\r\nsituation.", "\n \nMartin\r\ncontends that his counsel was constitutionally ineffective for not objecting to\r\nthis opinion testimony, relying upon Mares v. State, 52 S.W.3d 886, 893\r\n(Tex. ", "App.", "CSan Antonio\r\n2001, pet. ", "ref=d).", " \r\nThere, the San Antonio Court held that defense counsel was constitutionally\r\nineffective for not objecting to a probation officer=s testimony that in his opinion the defendant\r\nwas not a good candidate for community supervision.", "  The court noted that trial\r\ncounsel=s sole\r\nstrategy was to obtain a probated sentence, concluded that this strategy was\r\nnot served by failing to object, and found, therefore, that counsel was\r\nconstitutionally ineffective.", "  Id.  Martin concludes that, because his\r\ntrial counsel=s only\r\nstrategy was to obtain community supervision, the failure to object constitutes\r\nineffective assistance of counsel.", "  There may be instances in which sound\r\nstrategy would not require an objection to a probation officer=s opinion, such as where\r\ncounsel decides not to emphasize a minor statement, but we agree with the San\r\nAntonio Court that in this instance no discernable trial strategy was served.", "\n\n\n\n\nHowever,\r\nbefore we can find that counsel was constitutionally ineffective, we must first\r\nconsider whether the failure to object was harmful.", "  Harm exists when there is\r\na reasonable probability that, but for counsel=s\r\nunprofessional error, the result of the proceeding would have been different.", "  In\r\nre M.S., 115 S.W.3d 534, 549-50 (Tex. ", "2003).", "  A reasonable probability is a\r\nprobability sufficient to undermine confidence in the outcome.", "  Thompson,\r\n9 S.W.3d at 812.", "  Our review must consider the totality of the evidence.", "  A\r\nverdict or conclusion only weakly supported by the record is more likely to\r\nhave been affected by errors than one with overwhelming record support.", "  Ex\r\nparte Guzman, 730 S.W.2d 724, 734 (Tex. ", "Crim. ", "App. ", "1987).", "\nIn\r\nMares, the court found harm because of the emphasis placed upon the\r\nprobation officer=s\r\ntestimony by the prosecution during closing argument.", "  52 S.W.3d at 893.", "  Mares\r\nhad a prior conviction for aggravated assault with a deadly weapon and was\r\nconvicted of sexual assault.", "  The probation officer described both offenses as\r\nviolent crimes and then testified that, in her opinion, someone who had\r\npreviously been on probation for aggravated assault would not be a good\r\nprobation candidate for a sexual assault offense.", "  Id.  The prosecution\r\nemphasized this testimony during closing argument, pointing out to the jury\r\nthat the probation officer was an expert; that, in her opinion, the defendant\r\nwas not a good candidate for probation; and that she was essentially telling\r\nthe jury not to place the defendant on probation.", "  The prosecutor then asked\r\nthe jury to follow the officer=s\r\nadvice because of her expertise.", "  Id.  \nHetrick=s opinion testimony was far\r\nmore substantial than was offered in Mares.", "  But, the State=s reliance upon it was far\r\nless substantial.", "  The State pointed out in closing argument that Martin had\r\npleaded guilty to two drug offenses and that, during the pendency of his\r\nindictment for them, he committed four other drug offenses within a three-month\r\ntime span.", "  The State then referred to Hetrick=s\r\ntestimony and said:  AYou\r\nhave heard the probation department say they don=t\r\nlike this -- the most that they have is three cases and they laid it all out\r\nfor you.@  The\r\nprosecutor made no other reference to Hetrick=s\r\ntestimony.", "  \n\n\n\n\nThe\r\npotential for harm in Mares was apparent because the defendant was\r\nsentenced to ten years confinement and, therefore, was eligible for probation.[3] \r\n52 S.W.3d at 888.", "  The jury, however, denied his probation request.", "  Id.\r\nat 893.", "  It is logical to conclude that the jury relied upon the probation\r\nofficer=s testimony to\r\ndeny probation, particularly when that testimony was so strenuously emphasized\r\nby the prosecution.", "  In this case, however, we cannot say that the jury relied\r\nupon Hetrick=s\r\ntestimony because the jury did not reach Martin=s\r\nrequest.", "  Martin was sentenced to twenty years confinement.", "  Consequently, he\r\nwas not eligible for community supervision.[4] \r\nFurthermore, in light of the overwhelming evidence of Martin=s past criminal behavior,\r\ntestimony that he was not a good candidate for community supervision was hardly\r\npersuasive.", "  Finally, because the jury gave Martin the maximum permissible\r\nperiod of incarceration and the maximum permissible fine for a second degree\r\nfelony, it is clear that probation was not a consideration.", "  Martin was not\r\nharmed by trial counsel=s\r\nfailure to object.", "  His first issue is overruled.", "\nTrial\r\nCounsel=s\r\nClosing Argument.", "\nMartin\r\nnext argues that his trial counsel was ineffective during closing argument\r\nbecause counsel alienated jurors and cast further disdain upon him.", "  The State\r\nopened and emphasized that Martin was a drug dealer; that he had been\r\nrepeatedly arrested for drugs; that he was given an opportunity as a juvenile\r\nto straighten up but did not do so; and that, even though he was under\r\nindictment, he was still committing crimes until shortly before the start of\r\ntrial.", "  The State concluded by arguing that Martin was a curse to the community\r\nand no longer belonged there.", "  In response, Martin=s counsel began his closing argument by\r\nsaying:\nWe\r\ndon=t want Ricky\r\nMartin in this society.", "  We don=t\r\nwant him mingling with the child molesters on probation.", "  And we don=t want him mingling with\r\nthe sex offenders on probation.", "  We don=t\r\nwant him mingling with people that have shot people who are on probation.", "  That\r\nhave committed violent offenses and are on probation.", "  We don=t want that.", "\n \nWe\r\nwant him to mingle with rapists, with robbers.", "  We want to throw him away like\r\nthat trash.", "  Like that.", "  You know.", "\n \nLet\r\nsomeone treat him like a punk, you know.", "  You got a pretty mouth.", "  Is that what\r\nwe want?", "  We want years and years of that.", "\n \n. . . . ", "\n \nOkay.", " \r\nWhat are we going to do with him?", "  Are we done with him?", "  Do we throw him away\r\nlike trash?", "  You know, trash goes somewhere and doesn=t come back.", "  We give up on this piece of\r\ntrash, on this animal.", "  This disgusting, nothing.", "\n \n\n\n\n\nHe is\r\ncoming back.", "  Coming back to this city.", "  He=s\r\ngoing to be released to here when he gets out.", "  You want him back? ", "\n \n. . . .", "\n \nYou\r\nknow, he=s trash and\r\nhe needs to be in there and not out here on probation.", "  You know, we can=t have that because we\r\nreserve that for sex offenders.", "  Does that make sense?", "\n \nYou\r\nknow, look at him.", "  He=s\r\n18.", "  Look. ", "This is as pretty as I can make him.", "  I couldn=t even get his mom up here\r\nto come testify, you know.", "  She gave up on him.", "  Do we?", "  Look.", "  F--k him.", "  Let\r\nhim get raped.", "\n \nThe trial court\r\nintervened and asked trial counsel to Ahold\r\nit down.@  Martin=s counsel then continued:\nYes,\r\nsir.", "  He had his chance.", "  He=s\r\n18.", "  He=s worthless.", " \r\nHe=s trash.", "  He should\r\nget raped.", "  We should forget about him.", "  Let him get raped and brutalized.", "\n \nIf\r\nwe are lucky he just learns when he joins a gang in prison to get protected and\r\nhe makes some really good contacts.", "  He learns how to be the high level drug\r\ndealer.", "  He meets the big time criminals now.", "  He makes connections.", "  And that=s if we are lucky.", "  He\r\ncomes out of prison knowing the real players.", "  Knowing the real drug dealers.", " \r\nHe comes out of prison knowing how fun it is to rape little girls.", "\n \nThe State\r\nobjected, and the trial court sustained the objection and instructed counsel to\r\nAmove on.@  Martin=s counsel then stated:\nYou\r\nknow what he=s going\r\nto learn in prison.", "  You know what type of people that there are in prison.", "\n \nDo\r\nwe want that thing that is so worthless now that we can=t even give him a chance, that is so pathetic,\r\nthat is such a piece of trash that we can=t\r\neven give him a chance today, do we want him back out here after having been\r\nsubjected to this?", "  Is that better for society?", "  Is that the right thing to do\r\nto give up right now?", "\n \nYou\r\nknow, that is the easy thing to do.", "  I mean, it=s\r\nfunny but the easy thing to do is send someone to prison sometimes.", "\n \nYou\r\nknow, there=s going to\r\nbe some people there in the jury box that say, well, he sold drugs.", "  He is a\r\ndrug dealer.", "  They need to go to prison.", "  Maybe they do.", "\n \n\n\n\n\nYou\r\nknow, if he had decided to snitch people off he wouldn=t be here.[5] \r\nAs bad and as trashy as he is, if he had of decided to do that he wouldn=t be here.", "\n \nYou\r\nknow, as dangerous, as vile, as worthless as he is, if he had worked for them\r\nit would have been all right regardless of what he does.", "\n \nWell,\r\nhe is not that dangerous.", "  He is helpless.", "\n \nThe\r\nreason that I brought the probation officer up here is for y=all to understand the type\r\nof people that are on probation.", "\n \nThe\r\ntype of people that are on probation are criminals.", "  They have committed bad\r\nacts.", "  They are bad people.", "  It=s\r\nnot us that are on probation.", "  It=s\r\nnot people like me and like you that are on probation.", "  It=s people like Ricky that\r\nare on probation.", "  \n \n. . . .", "\n \nDrugs\r\nare rotten and he=s\r\ndone rotten things and has lived a rotten life.", "  And he is 18.", "  But he hasn=t raped little girls.", "  He\r\nhasn=t shot people.", " \r\nHe hasn=t beat people\r\nup.", "  He hasn=t\r\nassaulted people.", "  He hasn=t\r\ndone those things.", "  Which is worse?", "\n \nSomehow\r\nbecause he=s 18 and\r\njust went, you know, kind of C\r\nwhat do you expect?", "  You saw his mom selling drugs, being involved in drugs.", " \r\nShe got more upset because they mentioned her boyfriend=s name than anything that has happened to\r\nRicky.", "\n \nYou\r\nknow, she didn=t even\r\nbother to C she was\r\nbringing his clothes.", "  Was the reason that we were late yesterday.", "  She didn=t even bother to show up to\r\ntestify today.", "\n \nI\r\ndidn=t think to\r\nsubpoena her but I should have.", "  You know, you wouldn=t think that you would need to subpoena\r\nsomebody=s mother.", "\n \nYou\r\nknow sometimes there=s\r\na reason people turn out rotten. . . .", "\n \nProbation.", " \r\nWhat can we do?", "  Is he somebody that we just cannot help on probation? ", "\n \n. . . .", "\n \n\n\n\n\nFirst\r\nof all, Ricky is going to have to get a job other than this job, you know.", "  If\r\nhe doesn=t have a job\r\nand he=s able to and I\r\npromise that he is able to, he can get revoked.", "\n \nIf\r\nhe hangs around those people that you heard about, that=s another rule of probation. . . . ", "\n \n. . . .", "\n \nIf\r\nhe tests dirty for drugs, keeps smoking that marijuana.", "  They will pee test\r\nyou. . . .", "\n \n            . . . .", "\n \nIf\r\nhe commits another crime, if gets another one of these, I can guarantee you he=s going to prison. . . .", "\n \n            . . . .", "\n \nWhat\r\nit comes down to is, what is going to serve society best?", "  What is the best\r\nthing tomorrow?", "  You know, a month from now, five years from now?", "\n \nLet\r\nhim be supervised by probation officers.", "  Let them try to teach him something.", "\r\n. . .", "\n \n. .", "\r\n.", "  I just ask that you give the probation office, this Court, a chance to do\r\nsomething before we admit failure. ", "\n \n. . . .", "\n \nAnd\r\nhe=s not a piece of\r\ntrash.", "  He=s not an\r\nanimal.", "  He is a stupid, idiot kid.", "  He has never known anything but this.", "  He\r\nwas never told by his mother this isn=t\r\nthe way that you live.", "\n \n. . . .", "\n \nIf\r\n[probation officers] can make a difference, if they are making a difference, if\r\nthey had made a difference, I ask y=all\r\nto give them a chance to make a difference with Ricky.", "  With an 18 year old.", " \r\nWho is eligible for probation.", "  That was the stipulation.", "\n \nI\r\njust ask that you give them a chance.", "  If he fails it, he fails it.", "  But give\r\nthem a chance, give him a chance.", "\n\n\n\n\nWe\r\nhave quoted large portions of counsel=s\r\nargument in an effort to place the challenged statements in context.", "  It is\r\ndifficult for an appellate court to fully appreciate the impact of a closing\r\nargument because we are limited to a review of the record.", "  What counsel says\r\nis important.", "  The record illuminates us with this.", "  But, how it is said is\r\nvitally important because the tone and manner of delivery can significantly\r\nimpact the message conveyed, particularly in instances such as this where\r\ncounsel was clearly attempting to use sarcasm.", "  The record fails to provide us\r\nwith this critical information and, therefore, makes it difficult for us to\r\ndetermine the effect of counsel=s\r\nargument on the jury.", "\nThe\r\nuse of sarcasm or a dramatic shocking statement at the beginning of a closing\r\nargument can be sound trial strategy.", "  The jury heard eight police officers and\r\none probation officer testify that Martin was a crack dealer with a bad reputation.", " \r\nSome police officers even described him as  more than a street dealer, but as a\r\nmid or upper level dealer.", "  The jury knew that Martin had been arrested six\r\ntimes as an adult for selling crack cocaine and that his last arrest was two\r\nweeks before trial.", "  They knew that he failed probation as a juvenile because\r\nof an arrest for selling crack cocaine.", "  The State=s overwhelming evidence that Martin was a\r\ncareer drug dealer with little regard for the law and that he was selling\r\nthousands of dollars of crack cocaine in the community undoubtedly weighed\r\nheavily against any consideration of probation.", "  Counsel=s challenge was to overcome the perception\r\nthat Martin was a threat to society and to convince the jury to consider him as\r\na person and view him as someone worthy of a chance at rehabilitation.", "  That\r\nchallenge called for a dramatic closing argument.", "\n\n\n\n\nBut\r\nif counsel=s strategy\r\nwas appropriate, his execution was not.", "   The State concedes that counsel=s attempt at sarcasm was Abeyond the pale.@  Texas courts have held\r\nthat counsel=s\r\nprejudicial references to his own client can constitute ineffective\r\nassistance.", "  See, e.g., Ex parte Guzman, 730 S.W.2d at 726 (referring to\r\nclient as a Awet-back@); see also Ramirez v.\r\nState, 65 S.W.3d 156, 157 (Tex. ", "App.", "CAmarillo\r\n2001, pet. ", "ref=d)\r\n(counsel told jury during closing argument that he did not want them to\r\nperceive his client Aas\r\na drunk Mexican@).", " \r\nTexas courts have also held that offensive conduct by counsel can constitute\r\nineffective assistance.", "  See, e.g., Miller v. State, 728 S.W.2d 133, 135\r\n(Tex. ", "App.", "CHouston\r\n[14th Dist.] ", "1987, pet. ", "ref=d)\r\n(counsel referred to complainant, who was from Nigeria, as Aa man swinging from limb to\r\nlimb with a banana or coconut in one hand@).", "\nThe\r\ndistinction between these cases and the one before us is the contrast between\r\nthe challenged conduct and the remainder of the trial. ", " For example, in Guzman,\r\n730 S.W.2d at 725-27, 730-31, trial counsel repeatedly and intentionally\r\nreferred to his client as a Awet-back.@  In Ramirez,\r\ncounsel affirmatively introduced evidence that his client was an illegal alien,\r\nraised no objection when the State inquired into the extent of his client=s drinking B even though there was no\r\nevidence that alcohol played a part in the incident, and then used and allowed\r\nthe State to use a racial slur to describe his client.", "  65 S.W.3d at 157-58. ", "\nIn\r\nthis instance, Martin complains of his counsel=s\r\nreferences to him as Atrash@ and Athis disgusting nothing@ and of his statements, AF--k him.", "  Let him get\r\nraped.@  When the\r\nentirety of counsel=s\r\nclosing argument is considered, it is clear that he did not think his client\r\nwas trash nor did he want the jury to do so.", "  Instead, he was attempting to\r\nutilize a dramatic opening to his argument to overcome the effect of the State=s overwhelming evidence and\r\nto convince the jury that his client was a person, that he deserved another\r\nchance, and that placing him on probation instead of sending him to prison\r\nprovided the best opportunity to salvage a life worth saving. ", "\n\n\n\n\nWhen\r\ncounsel=s plea for\r\nprobation is contrasted with the entire record, it is clear that he was not\r\nengaged in a pattern of offensive conduct but was desperately trying to obtain\r\none last chance for a defendant with a poor track record.", "  During voir dire, he\r\nprepared the panel to consider probation for an individual guilty of multiple\r\ndrug offenses.", "  He verified that jurors could consider probation, and he\r\nsuccessfully challenged those for cause who indicated that they could not do\r\nso.", "  He told potential jurors that people who were guilty of very serious\r\noffenses, including murder, were on probation, and he explained how probation\r\nworked.", "  During his opening statement, counsel contended that probation was a\r\nviable option because of the services and programs available, that Martin was\r\neligible for probation, and that this would be his one and only chance.", " \r\nCounsel advised the jury that he would be Aforceful@ about this being Martin=s one last chance.", "  During\r\ntrial, counsel vigorously cross-examined each of the State=s witnesses, and he\r\nobtained the full benefit of Section 12.45 by having four unadjudicated\r\noffenses dismissed without Martin having to admit to them.", "  In fact, he\r\nsuccessfully obtained a reasonable doubt instruction when the first evidence of\r\nan unadjudicated offense was introduced.", "\nCounsel\r\ncould have utilized more professional language during closing argument and\r\nshould have been more mindful of the trial court=s\r\nadmonitions.", "  Whether that falls below an objective standard of reasonableness,\r\nwe need not decide because, when the record is viewed as a whole, we cannot say\r\nthat counsel=s poor\r\nchoice of language was harmful.", "  Martin received incarceration because of his\r\nown conduct and not because of his counsel=s\r\nclosing argument.", "  Martin=s\r\nsecond issue is overruled.", "\n                                                                        Holding\n            The\r\njudgment of the trial court is affirmed.", "\n                                                                              \n \nRICK STRANGE\nJUSTICE\n \nFebruary 7, 2008\nDo not publish.", " \r\nSee Tex. ", "R. App. ", "P. 47.2(b).", "\nPanel consists of:  Wright, C.J.,\nMcCall, J., and Strange, J.\n\n\n\n\n     [1]Tex. ", "Penal Code Ann. ' ", "12.45 (Vernon 2003).", "\n\n\n     [2]Section 12.45\r\nprovides in part: \n \n(a) A person may, with the consent of the attorney for\r\nthe state, admit during the sentencing hearing his guilt of one or more\r\nunadjudicated offenses and request the court to take each into account in\r\ndetermining sentence for the offense or offenses of which he stands adjudged\r\nguilty.", "\n \n(c) If a court lawfully takes into account an admitted offense,\r\nprosecution is barred for that offense.", "\n\n\n     [3]Tex. ", "Code Crim. ", "Proc. ", "Ann. ", "art. ", "42.12, ' 4 (Vernon Supp. ", "2007).", "\n\n\n     [4]Article 42.12, ' 4(d)(1).", "\n\n\n     [5]This was a\r\nreference to one of the State=s witnesses,\r\nJesse Saldana.", "  The State agreed to dismiss a prostitution charge against\r\nSaldana if he agreed to help the police Amake\r\na case.@  Saldana helped arrange the August 8 drug buy.", "\n\n\n" ]
{ "pile_set_name": "FreeLaw" }
[ 0.0022222222222222222, 0.007662835249042145, 0.00847457627118644, 0, 0, 0.013157894736842105, 0.007407407407407408, 0.008547008547008548, 0, 0, 0.009433962264150943, 0.011299435028248588, 0.01020408163265306, 0.017142857142857144, 0, 0.004878048780487805, 0, 0.017391304347826087, 0.006622516556291391, 0.01282051282051282, 0, 0, 0, 0, 0, 0, 0.009009009009009009, 0.009615384615384616, 0.008928571428571428, 0.022727272727272728, 0, 0, 0.01098901098901099, 0.011904761904761904, 0, 0.006134969325153374, 0.006230529595015576, 0.007751937984496124, 0.008547008547008548, 0, 0.004975124378109453, 0.02912621359223301, 0.16666666666666666, 0, 0, 0, 0.044444444444444446, 0.16666666666666666, 0, 0, 0, 0.061224489795918366, 0.16666666666666666, 0, 0.01639344262295082, 0.16666666666666666, 0, 0, 0, 0.04878048780487805, 0.16666666666666666, 0, 0, 0, 0.025, 0.16666666666666666, 0, 0.02040816326530612, 0.16666666666666666, 0, 0.14285714285714285, 0.007936507936507936, 0.019417475728155338, 0.004329004329004329, 0.02054794520547945, 0.00641025641025641, 0, 0, 0, 0, 0, 0, 0, 0, 0, 0, 0, 0, 0, 0, 0, 0, 0, 0, 0, 0, 0, 0, 0, 0, 0, 0, 0, 0, 0, 0.025, 0, 0.017543859649122806, 0, 0.04, 0.14285714285714285, 0.004329004329004329, 0, 0.005555555555555556, 0.0034965034965034965, 0, 0, 0, 0, 0, 0.06896551724137931, 0, 0, 0.021739130434782608, 0.16666666666666666, 0, 0, 0.006756756756756757, 0.05263157894736842, 0.008849557522123894, 0, 0, 0.010526315789473684, 0, 0.016129032258064516, 0.01327433628318584, 0.011029411764705883, 0.0055248618784530384, 0, 0, 0, 0.014705882352941176, 0, 0.004016064257028112, 0.009900990099009901, 0, 0, 0.027777777777777776, 0, 0.006269592476489028, 0.019230769230769232, 0.034482758620689655, 0, 0, 0, 0, 0, 0, 0, 0, 0, 0, 0, 0, 0, 0, 0, 0, 0, 0, 0, 0, 0, 0, 0, 0, 0, 0, 0, 0, 0, 0.038461538461538464, 0, 0, 0, 0, 0, 0, 0, 0, 0, 0.008403361344537815, 0, 0, 0, 0, 0, 0, 0, 0, 0, 0, 0, 0, 0, 0, 0, 0.01092896174863388, 0, 0, 0, 0, 0, 0, 0, 0, 0, 0, 0, 0, 0, 0, 0, 0, 0, 0, 0, 0, 0.020833333333333332, 0, 0.01282051282051282, 0, 0, 0, 0, 0, 0, 0, 0, 0, 0.009259259259259259, 0, 0, 0, 0, 0, 0, 0, 0, 0, 0, 0.011363636363636364, 0, 0, 0, 0, 0, 0, 0, 0, 0, 0, 0, 0, 0, 0, 0, 0, 0.008928571428571428, 0, 0, 0, 0, 0, 0, 0, 0.00546448087431694, 0, 0, 0, 0, 0, 0, 0, 0, 0, 0, 0, 0, 0, 0.007874015748031496, 0, 0.006756756756756757, 0, 0.007905138339920948, 0.00975609756097561, 0, 0, 0.01, 0.028368794326241134, 0, 0.045454545454545456, 0.016129032258064516, 0, 0.03508771929824561, 0, 0.043478260869565216, 0, 0.0070921985815602835, 0, 0.006224066390041493, 0.043478260869565216, 0.013605442176870748, 0, 0.0028089887640449437, 0, 0, 0, 0, 0.0045045045045045045, 0.01020408163265306, 0.00904977375565611, 0, 0, 0, 0, 0, 0, 0.0072992700729927005, 0, 0.125, 0.09090909090909091, 0.0625, 0, 0, 0, 0, 0, 0.09090909090909091, 0, 0, 0, 0.04, 0, 0, 0.024691358024691357, 0.03067484662576687, 0 ]
0.010061
5
[ "Alfred Gray\n\nAlfred Gray may refer to:\n\nAlfred Gray (mathematician) (1939–1998), American mathematician\nAlfred M. Gray, Jr. (born 1928), US Marine Corps general \nAlfred Gray (Kansas politician) (1830–1880), American politician from Kansas\nAlf Gray (1910–1974), English football half back\nAlf Gray (Australian footballer) (1874–1931), played for Essendon\n\nSee also\nAlf Grey (born 1935), English former football referee" ]
{ "pile_set_name": "Wikipedia (en)" }
[ 0.02877697841726619 ]
0.028777
5
[ "FDA proposes major hike in drug industry user fees\n\nThe US Food and Drug Administration has proposed recommendations to Congress for the next reauthorisation of the Prescription Drug User Fee Act (PDUFA) which, if adopted, “would significantly broaden and upgrade the agency's drug safety programme,” increase resources for review of TV drug advertising and “facilitate more efficient development of safe and effective new medications for the American public.”", "\n\nThe FDA has recommended that annual user fees paid by drugmakers to have new products reviewed will increase by almost 30%, to $392.8 million, which it says would enable it to upgrade its drug safety programme and increase its resources to review television drug advertisements.", "\n\nNearly $38 million would be used to \"ensure the safety\" of approved drugs, and would enable the agency to hire 82 staff to perform post-marketing safety activities. ", "About $8.6 million would be allocated to help the agency develop guidelines on designs for clinical trials and to move the FDA towards an \"all-electronic environment.\"", "\n\nThe agency also plans to recommend that a separate new user-fee programme be created for pharmaceutical firms who want the agency to review their direct-to-consumer TV ads. ", "This scheme would bring in around $6.2 million in fees in the first year, which would be used to employ 27 people to carry out the reviews.", "\n\nDepartment of Health and Human Services' Secretary Mike Leavitt said the recommendations would greatly help the FDA’s “ability to monitor and respond to emerging drug safety issues,” as well as continuing the agency’s “commitment to scientific improvements and streamlining the drug approval process.\"", "\n\nThe chief executive of the Pharmaceutical Research and Manufacturers of America, Billy Tauzin, backed the PDUFA proposals, saying they call for “comprehensive improvements in all areas that are essential to timely safe and effective use of new medicines by millions of American patients.”", "\n\n\"Tainted and comprimised\"\n\nMuch less happy was the US consumer group Public Citizen. ", "Sidney Wolfe, director of its health research group, said that “the FDA's crucial drug regulatory functions are too important to be tainted and compromised by direct funding from the very companies whose drugs the agency reviews for safety.” ", "He added that the FDA \"should request these additional funds through the same process that funded the agency from 1906 through 1992 – that is, the money should come from the federal Treasury, not the pharmaceutical industry.”", "\n\nMr Wolfe concluded by saying that the entire annual amount of funds now sought from industry under PDUFA “is equivalent to fewer than two days of the current expenditures for the disastrous war in Iraq. ", "Where are the congressional priorities?”", "\n\nIf approved the measures would come into effect from October 1 this year and the public will be able to comment on the draft proposal at a meeting scheduled for February 16." ]
{ "pile_set_name": "Pile-CC" }
[ 0.008695652173913044, 0.0035714285714285713, 0, 0.005988023952095809, 0, 0, 0.006600660066006601, 0.010344827586206896, 0.011494252873563218, 0.008264462809917356, 0.008888888888888889, 0, 0, 0 ]
0.004561
5
[ "Saeideh Aletaha, 26, died on Sunday after reportedly suffering a brain injury during an amateur kickboxing fight in Southampton, England (per Sky News). ", "Aletaha had been competing at Fast & Furious 19, an event that hosted MMA, kickboxing and boxing.", "\n\nAccording to the BBC Aletaha, who worked as a product design engineer and lived in Salisbury, collapsed at the event and was taken to Southampton General Hospital at around 9:00 PM local time. ", "The Sun reported that Aletaha died the following day as a result of her injuries.", "\n\nFollowing the show Saturday we regret to announce that following one of our matches, one of the competitors, Sai Aletaha... Posted by Fast & Furious Fight Series on Monday, November 18, 2019\n\nFast & Furious Fight Series wrote about Aletaha on their official Facebook page. ", "That statement reads as follows:\n\nFollowing the show Saturday we regret to announce that following one of our matches, one of the competitors, Sai Aletaha from Lookborai who lost, unfortunately suffered an injury leaving her in a critical state that she tragically has not recovered from. ", "All competitors get in prepared that they may be injured and this is something not expected to happen 99.9% of the time. ", "But, it can and in this we make the environment as safe as possible with pre and post medicals from a Doctor, and full medical cover through out with Doctor, Paramedic and Ambulance on site alongside our experienced team of staff with numerous first aiders, safety is not something ever skimped on in any of our 19 shows and all matches are made equal. ", "Thoughts of course go out to all family, friends and team mates, if anyone needs any support please come to us or your coaches. ", "Thank you to our team on the night who acted with utmost professionalism and carried their duties out above and beyond to react to the situation, and again we are here for any of you that needs it in this sad time.", "\n\nLookborai, the gym where Aletaha trained, also posted about Aletaha on Facebook. ", "The gym stated that, “Aletaha was a lovely character with a beautiful soul. ", "Her dedication to the sport was 110% traveling miles every day just to train. ", "She found her place with us just a few months ago but has become apart of the family and will be sorely missed.”", "\n\nFollowing the show Saturday we regret to announce that one of our team mates unfortunately suffered an injury leaving... Posted by Lookborai on Monday, November 18, 2019\n\nStonehenge CrossFit, another gym were Aletaha trained, also paid tribute to Aletaha via a Facebook post. ", "That gym wrote that Aletaha was “popular, kind and liked by everyone in the community.” ", "That post stated that around 30 members of the gym visited Aletaha’s bedside on Sunday to “say goodbye to their friend.”", "\n\nSaeideh Aletaha was just 26 when she died on Sunday. ", "She had been a member at Stonehenge CrossFit for more than three... Posted by Stonehenge CrossFit on Monday, November 18, 2019\n\nAccording to a statement from Hampshire Police, law enforcement were contacted at around 1:00 AM on Sunday by the Southampton General Hospital informing them that Aletaha was receiving treatment for a life-threatening brain injury.", "\n\nA police investigation into Aletaha’s death is ongoing." ]
{ "pile_set_name": "OpenWebText2" }
[ 0.006535947712418301, 0.030927835051546393, 0.010256410256410256, 0.012345679012345678, 0.01090909090909091, 0.0034602076124567475, 0, 0, 0, 0, 0.024096385542168676, 0.013157894736842105, 0, 0, 0.007194244604316547, 0.011363636363636364, 0.008333333333333333, 0.01818181818181818, 0.013927576601671309, 0.017543859649122806 ]
0.009412
5
[ "14*y for y.\n30\nSolve 0 = 6643*l - 6465*l + 6586 for l.\n-37\nSolve 88*p + 891*p + 749*p = 226*p - 106642 for p.\n-71\nSolve -5375 = 17120*h - 16995*h for h.\n-43\nSolve 3210*t = 8333*t - 522546 for t.\n102\nSolve 0 = 4100*y - 4219*y - 1071 for y.\n-9\nSolve -531*n + 486*n - 22 = 1082*n + 16883 for n.\n-15\nSolve 1715*q + 2735*q = -186*q - 578*q + 20856 for q.\n4\nSolve 6*h - 240*h - 182*h - 11232 = 0 for h.\n-27\nSolve -1441*z - 4219*z + 132261 + 377086 = -409*z for z.\n97\nSolve 109*s + 98070 - 102345 = -4166*s for s.\n1\nSolve 132*b - 120*b = 125*b + 121*b + 9828 for b.\n-42\nSolve -2692*g - 220080 = -1140*g + 619*g + 2414*g for g.\n-48\nSolve 1812 = 208*m - 7132 for m.\n43\nSolve 402*k - 5002*k = -271400 for k.\n59\nSolve 157*p = 227*p - 272*p + 19190 for p.\n95\nSolve 193175 + 9326 = -5473*w for w.\n-37\nSolve 230*z + 136*z = -8418 for z.\n-23\nSolve 1662 = 179*y - 5800 - 2383 for y.\n55\nSolve -808*i = 2557*i + 77395 for i.\n-23\nSolve 4384*d - 4588*d - 15708 = 0 for d.\n-77\nSolve -400*c = 6407762 - 6362562 for c.\n-113\nSolve 4608*x - 4175*x = -12990 for x.\n-30\nSolve 592 = 663*t - 595*t - 1220 - 1792 for t.\n53\nSolve -99*n + 51*n + 30*n - 902 = -40*n for n.\n41\nSolve 1219*k + 2552 = -25485 for k.\n-23\nSolve -2244*j = -2138*j + 954 for j.\n-9\nSolve -883*v - 721*v + 1505*v = 1485 for v.\n-15\nSolve -3134 - 773 = 201*a + 5945 + 2610 for a.\n-62\nSolve 1898*n = -31191 + 8415 for n.\n-12\nSolve 8 = 530*u - 331 - 191 for u.\n1\nSolve -271*p + 1487 = -9428 - 13380 - 3347 for p.\n102\nSolve -3124*q = 453*q + 164542 for q.\n-46\nSolve 78*m = -72*m + 182*m + 448 for m.\n-14\nSolve -39*a - 31*a - 98*a = 107*a - 9350 for a.\n34\nSolve 2544*h - 224496 = -2133*h for h.\n48\nSolve -9065*o - 212337 = 75907 + 91752 + 91384 for o.\n-52\nSolve -73781 = -6724*o + 7553*o for o.\n-89\nSolve 11507*v + 4581*v = -1464008 for v.\n-91\nSolve -750*n + 4639 = -2565*n - 18956 for n.\n-13\nSolve 1345*k = 521*k + 6965 + 1275 for k.\n10\nSolve 2056 + 8068 = 203*r - 5101 for r.\n75\nSolve 211653*b = 211453*b + 200 for b.\n1\nSolve 516*d = 497*d + 779 for d.\n41\nSolve -84*d + 19*d - 55*d + 2514 + 6 = 0 for d.\n21\nSolve 8036*z = 7573*z - 2315 for z.\n-5\nSolve -8947*d + 713 = -8978*d for d.\n-23\nSolve -469*p + 884*p - 468*p + 6033 = -486 for p.\n123\nSolve -50*v + 8762 = 287*v for v.\n26\nSolve -898 = -159*b + 3236 for b.\n26\nSolve 138*v + 13940 = -199*v - 3*v for v.\n-41\nSolve 44*c + 20*c - 428*c + 9648 = -163*c for c.\n48\nSolve -3785*i = -1397*i - 51862 - 15002 for i.\n28\nSolve 1199*c + 45121 = 27136 for c.\n-15\nSolve 144*r + 59*r + 9542 = -143*r - 21*r for r.\n-26\nSolve 0 = 165*m - 28641 + 23691 for m.\n30\nSolve 10*g + 44838 = 90996 - 45278 for g.\n88\nSolve -45527 + 131707 = -4309*g for g.\n-20\nSolve 0 = 190*z - 3902 + 2235 - 3083 for z.\n25\nSolve -1478*p + 37538 = -229*p - 17418 for p.\n44\nSolve 149908 = -3418*h - 154294 for h.\n-89\nSolve -975*a = 790*a + 1212 + 23498 for a.\n-14\nSolve -236*l - 372*l - 22496 = 0 for l.\n-37\nSolve -24835*q - 1320 = -24890*q for q.\n24\nSolve 58681*f - 58813*f + 2605 = 10393 for f.\n-59\nSolve 2366 + 2708 - 4694 = 20*j for j.\n19\nSolve -968*c = -2541*c + 8234 + 26372 for c.\n22\nSolve 0 = 10766114*g - 10766202*g - 1584 for g.\n-18\nSolve 2347*f + 363*f = -233060 for f.\n-86\nSolve 163*i = 7416 + 734 for i.\n50\nSolve 0 = 263535*b - 131756*b - 131763*b + 896 for b.\n-56\nSolve -3889*b + 3439*b - 13050 = 0 for b.\n-29\nSolve 4732*g - 73587 + 19437 + 6458 - 61144 = 0 for g.\n23\nSolve -914 + 61 = -15*i + 24*i - 160 for i.\n-77\nSolve 0 = 450*y + 99983 - 132833 for y.\n73\nSolve 17*u = 783*u - 27576 for u.\n36\nSolve 80339 = 109*d + 86116 for d.\n-53\nSolve -63*w + 704*w - 98*w - 477 = 66 for w.\n1\nSolve -1947*t + 5983*t = -2090*t - 214410 for t.\n-35\nSolve 3175*y = -36127 - 233748 for y.\n-85\nSolve 5840592 = 163*v + 5828856 for v.\n72\nSolve -43*c + 23*c + 108 = -11*c + 387 for c.\n-31\nSolve -917 + 2983 = 107*y - 1462 - 966 for y.\n42\nSolve -16722 - 7861 = -793*w for w.\n31\nSolve -70380 = 4*v - 70428 for v.\n12\nSolve -986 = -106*j - 33 + 1591 for j.\n24\nSolve -764 - 6630 = -619*x + 2510 for x.\n16\nSolve -2940*y + 2419*y - 8857 = 0 for y.\n-17\nSolve -2122*k + 1969*k = 765 for k.\n-5\nSolve -1087*j + 64*j + 128773 = -40*j for j.\n131\nSolve 254*r + 93*r = 161*r + 5022 for r.\n27\nSolve 109*y = -28*y + 11325 + 2786 for y.\n103\nSolve 795*s + 204*s + 507*s = -24096 for s.\n-16\nSolve 289541 = 1732*b + 2777*b + 1400*b for b.\n49\nSolve 7450 - 6268 = 63*k - 166*k - 14165 for k.\n-149\nSolve 181*k - 22*k - 47*k = -571*k + 56689 for k.\n83\nSolve 92*r + 475*r - 14742 = 0 for r.\n26\nSolve -427*n + 4638 + 3376 = -4306 - 33369 for n.\n107\nSolve 4762*i - 595*i - 336549 = -2432*i for i.\n51\nSolve 0 = -155*f - 1308 + 2548 for f.\n8\nSolve 21061 = -731*h - 5986 for h.\n-37\nSolve -109345 - 23365 = -1154*i for i.\n115\nSolve 87*v - 698*v + 108*v + 57915 = 784*v for v.\n45\nSolve -224*s - 63*s - 13860 = -56*s for s.\n-60\nSolve 477*u = -511*u + 1054*u - 2772 for u.\n42\nSolve 8*b + 40*b - 104*b - 1258 = 18*b for b.\n-17\nSolve -226*t + 1479 = -4623 for t.\n27\nSolve 26*f - 193*f - 216*f + 79*f - 6384 = 0 for f.\n-21\nSolve 12783 = 45*b + 6163 + 6350 for b.\n6\nSolve 382*j - 406*j = -509*j - 20855 for j.\n-43\nSolve -66*y - 3237907 = -3241207 for y.\n50\nSolve 1481*d + 40180 = 333*d for d.\n-35\nSolve 3000 = -45*q - 75*q for q.\n-25\nSolve -3644 = -39*t - 4580 for t.\n-24\nSolve 1272*l - 41713 = -7369 for l.\n27\nSolve 1844 = 70*i - 161 + 395 for i.\n23\nSolve 638*b = -5834 + 858 - 5870 for b.\n-17\nSolve -1591*w + 1236 + 252 = -1498*w for w.\n16\nSolve 1311*t + 10160 = 2225*t - 1549*t for t.\n-16\nSolve -322 = -384914*g + 384900*g for g.\n23\nSolve 5218*g + 12134 = 4634*g + 32574 for g.\n35\nSolve 56*y + 3200 + 7554 = 237*y - 1011 for y.\n65\nSolve -163*f = -1254 - 2332 for f.\n22\nSolve -14*u - 226*u + 26*u + 83 - 297 = 0 for u.\n-1\nSolve 492*a = -1941*a - 38928 for a.\n-16\nSolve 1952*d = 2325*d + 17531 for d.\n-47\nSolve 124*q = 328*q - 6936 for q.\n34\nSolve 86*b + 144*b + 1068 = -1931 - 681 for b.\n-16\nSolve -172*k - 12882 - 10391 = 185*k + 272*k for k.\n-37\nSolve 8336*n = -168*n - 442208 for n.\n-52\nSolve 2827*z = -602*z - 89154 for z.\n-26\nSolve -1515*p - 111941 = 3242*p - 1146*p for p.\n-31\nSolve 8471*v = 8318 + 643949 for v.\n77\nSolve 669288022*k + 5192 = 669288081*k for k.\n88\nSolve -895*x + 17304 - 49524 = 0 for x.\n-36\nSolve 131*y + 182*y - 93*y = -70*y + 13340 for y.\n46\nSolve 2780*k - 705*k + 147867 = -1740*k - 2614*k for k.\n-23\nSolve -8215397 + 8207339 = 237*l for l.\n-34\nSolve 12*a + 7 - 2 = 21*a - 13 for a.\n2\nSolve 1932*z + 2344 = -436*z + 35496 for z.\n14\nSolve 0 = -45*r - 408*r - 896*r + 51262 for r.\n38\nSolve -282*d + 982*d - 10385 - 26715 = 0 for d.\n53\nSolve 728*t - 232*t - 214*t - 250*t = -1696 for t.\n-53\nSolve 0 = 7*x - 11*x + 27*x + 4*x + 1053 for x.\n-39\nSolve -5*v + 56*v - 1595 = -1646 for v.\n-1\nSolve 1408*l = 790*l + 10470 - 40134 for l.\n-48\nSolve -657268*k = -655616*k - 8260 for k.\n5\nSolve 679*s + 39364 + 24799 = -11885 for s.\n-112\nSolve -4212*q + 3966*q - 16773 = -291 for q.\n-67\nSolve -105*g = -50*g - 317*g - 27*g + 6069 for g.\n21\nSolve 686 = -252*r + 683 + 1263 for r.\n5\nSolve -130317274*a - 22140 = -130317643*a for a.\n60\nSolve 3586*l + 2194*l + 104420 = 559*l for l.\n-20\nSolve 47735*h = 45387*h - 152620 for h.\n-65\nSolve 162*x + 60*x + 6490 + 4475 = -1689 for x.\n-57\nSolve 1258*i - 15430 = 7214 for i.\n18\nSolve 49260 = -308*s + 16304 for s.\n-107\nSolve -461*y + 29045 = 230*y + 207*y - 113*y for y.\n37\nSolve 2983*t - 2877*t - 212 = 0 for t.\n2\nSolve -64*t - 7*t - 1130 + 4680 = 0 for t.\n50\nSolve -18*p + 52*p = -65*p - 3465 for p.\n-35\nSolve -56*c = -200*c + 73*c - 88*c - 8427 for c.\n-53\nSolve 0 = 1142*c + 4049 - 694 + 3552 - 55 for c.\n-6\nSolve -133742*s = -133808*s - 726 for s.\n-11\nSolve -35513 + 37675 = 46*c for c.\n47\nSolve -55386 = -195*z - 1428*z - 121*z + 5654 for z.\n35\nSolve -956*i = 4555 + 6917 for i.\n-12\nSolve 400 + 301 = 8*k - 16*k + 149 for k.\n-69\nSolve 1867702*l = 1867437*l + 265 for l.\n1\nSolve 89*i = 160*i + 131*i + 9090 for i.\n-45\nSolve -286*f - 330*f + 10660 = -411*f for f.\n52\nSolve 9797*h + 1440 = 9765*h for h.\n-45\nSolve 13*q - 13*q = 8*q + 370 + 30 for q.\n-50\nSolve -2285*w + 50908 = 16867 - 36794 for w.\n31\nSolve -2547 = -707*n - 1190 - 4834 - 4300 for n.\n-11\nSolve 116*u - 3012 = 4064 for u.\n61\nSolve -826*v + 42880 - 16606 = -43110 for v.\n84\nSolve 152*d - 39*d + 144*d - 73*d = -2944 for d.\n-16\nSolve -141*k + 95*k - 1118 = -72*k for k.\n43\nSo" ]
{ "pile_set_name": "DM Mathematics" }
[ 0.006834268977300464 ]
0.006834
5
[ "The present invention relates in general to online Internet advertising optimization, and more specifically to analysis of relationships between web browser agent data and characteristics and behavior of website visitors.", "\nThere are many advertising systems and methods which are used to select advertisements for display on Internet websites. ", "These advertising systems use various strategies and logic to select which products and/or services may be of interest to an individual website visitor and how advertisements for the selected products should appear within a website. ", "There are many competing ideas and many different approaches to designing the logic which is used to select and display the advertising.", "\nOne such strategy is based on data which is collected for each website visitor. ", "In this strategy, a unique identifier (commonly, an Internet browser cookie) is downloaded from the website to the visitors computer. ", "This unique identifier allows the advertising system to tag the visitor and recognize the visitor's visits to the website as a discrete individual. ", "Further, data observed during the visit to the website is collected and stored in a data base and indexed to the unique identifier. ", "This allows the advertising network to cross reference the stored data about the visitor in the database each time the visitors computer requests a web page. ", "Observing a visitor's habits allows the advertising network to better determine which ads to display based on the stored data. ", "A website may also retrieve visitor preferences and interests stored at the website by identifying a returning visitor. ", "The different kinds of data which may be gathered and the means of referencing the data based on the users identifier are important aspects of the strategy.", "\nOne common way to gather visitor interests is to observe the visitor's path through the website and noting the topics of the pages which the visitor views. ", "Another way to gather data is to request that the visitor fill out a survey and then store the survey information for future use by the advertising system when the visitor returns. ", "A third common way to gather this data is by saving information supplied by the visitor when purchasing goods. ", "Oftentimes, the billing address given at the conclusion of an e-commerce transaction can be used to purchase demographic data from companies which compile such information on a wide basis. ", "Thus, there are several existing approaches to collecting and referencing data for online advertising systems.", "\nThere are several concerns and problems with known methods of data collection and indexing. ", "One overarching issue is that the visitor's privacy is threatened by the combined data gathering. ", "Another potential issue is that the cookie used to store the visitor's unique identifier resides on the visitor's computer system. ", "Visitors often delete these cookies and thereby defeat the ability to recognize repeat visits. ", "In addition, due to privacy concerns, a market has developed for software applications which remove cookies placed by advertising systems. ", "The result of removing the unique identifying cookie is that the advertising network can no longer reference information stored in the database for that visitor, and may incorrectly identify future visits by the same visitor as an additional visitor." ]
{ "pile_set_name": "USPTO Backgrounds" }
[ 0, 0, 0, 0, 0, 0, 0, 0, 0, 0, 0, 0, 0, 0, 0, 0, 0, 0, 0, 0, 0, 0, 0 ]
0
5
[ "#\n# Copyright:: Copyright (c) Chef Software Inc.\n# License:: Apache License, Version 2.0\n#\n# Licensed under the Apache License, Version 2.0 (the \"License\");\n# you may not use this file except in compliance with the License.", "\n# You may obtain a copy of the License at\n#\n# http://www.apache.org/licenses/LICENSE-2.0\n#\n# Unless required by applicable law or agreed to in writing, software\n# distributed under the License is distributed on an \"AS IS\" BASIS,\n# WITHOUT WARRANTIES OR CONDITIONS OF ANY KIND, either express or implied.", "\n# See the License for the specific language governing permissions and\n# limitations under the License.", "\n#\n\nrequire_relative \"event_dispatch/base\"\n\nclass Chef\n class ActionCollection < EventDispatch::Base\n include Enumerable\n extend Forwardable\n\n class ActionRecord\n\n # @return [Chef::Resource] The declared resource state.", "\n #\n attr_accessor :new_resource\n\n # @return [Chef::Resource] The current_resource object (before-state). ", " This can be nil\n # for non-why-run-safe resources in why-run mode, or if load_current_resource itself\n # threw an exception (which should be considered a bug in that load_current_resource\n # implementation, but must be handled), or for unprocessed resources.", "\n attr_accessor :current_resource\n\n # @return [Chef::Resource] the after_resource object (after-state). ", " This can be nil for\n # non custom-resources or resources that do not implement load_after_resource.", "\n attr_accessor :after_resource\n\n # @return [Symbol] # The action that was run (or scheduled to run in the case of \"unprocessed\" resources).", "\n attr_accessor :action\n\n # @return [Exception] The exception that was thrown\n attr_accessor :exception\n\n # @return [Hash] JSON-formatted error description from the Chef::Formatters::ErrorMapper\n attr_accessor :error_description\n\n # @return [Numeric] The elapsed time in seconds with machine precision\n attr_accessor :elapsed_time\n\n # @return [Chef::Resource::Conditional] The conditional that caused the resource to be skipped\n attr_accessor :conditional\n\n # The status of the resource:\n # - updated: ran and converged\n # - up_to_date: skipped due to idempotency\n # - skipped: skipped due to a conditional\n # - failed: failed with an exception\n # - unprocessed: resources that were not touched by a run that failed\n #\n # @return [Symbol] status\n #\n attr_accessor :status\n\n # The \"nesting\" level. ", " Outer resources in recipe context are 0 here, while for every\n # sub-resource_collection inside of a custom resource this number is incremented by 1.", "\n # Resources that are fired via build-resource or manually creating and firing\n #\n # @return [Integer]\n #\n attr_accessor :nesting_level\n\n def initialize(new_resource, action, nesting_level)\n @new_resource = new_resource\n @action = action\n @nesting_level = nesting_level\n end\n\n # @return [Boolean] true if there was no exception\n def success?", "\n !", "exception\n end\n end\n\n attr_reader :action_records\n attr_reader :pending_updates\n attr_reader :run_context\n attr_reader :consumers\n attr_reader :events\n\n def initialize(events, run_context = nil, action_records = [])\n @action_records = action_records\n @pending_updates = []\n @consumers = []\n @events = events\n @run_context = run_context\n end\n\n def_delegators :@action_records, :each, :last\n\n # Allows getting at the action_records collection filtered by nesting level and status.", "\n #\n # TODO: filtering by resource type+name\n #\n # @return [Chef::ActionCollection]\n #\n def filtered_collection(max_nesting: nil, up_to_date: true, skipped: true, updated: true, failed: true, unprocessed: true)\n subrecords = action_records.select do |rec|\n ( max_nesting.nil? ", "|| rec.nesting_level <= max_nesting ) &&\n ( rec.status == :up_to_date && up_to_date ||\n rec.status == :skipped && skipped ||\n rec.status == :updated && updated ||\n rec.status == :failed && failed ||\n rec.status == :unprocessed && unprocessed )\n end\n self.class.new(events, run_context, subrecords)\n end\n\n # This hook gives us the run_context immediately after it is created so that we can wire up this object to it.", "\n #\n # This also causes the action_collection_registration event to fire, all consumers that have not yet registered with the\n # action_collection must register via this callback. ", " This is the latest point before resources actually start to get\n # evaluated.", "\n #\n # (see EventDispatch::Base#)\n #\n def cookbook_compilation_start(run_context)\n run_context.action_collection = self\n # fire the action_colleciton_registration hook after cookbook_compilation_start -- last chance for consumers to register\n run_context.events.enqueue(:action_collection_registration, self)\n @run_context = run_context\n end\n\n # Consumers must call register -- either directly or through the action_collection_registration hook. ", " If\n # nobody has registered any interest, then no action tracking will be done.", "\n #\n # @params object [Object] callers should call with `self`\n #\n def register(object)\n consumers << object\n end\n\n # End of an unsuccessful converge used to fire off detect_unprocessed_resources.", "\n #\n # (see EventDispatch::Base#)\n #\n def converge_failed(exception)\n return if consumers.empty?", "\n\n detect_unprocessed_resources\n end\n\n # Hook to start processing a resource. ", " May be called within processing of an outer resource\n # so the pending_updates array forms a stack that sub-resources are popped onto and off of.", "\n # This is always called.", "\n #\n # (see EventDispatch::Base#)\n #\n def resource_action_start(new_resource, action, notification_type = nil, notifier = nil)\n return if consumers.empty?", "\n\n pending_updates << ActionRecord.new(new_resource, action, pending_updates.length)\n end\n\n # Hook called after a current resource is loaded. ", " If load_current_resource fails, this hook will\n # not be called and current_resource will be nil, and the resource_failed hook will be called.", "\n #\n # (see EventDispatch::Base#)\n #\n def resource_current_state_loaded(new_resource, action, current_resource)\n return if consumers.empty?", "\n\n current_record.current_resource = current_resource\n end\n\n # Hook called after an after resource is loaded. ", " If load_after_resource fails, this hook will\n # not be called and after_resource will be nil, and the resource_failed hook will be called.", "\n #\n # (see EventDispatch::Base#)\n #\n def resource_after_state_loaded(new_resource, action, after_resource)\n return if consumers.empty?", "\n\n current_record.after_resource = after_resource\n end\n\n # Hook called after an action is determined to be up to date.", "\n #\n # (see EventDispatch::Base#)\n #\n def resource_up_to_date(new_resource, action)\n return if consumers.empty?", "\n\n current_record.status = :up_to_date\n end\n\n # Hook called after an action is determined to be skipped due to a conditional.", "\n #\n # (see EventDispatch::Base#)\n #\n def resource_skipped(resource, action, conditional)\n return if consumers.empty?", "\n\n current_record.status = :skipped\n current_record.conditional = conditional\n end\n\n # Hook called after an action modifies the system and is marked updated.", "\n #\n # (see EventDispatch::Base#)\n #\n def resource_updated(new_resource, action)\n return if consumers.empty?", "\n\n current_record.status = :updated\n end\n\n # Hook called after an action fails.", "\n #\n # (see EventDispatch::Base#)\n #\n def resource_failed(new_resource, action, exception)\n return if consumers.empty?", "\n\n current_record.status = :failed\n current_record.exception = exception\n current_record.error_description = Formatters::ErrorMapper.resource_failed(new_resource, action, exception).for_json\n end\n\n # Hook called after an action is completed. ", " This is always called, even if the action fails.", "\n #\n # (see EventDispatch::Base#)\n #\n def resource_completed(new_resource)\n return if consumers.empty?", "\n\n current_record.elapsed_time = new_resource.elapsed_time\n\n # Verify if the resource has sensitive data and create a new blank resource with only\n # the name so we can report it back without sensitive data\n # XXX?: ", "what about sensitive data in the current_resource?", "\n # FIXME: this needs to be display-logic\n if current_record.new_resource.sensitive\n klass = current_record.new_resource.class\n resource_name = current_record.new_resource.name\n current_record.new_resource = klass.new(resource_name)\n end\n\n action_records << pending_updates.pop\n end\n\n private\n\n # @return [Chef::ActionCollection::ActionRecord] the current record we are working on at the top of the stack\n def current_record\n pending_updates[-1]\n end\n\n # If the chef-client run fails in the middle, we are left with a half-completed resource_collection, this\n # method is responsible for adding all of the resources which have not yet been touched. ", " They are marked\n # as being \"unprocessed\".", "\n #\n def detect_unprocessed_resources\n run_context.resource_collection.all_resources.select { |resource| resource.executed_by_runner == false }.each do |resource|\n Array(resource.action).each do |action|\n record = ActionRecord.new(resource, action, 0)\n record.status = :unprocessed\n action_records << record\n end\n end\n end\n end\nend\n" ]
{ "pile_set_name": "Github" }
[ 0.017937219730941704, 0.00974025974025974, 0.009708737864077669, 0.017094017094017096, 0.008264462809917356, 0, 0.008771929824561403, 0.009523809523809525, 0.006666666666666667, 0.006514657980456026, 0, 0.012254901960784314, 0, 0.012544802867383513, 0.013071895424836602, 0.006198347107438017, 0, 0, 0.002066115702479339, 0, 0.0045662100456621, 0, 0, 0, 0, 0, 0, 0, 0, 0, 0.007042253521126761, 0.006578947368421052, 0, 0, 0, 0, 0, 0, 0, 0, 0.0038022813688212928, 0, 0, 0, 0, 0.0013966480446927375, 0, 0.002544529262086514 ]
0.003464
5
[ "A beautiful favor isn't hard to find, just take a little inspiration from the natural beauty of your floral arrangements! ", "Give guests a stem to take home inside this Pretty Petals Porcelain Miniature Flower Wedding Favor Vase. ", "The petite vase is just the right size for a single flower. ", "It measures a dainty 2 1/2'' by 2 1/2'' by 3 1/8'' (H). ", "The bottom of the vase has a classic ball shape and the neck flares out in a double ring of stylized petals. ", "Alone, it makes a perfect accent to a table setting by mirroring the larger centerpiece. ", "Arranged in groups, they make a memorable floral display. ", "Personalize these little beauties with one of our custom created tags or a ribbon in your theme colors. ", "Make sure you order enough because everyone will want to take one home! ", "Sold in a set of 4." ]
{ "pile_set_name": "Pile-CC" }
[ 0, 0.009523809523809525, 0, 0, 0, 0, 0, 0, 0, 0 ]
0.000952
5
[ "Q:\n\n.htaccess Not Following Rule\n\nI have the following .htaccess file. ", "\nRewriteEngine on\nRewriteCond %{HTTP_HOST} !", "^www\\.mysite\\.com$\nRewriteRule ^(.*)$ http://www.mysite.com/$1 [R=301]\nRewriteCond %{REQUEST_FILENAME} !", "-f\nRewriteCond %{REQUEST_FILENAME} !", "-d\nRewriteCond %{REQUEST_URI} !(", "\\.[a-zA-Z0-9]{1,5}|/)$\nRewriteRule ^awesomeslash/ /somepage.php [NC,PT] \nRewriteRule ^([^/\\.]*)/?([^/\\.]*)/?([^/\\.]*)/?$ /template2.php?slash1=$1&slash2=$2&slash3=$3 [L]\n\nI've running into a problem with the rule: \n RewriteRule ^awesomeslash/ /somepage.php [NC,PT] \n\nWhen I go to http://www.mysite.com/awesomeslash/\nIt is not loading somepage.php instead it is following the run under it and going to template2.php\n\nA:\n\nI think adding L to [NC,PT] should fix it. ", "\n\"The [L] flag causes mod_rewrite to stop processing the rule set. ", "In most contexts, this means that if the rule matches, no further rules will be processed. ", "This corresponds to the last command in Perl, or the break command in C. Use this flag to indicate that the current rule should be applied immediately without considering further rules.\"", "\nhttp://httpd.apache.org/docs/current/rewrite/flags.html#flag_l \n\nA:\n\nFirstly: This does not make much sense if you want rewrite /awesomeslash/ to /somepage.php:\nRewriteCond %{REQUEST_FILENAME} !", "-f\nRewriteCond %{REQUEST_FILENAME} !", "-d\nRewriteCond %{REQUEST_URI} !(", "\\.[a-zA-Z0-9]{1,5}|/)$\nRewriteRule ^awesomeslash/ /somepage.php [NC,PT] \n\nRewriteRule ^([^/\\.]*)/?([^/\\.]*)/?([^/\\.]*)/?$ /template2.php?slash1=$1&slash2=$2&slash3=$3 [L]\n\nI assume you wanted this instead:\nRewriteRule ^awesomeslash/ /somepage.php [NC,PT] \n\nRewriteCond %{REQUEST_FILENAME} !", "-f\nRewriteCond %{REQUEST_FILENAME} !", "-d\nRewriteCond %{REQUEST_URI} !(", "\\.[a-zA-Z0-9]{1,5}|/)$\nRewriteRule ^([^/\\.]*)/?([^/\\.]*)/?([^/\\.]*)/?$ /template2.php?slash1=$1&slash2=$2&slash3=$3 [L]\n\nSecondly:\n\nYou need L flag next to [NC,PT]: [NC,PT,L] -- this will tell Apache to not to process other rules.", "\n\nYou also need to add $ at the end of match pattern to make the rule match only this URL and not /awesomeslash/something-else.", "\nRewriteRule ^awesomeslash/$ /somepage.php [NC,PT,L]\n\n" ]
{ "pile_set_name": "StackExchange" }
[ 0, 0, 0.019230769230769232, 0, 0, 0.004282655246252677, 0, 0, 0, 0.005128205128205128, 0, 0, 0.0033783783783783786, 0, 0, 0.004347826086956522, 0, 0 ]
0.00202
5
[ "Hypophosphatemic rickets accompanying McCune-Albright syndrome: evidence that a humoral factor causes hypophosphatemia.", "\nMcCune-Albright syndrome (MAS) is sometimes complicated by hypophosphatemia. ", "However, it remains unclear whether a humoral factor is associated with the cause of hypophosphatemia. ", "We isolated cells with mutations of the Gsalpha gene from fibrous bone dysplasia tissues of two MAS patients (MAS cells). ", "Severe combined immunodeficiency (SCID) mice were subjected to experiments using from one of these cells patients. ", "Effects of conditioned media (CM) isolated from MAS cells (MAS-CM) on phosphate transport were investigated by using rat renal slices, the renal cell line OK-B, rat intestinal rings and the human intestinal cell line Caco-2. ", "In addition, the effects of MAS-CM on human sodium-dependent phosphate transporter (NPT2) gene promoter activity expression were investigated in the renal cell line OK-B2400 and were compared with the effects of CM isolated from a patient with oncogenic hypophosphatemic osteomalacia (OHO). ", "MAS cells caused significant hypophosphatemia (P < 0.05) and elevated serum alkaline phosphatase activity (P < 0.05) in SCID mice. ", "The MAS-CM significantly inhibited phosphate uptake in everted intestinal rings (P < 0.01), whereas it had no effect on glucose uptake. ", "The MAS-CM had no effect on either phosphate uptake in the kidney or NPT2 gene promoter activity. ", "In contrast, the CM of the OHO patient significantly inhibited phosphate uptake and NPT2 gene promoter activity. ", "These results indicate that the humoral factor derived from fibrous dysplasia cells of the MAS patient is different to that from OHO patients, because the humoral factor from the MAS patient inhibited phosphate transport not in the kidney but in the intestine." ]
{ "pile_set_name": "PubMed Abstracts" }
[ 0.008403361344537815, 0.01282051282051282, 0, 0.02459016393442623, 0.008695652173913044, 0.017777777777777778, 0.010309278350515464, 0.015267175572519083, 0.007352941176470588, 0.01020408163265306, 0.008849557522123894, 0.011538461538461539 ]
0.011317
5
[ "The invention relates to apparatus for damping vibrations, especially torsional vibrations between the output element (e.g., a crankshaft) of an engine and the power train in a motor vehicle. ", "More particularly, the invention relates to improvements in apparatus of the type having at least two flywheels which are rotatable relative to each other against the opposition of damper means wherein one flywheel is the input member and the other flywheel is the output member of the damper means. ", "The output member can be coupled to the power train by a clutch, particularly a friction clutch.", "\nHeretofore known vibration damping apparatus of the above outlined type employ dampers which have energy storing elements acting in the circumferential direction of the flywheels and normally including coil springs which store elastic energy, and additional energy storing elements which act in the axial direction of the flywheels and cooperate with friction pads and/or linings to produce friction (i.e., hysteresis). ", "The means for generating friction operate in parallel with energy storing means which act in the circumferential direction of the flywheels.", "\nIt has been found that certain conventional vibration damping apparatus can operate satisfactorily (i.e., they are capable of damping torsional vibrations as well as noise) but only under specific circumstances. ", "Thus, the mode of operation of such conventional apparatus is not entirely satisfactory under many operating conditions because their design is a compromise due to an attempt to ensure satisfactory or acceptable operation under a variety of different conditions. ", "For example, a purely mechanical solution does not suffice to cover a wide spectrum of operating conditions entailing the development of many basically different stray movements and noise levels. ", "Moreover, purely mechanical solutions are quite expensive, especially if they are to adequately suppress stray movements and noise under a variety of different operating conditions. ", "This is due to the fact that, if a mechanically operated vibration damping apparatus is to counteract a wide range of amplitudes of undesirable stray movements of the flywheels relative to each other, such undertaking greatly increases the cost, bulk, complexity and sensitivity of the apparatus. ", "Moreover, even a very complex and expensive mechanical vibration damping apparatus is incapable of operating satisfactorily under any one of a wide range of different operating conditions because the individual damper stages (i.e., hystereses produced by individual energy storing elements which act in the circumferential direction of the flywheels) cannot be altered as a function of changes in operating conditions. ", "Still further, presently known apparatus are subject to extensive wear so that their useful life is relatively short, and they are also prone to malfunction." ]
{ "pile_set_name": "USPTO Backgrounds" }
[ 0, 0, 0, 0, 0, 0, 0, 0, 0, 0, 0, 0 ]
0
5
[ "Sharks Winger Back In The Fold by San Jose Sharks Staff / San Jose Sharks\n\nSan Jose Sharks Executive Vice President and General Manager Doug Wilson announced today that the club has re-signed Group II restricted free agent forward Devin Setoguchi to a one-year contract.", "\n\n\n\n\n\nThe 23-year-old Setoguchi finished the 2009-10 campaign tied for fourth on the team in goals (20). ", "He set a career-high with four game-winning goals and recorded 36 points in 70 games.", "\n\n\n\nDuring the postseason, he finished third on the team in goals (5), including the overtime game-winner in Game 2 of the Western Conference Quarter Finals vs. Colorado. ", "He also chipped in with four assists and was a +3 in 15 games.", "\n\n\n\n“We’re happy to have Devin under contract for next season,” said Wilson. “", "He is an important part of our team and we are looking forward to his on-going growth as a player. ", "We have very high expectations for him.”", "\n\n\n\nIn 2008-09, Setoguchi set career-highs in goals (31), assists (34) and points (65) and finished third on the team in points. ", "He was named to the Sophomore Team for the YoungStars Game presented by Upper Deck at the All-Star Game.", "\n\n\n\nThe six-foot, 200-pound right wing was selected by San Jose in the first round (8th overall) of the 2005 NHL Entry Draft.", "\n\n\n\n\n\nView Less" ]
{ "pile_set_name": "OpenWebText2" }
[ 0.014814814814814815, 0, 0, 0, 0, 0.02564102564102564, 0, 0, 0.007751937984496124, 0.028846153846153848, 0, 0 ]
0.006421
5
[ "Nathalie Delon\n\nNathalie Delon (born Francine Canovas on 1 August 1941, Oujda) is a French model and actress, former wife of Alain Delon, mother of Anthony Delon and grandmother of . ", "She was also known as Nathalie Barthélémy, due to her first marriage to Guy Barthélémy.", "\n\nDuring the 1960s Nathalie was considered one of the most beautiful women in the world. ", "She dated both Eddie Fisher and Richard Burton after each of them divorced Elizabeth Taylor.", "\n\nSelected filmography\n\nExternal links\n \n\nCategory:1941 births\nCategory:Living people\nCategory:French film actresses\nCategory:People from Oujda\nCategory:20th-century French actresses" ]
{ "pile_set_name": "Wikipedia (en)" }
[ 0.03278688524590164, 0.022988505747126436, 0, 0.03260869565217391, 0.005494505494505495 ]
0.018776
5
[ "AUSTRALIA has become the “walking dead of debt” due for a financial reckoning that could shock the housing market “bubble” within months.", "\n\nThat’s according to “anti-economist” Professor Steve Keen who defines Australia as a “zombie to be” given soaring personal debt that has created a government-induced property bubble ripe to burst.", "\n\n“Australia has simply delayed its day of reckoning,” he told news.com.au in reference to the global financial crisis that shocked many countries around the world from 2008 but left the lucky country relatively unscathed after a series of government interventions.", "\n\nThe Kingston University Professor claims first homeowners grants rolled out by successive governments have artificially kept prices high creating a form of “instant prosperity” that politicians are loath to stop.", "\n\n“The housing bubble makes the politicians look good because A, people are feeling wealthier, and B … people are borrowing money to spend,” he said.", "\n\n“Then the government runs a balanced budget and looks like it really knows what it’s doing”\n\n“It hasn’t got a f***ing clue frankly, because what’s actually happening is the reason it’s making that money is credit is expanding,” he said.", "\n\n“It’s the old classic story, you’re criticising a party because someone’s laced the punchbowl. ", "You try to take the punchbowl away from the party you’re a very unpopular person but you need to because what’s actually happening is people are getting intoxicated with credit”.", "\n\nHis latest book, Can We Avoid Another Financial Crisis? ", "argues Australia, along with Belgium, China, Canada and South Korea, is a “zombie” economy sleepwalking into a crunch that could come between 2017 and 2020.", "\n\n“Both [Australia and Canada] will suffer a serious economic slowdown in the next few years since the only way they can sustain their current growth rates is for debt to continue growing faster than GDP,” he writes.", "\n\nThe comments come amid national uproar around housing affordability following the government’s first home super saver scheme that will allow people to salary sacrifice into an account for a house deposit. ", "While it’s designed to help people by providing pre-tax savings, critics have dubbed it a “cruel hoax” that will benefit vendors.", "\n\nAustralia already has the fourth highest ratio of household debt as a percentage of net disposable income in the world at 212 per cent, according to the OECD with Sydney house prices more than 10 times the average income. ", "Sydney and Melbourne have also recently been ranked the 14th and 15th most expensive cities in the world by the Economist Intelligence Unit.", "\n\nAMP capital economist Shane Oliver warned this week he expects a “soft patch” in housing price growth at a time when cost of living is rising faster than real wages and consumer confidence is dipping.", "\n\n“The peak in home price growth in [Melbourne and Sydney] has likely been seen with the combination of bank rate hikes, tightening lending standards, surging unit supply and a reduction in expenses that can be claimed under negative gearing all likely to help drive a slowing going forward,” he wrote in a recent research note.", "\n\nReserve Bank Governor Dr Phil Lowe has already indicated high household debt levels have affected spending. ", "This, combined with lower wage growth is something the central bank was “learning” how to deal with, he said in February.", "\n\n“Households are carrying more debt than they have before and, at the same time, they are experiencing slower growth in their nominal incomes than they have for some decades,” he said.", "\n\n“For many, this is a sobering combination. ", "Reflecting this, our latest forecasts were prepared on the basis that growth in consumption was unlikely to run ahead of growth in household income over the next couple of years.”", "\n\nFor Prof Keen, the solution for governments to an overheated housing market is obvious: “Stop making housing into an asset.”", "\n\n“Make housing a place for people to actually live. ", "So you go back to saying ‘what’s desirable is affordable houses’ and affordable means it doesn’t cost a first homebuyer more than three or four years’ income to get a property,” he said.", "\n\nAs for those struggling to get on the ladder in the meantime?", "\n\n“The only thing you can do in the middle is say I’m just not going to join in, and if it happens on a collective level …. ", "it’s game over for the bubble because the bubble only works if more people keep taking out more leverage.”", "\n\nVictoria.Craw@news.com.au" ]
{ "pile_set_name": "OpenWebText2" }
[ 0, 0.005050505050505051, 0, 0, 0, 0, 0, 0, 0, 0, 0, 0, 0, 0.004464285714285714, 0.007142857142857143, 0.0049504950495049506, 0, 0.00909090909090909, 0, 0, 0, 0, 0, 0, 0, 0, 0, 0, 0.037037037037037035 ]
0.002336
5
[ "In exciting news, a recent study has published the first ever behavioral and ecological observations of Omura's Whales, expanding scientific knowledge of these rare whales dramatically.", "\n\nThe Omura's Whale (Balaenoptera omurai) was first described in the 1970s based on samples collected by Japanese whaling vessels and a few strandings--but were classified as \"pygmy Bryde's Whales\" and weren't recognized as a separate species until 2003. ", "Up until now, there were no confirmed field observations of these animals, and very little was known about their behavior and ecology. ", "Recent data collected near Madagascar by scientists from the Wildlife Conservation Society suggests that Omura's Whales are plankton feeding baleen whales, and that they feed and reproduce in the same habitat, unlike many other baleen whales which are known for their long range migrations between feeding and nursery areas. ", "Omura's Whales are related to Bryde's Whales and look similar enough that genetic testing was used to confirm identification of this population, although the authors have gathered data which will make future field observations easier. ", "Based on genetic analysis, the population in Madagascar is believed to be fairly small and isolated.", "\n\nOmura's Whales are considered a \"small\" tropical whale, with observed specimens all under 40 feet, and studies of diet suggest that they rely on filter feeding for zooplankton (and, from earlier data, small fish and crustaceans). ", "While they are rarely found in tightly knit groups (within several body lengths of each other), when one adult was found, more were often within a few hundred meter range. ", "This longer range association and communication means that studying social groups may be more difficult than with other baleen whales that tend to spend time closer together.", "\n\nIt's incredible that there are still animals this size that we know so little about, and these findings are an exciting step in learning more about non-migratory tropical baleen whales. ", "Unfortunately, as the authors point out, this population is threatened by oil and gas development off Northwestern Madagascar. ", "The full paper is available to read for free online here.", "\n\nThe ongoing severe California drought is bad news for wildlife, overall. ", "However, in a surprising conservation success, the drought may be helping to fuel the rebound of monarch butterflies along the Pacific coast. ", "As water restrictions increasingly limit how much homeowners are allowed to water landscaping plants, more people are turning to native plants for a drought-tolerant alternative--including milkweed, the species monarchs lay their eggs on. ", "Growing availability of plants that represent critical habitat may help speed a rebound from previous severe population declines.", "\n\nOf course, monarchs are also dependent on a range of other initiatives, from efforts to curb pesticide use to protection of their overwintering grounds in Mexico. ", "If you are interested in helping to create monarch habitat where you live, Monarch Joint Venture has a program to help you find seeds or plants of milkweed native to your area. ", "There are also a lot of opportunities to contribute to monarch research and conservation as a citizen scientist, and they've provided a helpful round-up of those programs here.", "\n\nIf you just want to learn more about monarchs and their amazing migrations (up to 3,000 miles!) ", "you can check out some spectacular photos or read more from National Geographic here.", "\n\nIn bad news for global corals, NOAA has reported that we are in the midst of a third major global coral bleaching event resulting from temperature stress. ", "Estimates suggest that by the end of 2015, almost 95% of corals in US waters will have been exposed to conditions that can cause them to bleach.", "\n\nIn 2005, the U.S. lost half of our Caribbean coral reefs to a major bleaching event as the result of warming seas. ", "While coral can survive bleaching if the conditions that led to the loss of symbiotic algae (called zooxanthellae) don't last that long, predictions suggest that the current bleaching event will continue into 2016 and so is likely to substantially impact reefs around the world. ", "Corals are one of the marine species we know will be harmed most severely by climate change, as the temperature and chemical composition of the ocean changes.", "\n\nIf you'd like to learn more about coral bleaching, there's additional information from NOAA here. You can also read the press release about the current bleaching event here, and if you are a recreational diver, you can help NOAA track bleaching (or the absence of bleaching) by contributing data through the Coral Reef Watch program." ]
{ "pile_set_name": "Pile-CC" }
[ 0.010810810810810811, 0.00392156862745098, 0, 0.009230769230769232, 0.01276595744680851, 0, 0.008620689655172414, 0, 0, 0, 0, 0, 0, 0, 0, 0, 0, 0.005649717514124294, 0, 0, 0.011764705882352941, 0.006369426751592357, 0, 0, 0, 0, 0.002967359050445104 ]
0.00267
5
[ "---\nabstract: 'We first propose and study a quantum toy model of black hole dynamics. ", "The model is unitary, displays quantum thermalization, and the Hamiltonian couples every oscillator with every other, a feature intended to emulate the color sector physics of large-$\\mathcal{N}$ matrix models. ", "Considering out of equilibrium initial states, we analytically compute the time evolution of every correlator of the theory and of the entanglement entropies, allowing a proper discussion of global thermalization/scrambling of information through the entire system. ", "Microscopic non-locality causes factorization of reduced density matrices, and entanglement just depends on the time evolution of occupation densities. ", "In the second part of the article, we show how the gained intuition extends to large-$\\mathcal{N}$ matrix models, where we provide a gauge invariant entanglement entropy for ‘generalized free fields’, again depending solely on the quasinormal frequencies. ", "The results challenge the fast scrambling conjecture and point to a natural scenario for the emergence of the so-called brick wall or stretched horizon. ", "Finally, peculiarities of these models in regards to the thermodynamic limit and the information paradox are highlighted.'", "\n---\n\n[ ]{}\n\n\\\n\n*Institute for Theoretical Physics *and* Center for Extreme Matter and Emergent Phenomena,\\\nUtrecht University, 3508 TD Utrecht, The Netherlands\\\n*\n\n[javier.magan@cab.cnea.gov.ar]{}\\\n\n6 mm\n\nIntroduction and description of results {#secI}\n=======================================\n\nThe purpose of this article is twofold. ", "We first want to propose and study a quantum model that captures several important features of black hole dynamics and it is still tractable from a unitary microscopic perspective. ", "The desire is to collect enough and robust intuition through a concrete solvable model. ", "The second objective is to extend this intuition to exact descriptions of black holes, such as large-$\\mathcal{N}$ Matrix Models[^1] [@thooft; @matrix; @adscft].", "\n\nThe essential features we want our toy model to possess are the following:\n\n- The model must display quantum thermalization [@useth].", "\n\n- The model must have the strongest form of non-locality: the Hamiltonian must couple every oscillator with every other oscillator through couplings of the same size [@susskind]. ", "Throughout the article we will call these type of models *democratic*, to distinguish between other possible forms of non-locality [^2].", "\n\nThe first bullet point is an obvious necessary feature. ", "Since the groundbreaking contributions of Bekenstein [@bekenstein] and Hawking [@hawking], it is widely accepted that black holes are thermal objects. ", "With the advent of the AdS/CFT correspondence [@adscft], the previous statement has a firm theoretical ground, since AdS/CFT maps black hole physics into a many-body quantum system at finite temperature [@witten].", "\n\nThe second bullet point is less obvious. ", "Within conjectured non-perturbative definitions of string theory [@matrix; @adscft], black holes are mapped to high energy states of large-$\\mathcal{N}$ matrix models. ", "These matrix models have interaction terms of the following form: $$\\label{matrixh}\n\\sum\\limits_{i,j=1}^{\\mathcal{N}}\\pi_{i}\\,A_{ij}\\,\\pi_{j}\\:,$$ comprising interactions between fundamental and adjoint (matrix valued) degrees of freedom, see [@matrixpol] for an explicit example applied to quantum black holes. ", "In the high energy sector of the model, the matrix-valued field $A_{ij}$ has all its entries thermally excited. ", "From the point of view of the vector field $\\pi_{i}$, the previous term connects every oscillator $\\pi_{i}$ with every other $\\pi_{j}$ democratically. ", "Another argument supporting the use of democratic models comes from the recently proposed microscopic model of certain black hole dynamics [@subir], in which a set of fermions interact through random quartic interactions.", "\n\nGiven the previous set of arguments and requirements, the toy model of black hole dynamics we propose in this article is defined by the following Hamiltonian: $$\\label{H}\nH=\\alpha \\sum\\limits_{i=1}^{N}c_{i}^{\\dagger}\\,c_{i}+\\eta\\sum\\limits_{i,j=1}^{N}c_{i}^{\\dagger}\\,V_{ij}\\,c_{j}\\;,$$ where $\\alpha$ and $\\eta$ are parameters with dimensions of energy, $c_{i}^{\\dagger}$ and $c_{i}$ are creation and annihilation operators of spinless free fermions (with the usual anticommutator relations), and $V_{ij}$ are independent random gaussian real numbers with zero mean and unit variance [^3]. ", "Therefore, the matrix $(\\eta V)_{ij}\\equiv\\eta V_{ij}$ is a random matrix taken from the Gaussian Orthogonal Ensemble (GOE) with deviation $\\sigma_{\\eta V}=\\eta$. For a beautiful and modern treatment of random matrices see [@tao]. ", "Notice that considering bosons instead of fermions is straightforward. ", "In second-quantized formalism all the computations are similar, and one obtains the same conclusions.", "\n\nThe eigenstate correlations and entanglement patterns of the family of Hamiltonians (\\[H\\]) were studied in [@useth], where it was analytically proved that the model satisfies the so-called Eigenstate Thermalization Hypothesis (ETH) [@srednickisub] [^4]. ", "This hypothesis concerns the origin of statistical mechanics in closed quantum systems. ", "As remarked in [@useth], the family of Hamiltonians (\\[H\\]) shows that ETH is ‘typical’ within the space of *gaussian* systems. ", "Non-linear interactions are not needed in this regard, contrary to common belief.", "\n\nBelow we will study two different types of non-equilibrium unitary processes which are particularly interesting from the point of view of black hole physics. ", "We first consider the typical black hole thought experiment, in which we ‘throw in’ some small system to a black hole. ", "This amounts to set an initial state in which a small subsystem is factored out from the thermal ensemble (\\[initial\\]). ", "We then compute the time evolution of two-point correlation functions and entanglement entropies (\\[S\\]), (\\[dS\\]),(\\[cort\\]), and (\\[entt\\]). ", "The fact that our Hamiltonian is gaussian implies that the computation of all the two-point correlation functions provides every $n$-point correlation function as well. ", "This will allow us to discuss properly any subtle question regarding information spreading. ", "The second type of initial states we consider are completely factorized states (\\[factorized\\]). ", "This non-equilibrium process intends to emulate ‘collapse’ scenarios in black hole physics, in which a set of particles with definite quantum numbers collapse beyond their Schwarzschild’s radius. ", "This situation turns out to be severely more complicated. ", "Although we are not able to solve for the evolution of occupation densities exactly, we will show interesting relations that the evolution of entanglement entropies satisfy, see (\\[scaling\\]), (\\[scaling1\\]), (\\[fermions\\]) and (\\[bosons\\]). ", "These special relations were argued to hold generically for democratic systems in [@uswalks].", "\n\nIn section (\\[secIII\\]) we apply the results to black hole physics. ", "We first analyze the toy model. ", "Without taking into account Poincaré recurrences, the model displays three types of thermalization time scales. ", "The first one is the local relaxation time scale, the analog of quasinormal decay. ", "It is given by $t_{\\textrm{local}}=1/R$, where $R^{2}\\equiv 4N\\eta^{2}\\sim \\mathcal{O}(\\alpha)$ can be taken to be independent of the system size. ", "The second one is the scrambling time scale [@hayden; @susskind], defined equivalently as the time by which entanglement entropies become thermal or the time by which information spreads through the whole system, as it was defined in the original reference [@susskind]. ", "It can also be defined as the time at which the mean field approximation ceases to apply. ", "It turns out to be given by $t_{\\textrm{scrambling}}=t_{\\textrm{local}}=1/R$, a result that challenges the fast scrambling conjecture, as we describe properly below. ", "Finally, we notice the existence of another time scale. ", "It is the time by which deviations from thermality are uniform over the entire system. ", "This ‘randomization’ time scale turns out to be $t_{\\textrm{random}}=N^{1/3}/R$ for our toy model.", "\n\nIn the second part of section (\\[secIII\\]), we extend the results and intuition to exact descriptions of black hole dynamics, i.e to large-$\\mathcal{N}$ matrix models or the SYK model [@subir]. ", "The extension is possible because the results of the toy model are rooted in a single feature: large-$N$ factorization during time evolution in high energy states. ", "This feature causes extensive entanglement dynamics. ", "For the SYK the extension is trivial. ", "For the matrix model it works in Fourier space, providing a gauge invariant notion of entanglement entropy associated to a given set of ‘generalized free field’ modes [^5]. ", "If we have a set $A$ of generalized free fields modes $\\mathcal{O}_{\\omega, k}^{i}$ with associated number operator $n_{\\mathcal{O}_{\\omega, k}^{i}}$, the entanglement entropy is given by: $$\\label{SO}\nS_{A}(t)=\\sum\\limits_{i\\in A}(\\,n_{\\mathcal{O}_{\\omega, k}^{i}}(t) +1\\,)\\,\\log (\\,n_{\\mathcal{O}_{\\omega, k}^{i}}(t) +1\\,)\\,-\\,n_{\\mathcal{O}_{\\omega, k}^{i}}(t)\\,\\log (\\,n_{\\mathcal{O}_{\\omega, k}^{i}}(t))\\;.$$\n\nAs for the toy model, the previous analysis shows the existence of three time scales. ", "The first is the well-known local relaxation time scale, associated to quasinormal ringing. ", "It is given by the inverse of the imaginary part of the quasinormal frequency, typically proportional to the inverse of the temperature. ", "The second is the scrambling time scale, which again turns out to be independent of the system size $N$. It is controlled by the lowest quasinormal frequency. ", "The third is the time by which deviations from equilibrium are uniform throughout the system. ", "This time scale turns out to be: $$t_{\\textrm{random}}=\\,\\frac{1}{\\Omega_{\\mathcal{O}_{\\omega, k}^{i}}^{^{\\textrm{min}}}}\\,\\log N\\;,$$ where $\\Omega_{\\mathcal{O}_{\\omega, k}^{i}}^{^{\\textrm{min}}}$ is the imaginary part of the lowest quasinormal frequency. ", "As we expand below, this time scale coincides with the time it takes a freely falling observer to cross the near horizon region until it hits the so-called brick wall [@brick] or stretched horizon [@complementarity], as pointed out in [@us1; @us3].", "\n\nIn the last section, we comment on the information paradox [@paradox] in the light of our results. ", "We show that the thermodynamic limit of democratic systems is essentially different from the thermodynamic limit of common local systems, being effectively described by non-unitary dynamics. ", "The large-N limit acts as a coarse-graining by neglecting information originating in subleading correlation functions. ", "It generates entropy without the need of tracing out part of the system. ", "This observation naturally reconciles the old solution described in [@paradox] with unitary microscopic descriptions of black holes in which the interaction structure is democratic.", "\n\nNon-equilibrium unitary processes with random free fermions {#secII}\n===========================================================\n\nIn this section, we study non-equilibrium dynamical processes using the family of Hamiltonians (\\[H\\]). ", "Within the context of many-body quantum mechanics, there are many out of equilibrium initial conditions one can consider. ", "Relaxation times, global or local (to be explicitly defined below), depend on the choice of the initial state. ", "In this article, we consider two important classes of initial states.", "\n\nThe first initial state we consider intends to emulate the typical black hole thought experiment which consists of ‘throwing in’ a small subsystem $\\textbf{A}$ to a black hole. ", "Subsystem $\\textbf{A}$ qualifies as an ‘unentangled perturbation’ of the black hole state. ", "The combined system is left to relax through unitary evolution. ", "Any information contained in $\\textbf{A}$ is shared with the large number of internal degrees of freedom of the black hole and radiation [@hayden; @susskind], and becomes only accessible through fine-grained measurements of the global quantum state. ", "Notice that the previous ‘natural’ and ‘physical’ assertion, concerning at the same time both information mixing and information conservation, has proven to be very difficult to observe in a specific model. ", "There exist an obvious tension between the need of complicated dynamics to chaotically mix the initial information and the ability of actually solving the proposed model. ", "As we show below, the family of Hamiltonians (\\[H\\]) overcome this tension.", "\n\nThe second initial state corresponds to a ‘collapse scenario’, in which there is no black hole whatsoever to start with. ", "In this context, an extreme situation arises by choosing as an initial state $N/2$ unentangled particles, a factorized state belonging to the highest entropic sector of the theory. ", "In this case, the evolution of entanglement entropy is significant for any subsystem size.", "\n\n“Throwing in” an unentangled particle\n-------------------------------------\n\nFor the first out-equilibrium scenario, we will consider the initially unentangled subsystem to be the first oscillator. ", "The initial quantum state is: $$\\label{initial}\n\\rho_{\\textrm{in}}=\\rho_{1}\\otimes\\rho_{\\beta}\\;,$$ defined by $$\\textrm{Tr}(\\,\\rho_{\\textrm{in}} \\,c^{\\dagger}_{k}c_{l})=(n-f)\\,\\delta_{k1}\\,\\delta_{k1}+f\\,\\delta_{kl}\\;.$$ In the previous expressions, $\\rho_{1}$ corresponds to the reduced density matrix of the first degree of freedom. ", "It is itself a thermal density matrix with occupation number $n$. The rest of the degrees of freedom form the ‘black hole’ and are set in the thermal ensemble $\\rho_{\\beta}=e^{-\\beta H}/Z$, associated to the Hamiltonian (\\[H\\]). ", "This thermal ensemble is fully characterized by its mean occupation number $f$. The computation of the average occupation number $f$ at temperature $\\beta$ for the Hamiltonian (\\[H\\]) can be found in [@useth]. ", "Notice that gaussianity ensures that the two-point correlation function characterizes the full evolution of the state (\\[initial\\]), including entanglement entropies [@peschel]. ", "Moreover, due to Wick’s decomposition principle, stabilization of two-point correlation functions to the stationary value, whatever it is, implies stabilization of all higher point correlation functions as well. ", "All time scales associated to the thermalization process are then encoded in the evolution of the time evolved two-point correlation matrix: $$\\mathcal{C}_{kl}(t)=\\textrm{Tr}(\\,\\rho (t)\\, c^{\\dagger}_{k}c_{l})=\\textrm{Tr}(\\,\\rho_{\\textrm{in}}\\, c^{\\dagger}_{k}(t)c_{l}(t))\\;,$$ where $\\rho (t) =U(t)\\,\\rho_{\\textrm{in}}\\,U^{\\dagger}(t)$ and $c_{k}(t)=U^{\\dagger}(t)\\,c_{k}\\,U(t)$. The unitary evolution operator is defined as usual by $U(t)=e^{-i\\, H\\, t}$.\n\nTo compute $\\mathcal{C}_{kl}(t)$, notice that the ‘free’ nature of the Hamiltonian (\\[H\\]) allows an exact solution via diagonalization of the matrix $V$. If $\\psi^{a}$, for $a=1,\\cdots , N$, are the eigenvectors of $V$ with eigenvalues $\\epsilon_{a}$: $$\\sum\\limits_{j=1}^{N}V_{ij}\\,\\psi^{a}_{j} = \\epsilon_{a}\\,\\psi^{a}_{i}\\;,$$ and we define new creation and annihilation operators $d_{a}^{\\dagger}$ and $d_{a}$ by: $$d_{a}=\\sum\\limits_{i=1}^{N}\\psi^{a}_{i}\\,c_{i}\\;,$$ then the Hamiltonian can be equivalently written as: $$H = \\sum\\limits_{a=1}^{N}(\\alpha +\\epsilon_{a})\\, d_{a}^{\\dagger}\\,d_{a}= \\sum\\limits_{a=1}^{N}E_{a}\\, d_{a}^{\\dagger}\\,d_{a}\\;.$$ In the basis $d_{a}$, the Hamiltonian is a set of decoupled fermionic oscillators. ", "Their time evolution is: $$d_{a}(t)=U^{\\dagger}(t)\\,d_{a}\\,U(t)=e^{-i\\,(\\alpha+\\epsilon_{b})\\,t}d_{a}\\;.$$ Since $V$ is a random matrix, the creation and annhilitation operators create ‘random particles’. ", "Their properties can be unravelled by using the theory of random matrices, which deals with the statistical properties of eigenvectors and eigenvalues of matrices such as $\\eta V$. For an extensive treatment of random matrices see [@tao] and [@haake]. ", "In relation to the eigenvalues, we will only need the widely known Wigner’s semicircle law, accounting for the probability distribution of having an eigenvalue equal to $\\lambda$. It reads: $$\\label{wigner}\nP(\\lambda)=\\frac{2}{\\pi R^{2}}\\,\\sqrt{R^{2}-\\lambda^{2}}\\;,$$ where $R^{2}\\equiv 4N\\eta^{2}$, and where we remind that it concerns the eigenvalues of $\\eta\\, V$, a matrix of size $N$ with deviation equal to $\\eta$, see the Hamiltonian (\\[H\\]).", "\n\nOn the other hand, the statistical properties of eigenvectors are less widely known. ", "The main assertion is that the orthogonal matrix of eigenvectors $(\\psi^{1},\\cdots ,\\psi^{N} )$ is distributed according to the Haar measure on the orthogonal group $O(N)$. This means that the eigenvectors have independent and random gaussian entries, up to normalization: $$\\label{statvec}\n[\\psi^{a}_{i}]=0 \\,\\,\\,\\,\\,\\,\\,\\, [\\psi^{a}_{i}\\,\\psi^{b}_{j}]=\\frac{1}{N}\\,\\delta_{ab}\\,\\delta_{ij}\\;,$$ wherein what follows we will use $[p]$ to denote the average of the random variable $p$ over the random matrix ensemble.", "\n\nWith this previous statistical information about eigenvalues and eigenvectors, we can compute the evolution of the correlation matrix $\\mathcal{C}_{kl}(t)$ for a ‘typical’ Hamiltonian belonging to the family (\\[H\\]). ", "Notice that: $$\\begin{aligned}\n&\\mathcal{C}_{kl}(t)=\\textrm{Tr}(\\rho_{\\textrm{in}} c^{\\dagger}_{k}(t)c_{l}(t))=\\sum\\limits_{a,b=1}^{N}\\psi^{a}_{i}\\psi^{b}_{j}\\,\\textrm{Tr}(\\rho_{\\textrm{in}}d^{\\dagger}_{a}(t)d_{b}(t))=&\\nonumber \\\\ &\\sum\\limits_{a,b=1}^{N}\\psi^{a}_{i}\\psi^{b}_{j}\\,e^{it(\\epsilon_{a}-\\epsilon_{b})}\\,\\textrm{Tr}(\\rho_{\\textrm{in}}d^{\\dagger}_{a}d_{b})=\\sum\\limits_{k,l,a,b=1}^{N}\\psi^{a}_{i}\\psi^{b}_{j}\\psi^{a}_{k}\\psi^{b}_{l}\\,e^{it(\\epsilon_{a}-\\epsilon_{b})}\\,\\textrm{Tr}(\\rho_{\\textrm{in}}c^{\\dagger}_{a}c_{b})=&\\nonumber \\\\ &\\sum\\limits_{k,l,a,b=1}^{N}\\psi^{a}_{i}\\psi^{b}_{j}\\psi^{a}_{k}\\psi^{b}_{l}\\,e^{it(\\epsilon_{a}-\\epsilon_{b})}((n-f)\\,\\delta_{k1}\\delta_{k1}+f\\delta_{kl})&\\;.\\end{aligned}$$ Defining $$S_{ij}(t)\\equiv\\sum\\limits_{a,b=1}^{N}\\psi^{a}_{i}\\,\\psi^{b}_{j}\\,\\psi^{a}_{1}\\,\\psi^{b}_{1}\\,e^{it(\\epsilon_{a}-\\epsilon_{b})}\\;,$$ the previous equation can be simply written as: $$\\label{cor}\n\\mathcal{C}_{ij}(t)=\\delta_{ij}\\,f+(n-f)\\,S_{ij}(t)\\;.$$ The intuition coming from the previous equation is the following. ", "At the initial time $S_{ij}(t_{\\textrm{in}})=\\delta_{i1}\\delta_{j1}$, we recover the unentangled initial state (\\[initial\\]), as we should. ", "As time evolves $S_{ij}(t)\\rightarrow 0$, and the final state is the global thermal distribution $\\rho_{\\beta}=e^{-\\beta H}/Z$, characterized by its mean occupation number $f$.\n\nThe next step is to compute the statistical properties of $\\mathcal{C}_{ij}(t)$. In terms of the statistical properties of $S_{ij}$, they are given by: $$[\\mathcal{C}_{ij}(t)]=\\delta_{ij}f+(n-f)[S_{ij}(t)]\\;,$$ and $$[\\mathcal{C}_{ij}(t)^{2}]-[\\mathcal{C}_{ij}(t)]^{2}=(n-f)^{2}([S_{ij}^{2}(t)]-[S_{ij}(t)]^{2})\\;.$$ To compute the mean value $[S_{ij}(t)]$ and the deviations from the mean $(\\sigma^{S}_{ij}(t))^{2}=[S_{ij}^{2}(t)]-[S_{ij}(t)]^{2}$ we use (\\[wigner\\]) and (\\[statvec\\]). ", "We want to remark that if the deviations from the mean are small (as they will), we are indeed computing the typical properties of single Hamiltonian realizations taken from the family (\\[H\\]). ", "This is due to self averaging, a known feature in random matrix theory.", "\n\nThe computation divides into different computations, as shown by the structure of the following matrix: $$\\mathcal{C}_{ij}(t)=\n\\begin{pmatrix}\n\\mathcal{C}_{11}(t)& \\begin{pmatrix}\\,\\,\\,\n\\cdots\\,\\,\\,&\\,\\,\\,\\mathcal{C}_{1j}(t)\\,\\,\\,&\\,\\,\\,\\cdots\\,\\,\\,\\\\\n\\end{pmatrix}\\\\[-3mm] \\\\\n\\begin{pmatrix}\n\\vdots\\\\ \\mathcal{C}_{1j}^{*}(t)\\\\ \\vdots\\\\\n\\end{pmatrix}&\\begin{pmatrix}\\ddots& & \\mathcal{C}_{ij}(t)\\\\ &\\mathcal{C}_{ii}(t)& \\\\ \\mathcal{C}_{ij}^{*}(t)& &\\ddots\\\\ \\end{pmatrix}\\\\\n\\end{pmatrix}\\;,$$ where we mean that $\\mathcal{C}_{1j}(t)=\\mathcal{C}_{1k}(t)$, $\\mathcal{C}_{jj}(t)=\\mathcal{C}_{kk}(t)$ and $\\mathcal{C}_{ij}(t)=\\mathcal{C}_{kl}(t)$ for $j,k,i,l\\neq 1$. Let’s start by $S_{11}(t)$. The mean is given by: $$[S_{11}(t)]=[\\sum\\limits_{a,b=1}^{N}\\psi^{a}_{1}\\psi^{b}_{1}\\psi^{a}_{1}\\psi^{b}_{1}\\,e^{it(\\epsilon_{a}-\\epsilon_{b})}]=\\sum\\limits_{a,b=1}^{N}[\\psi^{a}_{1}\\psi^{b}_{1}\\psi^{a}_{1}\\psi^{b}_{1}]\\,[e^{it(\\epsilon_{a}-\\epsilon_{b})}]\\;,$$ where in the second line we have used the fact that eigenvenctors and eigenvalues are not correlated in the large $N$ limit. ", "The leading term comes from the following contraction $[\\psi^{a}_{1}\\psi^{a}_{1}]\\,[\\psi^{b}_{1}\\psi^{b}_{1}]$, since in this case we do not kill any sum. ", "Noticing that the $a=b$ part of the sum is time independent: $$[S_{11}(t)]=\\frac{3}{N}+\\sum\\limits_{a\\neq b}\\frac{1}{N^{2}}\\,([\\,\\cos(t\\,(\\epsilon_{a}-\\epsilon_{b}))\\,]+i\\,[\\,\\sin(t\\,(\\epsilon_{a}-\\epsilon_{b}))\\,]\\,)\\;.$$ The averages over the spectrum are given by: $$[\\,\\cos(t(\\epsilon_{a}-\\epsilon_{b}))\\,]=\\int\\limits_{-R}^{R} d\\epsilon_{a}d\\epsilon_{b}\\,\\frac{4}{\\pi R^{4}}\\,\\sqrt{R^{2}-\\epsilon_{a}}\\,\\sqrt{R^{2}-\\epsilon_{b}}\\cos(t(\\epsilon_{a}-\\epsilon_{b}))=\\frac{4\\,(J_{1}(Rt))^{2}}{(Rt)^{2}}\\;,$$ where $J_{1}(x)$ is the Bessel function of the first kind. ", "The final result reads: $$[S_{11}(t)]=\\frac{3}{N}+\\frac{N(N-1)}{N^{2}}\\frac{4\\,(J_{1}(Rt))^{2}}{(Rt)^{2}}\\simeq \\frac{3}{N}+\\frac{4\\,(J_{1}(Rt))^{2}}{(Rt)^{2}}\\;.$$ We remind that the Bessel function $J_{1}(Rt)$ is an oscillatory decaying function. ", "For large argument $R\\,t\\gg 1$, the previous expression behaves as: $$[S_{11}(t\\gg 1/R)]\\simeq \\frac{3}{N}+\\frac{8}{\\pi (Rt)^{3}}\\cos^{2} (Rt-\\frac{3\\pi}{4})\\;.$$ By an analogous procedure, we can compute all the other mean values and deviations. ", "The computations are somewhat tedious, especially for $[S_{11}(t)^{2}]$, so we will just quote the results in matrix form. ", "The mean is given by:\n\n$$\\label{S}\n[S_{ij}(t)]=\n\\begin{pmatrix}\n\\frac{3}{N}+\\frac{4\\,(J_{1}(Rt))^{2}}{(Rt)^{2}}& \\begin{pmatrix}\n\\,\\,\\cdots&0&\\cdots\\,\\,\\\\\n\\end{pmatrix}\\\\[-3mm] \\\\\n\\begin{pmatrix}\n\\vdots\\\\ 0\\\\ \\vdots\\\\\n\\end{pmatrix}&\\begin{pmatrix}\\ddots& & 0\\\\ &0& \\\\ 0& &\\ddots\\\\ \\end{pmatrix}\\\\\n\\end{pmatrix}\\;,$$\n\nwhile the deviation reads: $$\\label{dS}\n[(\\sigma^{S}_{ij}(t))^{2}]=\n\\begin{pmatrix}\n\\frac{8}{N^{2}}+\\frac{16}{N}\\frac{(J_{1}(Rt))^{2}}{(Rt)^{2}}(1+\\frac{J_{1}(2Rt)}{Rt})& \\begin{pmatrix}\n\\,\\,\\cdots&\\frac{15}{N^{3}}+\\frac{4(J_{1}(Rt))^{2}J_{1}(2Rt)}{N\\,(Rt)^{3}}&\\cdots\\,\\,\\\\\n\\end{pmatrix}\\\\[-3mm] \\\\\n\\begin{pmatrix}\n\\vdots\\\\ \\frac{15}{N^{3}}+\\frac{4(J_{1}(Rt))^{2}J_{1}(2Rt)}{N\\,(Rt)^{3}}\\\\ \\vdots\\\\\n\\end{pmatrix}&\\begin{pmatrix}\\ddots& & \\frac{3}{N^{3}}+\\frac{4(J_{1}(2Rt))^{2}}{N^{2}\\,(Rt)^{2}}\\\\ &\\frac{2}{N^{2}}+\\frac{4(J_{1}(2Rt))^{2}}{N^{2}\\,(Rt)^{2}}& \\\\ \\frac{3}{N^{3}}+\\frac{4(J_{1}(2Rt))^{2}}{N^{2}\\,(Rt)^{2}}& &\\ddots\\\\ \\end{pmatrix}\\\\\n\\end{pmatrix}\\;.$$ Joining all results together, we can write the evolution of the correlation matrix $\\mathcal{C}_{ij}(t)$ schematically as: $$\\label{cort}\n\\mathcal{C}_{ij}(t)=f\\delta_{ij}+(n-f)\\begin{pmatrix}\n\\frac{4\\,(J_{1}(Rt))^{2}}{(Rt)^{2}}\\pm\\mathcal{O}(1/N)& \\begin{pmatrix}\n\\,\\,\\,\\,\\,\\,\\,\\,\\,\\cdots\\,\\,\\,\\,\\,\\,\\,&\\pm \\mathcal{O}(1/N^{\\frac{3}{2}})&\\,\\,\\,\\,\\,\\,\\,\\cdots\\,\\,\\,\\,\\,\\,\\,\\,\\,\\\\\n\\end{pmatrix}\\\\[-3mm] \\\\\n\\begin{pmatrix}\n\\vdots\\\\ \\pm \\mathcal{O}(1/N^{\\frac{3}{2}})\\\\ \\vdots\\\\\n\\end{pmatrix}&\\begin{pmatrix}\\ddots& & \\pm \\mathcal{O}(1/N^{\\frac{3}{2}})\\\\ &\\pm\\mathcal{O}(1/N)& \\\\ \\pm \\mathcal{O}(1/N^{\\frac{3}{2}})& &\\ddots\\\\ \\end{pmatrix}\\\\\n\\end{pmatrix}\\;.$$ From these formulas we conclude that the initial out of equilibrium state (\\[initial\\]) thermalizes and evolves towards a state with homogeneous average occupation given by $f$. The final state is a ‘global black hole’.", "\n\nBesides, since the system is gaussian, the entanglement entropy of any subsystem $A$ can be computed using the framework developed in [@peschel]. ", "This framework writes the entanglement entropy in terms of the eigenvalues of the correlation matrix of subsystem $A$. ", "If $\\mathcal{C}_{ij}$, for $i,j \\in A$, is such a correlation matrix and $\\lambda_{m}$ are its eigenvalues, the entanglement entropy can be written as: $$S_{A}=-\\textrm{Tr}(C\\log C + (1-C)\\log (1-C))=-\\sum\\limits_{m}(\\lambda_{m}\\log \\lambda_{m} + (1-\\lambda_{m})\\log (1-\\lambda_{m}))\\;.$$ The key point to observe now is that off-diagonal terms in the correlation matrix (\\[cort\\]) vanish in the thermodynamic limit. ", "Consider a subsystem $A$ composed of $M$ oscillators. ", "We can express the correlator matrix as: $$\\label{cex}\nC_{ij}=n_{i}(t)\\delta_{ij}+(\\Delta C)_{ij}\\;,$$ where $n_{ij}(t)$ is the occupation density of the oscillator $i$, and $\\Delta C$ is a random matrix taken from the GOE ensemble with zero mean and deviation $\\sigma_{C}\\sim \\mathcal{O}(1/N^{3/2})$. To compute the entropy we need to compute: $$\\label{averageent}\n[S_{A}]=-[\\textrm{Tr}(C\\log C + (1-C)\\log (1-C))]\\;.$$ This can be approximately computed by using the techinques of Appendix A in [@usrandom]. ", "The trace is a sum over the eigenvalues of $C$. ", "These can be expressed as $C_{i}=n_{i}+(\\Delta C)_{i}$, where $(\\Delta C)_{i}$ are the eigenvalues of $\\Delta C$. ", "Using the semicircle law (\\[wigner\\]), we obtain $[(\\Delta C)_{i}]=0$ and $[(\\Delta C)_{i}^{2}]\\propto M/N^{3/2}$. Taylor expanding (\\[averageent\\]) we obtain: $$\\label{averageent2}\n[S_{A}]=-\\sum\\limits_{i=1}^{M}(n_{i}(t)\\log n_{i}(t) + (1-n_{i}(t))\\log (1-n_{i}(t)))-\\mathcal{O}(\\frac{M^{2}}{N^{3/2}})\\;.$$ We conclude that the Von Neumann entropy in the thermodynamic limit is extensive at all times, even for big subsystems. ", "For a susbystem $B$ not including the first degree of freedom it is just the thermal entropy of $B$, independent of time. ", "For a subsystem $B\\equiv 1\\cup A$ including the first degree of freedom, it is just the thermal entropy of $A$ plus $S_{1}(t)$. Therefore, in this scenario, the most interesting entanglement entropy is the one associated to the first degree of freedom. ", "It is just given by: $$\\label{entt}\nS_{1}(t)=-\\textrm{Tr}(\\,\\rho_{1}(t)\\,\\log \\rho_{1}(t)\\,)=-\\mathcal{C}_{11}(t)\\log\\mathcal{C}_{11}(t)-(1-\\mathcal{C}_{11}(t))\\log(1-\\mathcal{C}_{11}(t))\\;.$$ Using (\\[cort\\]) we observe that $S_{1}(t)$ evolves towards the thermal entropy of the Fermi distribution, saturating at a distance of $\\mathcal{O}(1/N)$ from it. ", "Notice that stationarity, the time in which the initial factorization of the density matrix (\\[initial\\]) is broken, is reached in a time scale $t_{\\textrm{r}}=1/R$ independent of the system size. ", "It is clear from this result that mean field approximations of democratic systems in time dependent scenarios have a very limited range of applicability. ", "We will expand more on this issue and on its applications to black hole physics below.", "\n\nThe previous equations (\\[S\\]), (\\[dS\\]), (\\[cort\\]), and (\\[entt\\]) constitute the main results of this section. ", "In section (\\[secIII\\]) we will use them to make several remarks about information spreading in democratic systems, and comment on their applications to black hole physics.", "\n\nEntanglement dynamics from factorized initial states\n----------------------------------------------------\n\nThe second class of initial states we consider is the class of completely factorized states. ", "More concretely, we can have an initial state in which the first $\\bar{N}$ oscillators are excited, while the $N-\\bar{N}$ left over are not: $$\\label{factorized}\n\\vert\\Psi_{\\textrm{in}}\\rangle=c^{\\dagger}_{1}\\,c^{\\dagger}_{2}\\,\\cdots\\, c^{\\dagger}_{\\bar{N}}\\,\\vert 0\\rangle\\;.$$ This situation emulates a ‘collapse’ scenario, in which $\\bar{N}$ initially unentangled particles interact with each other, forming a black hole at late times. ", "To analyze the time evolution we need to write the state in terms of the decoupled oscillator basis $d_{a}^{\\dagger}$: $$\\label{evolvedst}\n\\vert\\Psi (t)\\rangle= \\sum\\limits_{j,k,\\cdots ,l}\\psi^{j}_{1}\\psi^{k}_{2}\\cdots \\psi^{l}_{\\bar{N}}\\,e^{i\\,t\\,(E_{j}+E_{k}+\\cdots +E_{l})}d^{\\dagger}_{j}\\,d^{\\dagger}_{k}\\cdots d^{\\dagger}_{l}\\,\\vert 0\\rangle\\;.$$ The time evolved correlator matrix is then: $$\\begin{aligned}\n\\mathcal{C}_{ij}(t)&=&\\langle\\Psi (t)\\vert \\,c_{i}^{\\dagger}c_{j}\\,\\vert\\Psi (t)\\rangle \\nonumber \\\\ &=&\\sum\\limits_{j,l;k,m;n,p}\\psi^{j}_{1}\\cdots \\psi^{l}_{\\bar{N}}\\psi^{k}_{1}\\cdots \\psi^{m}_{\\bar{N}}\\psi^{n}_{i}\\psi^{p}_{j}\\,e^{-i\\,t\\,(E_{j}+\\cdots +E_{l}-E_{k}-\\cdots -E_{m})}\\times\\nonumber\\\\ &\\times&\\langle 0\\vert \\,d_{j}\\,\\cdots d_{l}\\,d^{\\dagger}_{n}\\,d_{p}\\,d^{\\dagger}_{k}\\,\\cdots \\,d^{\\dagger}_{m}\\,\\vert 0\\rangle\\;.\\end{aligned}$$ The computation is severely more complicated. ", "There is an exponentially growing number of terms, with respect to the number of initial particles. ", "This is due to the vacuum ($2\\bar{N}+2$)-point correlation function, after applying Wick’s decomposition principle.", "\n\nAlthough we were not able to compute the full statistical properties of the previous quantity, it is possible to observe that it is diagonal on average: $$\\label{caver}\ni\\neq j\\longrightarrow [\\,\\mathcal{C}_{ij}(t)\\,]=0\\;.$$ On average, the only surviving entries are the diagonal ones, corresponding to the expectation values of the number operators. ", "This could have been expected from generic considerations, since the process must be similar to the previous case, in regards to information spreading. ", "The intuition is the following. ", "Given the randomness of $V$ in the Hamiltonian (\\[H\\]), from the point of view of a single degree of freedom, the rest of the system behaves as a thermal bath at all times. ", "Relation (\\[caver\\]) is an example of large-$N$ factorization in fully time-dependent scenarios.", "\n\nSince there is an effective permutation symmetry between the degrees of freedom in the Hamiltonian (\\[H\\]), the decay of the number operator of the initially excited particles is the same for all of them $[\\langle c_{\\uparrow}^{\\dagger}c_{\\uparrow}\\rangle] (t)= n_{\\uparrow} (t)$. The same can be said about the decays of the number operator associated to the oscillators that were initially non-excited $[\\langle c_{\\downarrow}^{\\dagger}c_{\\downarrow}\\rangle (t)]= n_{\\downarrow} (t)$. The average value of the correlator matrix has then the following diagonal form: $$\\label{diagonalC}\n[\\,\\mathcal{C}_{ij}(t)\\,]=\\textrm{Diag} (\\,n_{\\uparrow} (t),\\cdots,n_{\\uparrow} (t),n_{\\downarrow} (t),\\cdots,n_{\\downarrow} (t)\\,)\\;,$$ where there are $\\bar{N}$ entries with $n_{\\uparrow} (t)$ and $N-\\bar{N}$ entries with $n_{\\downarrow} (t)$.\n\nWe will assume in what follows that the off-diagonal deviations $\\sigma_{C}^{2}=[C_{ij}^{2}]-[C_{ij}]^{2}$ have the same structure as in the previous case, and are of $\\mathcal{O}(1/N^{ p})$, for some positive $p$ [^6]. ", "This is a natural assumption given the results of the previous section and the intuition coming from large-N factorization in matrix models. ", "At any rate, we stress that the following statements rely on such assumption.", "\n\nIf such assumption is correct, we can repeat the computation of the previous section, formulas (\\[cex\\]) and (\\[averageent2\\]). ", "Considering a subsystem $A$ composed of $M$ oscillators we obtain: $$[S_{A}]=-\\sum\\limits_{i=1}^{M}[n_{i}(t)\\log n_{i}(t) + (1-n_{i}(t))\\log (1-n_{i}(t))]-\\mathcal{O}(\\frac{M^{2}}{N^{p/2}})\\;.$$ From the previous relations we conclude that the deviation from entanglement extensivity is subleading for *any* $M\\lesssim N/2$ if $p\\geq 2$, which is a natural expectation as commented before. ", "Notice that for $M=a N$, the leading term is proportional to $a$ while the subleading term is proportional to $a^{2}$.\n\nTherefore, up to subleading corrections, the average entanglement entropy of a subset of $M=M_{\\uparrow}+M_{\\downarrow}$ oscillators satisfy the following scaling law: $$\\label{scaling}\nS_{M}=M_{\\uparrow}\\,S_{M_{\\uparrow} }(t)+M_{\\downarrow}\\,S_{M_{\\downarrow} }(t)\\;,$$ where $$\\label{scaling1}\nS_{n_{\\uparrow} }(t)=-n_{\\uparrow} (t)\\,\\log n_{\\uparrow} (t)-(1-n_{\\uparrow} (t)\\,)\\log(\\,1-n_{\\uparrow} (t)\\,)\\;,$$ is the entanglement entropy associated to one oscillator that was initially excited, and where a similar relation holds for $S_{n_{\\downarrow}} (t)$. The previous scaling relation was found by generic arguments concerning information spreading in democratic systems in [@uswalks]. ", "It seems to be a distinctive feature of democratic systems, when compared to local ones. ", "Relation (\\[scaling\\]) shows that the global properties of information spreading, such as the growth of entanglement entropies of large subsystems are fully controlled by the growth of entanglement entropies of each oscillator separately. ", "But from (\\[scaling\\]) we gain a further insight. ", "Since the single particle entanglement entropies are themselves controlled by the decay of the number operators $n (t)$, the global properties of information spreading are encoded in the local relaxation properties as well.", "\n\nGiven the previous results, it is of obvious interest to find $n(t)$ as a function of the $\\bar{N}$, in particular its characteristic time scale. ", "This can be computed without too much trouble for the case of one excitation $\\bar{N}=1$, in which: $$\\label{N1}\n[\\,\\langle \\,c_{\\uparrow}^{\\dagger}c_{\\uparrow}\\,\\rangle\\,]\\,(t)=\\frac{4\\, (J_{1}(Rt))^{2}}{(Rt)^{2}}\\;,$$ decaying to zero at large times. ", "This is expected since the final state has negligible energy. ", "The entanglement entropy associated to the first oscillator grows at intial times, attains its maximum possible value for $[\\,\\langle \\,c_{\\uparrow}^{\\dagger}c_{\\uparrow}\\,\\rangle\\,]\\,=1/2$ and then decays to zero for times $t\\gg 1/R$. Notice again that this simple calculation shows that entanglement is created in times of $\\mathcal{O}(1/R)$. Mean field methods fail to explain the dynamics at these early time scales.", "\n\nAlso, notice that the simple computation of $\\bar{N}=1$ could have been predicted with the solution of the previous section. ", "Namely, if we take (\\[cort\\]) and set $f=0$ and $n=1$ we obtain (\\[N1\\]) as the leading term in the thermodynamic expansion. ", "This suggests that each degree of freedom sees the rest of the system as a thermal bath already by times $t\\ll 1/R$ much shorter than the relaxation time scale. ", "Looking at the evolved state (\\[evolvedst\\]), we might expect that the state becomes similar to the random state in the $\\bar{N}$-particle sector[^7] when the phases randomize. ", "This gives a time scale of the order of the inverse of the energy sum in (\\[evolvedst\\]). ", "In turn, due to the central limit theorem, the energy sum is a gaussian random variable with squared deviation given by $\\sigma^{2}_{\\sum E}=\\bar{N}R^{2}$. The time scale for dephasing is $Rt\\sim\\mathcal{O}(1/\\sqrt{\\bar{N}})$, dying in the thermodynamic limit. ", "It seems natural to expect that at very early times $t\\sim R/\\sqrt{\\bar{N}}$, each degree of freedom sees the rest of the system as a thermal bath, and, therefore, decays according to (\\[cort\\]) [^8]\n\nFinally, notice that the results of this section are not expected to depend on the initial factorized state we consider. ", "For a generic factorized state of $N$ fermions: $$\\label{factgen}\n\\vert\\Psi_{\\textrm{in}}\\rangle=\\vert\\psi_{1}\\rangle\\otimes\\vert\\psi_{2}\\rangle\\otimes\\cdots\\otimes\\vert\\psi_{N}\\rangle;,$$ one can always make a set of local unitary transformations and bring the state towards the form (\\[factorized\\]). ", "Of course, this is not a symmetry of the Hamiltonian (\\[H\\]). ", "But given that the Hamiltonian is random, we know that the eigenstates are ‘non-locally’ spread through the whole system, see relation (\\[statvec\\]). ", "Given that we have preformed a set of local unitary transformations, the Hamiltonian in the new basis still randomly connects degrees of freedoms at different sites. ", "This again implies large-$N$ factorization (\\[caver\\]) for the initial state (\\[factgen\\]), and the evolution of entanglement entropy of a set $A$ of degrees of freedom is expected to be generically given by: $$\\label{fermions}\nS_{A}^{\\textrm{fermions}}=-\\sum\\limits_{i\\in A}(\\,n_{i}(t)\\log n_{i}(t) + (1-n_{i}(t)\\,)\\log (\\,1-n_{i}(t))\\,)\\;.$$ Finally, as commented in the introduction, we could have well considered bosons instead of fermions. ", "The only thing that would change is the relation between the entropy and the number operators. ", "In the case of bosons it would read: $$\\label{bosons}\nS_{A}^{\\textrm{bosons}}=\\sum\\limits_{i\\in A}(\\,(n_{i}(t)+1)\\log (\\,n_{i}(t)+1)-n_{i}(t)\\,\\log n_{i} (t)\\,)\\;.$$ Relations (\\[fermions\\]) and (\\[bosons\\]) have been seen to appear in [@useth; @usrandom] when considering eigenstates of fully random Hamiltonians or eigenstates of random free fermions. ", "They are rooted in the microscopic non-locality of the quantum state, allowing the computation of entanglement entropies for small enough subsystems solely by means of the diagonal entries of the reduced density matrices. ", "These diagonal entries are the expectation values of the number operators. ", "What relations (\\[cort\\]), (\\[caver\\]), (\\[fermions\\]) and (\\[bosons\\]) show is that these relations are not only valid at stationary states. ", "Due to the democratic structure of interactions, these relations are valid at all times in the thermodynamic limit.", "\n\nBlack hole physics, random particles and matrix models {#secIII}\n======================================================\n\nIn this section we apply the results of the previous section, concerning the dynamical behavior of random particles, to the physics of black holes and large-$\\mathcal{N}$ Matrix models. ", "These large-$\\mathcal{N}$ Matrix models hold key clues towards understanding the emergence of black hole geometric backgrounds [@matrix; @adscft; @papadodimas], and we would like to understand in more detail several aspects of their color dynamics.", "\n\nOne crucial aspect to analyze in this regard is how perturbations (and the information associated with them) spread through the system as time evolves [@susskind]. ", "A related aspect of primary interest in the community is to study these thermalization processes through entanglement entropy [@esperanza; @vijay] since entanglement entropy has a direct geometrical meaning [@takayanagi]. ", "Finally, it is interesting to see what are the peculiarities of democratic models in regards to the information paradox [@paradox].", "\n\nIn this section we analyze aspects of those three questions. ", "We first do it for the random particle toy model, the main objective being to gain the necessary intuition in a simplified solvable scenario. ", "Besides, as we will see, the results are interesting in their own right. ", "Later on, we show how the intuition naturally extends to large-$\\mathcal{N}$ matrix models, which furnish exact conjectured descriptions of quantum black holes.", "\n\nInformation transport and scrambling with random particles\n----------------------------------------------------------\n\nIn the context of black hole physics, information transport in thermal processes was coined ‘information scrambling’ in [@hayden; @susskind]. ", "A lower bound was conjectured for such information scrambling in physical systems: $$t_{\\textrm{scrambling}}\\sim \\beta \\,\\log N\\;,$$ where $\\beta$ is the temperature of the system or other significant time scale associated with the Hamiltonian or initial state considered, and $N$ the number of black hole degrees of freedom.", "\n\nLet’s begin with defining in various different ways *information scrambling*. ", "In [@susskind] it was broadly defined as the time scale by which a given perturbation spreads or contaminates the whole system. ", "In local theories the intuition leads naturally to consider diffusion processes. ", "It was then more rigorously (and abstractly) defined by the ‘Page’s test’ [@page], as the time by which subsystems are maximally entangled with the environment, see [@susskind; @lashkari; @us3]. ", "The logic behind this last definition is well known in the context of quantum thermalization, see the recent review [@eisertreview]. ", "If we choose a generic subsystem $A$ smaller than half of the system, with reduced density matrix $\\rho_{A}$, the non-equilibrium unitary process will drive this reduced state towards the thermal distribution at temperature $\\beta$. This implies that the entanglement entropy associated to $A$ runs towards the thermal entropy as well. ", "Asking for such an entanglement relaxation to all possible subsystems is a global notion of thermalization. ", "The problem with this abstract notion of thermalization is that its connection to actual ‘information transport’ is to some extent opaque. ", "This gap was filled in [@uscod; @uswalks], in which information transport was carefully defined by the evolution of the structure of correlations between subsystems in the global state. ", "In this setup, the scrambling time is naturally defined by the time scale in which correlations uniformize in size so that information is transported to the whole system democratically. ", "This democratic structure of correlations is a characteristic feature of random pure states [@uscod; @usrandom].", "\n\nWe can analyze most types of definitions with the global solution (\\[cort\\])[^9]. ", "The first thing we observe is that correlations spread instantaneously. ", "From (\\[cort\\]) it is clear that any property of global relaxation can be characterized by analyzing the number operator associated to the first degree of freedom: $$\\mathcal{C}_{11}(t)\\simeq f+(n-f)\\,(\\,\\frac{4\\,(J_{1}(Rt))^{2}}{(Rt)^{2}}\\pm\\mathcal{O}(1/N)\\,)\\;.$$ In $\\mathcal{C}_{11}(t)$ appears the essential time scale characterizing information spreading, which is given by $t_{\\textrm{relax}}=1/R$. Indeed, in the thermodynamic limit: $$\\label{diagonalCt}\nC_{ij}(\\,t\\gg 1/R\\,)= f\\,\\delta_{ij}+(\\,\\mathcal{C}_{11}(t)-f\\,)\\,\\delta_{i1}\\,\\delta_{j1}\\;.$$ For $t\\gg 1/R$ the correlation matrix is simply the global thermal density matrix: $$\\label{diagonalCt2}\nC_{ij}(\\,t\\gg 1/R\\,) \\simeq f\\,\\delta_{ij}\\;,$$ up to computed subleading corrections. ", "The information lost by the first degree of freedom is instantaneously spread through the whole system, since correlations are uniform through the system $C_{1j}(t)\\sim C_{1k}(t)$. This is of course due to the microscopic non-locality.", "\n\nThe fact that correlations are at all times uniform over the system does not immediately imply scrambling. ", "A clear and extreme example of this situation arises when all correlations are zero, as happens for the initial state considered before: $$\\label{initial2}\n\\rho_{\\textrm{in}}=\\rho_{1}\\otimes\\rho_{\\beta}\\;,$$ where there is a clear factorization between the first degree of freedom and the thermal ensemble. ", "Scrambling of the information associated with the first degree of freedom requires correlations to be uniform over the system *plus* the first degree of freedom to be maximally entangled with the environment. ", "Since any created correlations are homogeneously spread through the system, the time of entanglement production, or analogously the time in which the initial factorization (\\[initial2\\]) breaks down *is* the scrambling time itself.", "\n\nThe entanglement entropy was computed in the previous section to be: $$\\label{sc11}\nS_{1}(t)=-\\mathcal{C}_{11}(t)\\log\\mathcal{C}_{11}(t)-(1-\\mathcal{C}_{11}(t)\\,)\\,\\log(1-\\mathcal{C}_{11}(t)\\,)\\;,$$ which takes the Von Newmann entropy at the initial time: $$S(t_{\\textrm{in}})=-(\\,n\\log n + (1-n\\,)\\,\\log (1-n\\,)\\,)\\;,$$ to the Von Newmann entropy associated to the global thermal state: $$S(t\\gg t_{\\textrm{local}})=-(\\,f\\log f + (1-f\\,)\\,\\log (1-f\\,)\\,)\\;.$$ The characteristic time scale for stationarity of entanglement entropy is thus $t_{\\textrm{local}}=1/R$. Also, as shown in the previous section, notice that any other entanglement entropy associated to other subsystems reach the entanglement plateaux at this time scale as well.", "\n\nAt this point, we want to remark that this result, if correct, furnish a counterexample of the mean field bound presented in [@lashkari]. ", "In Ref. [", "@lashkari] it was claimed that there is a lower bound for the time scale of entanglement production in gaussian systems with fully non-local interactions. ", "This lower bound was claimed to be proportional to $\\log N$. More specifically, it is claimed that for models such as (\\[H\\]) the time evolution of the $N$ oscillator system can be approximated by: $$\\label{mf}\nU(t)=e^{-i H t}\\simeq \\prod\\limits_{i=1}^{N}U^{\\textrm{MF}}_{i}(t)\\;,$$ for some suitably defined ‘mean field local unitary evolution’ $U^{\\textrm{MF}}_{i}(t)$, acting in each oscillator separately. ", "The claim is that this approximation is valid until times proportional to $\\log N$. If we would apply such approximation to our initial state (\\[initial\\]), it would imply that until such large times entanglement could not be created. ", "We believe that our results, formulas (\\[cort\\]), (\\[entt\\]) and (\\[N1\\]), which follow from relatively simple computations, show that production and stationarity of entanglement entropy are attained in a time scale independent of the system size. ", "If correct, they furnish a counterexample to the arguments presented in [@lashkari], since this early time entanglement cannot be explained with a mean field approximation, such as (\\[mf\\]). ", "It is obvious that this issue deserves further consideration. ", "Finally, in view of our results, notice that there is indeed some mean field approximation applying here. ", "In the large-N limit, and before times of $\\mathcal{O}(\\beta \\log N)$, the evolution of the global state can be approximated by: $$\\label{mmf}\n\\rho (t)\\simeq \\prod\\limits_{i=1}^{N}\\rho_{i}(t)\\;,$$ where $\\rho (t)$ is not obtained from $\\rho (0)$ by unitary evolution but by some super-evolution operator which change the on-site Von Neumann entropy of the reduced on-site state. ", "This is another path to understand (\\[fermions\\]) and (\\[bosons\\]).", "\n\nComing back to scrambling, and to make the statement even clearer, we can also use the approach to information scrambling proposed in [@uscod; @uswalks]. ", "Due to the effective permutation symmetry of the random free fermions Hamiltonian (\\[H\\]), for any pair of subsystems $A$ and $B$ with the same size, the Mutual Information between the first degree of freedom and those subsystems is equal: $$\\label{spread}\n[\\,I(1,A)(t)\\,]\\equiv[\\,S_{1}+S_{A}-S_{1A}\\,]=[\\,I(1,B)(t)\\,]=[\\,I(1,B)(t)\\,]\\equiv [\\,S_{1}+S_{B}-S_{1B}\\,]\\;.$$ For a subsystem $A$ of $M$ degrees of freedom, using (\\[averageent2\\]) we obtain: $$\\label{mutualvanish}\nI(1,A)(t)\\sim \\mathcal{O}(M^{2}/N^{3/2})\\;,$$ We now join all features together. ", "The information lost by the first degree of freedom, quantified by $\\Delta S_{1}(t)$, satisfies two properties. ", "First, it is non-locally spread through the whole system, the Mutual Information being of the same size for any given pair of subsystems with the same size (\\[spread\\]). ", "This is due to the democratic structure of interactions in the microscopic Hamiltonian (\\[H\\]). ", "Second, this information can only be recovered by looking to big enough subsystems, since for small enough subsystems the Mutual Information vanishes in the thermodynamic limit (\\[mutualvanish\\]). ", "It is of obvious interest to quantify how big enough the subsystem must be to recover the information about the first degree of freedom. ", "However, we want to stress that subsystems with a size not scaling with $N$ in the thermodynamic limit will not achieve this goal. ", "From this precise point of view, the information lost by the first degree of freedom is fully scrambled over the environment. ", "Given (\\[sc11\\]), by $t_{\\textrm{local}}=1/R$ the first degree of freedom has shared an $\\mathcal{O}(1)$ amount of its information content with the environment. ", "Therefore, after $t_{\\textrm{local}}=1/R$ has elapsed, an $\\mathcal{O}(1)$ amount of information has been shared non-locally with an $\\mathcal{O}(N)$ amount of degrees of freedom.", "\n\nWe can repeat all the arguments for the factorized initial state (\\[factorized\\]). ", "The only thing we need to assume to arrive at the same set of conclusions is again that the mean squared deviation of the correlation matrix is of $\\mathcal{O}(1/N^{p})$, for $p\\geq 1$. As explained in the previous section this is a natural assumption given the large-N matrix structure of the model and given the exact results obtained with the first initial state. ", "If such assumption is correct, entanglement entropies evolve extensively, even for subsystems with extensive size $M\\lesssim N/2$. This implies that saturation at the plateaux of the subsystem occurs when one degree of freedom saturates by itself, i.e the time in which the on-site entanglement is created. ", "This is equivalent to the time by which the mean field approximation breaks down, which is just given by the local relaxation time scale.", "\n\nSummarizing, from the behavior of entanglement entropies, Mutual Information and correlation functions, we conclude that the time scale associated to:\n\n- Information lost by on-site degrees of freedom, quantified by their associated entanglement entropy,\n\n- Information spreading through the black hole degrees of freedom, characterized by correlations and Mutual Information being uniform through the system,\n\n- Relaxation of all entanglement entropies to thermal values at leading order,\n\n- Breakdown of the mean field approximation\n\nis size independent and given by: $$\\label{timescale}\nt_{\\textrm{local}}=t_{\\textrm{scrambling}}=1/R\\;.$$ We conclude that the family of Hamiltonians (\\[H\\]) studied in this article furnish an specific example of the generic statement described in [@uswalks]:\n\n- For democratic systems, information spreads instantaneously. ", "The characteristic time scales appear already at the level of local relaxation of perturbations.", "\n\nOn the other hand, these results challenge the fast scrambling conjecture [@susskind]. ", "The fast scrambling conjecture states that the time scale for a global spreading of information is bounded from below by $t_{\\textrm{scrambling}}\\gtrsim \\beta\\log N$, while here we find a time scale independent of the system size. ", "We want to stress again that the lower bounds presented in Ref.[@lashkari] seem not to be valid generically. ", "The breakdown of the mean field approximation, or analogously the breakdown of the factorization of the initial state (\\[initial2\\]), occurs on a time scale given by $t=1/R$, independent of the system size, see (\\[sc11\\]). ", "In such a situation, given that the spreading of information is instantaneous, the model (\\[H\\]) seem to furnish a counterexample of the claimed fast scrambling lower bound.", "\n\n### Scrambling vs randomization time scales\n\nIn the previous section we have shown that an $\\mathcal{O}(1)$ amount of information about the initial state is spread in a time of $\\mathcal{O}(1/R)$ over the entire system. ", "But has the thermalization process reach equilibrium in all possible senses?", "\n\nLooking to the exact form of the solution (\\[S\\]), (\\[dS\\]),(\\[cort\\]), we see that after the relaxation time scale $\\mathcal{O}(1/R)$ has elapsed, there is still a long way towards ‘*global equilibration of the deviations from thermality*’. ", "To be precise, the set of number operators corresponding to the black hole internal degrees of freedom (the oscillators with $i\\neq 1$), are shifted from the thermal expectation value by: $$\\mathcal{C}_{ii}-f\\sim \\mathcal{O}(1/N)\\;,$$ already at times of $\\mathcal{O}(1/R)$. On the other hand, the deviation from thermality of the number operator corresponding to the first degree of freedom is: $$\\label{c11}\n\\mathcal{C}_{11}-f =(n-f)\\, \\frac{4\\,(J_{1}(Rt))^{2}}{(Rt)^{2}}\\pm \\mathcal{O}(1/N)\\;.$$ It is now natural to define a ‘randomization’ time scale $t_{\\textrm{random}}$, by asking the deviations from thermality to be uniform over the system. ", "Notice that this is not equivalent to ask for correlations to be uniform over the system. ", "From a more physical perspective, it is defined as the time scale at which the leading term in (\\[c11\\]), the term driving the decay, ceases to be valid due to $\\mathcal{O}(1/N)$ corrections. ", "For random free fermions we obtain: $$\\mathcal{C}_{ii}-f\\,\\simeq\\, \\mathcal{C}_{11}-f\\,\\,\\,\\,\\,\\longrightarrow \\,\\,\\,\\,\\,t_{\\textrm{random}}\\sim \\frac{1}{R}N^{\\frac{1}{3}}\\;.$$ Summarizing, from the present perspective there are three natural time scales one can consider: local relaxation time scales, information transport or scrambling, and near equilibrium randomization. ", "For usual local systems, scrambling and randomization are equal, since the time that information takes to spread through the entire system is sufficiently big due to causality constraints. ", "For non-local systems, such as random free fermions, the situation is different. ", "Since information transport is instantaneous, local relaxation and scrambling are governed by the same time scale, while randomization takes a longer time.", "\n\nIn the next section we extend all this intuition to large-$\\mathcal{N}$ matrix models.", "\n\nInformation transport and scrambling with matrix models\n-------------------------------------------------------\n\nLet us recapitulate the important results obtained so far. ", "In the large-$N$ limit of the random free fermion model, the correlator matrix is diagonal, see (\\[cort\\]) and (\\[diagonalCt\\]). ", "The deviation from this diagonal behavior can be computed. ", "This large-$N$ factorization happens even in time-dependent scenarios. ", "It is rooted in the democratic structure of the microscopic Hamiltonian (\\[H\\]). ", "Since the diagonal entries are given by the local expectation values of number operators $\\langle c_{i}^{\\dagger}c_{i}\\rangle (t)=n_{i}(t)$, the entanglement entropy of a subset $A$ of $M\\lesssim N/2$ degrees of freedom is simply given by: $$\\label{sextensivefermions}\nS_{A}^{\\textrm{fermions}}=-\\sum\\limits_{i\\in A}(\\,n_{i}(t)\\log n_{i}(t) + (1-n_{i}(t)\\,)\\log (\\,1-n_{i}(t))\\,)\\;,$$ For random particles, the entanglement entropy is an *extensive* quantity even in time-dependent scenarios. ", "The deviation from extensivity can be computed with the deviation of the correlator matrix from its diagonal mean. ", "This computation shows that the deviation is subleading in the thermodynamic limit, and allows the study of entanglement in every subsystem, no matter its size, by the entanglement of on-site subsystems. ", "As commented in the introduction, the generalization of the Hamiltonian (\\[H\\]) to the bosonic case is straightforward. ", "The only change is the connection between entanglement entropy and occupation densities, the formula being slightly different. ", "For random free bosons it would read: $$\\label{sextensivebosons}\nS_{A}^{\\textrm{bosons}}=\\sum\\limits_{i\\in A}(\\,(n_{i}(t)+1)\\log (\\,n_{i}(t)+1)-n_{i}(t)\\,\\log n_{i} (t)\\,)\\;.$$ We claim that the previous formulas extend to large-$\\mathcal{N}$ matrix models and the SYK model [@subir]. ", "The reason is that this formula is *only* based on the effective gaussianity of reduced subsystems in democratic models. ", "In other words, formulas (\\[sextensivefermions\\]) and (\\[sextensivebosons\\]) are information theoretic versions of large-N factorization and therefore apply to any theory satisfying such property (such as large-$\\mathcal{N}$ matrix models and the SYK model). ", "The only difference between the toy model and real microscopic descriptions of black holes lies in the specific time dependence of occupation densities. ", "For random particles, we have a polynomial decay of number operators (\\[c11\\]) while for black holes we expect quasinormal ringing [@hubeny].", "\n\nTherefore, for the SYK model the extension is trivial. ", "Since the Hilbert spaces are isomorphic, the nature of subsystems is the same. ", "Entanglement evolution is just given by (\\[sextensivefermions\\]).", "\n\nFor large-$\\mathcal{N}$ field theories one has to take care due to mainly two reasons. ", "The first is that the democratic interactions appear only in color space. ", "The second is that subsystems are not defined by partitions of the Hilbert space, since these partitions are not gauge invariant. ", "Subsystems are better defined by operator algebras themselves. ", "To avoid these two issues, the easiest path is to consider the theory in Fourier space. ", "Our subsets will be sets of generalized free field modes[^10] $O_{\\omega,k}$. Generalized free fields are traces of products of the fields of the theory, in which we are not allowed to consider products with $\\mathcal{O}(N)$ fields. ", "Generalized free fields are gauge invariant operators by construction, due to the trace, a feature that avoids the problem of defining entanglement in a gauge invariant manner. ", "They are called ‘generalized free fields’ because they generate a Fock space of free particles in the large-$N$ limit, but they accomplish so without satisfying any free wave equation. ", "This last feature is possible due to large-$N$ factorization [@thooft]. ", "At finite temperature, for any set of generalized free fields $\\mathcal{O}$, large-$N$ factorization is the following statement[^11] $$\\label{thermalfact}\n\\textrm{Tr}(\\, \\rho_{\\beta}\\,\\mathcal{O}(x_{1})\\cdots \\mathcal{O}(x_{2n})) = \\frac{1}{2^{n}}\\sum\\limits_{\\pi}\\textrm{Tr}(\\, \\rho_{\\beta}\\, \\mathcal{O}(\\pi_{1})\\mathcal{O}(\\pi_{2}))\\cdots\\textrm{Tr}( \\,\\rho_{\\beta} \\,\\mathcal{O}(\\pi_{2n-1})\\mathcal{O}(\\pi_{2n})) +\\mathcal{O}(1/N)\\;.$$ As shown in [@papadodimas], relation (\\[thermalfact\\]) allows us to define creation and annihilitation operators $O_{\\omega,k}$ and $O_{\\omega,k}^{\\dagger}$ satisfying the usual algebra of thermal free oscillators. ", "Assuming for concretenes a bosonic generalized free field we have: $$C_{\\omega k,\\omega'k'}=Z^{-1}_{\\beta}\\,\\textrm{Tr}(\\, \\rho_{\\beta}\\,O_{\\omega',k'}^{\\dagger}O_{\\omega,k}\\,)=\\frac{1}{e^{\\beta}-1}\\,\\delta (w-w')\\,\\delta (k-k')=\\,n_{\\mathcal{O}_{\\omega,k}}^{\\beta}\\,\\delta (w-w')\\,\\delta (k-k')\\;,$$ and $$Z^{-1}_{\\beta}\\,\\textrm{Tr}(\\, \\rho_{\\beta}\\,O_{\\omega,k}O_{\\omega',k'}^{\\dagger}\\,)=(1+n_{\\mathcal{O}_{\\omega,k}}^{\\beta})\\,\\delta (w-w')\\,\\delta (k-k')\\;,$$ together with $$\\label{crossterms}\nZ^{-1}_{\\beta}\\,\\textrm{Tr}(\\, \\rho_{\\beta}\\,O_{\\omega',k'}O_{\\omega,k}\\,)=0 \\,\\,\\,\\,\\,\\,\\,\\,\\,\\,\\,\\,\\,\\,\\, Z^{-1}_{\\beta}\\,\\textrm{Tr}(\\, \\rho_{\\beta}\\,O_{\\omega',k'}^{\\dagger}O_{\\omega,k}^{\\dagger}\\,)=0\\;,$$ where connected higher point correlation functions vanish. ", "This implies that the two point correlation matrix $C_{\\omega k,\\omega'k'}$ is diagonal up to subleading corrections, sclaing as $\\mathcal{O}(1/N)$. This whole discussion does not change if the quantum state $\\rho$ is not the thermal density matrix, but an evolving pure/mixed $\\rho (t)$ [^12]: $$C_{\\omega k,\\omega'k'}(t)=\\,\\textrm{Tr}(\\, \\rho(t)\\,O_{\\omega',k'}^{\\dagger}O_{\\omega,k}\\,)=\\,n_{\\mathcal{O}_{\\omega,k}}(t)\\,\\delta (w-w')\\,\\delta (k-k')\\;.$$ So just by assuming (\\[thermalfact\\]), and repeating the argument of the previous section to bound the errors when computing entanglement entropy, the time dependent and gauge invariant entanglement entropy of *any* set $A$ of $M\\lesssim N/2$ generalized free fields is just given by: $$\\label{SO2}\nS^{A}(t)=\\sum\\limits_{i\\in A}(n_{\\mathcal{O}^{i}_{\\omega,k}}(t) +1)\\log (n_{\\mathcal{O}^{i}_{\\omega,k}}(t) +1)-n_{\\mathcal{O}^{i}_{\\omega,k}}(t) \\log (n_{\\mathcal{O}^{i}_{\\omega,k}}(t))\\pm \\mathcal{O}(M^{2}/N)\\;.$$ Notice that due to large-$N$ factorization we are able to define entanglement in a gauge invariant manner. ", "The clue is to consider entanglement entropy to be associated to observables or operator algebras [^13], such as generalized free fields $\\mathcal{O}$. Given an operator algebra, there are many observations one can do (all possible correlation functions). ", "Finding entanglement entropy from all correlator functions in the operator algebra can be quite complicated in general. ", "Crucially, for large-$\\mathcal{N}$ matrix models the situation simplifies due to large-$N$ factorization, and entanglement entropy can just be found by looking at the covariance matrix. ", "This covariance matrix is diagonal in Fourier space and it is controlled only by occupation densities (one point functions), see (\\[SO2\\]).", "\n\nAt it should, formula (\\[SO2\\]) is coherent with the expectations for random pure states, where it correctly provides the thermal entropy at the appropriate temperature: $$S^{\\beta}_{A}=\\sum\\limits_{i\\in A}\\beta \\omega_{i}\\,\\frac{e^{-\\beta\\omega_{i}}}{1-e^{-\\beta\\omega_{i}}}-\\log (\\,1-e^{-\\beta\\omega_{i}})\\;.$$ Besides, formula (\\[SO2\\]) implies that the conclusion obtained for random particles extends to matrix models:\n\n- Entanglement entropies are fully controlled by the decay of occupation densities. ", "Global scrambling of information is controlled by local relaxation of perturbations.", "\n\nWe want to stress again at this point that the only assumption we used to derive this result is large-$N$ factorization in thermal ensembles (\\[thermalfact\\]). ", "This is well supported by the toy model of random particles and by the black hole geometric description of large-$N$ matrix models. ", "Given this feature, reduced subsystems are products of uncorrelated gaussian density matrices. ", "We can then use the covariance matrix approach for gaussian systems [@peschel], since this is the only variable surviving the large-N limit. ", "Finally, bounds on subleading terms can be found as in the previous section, from the size of the corrections to the two-point functions.", "\n\nFormula (\\[SO2\\]) is a remarkably simple result with many potential consequences. ", "In what follows, we will just focus on its implications to information transport, scrambling, and randomization. ", "To this end we first need to argue for the actual functional form of $n_{\\mathcal{O}^{i}_{\\omega,k}}(t)$ in large-$\\mathcal{N}$ matrix models. ", "Notice that on generic grounds, quasinormal ringing implies that the field mode decays at late times as: $$\\delta \\mathcal{O}\\sim \\,e^{-\\Omega_{\\omega,k}^{\\textbf{I}}t}\\,(\\,A'\\,e^{i\\,\\Omega_{\\omega,k}^{\\textbf{R}}\\,t}+B'\\,e^{-i\\,\\Omega_{\\omega,k}^{\\textbf{R}}\\,t})\\;,$$ where $\\Omega_{\\omega,k}^{\\textbf{I}}$ and $\\Omega_{\\omega,k}^{\\textbf{R}}$ are the real and imaginary parts of the quasinormal frequency $\\Omega_{\\omega,k}$. For a harmonic oscillator satisfying (\\[crossterms\\]), the number operator is basically the square of the field, so that we expect: $$\\label{nmatrix}\nn_{\\mathcal{O}_{\\omega,k}}(t)\\simeq n_{\\mathcal{O}_{\\omega,k}}^{\\beta}+\\, A\\,e^{-2\\,\\Omega_{\\textrm{I}}\\,t}\\,(\\,\\cos (2\\,\\Omega_{\\omega,k}^{\\textbf{R}}\\,t\\,+\\theta)\\, +1\\,)\\;.$$ Although the argument leading to (\\[nmatrix\\]) is heuristic, a similar law has indeed been found recently in [@occupation] for a related occupation density described first in the context of holography in [@vijayoccupation]. ", "More importantly, we have recently verified that this type of law describes the evolution of Bekenstein-Hawking entropy [@aron].", "\n\nPlugging the previous expression (\\[nmatrix\\]) into (\\[SO2\\]), we obtain the evolution of entanglement entropy of a subset $A$ of generalized free fields. ", "The formula is extensive, but each mode decays with a different $\\Omega_{\\omega,k}$. To analyze its properties we just need to analyze the evolution of single perturbations. ", "Defining $f_{\\omega,k}(t)\\equiv \\,A\\,(\\,\\cos (2\\,\\Omega_{\\omega,k}^{\\textbf{R}}\\,t\\,+\\theta)+1\\,)$, the evolution of entanglement entropy associated to a single mode is given by: $$\\begin{aligned}\n\\label{entmatrix}\nS^{\\mathcal{O}_{\\omega,k}}(t)&=&(\\,n_{\\mathcal{O}_{\\omega,k}}^{\\beta}+\\,e^{-2\\,\\Omega_{\\textrm{I}}\\,t}\\,f_{\\omega,k}(t) \\,+1\\,)\\log (\\,n_{\\mathcal{O}_{\\omega,k}}^{\\beta}+\\,e^{-2\\,\\Omega_{\\textrm{I}}\\,t}\\,f_{\\omega,k}(t) \\,+1\\,)-\\nonumber\\\\ &-&(\\,n_{\\mathcal{O}_{\\omega,k}}^{\\beta}+\\,e^{-2\\,\\Omega_{\\textrm{I}}\\,t}\\,f_{\\omega,k}(t)\\,) \\log (\\,n_{\\mathcal{O}_{\\omega,k}}^{\\beta}\\,+e^{-2\\,\\Omega_{\\textrm{I}}\\,t}\\,f_{\\omega,k}(t)\\,)\\;.\\end{aligned}$$ We cannot trust the previous solution to small enough times since in that regime we should include higher quasinormal frequencies into (\\[nmatrix\\]). ", "But notice that for $2\\Omega_{\\textrm{I}}t\\ll 1$, the difference between the entanglement entropy at time $t$ and the one at time $t=0$ grows linearly with time: $$S^{\\mathcal{O}_{\\omega,k}}\\,(\\,t\\,\\ll \\,\\frac{1}{2\\,\\Omega_{\\textrm{I}}}\\,)\\propto \\Omega_{\\textrm{I}}\\,t\\;.$$ On the other hand, for large times $2\\,\\Omega_{\\textrm{I}}t\\gg 1$, where formula (\\[nmatrix\\]) can be trusted, we enter the so-called entanglement plateaux, which is predicted to be: $$\\begin{aligned}\n\\label{entmatrix1}\nS^{\\mathcal{O}_{\\omega,k}}\\,(\\,t\\gg \\,\\frac{1}{2\\,\\Omega_{\\textrm{I}}}\\,) &\\simeq & S^{\\beta}_{\\mathcal{O}_{\\omega,k}}+\\,\\log \\,(\\,\\frac{n+1}{n}\\,)\\, e^{-2\\,\\Omega_{\\textrm{I}}\\,t}\\,A\\,(\\cos (2\\,\\Omega_{\\omega,k}^{\\textbf{R}}\\,t\\,+\\theta)+1\\,)\\,=\\nonumber\\\\ &=& S^{\\beta}_{\\mathcal{O}_{\\omega,k}}+ \\,e^{-2\\,\\Omega_{\\textrm{I}}\\,t}\\,\\bar{A}\\,(\\cos (2\\,\\Omega_{\\omega,k}^{\\textbf{R}}\\,t\\,+\\theta)+1\\,)\\;.\\end{aligned}$$ Relations (\\[entmatrix\\]) and (\\[entmatrix1\\]) are analogous to relations (\\[fermions\\]) and (\\[sc11\\]) for the case of random free fermions, and are part of the main results of the article. ", "The difference between the toy model and the real one lies only in the functional form of the decay of occupation densities. ", "A similar law was recently found to describe Bekesntein-Hawking entropy evolution, see Ref. [", "@aron].", "\n\nFinally, just for completeness, notice that the Mutual Information between gauge invariant generalized free field modes is again zero *at all times*: $$I\\,(\\,\\mathcal{O}_{\\omega,k}\\, ,\\mathcal{O}'_{\\omega',k'}\\,)\\,(t)\\sim\\mathcal{O}(1/N)\\:,$$ due to the extensivity of entanglement entropies through unitary evolution. ", "This is an information theoretic expression of large-N factorization. ", "For the Mutual Information between bigger subsets of generalized free field modes one can use (\\[SO2\\]).", "\n\nJoining all results together, the conclusions regarding scrambling and local relaxation are universal to all democratic models, including SYK and large-N field theories. ", "The time scale associated to:\n\n- Information lost by the generalized free field mode $\\mathcal{O}_{\\omega,k}$, quantified by its associated entanglement entropy,\n\n- Information spreading through the matrix model, characterized by the vanishing of connected correlators and Mutual Information between small subsystems at all times,\n\n- Relaxation of all entanglement entropies to thermal values at leading order,\n\n- Breaking of the mean field approximation, characterized by the growth of entanglement of one generalized free field mode.", "\n\nis operator dependent, size independent and given by: $$t_{\\textrm{relax}}=t_{\\textrm{scrambling}}=\\frac{1}{2\\,\\Omega_{\\textrm{I}}}\\;,$$ a result which put under question marks the fast scrambling conjecture [@susskind]. ", "We want to stress again that although some aspects seem complicated, everything is based on large-N factorization, which implies factorization of reduced subsystems, and gaussianity. ", "Information theoretically, large-N factorization translates into equations (\\[sextensivebosons\\]), (\\[sextensivefermions\\]) and (\\[SO2\\]).", "\n\n### Scrambling vs randomization time scales\n\nWe have shown that random particles, SYK and large-$N$ matrix models behave very similarly in regards to information spreading. ", "Entanglement entropy evolution is extensive for all of them, and it is fully controlled by the decay of occupation densities. ", "This is rooted in large-N factorization. ", "What differs in each model is the time dependence of the occupation densities themselves. ", "Quite generically, we expect the occupation densities to decay exponentially fast, while for random particles we found only a polynomial decay[^14].", "\n\nThis difference can be seen through the randomization time. ", "This was defined as the time scale by which deviations from thermality are uniform throughout the system. ", "It can be found by looking at the properties of entanglement entropies at the entanglement plateaux, or equivalently by looking at the occupation densities. ", "Physically, it is the time scale by which we cannot trust the time decay, since $\\mathcal{O}(1/N)$ corrections kick in. ", "For large-$N$ matrix models, it is just given by: $$n_{\\mathcal{O}_{\\omega,k}}(t)-n_{\\mathcal{O}_{\\omega,k}}^{\\beta}=\\,e^{-2\\,\\Omega_{\\textrm{I}}\\,t}\\,f_{\\omega\\, , k}\\, (t)\\sim \\,\\mathcal{O}(\\frac{1}{N})\\;,$$ so that: $$\\label{randommatrix}\nt_{\\textrm{random}}=\\frac{1}{2\\,\\Omega_{\\textrm{I}}}\\log N \\;.$$ Taking into account the geometric observations done in [@us1; @us3] concerning black hole geometric backgrounds, this randomization time scale is of the same order of magnitude as the flight crossing time through the near horizon region until the so-called brick wall [@brick]. ", "Our results suggest that dynamical processes in the near horizon region correspond to near equilibrium evolution, fully controlled by quasinormal frequencies, and cut *dynamically* at the time scale (\\[randommatrix\\]). ", "The remarkable point is that the saturation of the process at such randomization time scale is not a phenomenological assumption, unlike in the gravity side. ", "In the present perspective, the saturation at (\\[randommatrix\\]) is a purely *dynamical* feature, due to finite $N$ effects coming from unitarity of the microscopic model. ", "Notice also that we expect a different randomization time for each perturbation. ", "The emergence of the brick wall might be not universal after all. ", "In this framework, it definitely depends on the specific observable we use to probe the black hole.", "\n\nComments on unitarity breaking in the thermodynamic limit\n---------------------------------------------------------\n\nIn this section we comment on another interesting and distinguishing feature of democratic systems. ", "Going to (\\[cort\\]), and taking the large-$N$ limit we obtain: $$\\mathcal{C}_{ij}(t)^{\\infty}\\equiv\\lim_{N\\to\\infty}\\mathcal{C}_{ij}(t)=f\\delta_{ij}+(n-f)\\,\\frac{4\\,(J_{1}(Rt))^{2}}{(Rt)^{2}}\\,\\delta_{i1}\\,\\delta_{j1}\\;.$$ Since the resulting correlation matrix is diagonal, the Von Neumann entropy associated to it is given by: $$S(\\rho)=S_{1}(t)+S_{BH}=S(t)\\;,$$ a time-dependent function. ", "In the large-$N$ limit, the entropy associated with the *global* state changes with time. ", "Its evolution cannot be driven by a unitary operator, and should be represented by some super-evolution operator $\\$ $, in the lines of [@paradox]. ", "In this precise sense, unitarity is *globally* broken in the thermodynamic limit. ", "We stress that this is essentially different from local theories, where superoperators and non-unitary behavior appear only *locally* when tracing out some part of the system. ", "Taking the thermodynamic limit $N\\to\\infty$ does not affect the entropy globally as time evolves in local systems.", "\n\nWe conclude that there is in principle no contradiction whatsoever between unitarity of quantum gravity and non-unitarity of the Einstein-Hilbert action (its semiclassical limit), whenever the microscopic theory is democratic, such as Matrix Models [@matrix], AdS/CFT [@adscft], or the recently introduced infinite range fermionic model [@subir].", "\n\nConclusions {#secIV}\n===========\n\nIn this article we first have proposed and studied a quantum toy model of black hole physics. ", "While capturing essential features of quantum black holes, the model is still exactly solvable. ", "The two properties we wanted our system to possess are quantum thermalization and democracy of interactions. ", "As described in the introduction, ‘democracy of interactions’ means the strongest form of non-locality, which amounts to have every oscillator interacting with every other oscillator through couplings of the same size. ", "The first requirement is an obvious necessary feature since there are very strong reasons to consider black holes as many body systems at finite temperature [@hawking; @witten]. ", "The second property intends to emulate realistic black hole models, such as large-$\\mathcal{N}$ matrix models [@matrix; @adscft; @susskind; @subir]. ", "The color sector physics of large-$\\mathcal{N}$ matrix models furnish an example of such democratic systems.", "\n\nThe proposed toy model was defined by the Hamiltonian (\\[H\\]). ", "It is a gaussian system of coupled fermionic oscillators in which the mass matrix belongs to the Gaussian Orthogonal Ensemble of random matrices [^15]. ", "This system was described in [@useth], as an analytical example of the Eigenstate Thermalization Hypothesis [@srednickisub; @ETH; @eisertreview]. ", "The advantages of choosing such a model are the following:\n\n- The model is exactly solvable. ", "Corrections of $\\mathcal{O}(1/N)$ can be properly taken into account.", "\n\n- The model has enough complexity to display quantum thermalization. ", "It allows a precise study of information spreading in democratic systems, at the global and local levels.", "\n\n- The extension to bosons is straightforward.", "\n\nWithin the toy model, we have chosen to analyze two generic physical processes. ", "The first scenario is the usual black hole thought experiment, in which one throws in a particle initially unentangled with the black hole (\\[initial\\]). ", "Subsequent evolution entangles both particle and hole, and information about the initial state $n$ is mixed/spread through the system (but conserved due to unitarity). ", "This unitary time evolution can be fully accounted by the two-point correlation functions of the theory, equations (\\[S\\]), (\\[dS\\]),(\\[cort\\]), and (\\[entt\\]). ", "The second scenario is akin to a ‘collapse’ scenario, in which the initial state is a factorized many-particle state (\\[factorized\\]). ", "For this case, the computations are harder, but we could show that the typical correlation matrix is again diagonal (\\[diagonalC\\]).", "\n\nAs described in the second part of the article, this type of covariance matrix appears also in realistic models of black holes, such as SYK or large-$N$ matrix models. ", "The property that underlies this covariance matrix is large-N factorization, a property characteristic of both SYK and large-$N$ matrix models that seems to be a defining feature of democratic models.", "\n\nGiven large-N factorization, extensivity of entanglement evolution follows naturally, with subleading computable corrections to it. ", "The formulas for all the models are equivalent, and given by (\\[sextensivefermions\\]) for random free fermions or the SYK model, by (\\[sextensivebosons\\]) in the random bosonic cases and by formula (\\[SO2\\]) for matrix models, where it is more convenient to work with generalized free field modes in Fourier space. ", "An intuitive explanation of the extensivity of entanglement evolution is the following. ", "Due to the non-locality of the theory, correlations between a given degree of freedom and any other are all of the same size. ", "Since there are $\\mathcal{O}(N)$ degrees of freedom interacting at the same time and given monogamy of entanglement, the shared correlation between any pair of them must be of $\\mathcal{O}(1/N)$. Large-$N$ factorization seems thus to be related with monogamy of entanglement in democratic systems.", "\n\nIn every case, the main conclusions are:\n\n- In the thermodynamic limit, the behavior of entanglement entropies is extensive at all times, see (\\[sextensivebosons\\]) and (\\[sextensivefermions\\]). ", "The evolution of entanglement entropies is fully controlled by local relaxation of occupation numbers.", "\n\n- Information spreading is instantaneous. ", "Any lost information is instantaneously scrambled since the Mutual Information between small enough subsystems is zero at all times.", "\n\n- The time scale by which the entanglement entropy of a subset $A$ reach the entanglement plateaux is independent of the system size and it is controlled by on-site quantities. ", "The size $A$ can scale with the size of the total system, as long as it is not bigger than half of it.", "\n\n- The mean field approximation, defined as the time of on-site entanglement production is broken at a time independent of the system size, although another kind of mean field approximation is valid for longer times (\\[mmf\\]).", "\n\nThe previous results challenge the fast scrambling conjecture [@susskind]. ", "As described in section (\\[secIII\\]), this conjecture states that there is a minimum lower bound for the time scale associated to ‘information scrambling’. ", "The conjectured lower bound is of order $\\mathcal{O}(\\beta\\log N)$. Information scrambling was defined by global equilibration of entanglement entropies, together with information spreading through the entire system. ", "The previous results suggest that both requirements are fulfilled in a time $t_{\\textrm{scrambling}}=\\frac{1}{2\\,\\Omega_{\\textrm{I}}}$, where $\\Omega_{\\textrm{I}}$ is the lowest quasinormal frequency associated with the subset $A$ that has been brought out of equilibrium. ", "Although there is still room for discussion of the present results, and probably subtle aspects concerning information theory might appear given that we are dealing with very unconventional Hamiltonians, the present results seem to be making a strong case against the fast scrambling conjecture and they invalidate the mean field bounds presented in [@lashkari].", "\n\nWe conclude that the only difference between the different models lies in the specific time dependence of number operators. ", "Whereas for random particles we found polynomial decays, for matrix models we have exponential decays. ", "Therefore, the time scale by which $\\mathcal{O}(1/N)$ corrections need to be taken into account is different in each case. ", "This time scale was coined ‘randomization time’. ", "It is the time scale by which the deviations from thermality are uniform over the system. ", "For random particles, we found a randomization time scaling polynomially with $N$. For matrix models, the result turns out to be: $$t_{\\textrm{random}}=\\frac{1}{2\\,\\Omega_{\\textrm{I}}}\\log N\\;.$$ Interestingly, this time scale sets the causality bounds for crossing the near horizon region until the so-called brick wall or stretched horizon, see [@us1; @us3]. ", "It is tempting to conclude that the emergence of the near horizon geometry is related to near equilibrium evolution, characterized by the properties of the entanglement plateaux, or analogously by the properties of the occupation densities. ", "These two quantities are controlled by the famous quasinormal modes, so it is tempting to conclude that the emergence of near horizon regions is fully encoded in the quasinormal ringing of the dual field theory, i.e in the poles of retarded correlation functions [@hubeny; @ivosachs]. ", "This speculative proposal provides a *dynamical* reason for the appearance of the brick wall, as the time by which the evolution of occupation densities and entanglement entropies is affected by $\\mathcal{O}(1/N)$ corrections.", "\n\nIn the last section, we made a remark about the information paradox [@paradox] in the light of the toy model. ", "We noticed that unitarity of the toy model, characterized for example by a time independent Von Neumann entropy of the global state, is lost in the large-$N$ limit *at all times*. ", "Taking the large-N limit affects the entropy globally. ", "This is a characteristic feature of democratic systems, whose semiclassical approximation at high energies is effectively described by non-unitary dynamics. ", "The old and famous proposal of [@paradox] seems to naturally emerge in theories of quantum gravity which have a democratic structure of interactions.", "\n\nAcknowledgements {#acknowledgements .unnumbered}\n================\n\nIt is a pleasure to thank Aron Jansen and Stefan Vandoren for interesting discussions. ", "This work was supported by the Delta-Institute for Theoretical Physics (D-ITP) that is funded by the Dutch Ministry of Education, Culture and Science (OCW).", "\n\n[00]{}\n\nG. ’t Hooft [*A planar diagram theory for strong interactions*]{}, Nuc.", " Phys.", " B. [**72**]{} (1974) 461.", "\n\nT. Banks, W. Fischler, S. H. Shenker and L. Susskind, [*M theory as a matrix model: A Conjecture*]{}, Phys.", " Rev. D [**55**]{} (1997) 5112, \\[arXiv:9610043 \\[hep-th\\]\\].", "\n\nJ. M. Maldacena, [*The Large N limit of superconformal field theories and supergravity*]{}, Adv.", " Theor.", " Math.", " Phys [**2**]{} (1998), \\[arXiv:9711200 \\[hep-th\\]\\].", "\n\nJ. M. Mag' an, [*Random free fermions: an analytical example of eigenstate thermalization*]{}, Phys.", " Rev. Lett [**116**]{} (2016) 030401. ", "\\[arXiv:1508.05339 \\[hep-th\\]\\].", "\n\nY. Sekino and L. Susskind, [*Fast Scramblers*]{}, JHEP [**0810**]{} (2008) 065, \\[arXiv:0808.2096 \\[hep-th\\]\\].", "\n\nR. J. Szabo [*Quantum field theory on noncommutative spaces*]{} Phys.", " Rep. [**378**]{} 4 (2003), \\[arXiv:0109162 \\[hep-th\\]\\].", "\n\nG. Gori, S. Paganelli, A. Sharma, P. Sodano and A. Trombettoni, Phys.", " Rev. B [**91**]{} (2015) 245138, \\[arXiv:1405.3616 \\[cond-mat\\]\\].", "\n\nJ. D. Bekenstein, [*Black holes and entropy*]{}, Phys.", " Rev. D [**7**]{} (1973) 2333.", "\n\nS. W. Hawking, [*Particle Creation by Black Holes*]{}, Commun.", " Math.", " Phys.", "  [**43**]{} (1975) 199 \\[Erratum-ibid.", "  [**46**]{} (1976) 206\\].", "\n\nE. Witten, [*Anti-de Sitter space, thermal phase transition, and confinement in gauge theories*]{}, Adv.", " Theor.", " Math.", " Phys.", "  [**2**]{} (1998) 505, \\[arXiv:9803131 \\[hep-th\\]\\].", "\n\nN. Lizuka and J. Polchinski, [*A matrix model for black hole thermalization*]{}, JHEP [**0810**]{} (2008) 028, \\[arXiv:0801.3657 \\[hep-th\\]\\].", "\n\nS. Sachdev, [*Bekenstein-Hawking Entropy and Strange Metals*]{}, Phys.", " Rev. X [**5**]{} (2015) 041025, \\[arXiv:1506.05111 \\[hep-th\\]\\].", "\n\nT. Tao, [*Topics in random matrix theory*]{}, Graduate Studies in Mathematics [**132**]{} (2011).", "\n\nT. Tao and V. Vu [*Random matrices: Universal properties of eigenvectors*]{}, Random Matrices: Theory Appl. [**", "01**]{} (2012) \\[arXiv:1103.2801 \\[math\\]\\]. ", "M. Srednicki, [*Chaos and quantum thermalization*]{}, Phys.", " Rev. E [**50**]{} (1994) 888, \\[arXiv:9403051 \\[cond-mat\\]\\].", "\n\nR. V. Jensen and R. Shankar, [*Statistical behavior in deterministic quantum systems with few degrees of freedom*]{}, Phys.", " Rev. Lett [**54**]{} (1985) 1879.", "\n\nC. Gogolin and J. Eisert, [*Equilibration, thermalisation, and the emrgence of statistical mechanics in closed quantum systems*]{}, \\[arXiv:1503.07538 \\[quant-ph\\]\\].", "\n\nJ. M. Mag' an, [*Fast scramblers, democratic walks and information fields*]{}, \\[arXiv:1507.02477 \\[hep-th\\]\\].", "\n\nP. Hayden and J. Preskill, [*Black holes as mirrors: Quantum information in random subsystems*]{}, JHEP [**0709**]{} (2007) 120, \\[arXiv:0708.4025 \\[hep-th\\]\\].", "\n\nK. Papadodimas and S. Raju, [*An infalling observer in AdS/CFT*]{}, JHEP [**1310**]{} (2013) 212, \\[arXiv:1211.6767 \\[hep-th\\]\\].", "\n\nK. Papadodimas and S. Raju, [*State-dependent bulk to boundary maps and black hole complementarity*]{}, Phys.", " Rev. D [**89**]{} (2014) 086010, \\[arXiv:1310.6335 \\[hep-th\\]\\].", "\n\nJ. M. Mag' an and S. Vandoren, [*Entanglement in Fock space of random QFT states*]{}, JHEP [**1507**]{} (2015) 016, \\[arXiv:1504.01346 \\[hep-th\\]\\].", "\n\nG. ’t Hooft, [*On the quantum structure of a black hole*]{}, Nuclear.", " Phys.", " B.  [**256**]{} (2005) 0550-3213.", "\n\nG. ’t Hooft, [*The Scattering matrix approach for the quantum black hole: An Overview*]{}, Int.", " J. Mod.", " Phys. [**", "A11**]{} (1996), \\[arXiv:9607022 \\[gr-qc\\]\\].", "\n\nL. Susskind, L Thorlacius and J. Uglum, [*The Stretched horizon and black hole complementarity*]{}, Phys.", " Rev. D.  [**48**]{} (1993) 3743, \\[arXiv:9306069 \\[hep-th\\]\\].", "\n\nJ. .L. F. Barbon and J. M. Magan, [*Chaotic Fast Scrambling At Black Holes*]{}, Phys.", " Rev. D.  [**84**]{} (2011) 106012, \\[arXiv:1105.2581 \\[hep-th\\]\\].", "\n\nJ. .L. F. Barbon and J. M. Magan, [*Fast Scramblers Of Small Size*]{}, JHEP [**1110**]{} (2011) 035, \\[arXiv:1106.4786 \\[hep-th\\]\\].", "\n\nJ. .L. F. Barbon and J. M. Magan, [*Fast Scramblers, Horizons and Expander Graphs*]{}, JHEP [**1208**]{} (2012) 016, \\[arXiv:1204.6435 \\[hep-th\\]\\].", "\n\nS. W. Hawking, [*Breakdown of Predictability in Gravitational Collapse*]{}, Phys.", " Rev. D [**14**]{} (1976) 2460.", "\n\nI. Peschel, [*Entanglement in solvable many-particle models*]{}. ", "Braz.", " J. Phys [**42**]{} (2012) 267.", "\n\nF. Haake, [*Quantum Signatures of Chaos*]{}. ", "Springer-Verlag Berlin Heidelberg, [**54**]{} (2010).", "\n\nJ. Abajo-Arrastia, J. Aparicio and E. Lopez, [*Holographic Evolution of Entanglement Entropy*]{}, JHEP [**1011**]{} (2010) 149, \\[arXiv:1006.4090 \\[hep-th\\]\\].", "\n\nV. Balasubramanian, A. Bernamonti, J. de Boer, N. Copland, B. Craps, E. Keski-Vakkuri, B. Muller, A. Schafer, M. Shigemori and W. Staessens, [*Holographic Thermalization*]{}, Phys.", " Rev. D [**84**]{} (2011) 026010, \\[arXiv:1103.2683 \\[hep-th\\]\\].", "\n\nS. Ryu and T. Takayanagi, [*Holographic derivation of entanglement entropy from AdS/CFT*]{}, Phys.", " Rev. Lett.", "  [**96**]{} (2006) 181602 \\[hep-th/0603001\\].", "\n\nD. N. Page, Phys.", " Rev. Lett [**71**]{} (1993) 1291, \\[arXiv:9305007 \\[gr-qc\\]\\].", "\n\nN. Lashkari, D. Stanford, M. Hastings, T. Osborne and P. Hayden, [*Towards the Fast Scrambling Conjecture*]{}, JHEP [**1304**]{} (2013) 022, \\[arXiv:1111.6580 \\[hep-th\\]\\].", "\n\nJ. M. Magan and S. Paganelli, [*Codification volume of an operator algebra and its irreversible growth through thermal processes*]{}, Phys. ", " Rev.  A [**90**]{} (2014) 032103, \\[arXiv:1404.2208 \\[quant-ph\\]\\].", "\n\nG. T. Horowitz and V. E. Hubeny, [*Quasinormal modes of AdS black holes and the approach to thermal equilibrium*]{}, Phys. ", " Rev.  D [**62**]{} (2000) 024027, \\[arXiv:9909056 \\[hep-th\\]\\].", "\n\nA. P. Balachandranm, T. R. Govindarajan, A. Q. Queiroz and A. F. Reyes-Lega, [*Entanglement and Particle Identity: A Unifying Approach*]{}, Phys.", " Rev. Lett.", "  [**110**]{} (2013) 080503 , \\[arXiv:1303.0688 \\[hep-th\\]\\].", "\n\nA. P. Balachandranm, T. R. Govindarajan, A. Q. Queiroz and A. F. Reyes-Lega, [*Algebraic Approach to Entanglement and Entropy*]{}, Phys.", " Rev. A.  [**88**]{} (2013) 022301, \\[arXiv:1301.1300 \\[math-ph\\]\\].", "\n\nV. Karanen and P. Kleinert, [*Thermalization of Wightman functions in AdS/CFT and quasinormal modes*]{}, \\[arXiv:1511.08187 \\[hep-th\\]\\].", "\n\nV. Balasubramanian, A. Bernamonti, B. Craps, V. Kernanen, E Keski-Vakkuri, B Muller, L. Thorlacius and J. Vanhoof, [*Thermalization of the spectral function in strongly coupled two dimensional conformal field theories*]{}, JHEP [**4**]{} (2013) 69, \\[arXiv:1212.6066 \\[hep-th\\]\\].", "\n\nA. Jansen and J. Magan, [*Black hole collapse and democratic models*]{}, \\[arXiv:1604.03772 \\[hep-th\\]\\].", "\n\nR. H. Price, [*Nonspherical perturbations of relativistic gravitational collapse. ", "I. Scalar and gravitational perturbations*]{}, Phys.", " Rev. D.  [**5**]{} (1972) 2419.", "\n\nD. Birmingham, I. Sachs and S. .L Solodukhin, [*Conformal field theory interpretation of black hole quasinormal modes*]{}, Phys.", " Rev. Lett [**88**]{} (2002) 151301. ", "\\[arXiv:0112.055 \\[hep-th\\]\\].", "\n\n[^1]: In this article we will use $\\mathcal{N}$ to denote the size of the matrices in the matrix model. ", "The number of degrees of freedom $N$ will scale as $N\\sim \\mathcal{N}^{2}$. We choose this uncommon convention so that $N$ measures the entropy both in the toy model, in which it just counts the number of oscillators, and in the matrix model.", "\n\n[^2]: Non-locality can appear in diverse non-equivalent forms. ", "One famous example is noncommutative geometry and noncommutative quantum field theory [@ncom]. ", "Other examples are systems with long range couplings preserving translational symmetry [@simone].", "\n\n[^3]: The gaussian nature of each matrix entry can be relaxed. ", "As customary in random matrices, the only thing that matters is the mean and the variance of each entry. ", "What it is important in our approach is that all entries are random with the same variance. ", "Relaxing this condition is an interesting open problem we leave for future work. ", "In particular, regarding [@useth], it is interesting to ask how many couplings can we switch off (or correlate with other couplings) so that ergodicity is broken. ", "For the concerns of this article, the most important feature the model possess is large-N factorization. ", "It would be interesting to see to what extent we can make the model less random but still mantaining large-N factorization. ", "We thank an anonymous referee for pointing this aspect to us.", "\n\n[^4]: See [@ETH] for an earlier discovery of ETH and [@eisertreview] for an extensive review of the subject of quantum thermalization.", "\n\n[^5]: Generalized free fields are gauge invariant operators that generate a free Fock space in the large-N limit. ", "See section (\\[secIII\\]) below for a brief description and Ref.", " [@papadodimas] for a detailed treatment.", "\n\n[^6]: Indeed, from the results of the previous section we expect $p= 3$. If we take the static results of [@useth] we expect at least $p\\geq 1$. For large-N matrix models typically we have $p\\geq 1$. The ultimate reason for the suppression is the well-known fact that the connected part of a correlator in a large-N theory kills at least one sum more\n\n[^7]: For a characterization of random states in sectors with a fixed number of particles see [@useth].", "\n\n[^8]: The previous assertion needs to be rigorously proven. ", "Indeed, the previous argument might be supporting the claim that the global time scale for relaxation is $Rt\\sim\\mathcal{O}(1/\\sqrt{\\bar{N}})$ in these collapse scenarios. ", "We leave this issue for future work.", "\n\n[^9]: Let us stress a technical issue at this point. ", "The fact that we perturb only one degree of freedom (\\[initial\\]) does not mean we cannot discuss information scrambling. ", "Information about the first degree of freedom, i.e its initial occupation number, is shared with the rest of the system as time evolves. ", "This spreading can be defined more properly by using Mutual Information, as was layed out in [@uscod].", "\n\n[^10]: We refer to Ref.[@papadodimas] for more details and a complete set of references.", "\n\n[^11]: We remark here that large-N factorization is rigorously proven in the vacuum [@thooft]. ", "In what follows we assume its validity at finite temperature as well, as done in [@papadodimas]. ", "This assumption is clearly supported by the AdS/CFT correspondence [@adscft].", "\n\n[^12]: In an AdS/CFT this is predicted from the gravitational perspective, since interactions in the bulk are suppressed in the large-$N$ limit\n\n[^13]: This might be of interest for the approach to entanglement entropy developed in [@amilcar].", "\n\n[^14]: Notice that the polynomial decay might be connected with black holes in flat space, given the old results of Ref. [", "@flatold]. ", "This is an exciting connection worth to explore.", "\n\n[^15]: See [@tao; @haake] for excellent and complete treatments of Random Matrix Ensembles from a mathematical and physical perspective respectively. ", "See the third footnote of the article for comments on modifying the nature of the randomness of the model.", "\n" ]
{ "pile_set_name": "ArXiv" }
[ 0, 0.004739336492890996, 0, 0, 0, 0, 0, 0.005970149253731343, 0, 0, 0.018633540372670808, 0.0072992700729927005, 0.00546448087431694, 0, 0, 0.019867549668874173, 0.018779342723004695, 0, 0.011904761904761904, 0.009615384615384616, 0, 0, 0.004524886877828055, 0, 0.008658008658008658, 0, 0, 0.01556420233463035, 0.011363636363636364, 0.015625, 0, 0, 0, 0, 0, 0, 0, 0, 0.00510204081632653, 0, 0, 0.010752688172043012, 0, 0, 0, 0, 0, 0.011111111111111112, 0, 0, 0, 0, 0, 0.015306122448979591, 0, 0, 0.02631578947368421, 0, 0.00998003992015968, 0, 0, 0, 0, 0.0038910505836575876, 0.016129032258064516, 0.009900990099009901, 0, 0, 0, 0.0055248618784530384, 0, 0.00819672131147541, 0, 0, 0, 0, 0, 0.008, 0, 0, 0, 0, 0, 0, 0, 0, 0, 0.004761904761904762, 0.0056179775280898875, 0.0047169811320754715, 0.0008326394671107411, 0, 0.007936507936507936, 0.0022222222222222222, 0, 0.0038684719535783366, 0, 0.0028544243577545195, 0, 0, 0, 0, 0.000925925925925926, 0, 0, 0, 0, 0, 0.0016137708445400753, 0.006756756756756757, 0, 0, 0, 0.0058823529411764705, 0, 0, 0.002336448598130841, 0, 0, 0.0028089887640449437, 0, 0, 0, 0, 0, 0, 0, 0.004419889502762431, 0, 0.008695652173913044, 0, 0, 0, 0, 0, 0.004730368968779565, 0, 0, 0, 0, 0.00245398773006135, 0, 0, 0, 0.004484304932735426, 0, 0, 0, 0, 0, 0, 0, 0, 0, 0, 0, 0, 0, 0, 0, 0.0022471910112359553, 0, 0.00847457627118644, 0, 0, 0, 0, 0, 0.012096774193548387, 0.006024096385542169, 0.013513513513513514, 0.007633587786259542, 0, 0, 0, 0.00625, 0.0076045627376425855, 0, 0, 0.0078125, 0, 0.020512820512820513, 0.015037593984962405, 0, 0, 0, 0.010752688172043012, 0, 0.017857142857142856, 0, 0, 0, 0, 0, 0, 0, 0, 0, 0.007142857142857143, 0, 0.0064516129032258064, 0, 0, 0, 0.005235602094240838, 0, 0, 0.005277044854881266, 0, 0.01282051282051282, 0.0017953321364452424, 0, 0, 0, 0, 0, 0, 0, 0, 0.00558659217877095, 0, 0, 0, 0, 0.0034403669724770644, 0, 0.011235955056179775, 0, 0.009174311926605505, 0.004484304932735426, 0, 0, 0, 0, 0.0015360983102918587, 0, 0.005208333333333333, 0.005319148936170213, 0, 0, 0, 0, 0, 0, 0, 0, 0, 0.006085192697768763, 0, 0, 0, 0, 0.007017543859649123, 0, 0.003861003861003861, 0, 0.0070921985815602835, 0.017543859649122806, 0.012658227848101266, 0, 0, 0, 0.007692307692307693, 0, 0, 0.004291845493562232, 0, 0, 0.013888888888888888, 0.006097560975609756, 0.01038961038961039, 0.01021355617455896, 0, 0, 0, 0, 0.001949317738791423, 0, 0, 0, 0, 0.0070921985815602835, 0, 0, 0, 0.006993006993006993, 0.010193679918450561, 0.0078125, 0, 0, 0.013530135301353014, 0.007246376811594203, 0, 0.021505376344086023, 0.14285714285714285, 0.006230529595015576, 0, 0.009615384615384616, 0.005813953488372093, 0.003683241252302026, 0.004484304932735426, 0, 0, 0.005714285714285714, 0, 0, 0, 0, 0, 0, 0, 0, 0.008547008547008548, 0, 0, 0, 0.012345679012345678, 0, 0, 0, 0, 0, 0.006756756756756757, 0, 0, 0, 0.014367816091954023, 0, 0.010416666666666666, 0, 0, 0.011235955056179775, 0.026845637583892617, 0, 0, 0.006578947368421052, 0.03424657534246575, 0, 0, 0.0136986301369863, 0, 0, 0, 0, 0, 0, 0, 0, 0.0058823529411764705, 0.005, 0, 0.0031746031746031746, 0, 0, 0, 0, 0, 0, 0.007575757575757576, 0, 0, 0, 0.012987012987012988, 0, 0, 0, 0.0027624309392265192, 0, 0, 0.008130081300813009, 0, 0, 0.008310249307479225, 0, 0.007017543859649123, 0.004424778761061947, 0.008928571428571428, 0.005555555555555556, 0, 0, 0.006711409395973154, 0.01282051282051282, 0.019230769230769232, 0, 0, 0, 0, 0, 0, 0, 0, 0.018867924528301886, 0, 0.02631578947368421, 0.03125, 0, 0.014084507042253521, 0.017543859649122806, 0.04225352112676056, 0.014925373134328358, 0, 0, 0.015625, 0, 0, 0, 0, 0, 0, 0, 0, 0, 0.013888888888888888, 0, 0.03076923076923077, 0.010101010101010102, 0.008849557522123894, 0, 0.01694915254237288, 0, 0.016, 0, 0.011904761904761904, 0.008849557522123894, 0.018518518518518517, 0.015267175572519083, 0, 0.015384615384615385, 0.006666666666666667, 0, 0, 0, 0, 0, 0, 0, 0.018691588785046728, 0.015873015873015872, 0.011494252873563218, 0, 0.007462686567164179, 0.006666666666666667, 0, 0, 0, 0, 0, 0.02127659574468085, 0.018867924528301886, 0, 0.02197802197802198, 0, 0.02, 0, 0, 0, 0, 0.011494252873563218, 0, 0.014705882352941176, 0.008, 0.015625, 0.006802721088435374, 0, 0.01639344262295082, 0.007246376811594203, 0.014705882352941176, 0.014388489208633094, 0.010638297872340425, 0.009345794392523364, 0, 0.019230769230769232, 0, 0.015384615384615385, 0, 0.03333333333333333, 0, 0, 0, 0.010526315789473684, 0.010309278350515464, 0, 0, 0, 0, 0.006134969325153374, 0, 0, 0, 0.029411764705882353, 0, 0.015873015873015872, 0.024390243902439025, 0.00437636761487965, 0, 0, 0, 0, 0, 0, 0.0196078431372549, 0.011111111111111112, 0.010309278350515464, 0.010309278350515464, 0.025974025974025976, 0.00816326530612245, 0.008064516129032258, 0.09090909090909091, 0, 0.013157894736842105, 0, 0 ]
0.004305
5
[ "The present invention relates to a blocking sled with pivotable auxiliary pad. ", "In the prior art, such devices are typically employed by offensive linemen in the game of American football to practice blocking techniques. ", "In particular, such sleds typically include a frame that may glide over a grassy field and a bracket to which is affixed a pad that may be hit by the practicing player. ", "Such a pad may be fixedly mounted to a frame via the bracket or may be movable with respect thereto.", "\nWhen coaching an offensive lineman or even a running back or receiver in the proper blocking techniques, one technique that is often taught is that the player should maintain a low center of gravity for as long a period of time as possible so that the player has maximum leverage against a player they are blocking. ", "Human nature being what it is, typically, an offensive lineman either practicing blocking techniques or blocking during an actual football game has a tendency to rise up out of their initial three point stance and elevate their center of gravity which reduces their effectiveness.", "\nAs such, a need has developed for a blocking sled that will facilitate practicing of blocking techniques including maintenance of a low center of gravity. ", "It is with this need in mind that the present invention was developed.", "\nThe following prior art is known to Applicant:\nU.S. Pat. ", "No. ", "3,578,324 to Alvey et al. ", "discloses a football blocking apparatus having a frame with a pad mounted thereon as well as a pivotable arm having a proximal end behind the pad. ", "The present invention differs from the teachings of Alvey et al. ", "as contemplating a blocking sled having a main pad and an auxiliary pad overlying and extending forward of the main pad.", "\nU.S. Pat. ", "No. ", "3,649,016 to Kelley, Jr. discloses a football blocking trainer including a dummy that may be moved to either side or extended, retracted or swung to simulate movements of an opponent. ", "The present invention differs from the teachings of Kelley, Jr. as contemplating a blocking sled having a main pad and an auxiliary pad overlying and extending forward of the main pad.", "\nU.S. Pat. ", "No. ", "3,684,283 to Forrest (Applicant herein) discloses a wheeled football training sled having an overhead chute. ", "The overhead chute is vertically adjustable, but has no actuator to move it responsive to movements of the main pad.", "\nU.S. Pat. ", "No. ", "3,942,796 to Bowen discloses a football practice blocking and tackling reaction machine in which a dummy is mounted on a pivotable support. ", "The present invention differs from the teachings of Bowen as contemplating a blocking sled having a main pad and an auxiliary pad overlying and extending forward of the main pad.", "\nU.S. Pat. ", "No. ", "5,385,523 to Forrest discloses a dual motion blocking sled in which a pad is mounted on a frame that allows the pad to both reciprocate and pivot. ", "The present invention differs from the teachings of Forrest as contemplating a blocking sled having a main pad and an auxiliary pad overlying and extending forward of the main pad.", "\nU.S. Pat. ", "No. ", "5,462,272 to Staten discloses a football training sled in which a pad is mounted on a spring-biased telescoping support. ", "The present invention differs from the teachings of Staten as contemplating a blocking sled having a main pad and an auxiliary pad overlying and extending forward of the main pad.", "\nU.S. Pat. ", "No. ", "6,599,206 to Forrest, Sr. ", "et al. ", "discloses a triple reactor linemen trainer in which a plurality of pads are mounted on a frame and certain ones of the pads may be pivoted laterally based upon actuations by an operator. ", "The present invention differs from the teachings of Forrest, Sr. ", "et al. ", "as contemplating a blocking sled having a main pad and an auxiliary pad overlying and extending forward of the main pad.", "\nU.S. Pat. ", "No. ", "6,685,581 to Krause et al. ", "discloses an athletic training device consisting of a blocking sled having a plurality of pads that are movable against the force of spring bias. ", "The present invention differs from the teachings of Krause et al. ", "as contemplating a blocking sled having a main pad and an auxiliary pad overlying and extending forward of the main pad." ]
{ "pile_set_name": "USPTO Backgrounds" }
[ 0, 0, 0, 0, 0, 0, 0, 0, 0.034482758620689655, 0, 0.038461538461538464, 0, 0.015384615384615385, 0, 0.09090909090909091, 0, 0.010869565217391304, 0.005434782608695652, 0.09090909090909091, 0, 0.009174311926605505, 0, 0.09090909090909091, 0, 0.007142857142857143, 0.0056179775280898875, 0.09090909090909091, 0, 0.006802721088435374, 0.005555555555555556, 0.09090909090909091, 0, 0, 0, 0.09090909090909091, 0, 0, 0, 0, 0, 0, 0, 0.09090909090909091, 0, 0, 0, 0, 0 ]
0.016152
5
[ "Q:\n\nDialogflow cannot test on slack anymore?", "\n\nI got this error every time I try to click 'test in slack' in the Dialogflow Integration:\nTest bot activation error: undefined\nI don't know what happened here. ", "It works like two weeks ago and I didn't change anything. ", "Is it just me or everyone?", "\n\nA:\n\nI've tried to Test integration with Slack today and looks like it starts to work again. ", "Shutdown current test bot if possible and do OAuth again to see if the test @Dialogflow Bot comes back.", "\n\n" ]
{ "pile_set_name": "StackExchange" }
[ 0, 0, 0, 0, 0.010638297872340425, 0.009708737864077669, 0 ]
0.002907
5
[ "East Hastings and Main\n\nGastown\n\nSun Ah Hotel\n\nCrab Park\n\nEveryone’s cold and scared.", "\n\nScrawlings in the back place speak of heroin hidden. ", "This is a place, the kind that looks dark, where you can mistake trees for shadows depending on what bipolar plane you’re looking at. ", "Where you think branches are charcoal. ", "The Eastside, a place where you look back so you can look forward.", "\n\nThis is not a place where consciousness can be placed as a brick, but it is a place of threes: systematic piss and prick and alleys. ", "There’s happenings, invisibilities, easiness, and pipes that invoke a glitch.", "\n\nRed like scaly rabbits that breed sores in a place black.", "\n\nBlue is where one talks.", "\n\nCheap skyline. ", "Vancouver and of you. ", "Coffee up. ", "Quiet. ", "You are in the clouds. ", "Mortar brick, cheap. ", "Water, rent, beckons of Diet Coke cans in diners that stirs, that benefits. ", "Sometimes, at Vancouver Harbour, someone can see it all coming in amidst the choppy deafening grey.", "\n\nRally, comfort from some plastered poster — paper doesn’t blush — on every telephone pole. ", "The publicity, the glamour.", "\n\nPurple smoggy, on days that the dusk has a hue.", "\n\nSettle the women as they attempt business with hollow-faced men who walk.", "\n\nThe cold creates an edge typical. ", "Damns are taken. ", "Why? ", "is thought. ", "Why not step off somewhere?", "\n\nTourists have a different motive entirely, they walk as if it’s a zoo, as if everyone who lives there is entertainment, meant to be gawked at, men and women different enough to be nobody.", "\n\nSome people may go fast, some think they are ghosts, some have a fear and therefore crawl because they can feel implants.", "\n\nThere are some who smear their eyeballs as with lipstick and happenstance, the moon red with blood, dark as a youth who blushes and spatters, for they see dreams of blue farms and undiscovered fragments of pink girls, girls who make statements of skulls who see shadows. ", "Tomorrow, may be bold and may be red.", "\n\nEveryone here is creased like paper, the girls’ skin applicators failing at their job.", "\n\nIt is getting darker, darker still.", "\n\nThe vampires come out, eating themselves." ]
{ "pile_set_name": "OpenWebText2" }
[ 0, 0, 0, 0, 0.015151515151515152, 0, 0, 0, 0, 0, 0, 0, 0, 0, 0.047619047619047616, 0, 0, 0, 0, 0, 0.013333333333333334, 0, 0, 0, 0, 0, 0, 0, 0, 0, 0, 0, 0 ]
0.002306
5
[ "Benomyl resistance of Colletotrichum acutatum is caused by enhanced expression of beta-tubulin 1 gene regulated by putative leucine zipper protein CaBEN1.", "\nColletotrichum acutatum, the fungus causing anthracnose disease of various fruits and crops, has inherent resistance to benomyl. ", "The mechanism underlying its resistance to benomyl remains unclear. ", "We generated a benomyl-sensitive mutant CAT7-150 by transformation-mediated insertional mutagenesis. ", "Subsequently, a CaBEN1 gene was isolated from CAT7-150 by plasmid rescue and sequenced. ", "CaBEN1 encodes a protein of 818 amino acids containing a leucine zipper motif, and the amino acid sequence has significant similarity to hypothetical proteins found in Gibberella zeae and other fungi. ", "Disruption and complementation of the CaBEN1 gene demonstrated that CaBEN1 was necessary for resistance to benzimidazole fungicides, but was not essential for mycelial growth. ", "Northern analysis suggests that CaBEN1 has an important role in transcriptional activation of CaTUB1 in response to benomyl. ", "Benomyl resistance of CaBEN1-disrupted transformant was recovered by overexpression of CaTUB1. ", "Our results clearly demonstrate that the benomyl resistance is due to enhanced expression of CaTUB1, which is controlled by CaBEN1." ]
{ "pile_set_name": "PubMed Abstracts" }
[ 0.006493506493506494, 0, 0, 0, 0, 0.004975124378109453, 0, 0, 0, 0 ]
0.001147
5
[ "INTRODUCTION {#sec1-1}\n============\n\nDermatofibrosarcoma protuberans (DFSP) is a rare tumor involving the dermis of the skin with the incidence of 3--4/million.\\[[@ref1][@ref2]\\] These tumors are locally aggressive; however, distant metastases are unusual.\\[[@ref3][@ref4][@ref5]\\] The hallmark of this cancer is a specific translocation of COL1A1 and PDGFB (around 90% of the lesions).\\[[@ref6][@ref7]\\] This makes the tumors susceptible to treatment with tyrosine kinase inhibitor imatinib.\\[[@ref8]\\] F-18 fluorodeoxyglucose positron emission tomography/computed tomography (FDG PET/CT) may be a useful modality in treatment response evaluation of metastatic DFSP.", "\n\nCASE REPORT {#sec1-2}\n===========\n\nThe index case is a 62-year-old male patient. ", "He presented for the 1^st^ time 17 years back with a lesion in the left thigh which was completely excised and diagnosed as DFSP. ", "He presented with local recurrence after 15 years and again underwent wide local resection with negative margins of excision. ", "Imaging with CT scan did not show any distant metastases. ", "There was no adjuvant therapy. ", "There was a local recurrence a 2^nd^ time after 1 year. ", "An excision biopsy was repeated and revealed fibrosarcomatous components within the DFSP \\[Figure [1a](#F1){ref-type=\"fig\"} and [b](#F1){ref-type=\"fig\"}\\]. ", "An ^18^F-FDG PET/CT scan done to evaluate the disease extent showed extensive metastases in the both lungs \\[Figure [2a](#F2){ref-type=\"fig\"}, [c](#F2){ref-type=\"fig\"} and [e](#F1){ref-type=\"fig\"}\\] along with bone metastasis involving the left acetabulum. ", "A review of the previous chest CT scan done at the time of the first recurrence a year prior confirmed that the lung lesions were a new development. ", "The patient was started on imatinib 400 mg once a day. ", "The therapy was well tolerated without any major toxicity. ", "A repeat PET scan was done 5 months after initiation of treatment and shows complete metabolic resolution of all the lesions while the CT reveals only partial response \\[Figure [2b](#F2){ref-type=\"fig\"}, [d](#F2){ref-type=\"fig\"} and [f](#F2){ref-type=\"fig\"}\\]. ", "The patient is currently on follow-up and has completed 7 months of therapy with imatinib.", "\n\n![(", "a) The low power view of the tumor showing herring bone pattern (b) high power view showing spindle shaped tumor cells arranged in fascicles and exhibiting moderate pleomorphism](IJNM-31-191-g001){#F1}\n\n![(", "a) Whole body ^18^F-fluorodeoxyglucose positron emission tomography image prior to treatment showing lesions in the left thigh, left iliac bone and lungs. (", "b) Whole body ^18^F-fluorodeoxyglucose positron emission tomography image posttreatment showing complete metabolic response of all lesions. (", "c) Pretreatment transaxial image of the lung showing intensely hypermetabolic lung metastases. (", "d) Posttreatment transaxial image of the lung showing complete metabolic resolution and partial anatomical resolution of the lung lesions. (", "e) Pretreatment transaxial image showing hypermetabolic left iliac bone lesion. (", "f) Posttreatment transaxial image showing complete metabolic resolution of the iliac lesion](IJNM-31-191-g002){#F2}\n\nDISCUSSION {#sec1-3}\n==========\n\nOur case report has several noteworthy features.", "\n\nDFSP is a rare tumor of the skin with intermediate malignant potential. ", "As exemplified in our case, the tumor typically presents with multiple local recurrences despite adequate wide local excision. ", "The best surgical outcomes have been achieved with Moh\\'s microsurgical procedure which is practiced only at specialized centers.\\[[@ref9]\\]\n\nThe tumor in our patient demonstrated fibrosarcomatous transformation within the classical DFSP which is an unusual feature.\\[[@ref10][@ref11]\\] The patient had rapid development of extensive pulmonary and skeletal metastases within a span on 1 year and the role of the transformation as a contributory factor is an intriguing question. ", "Though pulmonary and lymph nodal metastases have been described previously in DFSP, bone metastasis are rare \\[[Figure 2e](#F2){ref-type=\"fig\"}\\].", "\n\nThe third noteworthy feature of the case is the radiological response to treatment with imatinib. ", "More than 90% of cases of DFSP harbor translocation of 17 and 22 genes.\\[[@ref5][@ref12]\\] Imatinib being a tyrosine kinase inhibitor has been shown to have definite role in DFSP in the following situations:\\[[@ref5][@ref12]\\]\n\nlocally advanced disease that is inoperable,locally advanced disease where a reduction in the size of the tumor would aid in surgical resection, andMetastatic disease not amenable to surgical resection.", "\n\nThere are very few case reports on the role of ^18^F-FDG PET in detection and follow-up of DFSP in the literature.\\[[@ref13][@ref14][@ref15][@ref16]\\] Our patient demonstrated a drastic metabolic response on ^18^F-FDG PET with the morphological response lagging behind \\[[Figure 2e](#F2){ref-type=\"fig\"}\\]. ", "It is well-known that all cases of DFSP may not harbor the same genetic translocation\\[[@ref17]\\] and hence might not respond to imatinib. ", "We would like to propose that demonstration of response on ^18^F-FDG PET/CT can act as a surrogate marker for patients who harbor gene rearrangement in DFSP and similar to the situation in nonsmall cell lung cancer, an early response evaluation PET may be able to predict the outcome in these patients.", "\n\nFinancial support and sponsorship {#sec2-1}\n---------------------------------\n\nNil.", "\n\nConflicts of interest {#sec2-2}\n---------------------\n\nThere are no conflicts of interest.", "\n" ]
{ "pile_set_name": "PubMed Central" }
[ 0.017991004497751123, 0, 0, 0, 0, 0, 0, 0, 0.0038910505836575876, 0, 0, 0, 0.0038314176245210726, 0, 0, 0, 0, 0, 0, 0, 0, 0, 0, 0, 0.008350730688935281, 0, 0, 0.009302325581395349, 0.019417475728155338, 0.007194244604316547, 0.006622516556291391, 0, 0.010869565217391304, 0 ]
0.002573
5
[ "The latest Jolicloud kernel is now supporting the latest netbooks based on the Intel Pine Trail platform. ", "To perform the update you need to go to My Jolicloud and click the ‘Update All’ button. ", "Having a look through the list, there doesn’t seem to be any major Pine trail netbook that has been excluded. ", "Check out the list for yourself after the break.", "\n\nAccording to , it’s based on Ubuntu Netbook Edition. ", "Upgrades come out periodically and are dowaolnded through Joli’s own update manager. ", "And the company behind it is called Jolicloud also (and has renamed the OS Joli OS ).As if what happens if they cease to exist well, not much, really. ", "The only thing it really uses the cloud for in and of itself is to synchronize what applications you have installed on your desktop. ", "If you don’t run it on more than one computer, you don’t even notice." ]
{ "pile_set_name": "Pile-CC" }
[ 0.018867924528301886, 0.011363636363636364, 0.00909090909090909, 0, 0, 0.011764705882352941, 0.006622516556291391, 0, 0 ]
0.006412
5
[ "fileFormatVersion: 2\nguid: 87377c86d84f49a4e912d37d28353e7f\ntimeCreated: 1485179854\nlicenseType: Store\nMonoImporter:\n serializedVersion: 2\n defaultReferences: []\n executionOrder: 0\n icon: {instanceID: 0}\n userData: \n assetBundleName: \n assetBundleVariant: \n" ]
{ "pile_set_name": "Github" }
[ 0 ]
0
5
[ "Olivier, Jane, and Louisiana) First Church of St. Charles (Grand Coteau. ", "The Church of St. Charles, Grand Coteau, Louisiana: index of baptisms, 1819-1930, volumes I-IV. (", "Grand Coteau, Louisiana: J. Olivier, c1981)." ]
{ "pile_set_name": "Pile-CC" }
[ 0.0273972602739726, 0.010309278350515464, 0.045454545454545456 ]
0.02772
5
[ "A former residential drug and alcohol rehab facility at 1072 and 1082 Vermont St. in San Jose’s Rose Garden neighborhood is being converted to a house for 16 homeless veterans. ", "The two homes are collectively known as the Vermont House.", "\n\nTwo years after work began on transforming a former residential drug and alcohol rehabilitation facility in the Rose Garden neighborhood into a home for 16 homeless veterans, the time has arrived for the new residents to move in.", "\n\nThe Vermont House, as the two homes at 1072 and 1082 Vermont St. are collectively known, was acquired by the city of San Jose through foreclosure in 2009 when the rehab facility defaulted on its loan. ", "The conversion project involves a multi-agency partnership that includes the city and Santa Clara County Housing Authority.", "\n\nThe City Council approved a $3 million grant for renovations that were sorely needed, as the building had mold, a broken heating system, leaking ceilings, foundation issues and a host of other problems.", "\n\nWorkers, including volunteers from Habitat for Humanity, literally helped rebuild the houses from the ground up, according to Patrick Heisinger of the San Jose Housing Department.", "\n\n“The property was built in the 1920s,” Heisinger said in an interview. “", "It’s a very nice architectural design, but there were some foundational issues that had to be addressed.”", "\n\nWith the homes waiting to receive certificates of occupancy any day, Heisinger said the target move-in date is Dec. 15. ", "About half of the residents have already been selected with help from the VA Palo Alto Health Care System and nonprofit group Abode Services, which will provide on-site management.", "\n\n“We’re moving ahead; we have about half of the veterans selected,” Abode Executive Director Louis Chicoine said. “", "We’ll have them all selected by the time we have our certificate.”", "\n\nSince 2015, the countywide All the Way Home campaign, which aims to find housing for the county’s homeless veterans, has managed to house about 510 of them. ", "More than half were given Veterans Affairs Supportive Housing vouchers, which will also be used by Vermont House residents.", "\n\n“Having all entities at the table is really what it takes to house our most vulnerable,” Heisinger said.", "\n\n“Without the vouchers, there wouldn’t be any income to provide ongoing services,” he said. “", "The project-based vouchers, the Housing Authority’s commitment to this project, allow Abode the owner to collect (and), in turn, use them to keep the property up and also have case managers on site.”", "\n\nStakeholders have managed to find permanent housing for veterans in a variety of places, but Chicoine said the Vermont House is meant to provide a supportive environment.", "\n\n“The issue with vets is their needs are so intense because of their experiences,” often resulting in post-traumatic stress disorder and issues involving reintegrating into society, he said. ", "Many also often struggle with mental illness and drug and alcohol abuse to cope with their problems, and being with others in the same position can help, he added.", "\n\n“The focus is on the strengths veterans have; they want to work together in a unit,” Chicoine said. “", "They’re working in a community towards them all being in a better situation.”", "\n\nHundreds of high school girls browse through clothes racks for the perfect prom dress during the \"Dressed by West\" event created by Lesley West at the Golden State Warriors practice facility in Oakland.", "\n\nFor the second time this year, the Cupertino community and other interested stakeholders got a chance to help shape the future of Vallco. ", "A March 13 presentation at Cupertino Community Hall led by Dan Parolek of Berkeley-based architecture and urban design firm Opticos included live polling as well as breakout sessions where people were invited to write down their ideas...\n\nSunnyvale residents are being asked to help the city name its newest park. ", "Dedicated to the city as part of The Vale residential development at 925 Deguigne Drive, the new 0.8-acre park includes a basketball court, play equipment, open grass area, barbecues and shaded picnic tables. ", "The park isn't finished yet, according to city spokeswoman Jennifer Garnett, adding it should be..." ]
{ "pile_set_name": "Pile-CC" }
[ 0, 0, 0, 0.0049261083743842365, 0, 0.004901960784313725, 0.016574585635359115, 0, 0, 0, 0.011111111111111112, 0.008620689655172414, 0, 0.006289308176100629, 0.024390243902439025, 0, 0, 0.010050251256281407, 0.011627906976744186, 0, 0, 0.009708737864077669, 0, 0.004901960784313725, 0.007142857142857143, 0.0031847133757961785, 0.004784688995215311, 0.010101010101010102 ]
0.00494
5
[ "Enlarge ESO/AP This artist rendering provided by the European South Observatory shows some of the 32 new planets astronomers found outside our solar system. ", "Outer space just got a little more crowded Monday as European astronomers unveiled 32 newly detected planets orbiting nearby stars. ", "The discovery by the High Accuracy Radial Velocity Planet Searcher (HARPS) team includes four \"Super-Earths\" only five to six times heavier than our own world. \"", "None are in a habitable zone\" — they're too hot to support life — says team scientist Stephane Udry of Switzerland's University of Geneva. ", "DISCUSSION: Astronomy expert, readers talk habitation, life forms on other planets About 400 \"extrasolar\" planets orbiting nearby stars have been detected since 1995, starting with a discovery made by the same team, led by Switzerland's Michel Mayor, of a Jupiter-sized planet orbiting the star 51 Pegasi, 50 light-years away. (", "One light-year is about 5.9 trillion miles.) \"", "We are on the good track to detect (indirectly) Earth-type planets within the next five to 10 years,\" Udry says. ", "Using the European Southern Observatory's 11.8-foot-wide telescope at Chile's La Silla Observatory, the team detects planets by \"radial velocity\" measure, which reveals the gravitation wobbles induced on stars by their planets. \"", "Wow — 32 or so planets at once — that certainly is a record for the largest number of new planets announced at the same time,\" says planetary scientist Alan Boss of the Carnegie Institute of Washington. \"", "It really shows that the Europeans have taken the lead\" in radial velocity planet hunting, he adds. ", "The team announced the newly discovered planets at a science meeting in Porto, Portugal. ", "They range in size from 5.4 times more massive than Earth to 7.1 times heavier than Jupiter, the largest planet in our solar system, and their host stars ranged from about 30 to 150 light-years away. ", "Two of the \"Super-Earths\" — thought to be rocky planets like Earth and not gas giants — orbit stars like our sun, and the other two orbit smaller \"M\" class stars, dimmer and redder than the sun. ", "So \"we have yet to find firm evidence for a habitable, Earth-mass planet,\" Boss says. ", "But he says the Super-Earth detections suggest that upcoming planet hunts, including NASA's Kepler spacecraft, should find \"lots of Earths.\" ", "Dr. Alan P. Boss, a research staff member at the Carnegie Institution's Department of Terrestrial Magnetism in Washington, D.C., and author of The Crowded Universe: The Search for Living Planets, answered your questions on habitation and life forms on other planets in the discussion below. ", "Discuss new planets with Dr. Alan P. Boss Copyright 2009 The Associated Press. ", "All rights reserved. ", "This material may not be published, broadcast, rewritten or redistributed. ", "Guidelines: You share in the USA TODAY community, so please keep your comments smart and civil. ", "Don't attack other readers personally, and keep your language decent. ", "Use the \"Report Abuse\" button to make a difference. ", "You share in the USA TODAY community, so please keep your comments smart and civil. ", "Don't attack other readers personally, and keep your language decent. ", "Use the \"Report Abuse\" button to make a difference. ", "Read more" ]
{ "pile_set_name": "OpenWebText2" }
[ 0.012738853503184714, 0, 0.006211180124223602, 0.014388489208633094, 0.006097560975609756, 0, 0.008849557522123894, 0.004366812227074236, 0.00980392156862745, 0, 0, 0, 0, 0, 0.028368794326241134, 0.010309278350515464, 0.02531645569620253, 0, 0, 0.010416666666666666, 0, 0, 0.011904761904761904, 0, 0, 0 ]
0.005722
5
[ "Isotope dilution high-performance liquid chromatography-electrospray tandem mass spectrometry assay for the measurement of 8-oxo-7,8-dihydro-2'-deoxyguanosine in biological samples.", "\nA sensitive and specific assay aimed at measuring 8-oxo-7,8-dihydro-2'-deoxyguanosine (8-oxodGuo) has been developed by associating a reversed-phase liquid chromatographic separation with an electrospray tandem mass spectrometric detection. ", "The HPLC-MS approach in the single ion monitoring (SIM) mode and the HPLC-MS/MS assay in the multiple reaction monitoring (MRM) mode have been compared, using isotopically labeled [M+4] 8-oxodGuo as the internal standard. ", "The limit of detection of 8-oxodGuo was found to be around 5 pmol and 20 fmol for the HPLC-MS and HPLC-MS/MS methods, respectively. ", "The HPLC-MS/MS assay is sensitive enough to allow the determination of the level of 8-oxodGuo in cellular liver DNA and in urine samples." ]
{ "pile_set_name": "PubMed Abstracts" }
[ 0, 0, 0.009009009009009009, 0.007575757575757576, 0.0072992700729927005 ]
0.004777
5
[ "A-One (TV channel)\n\nA-One Hip-Hop Music Channel – Russian music channel about the most important and actual trends of the youth culture. ", "The musical directions represented on the channel are hip-hop, r’n’b, rap, electronic music, funk, reggae, soul, indie and everything that blended in between.", "\n\nAs of 2015 A-One is broadcast with the help of the satellite ABS-2 (FTA), from the platform “Tricolor TV” in the networks of the cabel TV and IP-TV, mobile communication, OTT-platforms and Smart-TV and also in the TVEVT. ", "The channel is represented in 850 packages of the Russian cabel operators.", "\n\nHistory of the channel \n\n1 August 2005 is considered to be the beginning of the A-One twenty-four-hour broadcasting. ", "\nFrom 2005 to 2010 the channel established the annual RAMP (Rock Alternative Music Prize). ", "The foreign rock-singers as well as Russian were the nominees of the prize. ", "The winner was picked out by the audience votes. ", "The ceremonies in different years saw a lot of stars such as Korn (2005), Stone Sour (2006), UNKLE and Marilyn Manson (2007), Klaxons (2008) and Franz Ferdinand (2009).", "\n\nA-One is the organizer of the world star’s concerts in Moscow and St. Petersburg including Linkin Park, Apocalyptica, Tricky, The Roots, Manu Chao, Faith No More and the others.", "\n\nThe channel co-operates directly with such companies-distributors of the musical content as Universal, Sony Music, Warner, EMI, Soyuz and the others which allows getting the new music videos in a short period of time.", "\n\nIn October, 2012 A-One won the prize “Golden (Ray)” for the best Russian music channel. ", "In 2013 A-One Hip-Hop Music Channel entered the final of the international prize, The Eutelsat TV Awards. ", "In 2010 and 2014 the channel won the prize for “Music channel” at the prestige “Big Digit”.", "\n\nIn 2014 the channel launched the series of the night parties all over the country, A-ONE HYPE NIGHTS, with the most popular foreign and Russian artists. ", "The first night’s guest was Tyga (22 November 2014. ", "Space Moscow).", "\n\nVJ's and hosts \n\n VJ Yuklya (Julia Vorontsova)\n VJ gleBasta (Gleb Bolelov)\n\nPrograms\n\nTrendy\n\nMusic spots \n\n «Only New» — the hottest new music videos on air.", "\n “Only eggs are harder”– the most brutal music videos, music extreme to a fault. ", "The other channels don’t risk broadcasting these videos despite their popularity in the net.", "\n «Chill-Out» - the hour of beautiful and easy-tempered music before going to bed. ", "Relieve your stress; watch “Chill-Out”. ", "Every Thursday and Sunday at 23:00 (Msc time).", "\n «Mash Up»– the selection of energetic club music, the most fashionable dance floor in your TV, mobile phone and computer! ", "Watch and have fun every Friday and Saturday at 22:00.", "\n «Line Up» – the most popular hip-hop and r’n’b music videos and the best hits of the music mainstream. ", "Foreign and Russian artists, celebrities of the global scale and new names!", "\n «Wake Up» — start your day with us listening to the energetic music! ", "Bright, fresh and positive! ", "Let’s shake up the country! ", "Every day from 6 to 9 o’clock in the morning.", "\n «HIP-HOP.RU» — the hip-hop in Russian. ", "The collection of the tough and honest works of Russian hip-hop artists including the young stars. ", "Every day from Monday to Thursday at 01:00 o’clock.", "\n\nCharts \n\n «A-One Hip Hop Top 20» — the main chart of the channel. ", "Only the hottest music videos of popular groups and favorite artists. ", "Saturday, 19:00.", "\n «Urban Hits Top 10» — fashionable sound, music with no limits. ", "The chart of trendy music. ", "Saturday, 21:00.", "\n «All Russian» — chart of the Russian hip-hop. ", "Saturday, 15:00.", "\n «Dance Hits Top 10» — ten best dance music videos of the week. ", "The music of the most fashionable clubs all over the world. ", "Saturday, 18:00.", "\n\nInteresting facts \n\nOn 1 April 2011, some foreign names of the songs and artists were written in Russian and during the music spots under the logo there was an inscription “Эй-Уан” (A-ONE in Russian transliteration). ", "Despite it, the special emission “A-One News” was broadcast that day with the help of the channel’s staff. ", "This emission also took place in the chart “100 most stupid videos”, which was on air the 1st of April, 2009.", "\n\nBroadcasting \n\nThe channel is broadcast on the satellite platforms Tricolor TV, Raduga TV, Akado-Telecom, MTS, Dom.ru, TKT (Rostelekom), Beeline, OnLime, NETBYNET.", "\n'''\n\nExternal links\n A-One TV official site\n\nCategory:Music television channels\nCategory:Defunct television channels in Russia\nCategory:Television production companies of Russia\nCategory:Russian music awards\nCategory:Television channels and stations established in 2005\nCategory:Television channels and stations disestablished in 2016\nCategory:2005 establishments in Russia\nCategory:2016 disestablishments in Russia\nCategory:Music organizations based in Russia" ]
{ "pile_set_name": "Wikipedia (en)" }
[ 0, 0, 0.017937219730941704, 0, 0, 0.01098901098901099, 0, 0, 0.017857142857142856, 0.0111731843575419, 0.0182648401826484, 0, 0.009433962264150943, 0, 0.0064516129032258064, 0, 0, 0.0125, 0, 0, 0, 0, 0, 0, 0, 0, 0, 0, 0, 0, 0, 0, 0, 0, 0, 0, 0, 0, 0, 0, 0, 0, 0, 0, 0, 0, 0, 0, 0.04242424242424243, 0.0021691973969631237 ]
0.002984
5
[ "Bere Bay\n\nBere Bay is an Arctic waterway in the Qikiqtaaluk Region, Nunavut, Canada. ", "It is located in Norwegian Bay, off Devon Island's Grinnell Peninsula. ", "Triton Bay is to the southeast.", "\n\nExternal links\n Bere Bay, Nunavut at Atlas of Canada\n\nCategory:Bays of Qikiqtaaluk Region" ]
{ "pile_set_name": "Wikipedia (en)" }
[ 0.011764705882352941, 0, 0, 0.01098901098901099 ]
0.005688
5
[ "Q:\n\nЧерная иконка меню в toolbar'е\n\nНа всех устройствах с Android 4.4 и ниже (Minimum SDK 17 стоит) проявляется такая проблема: иконка меню черная. ", "Для всего остального, что есть в Toolbar, я задал цвета в разметке или в коде и все стало нормально. ", "А с этой иконкой прямо что-то не так...\n \nДа, возможно вопрос является дубликатом другого, но решение, предложенное там, мне не помогло. ", "Атрибут colorControlNormal действительно меняет цвет этой иконки, но он меняет и Background EditText'ов, а мне это совсем не нужно. ", "Как это исправить?", "\nitem'ы меню:\n<menu xmlns:android=\"http://schemas.android.com/apk/res/android\"\nxmlns:app=\"http://schemas.android.com/apk/res-auto\"\nxmlns:tools=\"http://schemas.android.com/tools\"\ntools:context=\".", "MainActivity\">\n<item\n android:id=\"@+id/action_settings\"\n android:orderInCategory=\"100\"\n android:title=\"@string/settings\"\n app:showAsAction=\"never\" />\n<item\n android:id=\"@+id/action_aboutapp\"\n android:orderInCategory=\"100\"\n android:title=\"@string/aboutapp\"\n app:showAsAction=\"never\" />\n<item\n android:id=\"@+id/action_floatbutton\"\n android:title=\"@string/edit_action\"\n android:icon=\"@drawable/ic_action_edit\"\n app:showAsAction=\"ifRoom\"/>\n\nМеню инициализируется в onCreateOptionsMenu.", "\nРазметка toolbar:\n<?", "xml version=\"1.0\" encoding=\"utf-8\"?", ">\n<android.support.design.widget.", "CoordinatorLayout \nxmlns:android=\"http://schemas.android.com/apk/res/android\"\nxmlns:app=\"http://schemas.android.com/apk/res-auto\"\nxmlns:tools=\"http://schemas.android.com/tools\"\nandroid:layout_width=\"match_parent\"\nandroid:layout_height=\"match_parent\"\ntools:context=\"com.churkin.myapplication.", "MainActivity\">\n\n<android.support.design.widget.", "AppBarLayout\n android:layout_width=\"match_parent\"\n android:layout_height=\"wrap_content\"\n android:theme=\"@style/AppTheme.", "AppBarOverlay\">\n\n <android.support.v7.widget.", "Toolbar\n android:id=\"@+id/toolbar\"\n android:layout_width=\"match_parent\"\n android:layout_height=\"?attr/actionBarSize\"\n android:background=\"?attr/colorPrimary\"\n app:popupTheme=\"@style/AppTheme.", "PopupOverlay\"\n app:titleTextColor=\"@android:color/white\" />\n\n</android.support.design.widget.", "AppBarLayout>\n\n<include layout=\"@layout/content_main\" />\n\n</android.support.design.widget.", "CoordinatorLayout>\n\nЕго стиль:\n <style name=\"AppTheme.", "AppBarOverlay\" parent=\"ThemeOverlay.", "AppCompat.", "Dark.", "ActionBar\" />\n <style name=\"AppTheme.", "PopupOverlay\" parent=\"ThemeOverlay.", "AppCompat.", "Light\" />\n\nA:\n\nДело было в том, что объявление и \"перекрашивание\" тулбара выполнялось после super.onCreate(savedInstanceState). ", "Не знаю как это связано, но теперь всё работает правильно\n\n" ]
{ "pile_set_name": "StackExchange" }
[ 0.013513513513513514, 0.019801980198019802, 0.021897810218978103, 0.007575757575757576, 0, 0.015463917525773196, 0.015473887814313346, 0.047619047619047616, 0.02857142857142857, 0, 0.010309278350515464, 0.02127659574468085, 0.007751937984496124, 0, 0.017699115044247787, 0.01, 0.011111111111111112, 0, 0, 0.1, 0, 0.025, 0, 0.1, 0.015625, 0 ]
0.018796
5
[ "Q:\n\nDisable \"internet access\" for JVM?", "\n\nI'd like to know if there is a way to tell the JVM that it cannot connect to any web resource for a certain Java program, or to immediately fail when doing so, i.e. to do a software equivalent of turning off internet access with a hardware switch. ", "\nThis is to assist an automated test, disabling the system's firewall is no option for me. ", "\nBackground: \nI'm currently working on a Java issue where XML identity transformation does not work with a DOCTYPE referenced in XML like this: \n<?", "xml version=\"1.0\" encoding=\"utf-8\"?", ">\n<!", "DOCTYPE svg PUBLIC \"-//W3C//DTD SVG 1.1//EN\" \"http://www.w3.org/Graphics/SVG/1.1/DTD/svg11.dtd\">\n<svg version=\"1.1\" xmlns=\"http://www.w3.org/2000/svg\" xmlns:xlink=\"http://www.w3.org/1999/xlink\" x=\"0px\" y=\"0px\" width=\"32px\"\n height=\"32px\" viewBox=\"0 0 32 32\" enable-background=\"new 0 0 32 32\" xml:space=\"preserve\">\n<!-- ", "content.... -->\n</svg>\n\nThe standard behavior of DocumentBuilderFactory, TransformerFactory etc. ", "is to access the web for the missing entities. ", "\nWhile the fix suggested https://stackoverflow.com/a/9398602/1143126 (a NullEntityResolver) resolved most of my problems, I'd like to test this functionality for regression in an automated way in an \"offline environment\". ", "\n\nA:\n\nThe comment by Dunes helped a lot (issue How to disable all network connections in Java is related), I overlooked that other question. ", "\nHere is what I will use to prevent network connections: \n\nStart the test case with JVM arguments -Djava.security.manager=default -Djava.security.policy=/java.policy\nAs java.policy, I use the default file I found with my Java installation, and added the following lines s.t. ", "stuff works with TestNG: \n// // additional permissions for running TestNG // // \n\n// TestNG reads a lot of properties...\npermission java.util.", "PropertyPermission \"*\", \"read\";\n\n// TestNG connects to a local port for debugging, and does some reflection magic\npermission java.net.", "SocketPermission \"127.0.0.1:*\", \"connect,resolve\";\npermission java.lang.", "RuntimePermission \"accessDeclaredMembers\";\npermission java.lang.reflect.", "ReflectPermission \"suppressAccessChecks\";\n\n// needs at least read access to (default) test suite folder location\npermission java.io.", "FilePermission \"C:/Users/<me>/AppData/Local/Temp/-\", \"read, write\";\n\n// if the test case (or, data provider) accesses any other files, add their location. ", "or do it the lazy way:\npermission java.io.", "FilePermission \"C:/-\", \"read, write\";\n\nWith that configuration, any access to an external source such as www.w3.org will result in an AccessControlException: \njavax.xml.transform.", "TransformerException: java.security.", "AccessControlException: access denied (java.net.", "SocketPermission www.w3.org:80 connect,resolve)\n\n" ]
{ "pile_set_name": "StackExchange" }
[ 0.02631578947368421, 0.004, 0, 0.013605442176870748, 0.02857142857142857, 0, 0.01238390092879257, 0.020618556701030927, 0, 0.0045045045045045045, 0.014184397163120567, 0.0036363636363636364, 0, 0, 0, 0, 0, 0, 0, 0.0111731843575419, 0, 0, 0.02040816326530612 ]
0.006931
5
[ "# Finite State Entropy\r\n\r\nThis package provides Finite State Entropy encoding and decoding.", "\r\n \r\nFinite State Entropy (also referenced as [tANS](https://en.wikipedia.org/wiki/Asymmetric_numeral_systems#tANS)) \r\nencoding provides a fast near-optimal symbol encoding/decoding\r\nfor byte blocks as implemented in [zstandard](https://github.com/facebook/zstd).", "\r\n\r\nThis can be used for compressing input with a lot of similar input values to the smallest number of bytes.", "\r\nThis does not perform any multi-byte [dictionary coding](https://en.wikipedia.org/wiki/Dictionary_coder) as LZ coders,\r\nbut it can be used as a secondary step to compressors (like Snappy) that does not do entropy encoding. ", "\r\n\r\n* [Godoc documentation](https://godoc.org/github.com/klauspost/compress/fse)\r\n\r\n## News\r\n\r\n * Feb 2018: First implementation released. ", "Consider this beta software for now.", "\r\n\r\n# Usage\r\n\r\nThis package provides a low level interface that allows to compress single independent blocks. ", "\r\n\r\nEach block is separate, and there is no built in integrity checks. ", "\r\nThis means that the caller should keep track of block sizes and also do checksums if needed. ", " \r\n\r\nCompressing a block is done via the [`Compress`](https://godoc.org/github.com/klauspost/compress/fse#Compress) function.", "\r\nYou must provide input and will receive the output and maybe an error.", "\r\n\r\nThese error values can be returned:\r\n\r\n| Error | Description |\r\n|---------------------|-----------------------------------------------------------------------------|\r\n| `<nil>` | Everything ok, output is returned |\r\n| `ErrIncompressible` | Returned when input is judged to be too hard to compress |\r\n| `ErrUseRLE` | Returned from the compressor when the input is a single byte value repeated |\r\n| `(error)` | An internal error occurred. ", " |\r\n\r\nAs can be seen above there are errors that will be returned even under normal operation so it is important to handle these.", "\r\n\r\nTo reduce allocations you can provide a [`Scratch`](https://godoc.org/github.com/klauspost/compress/fse#Scratch) object \r\nthat can be re-used for successive calls. ", "Both compression and decompression accepts a `Scratch` object, and the same \r\nobject can be used for both. ", " \r\n\r\nBe aware, that when re-using a `Scratch` object that the *output* buffer is also re-used, so if you are still using this\r\nyou must set the `Out` field in the scratch to nil. ", "The same buffer is used for compression and decompression output.", "\r\n\r\nDecompressing is done by calling the [`Decompress`](https://godoc.org/github.com/klauspost/compress/fse#Decompress) function.", "\r\nYou must provide the output from the compression stage, at exactly the size you got back. ", "If you receive an error back\r\nyour input was likely corrupted. ", "\r\n\r\nIt is important to note that a successful decoding does *not* mean your output matches your original input. ", "\r\nThere are no integrity checks, so relying on errors from the decompressor does not assure your data is valid.", "\r\n\r\nFor more detailed usage, see examples in the [godoc documentation](https://godoc.org/github.com/klauspost/compress/fse#pkg-examples).", "\r\n\r\n# Performance\r\n\r\nA lot of factors are affecting speed. ", "Block sizes and compressibility of the material are primary factors. ", " \r\nAll compression functions are currently only running on the calling goroutine so only one core will be used per block. ", " \r\n\r\nThe compressor is significantly faster if symbols are kept as small as possible. ", "The highest byte value of the input\r\nis used to reduce some of the processing, so if all your input is above byte value 64 for instance, it may be \r\nbeneficial to transpose all your input values down by 64. ", " \r\n\r\nWith moderate block sizes around 64k speed are typically 200MB/s per core for compression and \r\naround 300MB/s decompression speed. ", "\r\n\r\nThe same hardware typically does Huffman (deflate) encoding at 125MB/s and decompression at 100MB/s. \r\n\r\n# Plans\r\n\r\nAt one point, more internals will be exposed to facilitate more \"expert\" usage of the components. ", "\r\n\r\nA streaming interface is also likely to be implemented. ", "Likely compatible with [FSE stream format](https://github.com/Cyan4973/FiniteStateEntropy/blob/dev/programs/fileio.c#L261). ", " \r\n\r\n# Contributing\r\n\r\nContributions are always welcome. ", "Be aware that adding public functions will require good justification and breaking \r\nchanges will likely not be accepted. ", "If in doubt open an issue before writing the PR. ", " " ]
{ "pile_set_name": "Github" }
[ 0.02197802197802198, 0.010948905109489052, 0, 0.0044444444444444444, 0.007194244604316547, 0, 0, 0, 0, 0.008, 0, 0.003278688524590164, 0, 0.005952380952380952, 0, 0, 0, 0.007751937984496124, 0, 0, 0, 0, 0.0072992700729927005, 0, 0, 0, 0, 0, 0, 0.0045871559633027525, 0, 0.008064516129032258, 0, 0, 0, 0 ]
0.002486
5
[ "The PragmaPro is based on a modular cartridge technology and supports various NDT\ninstrument modal\n\n...\n\nities such as UT, PAUT, ECT and many more. ", "This new platform is\nbased on a machined, powder-coated aluminum frame for shock-proofness, best sealing\nqualities and maximum heat dissipation. ", "This is practical to extend the outdoor\ntemperature range and/or to extend the power injected in the transducers. ", "The\nPragmaPro is aiming at a very wide range of applications, such as weld scanning,\ncorrosion mapping and composite testing.", "\n\n2-ch TOFD, which can maximize your\nefficiency for PA and TOFD.", "\n● Support PA/TOFD/UT\n● 32-ch PA is more suitable for inspection on extra-thick wall and\nhigh-attenuation material.", "\n● 32-ch PA and 2-ch TOFD work simultaneously.", "\n● Support PR mode for corrosion inspection." ]
{ "pile_set_name": "Pile-CC" }
[ 0.02702702702702703, 0, 0, 0, 0, 0, 0, 0 ]
0.003378
5
[ "Stander\n\nStander may refer to:\n\nStanding frame for assisting the physically challenged\nStander (surname)\nStander (film) 2003 film about bank robber Andre Stander" ]
{ "pile_set_name": "Wikipedia (en)" }
[ 0.006211180124223602 ]
0.006211
5
[ "// @flow\n\nimport React from 'react';\nimport { GameContainerMock } from '../../../../../../mocks/GameContainerMock';\nimport WellGrid from '..';\n\nconst grid = [\n [null, [5, '#fbb414'], [6, '#fbb414']],\n [null, [4, '#3993d0'], [7, '#fbb414']],\n [\n [1, '#3993d0'],\n [2, '#3993d0'],\n [3, '#3993d0'],\n ],\n];\n\nexport default (\n <GameContainerMock cols={6} rows={6}>\n <WellGrid grid={grid} blocksCleared={[]} blocksPending={[]} />\n </GameContainerMock>\n);\n" ]
{ "pile_set_name": "Github" }
[ 0.006437768240343348 ]
0.006438
5
[ "Two controversial bills making their way through the General Assembly that could harm the civil rights of many people including LGBT residents are receiving heavy resistance from local activists who have showed up in force to protest.", "\n\nHouse Bill 1023 and Senate Bill 377, both named the “Preservation of Religious Freedom Act,” are making their way through the legislature but what was expected is still unclear.", "\n\nGeorgia Equality Executive Director Jeff Graham told the GA Voice late Tuesday night, “According to various news sources, the House bill does appear dead at this point. ", "However, there are still several days where the committee could meet and move the bill forward.”", "\n\nThe bill’s House sponsors have until midnight on Monday, March 3, to take action and hope it “crosses over” to be voted on by the Senate before the community can breathe a sigh of relief at HB 1023 truly being stalled.", "\n\n“There is also the matter of Senate Bill 377 that has already passed out of committee and is sitting in the Senate Rules Committee,” Graham continued.", "\n\n“We must remain vigilant, but Delta’s public opposition to these bills and the quickly changing meeting schedule do indicate that these bills are losing support.”", "\n\nHouse Judiciary Committee Chair Wendell Willard (R-Sandy Springs) told WABE on Tuesday he did not think that HB 1023 would survive this session. ", "On Monday evening, a second hearing for the bill was scheduled and then quickly canceled.", "\n\nAlso on Tuesday, Delta Air Lines issued a statement against bills like HB 1023 and a similar bill recently passed in Arizona: “As a global values based company, Delta Air Lines is proud of the diversity of its customers and employees, and is deeply concerned about proposed measures in several states, including Georgia and Arizona, that would allow businesses to refuse service to lesbian, gay, bisexual and transgender individuals. ", "If passed into law, these proposals would cause significant harm to many people and will result in job losses.”", "\n\nAn online petition was also started to urge lawmakers to stop the bills and has nearly 30,000 signatures already.", "\n\nGAY ACTIVIST: BILLS LIKE A ‘SWORD’ MEANT TO INJURE PEOPLE\n\nRep. Sam Teasley (R-Marietta) has stated that amendments he made to HB 1023 would make it just like the Religious Freedom Restoration Act passed by President Clinton in 1993.", "\n\nAnthony Kreis, a PhD student in the School of Public and International Affairs at the University of Georgia, agrees that HB 1023 and the federal law are similar, but says the bigger concern should be SB 377, the Senate bill which already passed through committee and could get a floor vote before Crossover Day.", "\n\n“That bill is nothing like the bill President Clinton signed in the ’90s,” Kreis told GA Voice. “", "The Senate bill is a sword which can be used to injure people, versus the federal law which was intended to shield people from objections that if you accommodate people they don’t harm other parties.”", "\n\nKreis, also the political co-chair for Atlanta’s HRC Steering Committee, believes the House bill is dead this session but the Senate version still is very much alive and could have treacherous consequences if passed.", "\n\n“The House bill doesn’t look like it’s going to make progress. ", "The process is bogged down, they got a late start. ", "If either bill has a chance of success it would be the Senate version, which is why I’m so personally concerned — because that’s what I consider the dangerous bill — the one with better chances to succeed. ", "We need to keep an eye on that,” Kreis said.", "\n\n“The House bill as it stands now and the Senate bill are worlds apart. ", "The biggest difference is the Senate bill threatens to undermine our civil rights tradition, our religious liberty tradition and it raises church and state establishment concerns,” he added.", "\n\n‘CAN’T SEE IT HAPPENING’\n\nOn Monday, a capacity crowd that spilled out into the Capitol rotunda waited more than three hours for a subcommittee hearing on HB 1023 that could open Georgia’s LGBT residents to more discrimination.", "\n\nThe delay was good news to Rep. Karla Drenner (D-Avondale Estates). “", "I’m happy that they’re taking a moment to pause on it. ", "I know there’s going to be a substitute bill so I’m not sure at this juncture what’s going to happen with that,” she said Tuesday afternoon.", "\n\nHouse Judiciary Chair Rep. Wendell Willard (R-Sandy Springs) told WABE 90.1 FM on Tuesday that it was unlikely the bill would pass out of committee in time by Crossover Day on Monday, March 3.", "\n\n“Can’t see it happening,” Willard told WABE. “", "It came in rather late in the session. ", "Too many proponents and opponents.”", "\n\nRep. Drenner added she was also concerned with allegations that some LGBT community members were saying disrespectful remarks to the bill’s sponsor, Rep. Sam Teasley, at the subcommittee hearing.", "\n\n“That doesn’t help our cause at all,” she said. “", "You can come down and be upset, but after you spew all over someone and leave, that becomes the lasting impression of what gay people are like for that member in the General Assembly. ", "If given the opportunity to come back again, perhaps the behavior might be more professional.”", "\n\n‘IT’S NOT ABOUT CAKE’\n\nEmotions were high during the long day at the Georgia Capitol as LGBT activists waited for hours for the hearing to start, with many leaving before it began. ", "The delay was due to the full House Judiciary Committee hearing that ran long, pushing the 3 p.m. scheduled time back to past 5 p.m. Then an unrelated bill was heard first.", "\n\nPoliticians, community leaders, religious leaders, business owners and private citizens flooded the room throughout the night. ", "State Senator and gubernatorial candidate Jason Carter stopped in for a quick greeting to the crowd but did not make any remarks on the bill.", "\n\nGeorgia Equality’s Graham later told the GA Voice that he hadn’t seen the community come out like this on an issue since 2004.", "\n\n“If we could get this many people out to support pro-LGBT legislation like Rep. Karla Drenner’s Fair Employment Practices Act or Rep. Keisha Waites anti-bullying bill [HB 816] we would likely see movement on them,” he said.", "\n\nState Rep. Simone Bell (D-Atlanta), who is gay, came in for a portion of the meeting and made emotional remarks to committee members.", "\n\n“This is of great interest to our community,” she said. “", "It’s not about a cake, it’s not about a wedding event. ", "It’s about us being able to live our lives fully, not as second class citizens but being able to go through this world as the people we were created to be.”", "\n\nGraham also told committee members he values religious freedom, but religion is often the cause of of preventing equality for LGBT people.", "\n\n“Please understand that as someone with a background in theology and religious studies, I value religious freedom and applaud efforts to ensure that diverse views are always heard and respected,” Graham said.", "\n\n“However, as a gay man, I can assure you that the vast majority of derogatory comments I have heard and the primary arguments used to prevent my community from achieving legal equality are rooted in what people profess to be their religious beliefs or their personal interpretation of a few selected biblical passages.”", "\n\n‘DANGEROUS AND IMMEASURABLE IN SCOPE’\n\nThe state Democratic Party is calling the bills “dangerous and immeasurable in scope.”", "\n\n“Among many other things, [the bills would] open a Pandora’s box of LGBT discrimination and unfairly target women based on their health care decisions,” stated a press release from the Democratic Party of Georgia.", "\n\n“What makes this even more alarming is the fact that Republicans are using a tactic they seem to be quite fond of lately by scheduling the hearings in tandem to sidestep opposition testimony,” said Michael Smith, spokesperson for the Democratic Party of Georgia.", "\n\n“The pathetic part of this is that Republicans could have spent their time crafting legislation that put people back to work or armed Georgia’s future workforce with a first-class education,” Smith added. “", "Instead, the Georgia GOP has connived ways to manufacture right-wing primary fodder for the far fringe of their base that stealthily silences the voice of any mainstream voter who will find this legislation objectionable.”", "\n\nAccording to the state Democratic Party, the proposed bills would:\n\n• Allow individuals and private organizations to claim an imposition on their religious freedoms to make decisions otherwise protected by current law. ", "For instance, both of these bills would allow employers to discriminate against potential employees based on current protected legal statuses — like race, sex, age, pregnancy, nationality and even religion under the guise of protecting the employer’s religious freedom.", "\n\n• Also: “While Republicans state that the bills are designed to protect religious freedoms that are already guaranteed under the U.S. Constitution, the legislation is tailored in a way to limit some of our most basic protections — including access to birth control for women and protection from state-sanctioned discrimination against LGBT individuals.”", "\n\nIn an action alert sent out last week, Georgia Equality said:\n\n• This bill would allow a person to take advantage and use personal religious beliefs to claim the right to break important laws that are meant to protect all of us.", "\n\n• Expansion of religious freedoms should not come at the expense of Georgia’s existing civil rights protections, public safety provisions, and other laws that serve the common good. ", "These bills could allow:\n\n-An employer to fire a woman who remarried after a divorce or who is pregnant and not married.", "\n\n-A person whose religion demands pacifism and non-violence to refuse to rent an apartment to a tenant who owns a gun.", "\n\n-The owner of a sandwich shop to refuse to serve a gay customer.", "\n\n-A healthcare worker to refuse a woman a prescribed medication.", "\n\n-A counselor be exempted from state required licensing requirements.", "\n\n“The bill applies not just to individuals but also to for-profit and secular corporations and organizations. ", "These businesses entered the stream of commerce for monetary gain and thus should not be allowed to reap the benefits of the marketplace but not have to play by the same rules as everyone else,” Georgia Equality stated.", "\n\npsaunders@thegavoice.com | @patricksaunders" ]
{ "pile_set_name": "OpenWebText2" }
[ 0.004273504273504274, 0.0111731843575419, 0.023391812865497075, 0, 0.013636363636363636, 0.019736842105263157, 0.006097560975609756, 0.027210884353741496, 0, 0.006880733944954129, 0, 0, 0.01276595744680851, 0.01597444089456869, 0.030303030303030304, 0.005, 0.013761467889908258, 0.015384615384615385, 0, 0.0048543689320388345, 0.022727272727272728, 0.0273972602739726, 0.005263157894736842, 0.008733624454148471, 0.028169014084507043, 0, 0, 0.005154639175257732, 0.041666666666666664, 0, 0, 0.01015228426395939, 0, 0.005434782608695652, 0, 0, 0.005813953488372093, 0, 0.0070921985815602835, 0.015625, 0.013333333333333334, 0.007407407407407408, 0, 0, 0, 0.007142857142857143, 0.004761904761904762, 0, 0.007874015748031496, 0.004651162790697674, 0.007575757575757576, 0, 0.0045045045045045045, 0.004524886877828055, 0, 0, 0.004347826086956522, 0, 0, 0.008403361344537815, 0, 0.015384615384615385, 0.014285714285714285, 0, 0.0045662100456621, 0.044444444444444446 ]
0.008286
5
[ "/**\n * Copyright (c) UNA, Inc - https://una.io\n * MIT License - https://opensource.org/licenses/MIT\n *\n * @defgroup Payment Payment\n * @ingroup UnaModules\n *\n * @{\n */\n\nfunction BxPaymentProviderChargebeeV3(oOptions) {\n this.init(oOptions);\n}\n\nBxPaymentProviderChargebeeV3.prototype = new BxPaymentMain();\n\nBxPaymentProviderChargebeeV3.prototype.init = function(oOptions) {\n this._sProvider = oOptions.sProvider;\n this._sActionsUrl = oOptions.sActionUrl;\n this._sObjName = oOptions.sObjName == undefined ? '", "oPaymentProviderChargebeeV3' : oOptions.sObjName;\n this._sAnimationEffect = oOptions.sAnimationEffect == undefined ? '", "fade' : oOptions.sAnimationEffect;\n this._iAnimationSpeed = oOptions.iAnimationSpeed == undefined ? '", "slow' : oOptions.iAnimationSpeed;\n\n this._sObjNameCart = oOptions.sObjNameCart == undefined ? '' : ", "oOptions.sObjNameCart;\n this._iClientId = oOptions.iClientId == undefined ? ", "0 : oOptions.iClientId;\n this._iSellerId = oOptions.iSellerId == undefined ? ", "0 : oOptions.iSellerId;\n this._iModuleId = oOptions.iModuleId == undefined ? ", "0 : oOptions.iModuleId;\n this._iItemId = oOptions.iItemId == undefined ? ", "0 : oOptions.iItemId;\n this._sItemName = oOptions.sItemName == undefined ? '' : ", "oOptions.sItemName;\n this._iItemCount = oOptions.iItemCount == undefined ? ", "0 : oOptions.iItemCount;\n this._sItemAddons = oOptions.sItemAddons == undefined ? '' : ", "oOptions.sItemAddons;\n this._sRedirect = oOptions.sRedirect == undefined ? '' : ", "oOptions.sRedirect;\n this._sCustom = oOptions.sCustom == undefined ? '' : ", "oOptions.sCustom;\n\n this._rHandler = Chargebee.init({\n site: oOptions.sSite\n });\n this._rHandler = Chargebee.getInstance();\n};\n\nBxPaymentProviderChargebeeV3.prototype.subscribe = function(oLink) {\n var $this = this;\n var oDate = new Date();\n\n oLink = jQuery(oLink);\n if(oLink.hasClass('bx-btn-disabled'))\n return;\n\n oLink.addClass('bx-btn-disabled');\n\n this._rHandler.openCheckout({\n hostedPage: function() {\n return $.post({\n url: $this._sActionsUrl + 'call/' + $this._sProvider + '/get_hosted_page/' + $this._iClientId + '/' + $this._iSellerId + '/' + $this._sItemName + '/' + $this._sItemAddons + '/',\n dataType: 'json'\n });\n },\n success: function(sHostedPageId) {\n $this.loadingInPopup(oLink, true);\n\n var oParams = {\n seller_id: $this._iSellerId,\n seller_provider: $this._sProvider,\n module_id: $this._iModuleId,\n item_id: $this._iItemId,\n item_count: $this._iItemCount,\n item_addons: $this._sItemAddons,\n redirect: $this._sRedirect,\n custom: $this._sCustom,\n page_id: sHostedPageId,\n _t: oDate.getTime()\n };\n\n $.post(\n $this._sActionsUrl + 'subscribe_json/',\n oParams,\n function(oData){\n $this.loadingInPopup(oLink, true);\n\n processJsonData(oData);\n },\n 'json'\n );\n },\n close: function() {\n oLink.removeClass('bx-btn-disabled');\n }\n });\n\n return false;\n};\n\nBxPaymentProviderChargebeeV3.prototype.manage = function(oLink, iPendingId) {\n\tvar $this = this;\n var oDate = new Date();\n\n $(\".bx-popup-applied:visible\").dolPopupHide();\n\n\tthis._rHandler.setPortalSession(function() {\n \treturn $.post({\n \t\turl: $this._sActionsUrl + 'call/' + $this._sProvider + '/get_portal/' + iPendingId + '/',\n \t\tdataType: 'json'\n \t});\n });\n\n\tvar cbPortal = this._rHandler.createChargebeePortal();\n\tcbPortal.open({});\n\n\treturn false;\n};\n\n/** @} */\n" ]
{ "pile_set_name": "Github" }
[ 0.013307984790874524, 0, 0, 0, 0, 0, 0, 0, 0, 0, 0.01098901098901099, 0, 0, 0.0013507429085997298 ]
0.001832
5
[ "\nWhat if we paid for Facebook – instead of letting it spy on us for free? - ", "randomerr\nhttps://www.washingtonpost.com/news/the-switch/wp/2018/04/05/what-if-we-paid-for-facebook-instead-of-letting-it-spy-on-us-for-free/\n======\nstephenr\nWasn't there already an attempt at this? ", "app.net ?", "\n\nI'm sure a sizeable chunk of Facebook's user base would not be willing to pay\n_any_ amount.", "\n\nTake for example: an in-law, who uses Facebook to \"store her photos\" because\npaying (the equivalent of) $0.99 a month for more iCloud storage space, is not\nsomething even within the realm of possibility.", "\n\n------\ndotcoma\nWhat if we just got off the hook instead?", "\n\n" ]
{ "pile_set_name": "HackerNews" }
[ 0, 0.005025125628140704, 0, 0.010752688172043012, 0.004878048780487805, 0.017241379310344827, 0 ]
0.005414
5
[ "Introduction\n============\n\nBackground\n----------\n\nThe aim of the present work is to extensively evaluate the pharmaceutical attributes of currently available riluzole presentations.", "\n\nRiluzole is the only drug licensed for use in the management of amyotrophic lateral sclerosis (ALS), the most common form of motor neurone disease.[@b1-dddt-11-059]\n\nALS is a progressive, ultimately fatal neurodegenerative disease, marked by a gradual degeneration of nerve cells of the central nervous system that control voluntary muscle movement. ", "Degeneration of motor neurones is characterized by muscle weakness, typically impacting arms and legs, speech, swallowing and breathing. ", "Impairment of swallowing (dysphagia) is a feature of ALS resulting from weakness or spasticity of muscles affecting the tongue, lips, palate, jaw, pharynx, larynx and upper trunk, causing difficulties for patients in the oral consumption of dry, tough-textured or crumbly food and thin liquids, although the specific nature of the difficulties depends on the patient's individual clinical pathology.[@b1-dddt-11-059]\n\nDestruction of nerve cells in motor neurone disease may be caused by excess glutamate (neurotransmitter) in the brain and spinal cord. ", "Riluzole is a glutamate antagonist that reduces the release of glutamate, thus slowing the progression of the early disease.[@b2-dddt-11-059] The burden of disease associated with poor patient compliance and a more rapid degeneration of motor neurones upon patients, family members and caregivers is substantial, with increasing cost associated with increasing disability and the need for assisted medical care.[@b2-dddt-11-059] Riluzole 100 mg daily is a disease-specific therapy that has been shown to slow disease progression in patients with probable and definitive ALS with symptoms of \\<5 years. ", "A median survival prolongation of up to 3 months has been shown in some patients in clinical studies.[@b2-dddt-11-059] The same review article cited earlier observational studies, which suggested that treatment with riluzole may be associated with a delay in disease progression of up to 21 months.[@b2-dddt-11-059]\n\nThe National Institute for Health and Care Excellence (NICE) approved riluzole for use in motor neurone disease in 2001, and the drug is freely available for diagnosed patients in the UK.[@b3-dddt-11-059] Riluzole is also approved in the European Union and US for the treatment of ALS.", "\n\nChallenges for ALS patients\n---------------------------\n\nMore than 80% of ALS patients develop dysphagia, rendering the oral administration of riluzole tablets impossible.[@b1-dddt-11-059],[@b4-dddt-11-059] The inability of a patient to swallow a tablet formulation may result in poor patient compliance and early discontinuation of a life-prolonging treatment. ", "Swallowing dysfunction and dysphagia however is a challenge for oral drug therapy.[@b5-dddt-11-059] Swallowing is a complex function involving several nerves and muscles acting in a synchronized reflex mode upon voluntary initiation.", "\n\nManipulation of a solid oral drug product by a patient or caregiver, for example, by crushing of tablets to aid administration or ingestion, increases the likelihood of medication errors and promotes the possibility of incorrect and incomplete dosing, changes in drug product performance and safety concerns. ", "It is important to recognize the potential consequences of manipulating a medicinal product. ", "Changes to the way a dosage form is presented can impact its absorption characteristics, cause product instability, produce local irritant or anesthetic effects (particularly for riluzole), cause failure to reach the site of absorption, may produce occupational health and safety issues and could result in a preparation with an unacceptable taste and/or unpleasant mouthfeel.[@b6-dddt-11-059] From a safety perspective, crushed tablets may expose carers or health care professionals to health risks due to dust inhalation, ingestion or non-oral absorption. ", "Irritation may also arise as a consequence of powdered drug substance coming into contact with the skin, eyes or other mucous membranes.", "\n\nNational Health Service guidance for riluzole tablets states that tablets can be crushed and given in a spoonful of sugar, food puree or yoghurt, however that crushed tablets can have an anesthetic effect on the tongue.[@b7-dddt-11-059] Such an undesirable and untoward side effect may result in a patient refusing or being unable to take his/her prescribed medication.", "\n\nCrushing tablets for the purpose of administration via an enteral feeding tube typically falls outside of the scope of a marketing authorization. ", "Under such circumstances, the prescriber and practitioner accept liability for any adverse events arising from administration. ", "Potential issues of drug administration via an enteral feeding tube include cleanliness and infection control, tube blockage, incorrect or incomplete dosing and particularly in a community setting, safety of the carer. ", "Applicable guidance states that tablet crushing should be considered as a last resort.[@b8-dddt-11-059] Furthermore administration of crushed tablets to ALS patients exhibiting swallowing dysfunction or dysphagia may result in aspiration pneumonia and an increased risk of silent aspiration.[@b4-dddt-11-059]\n\nA recent article highlights the many deficiencies of administering crushed tablets to patients.[@b9-dddt-11-059]\n\nUntil recently, riluzole was only available as an oral tablet dosage form; however, the first oral liquid form of riluzole (Teglutik^®^, novel riluzole oral suspension 5 mg/mL), has now been launched in the UK (Martindale Pharma Limited, Romford, Essex).[@b10-dddt-11-059],[@b11-dddt-11-059]\n\nAdvantages and potential challenges of an oral liquid presentation of riluzole\n------------------------------------------------------------------------------\n\nThe availability of an oral suspension of riluzole precludes the need for manipulation of tablets to facilitate administration. ", "Riluzole oral suspension is presented as a 5 mg/mL palatable oral liquid formulation comprising finely divided particles of drug substance suspended in a thixotropic vehicle. ", "The oral suspension utilizes controlled flocculation technology and exhibits pseudoplastic flow (shear thinning), meaning that the apparent viscosity of the vehicle is relatively high when the applied shear stress is low (ie, upon standing), but that the apparent viscosity decreases as the applied shear stress increases (ie, upon shaking). ", "The novel composition of the aqueous vehicle is designed to minimize the local anesthetic effect of the drug in the mouth and the unpleasant metallic taste.", "\n\nRiluzole oral suspension is supplied with a plastic graduated oral dosing syringe for accurate and reproducible dose administration for ALS patients. ", "The syringe barrel is graduated in milliliters up to 10 mL, to provide enhanced flexibility of dosing. ", "Each 1 mL of riluzole oral suspension contains 5 mg of active drug, allowing accurate incremental dosing up to 50 mg riluzole per 10 mL dose.[@b12-dddt-11-059] Riluzole oral suspension is indicated for twice-daily administration, with a 10 mL dose exhibiting the consistency of single cream, which is thus easily ingested by ALS patients who may experience difficulties in managing thin liquids.[@b1-dddt-11-059] In contrast, riluzole film-coated tablets are only available as a single-dose strength of 50 mg.[@b13-dddt-11-059]\n\nPotential challenges of riluzole oral suspension may include the perception that oral liquids are intended \"for children\" and that some patients may not initially be expecting to be offered an oral liquid. ", "In addition, some patients may not like having to manage 2×300 mL glass bottles of their medication each month. ", "Conversely, these possible negatives are more than offset by having the utility of riluzole in a dosage form that patients can easily take and that caregivers can easily administer with no compromise on the effectiveness of the drug.", "\n\nProduct overview\n================\n\nRationale for an aqueous suspension\n-----------------------------------\n\nRiluzole is a lipophilic drug substance with a low aqueous solubility. ", "The drug is very slightly soluble in water at neutral pH (\\~0.3 mg/ml at pH 7), and although the solubility of the drug substance increases with decreasing pH (\\~12 mg/ml at pH 1.2), chemical stability decreases significantly under acidic conditions.[@b14-dddt-11-059],[@b15-dddt-11-059] An aqueous solution formulation of riluzole is therefore not feasible.", "\n\nThe solubility of riluzole may be increased by means of co-solvents or solubilizers to produce a physically and chemically stable drug solution. ", "However, palatability is found to be significantly compromised, and the prolonged local anesthetic effect in the mouth (\\>20--30 minutes) is exacerbated as a consequence of the intrinsic properties of this drug substance in solution.[@b14-dddt-11-059],[@b15-dddt-11-059]\n\nSuspensions as drug delivery systems\n------------------------------------\n\nA pharmaceutical suspension is a relatively complex disperse system in which insoluble particles, generally \\>1 µm in diameter, are dispersed in a liquid medium, usually aqueous. ", "The small particle size and large surface area of dispersed drug particles ensures a high availability for dissolution and hence absorption.", "\n\nAn acceptable suspension should typically exhibit the following properties: the suspended material should not settle too rapidly; particles that do settle must not form a hard mass and should be readily dispersed upon shaking; and the suspension must not be too viscous to pour freely and provide uniform dosing.[@b16-dddt-11-059]\n\nHowever, many insoluble solids are not easily wetted by water and exhibit varying degrees of hydrophobicity, especially when finely divided. ", "Particles may form large porous clumps within the liquid, while others remain on the surface and may become attached to the container. ", "Any foam produced on shaking may be slow to subside because of the stabilizing effect of the small particles at the liquid/air interface. ", "To ensure adequate wetting, the interfacial tension between the solid and the liquid must be reduced so that absorbed air is displaced from solid surface by the liquid. ", "The particles will then disperse readily through the liquid, particularly if intense shearing is used during mixing.[@b16-dddt-11-059]\n\nThus, key ingredients of suspensions are surfactants, substances that alter the conditions prevailing at an interface, causing, for example, a marked decrease in the surface and interfacial tension of water. ", "These substances are of importance in a wide variety of fields as emulsifying agents, detergents, solubilizing agents, wetting agents, foaming and antifoaming agents and flocculants and deflocculants, and in drug stability and drug absorption.", "\n\nAll surfactants are characterized by having two regions in their molecular structure: a lyophobic (or hydrophobic) group, such as a hydrocarbon chain that has no affinity for aqueous solvent; and a lyophilic (or hydrophilic) group that has affinity for water. ", "To have such an affinity, the group must possess an appreciable polar character, for example, an ion or group with a large permanent dipole. ", "A molecule or ion that possesses this type of structure is termed amphipathic.[@b16-dddt-11-059]\n\nThe hydrocarbon chains of a water-dispersible surfactant are adsorbed to the hydrophobic particle surfaces, while the polar groups project into the aqueous medium becoming hydrated. ", "Thus, wetting of solid will occur due to a fall in both the interfacial tension between the solid and the liquid, and to a lesser extent, between the liquid and air.", "\n\nEven when dispersion has been achieved, suspensions may be disadvantaged by numerous factors such as excessive foaming, rapid separation or caking, taste issues caused by solubilization and change in bioavailability.", "\n\nFormulation development of riluzole 5 mg/mL oral suspension\n-----------------------------------------------------------\n\nTo successfully develop a suspension formulation, it is critical that the drug substance must comprise finely divided particles of appropriate particle size in order to produce a homogenous dispersion. ", "This is achieved by the use of micron-sized drug particles in the suspension product. ", "Riluzole is a highly hydrophobic material, particularly when presented as a finely divided powder. ", "Excipient selection then becomes key to ensuring physical and chemical stability of riluzole oral suspension. ", "The qualitative composition is provided in [Table 1](#t1-dddt-11-059){ref-type=\"table\"}.[@b12-dddt-11-059],[@b17-dddt-11-059]\n\nMaterials such as alcohol, glycerol and glycols that are water-miscible solvents will reduce the liquid/air interfacial tension. ", "The riluzole suspension formulation contains sorbitol (D-glucitol), a hexahydric alcohol. ", "Sorbitol will penetrate the loose agglomerates of drug particles displacing the air from the pores of the individual drug particles, thus enabling wetting to occur by the aqueous medium.", "\n\nTo further facilitate wetting of the drug substance and reduce the interfacial tension between the lipophilic solid and the hydrophilic vehicle, a combination of anionic and nonionic surfactants has been used.[@b14-dddt-11-059],[@b15-dddt-11-059]\n\nSodium lauryl sulfate (anionic) and macrogol cetostearyl ether (nonionic) surfactants are used as wetting agents in appropriate concentrations in riluzole oral suspension. ", "These surfactants are classified according to the nature of the ionic type of the hydrophilic group ([Table 2](#t2-dddt-11-059){ref-type=\"table\"}).", "\n\nAs suspensions are thermodynamically unstable, dispersed particles tend to aggregate and/or sediment with time; in order to minimize settling and prevent caking of dispersed particles of riluzole, controlled flocculation has been implemented utilizing functional excipients to produce a stabilized suspension which is readily re-dispersed upon shaking. ", "Flocculation is the process where suspended particles agglomerate to form loosely structured flocs, which are held together in a high-volume-network-like structure. ", "Flocculated particles are therefore weakly bonded, and as such, they do not form a cake and are readily resuspended. ", "The supernatant liquid is typically clear as colloidal particles are trapped within the flocs and sediment with them.", "\n\nThe hydrophilic colloid xanthan gum, a naturally occurring cellulose polymer, has been utilized in the patented formulation as a protective colloid, which coats the hydrophobic drug particles with a multimolecular layer. ", "This layer imparts a hydrophilic character to the solid drug particles and promotes both wetting and suspension.", "\n\nControlled flocculation is dependent on a combination of particle size control, the use of electrolytes to control zeta potential and the addition of polymers to enable cross-linking between particles. ", "Flocculation is typically achieved in part by neutralizing the charge of the suspended particles. ", "Riluzole oral suspension thus remains in a controlled flocculated state, by the innovative combination of anionic and nonionic surfactants and hydrophilic polymers. ", "Ionic surface-active agents cause flocculation by neutralization of the charge on each particle, whereas nonionic surfactants will have little effect on charge density, but because of their linear configurations, they may absorb onto more than one particle to promote a loose flocculated structure.", "\n\nThe suspending polymer xanthan gum also contributes to controlling the degree of flocculation. ", "The polymer molecules promote a gel-like network within the system and adsorb onto the surface of the dispersed particles, thus holding them in a flocculated state. ", "Although some settling is likely, the sedimentation volume remains large and can persist for a prolonged period.", "\n\nAn ideal pharmaceutical suspension will exhibit a high apparent viscosity at low rates of shear so that on storage, the particles either settle very slowly or remain permanently suspended. ", "At higher rates of shear, such as those caused by moderate shaking, the apparent viscosity should fall sufficiently for the product to be readily and accurately dispensed.", "\n\nA flocculated system, in part, meets these criteria, in that in such a system, pseudoplastic or plastic flow is exhibited as the structure progressively breaks down under shear. ", "The product then shows time-dependent reversibility of the loss of structure, or thixotrophy. ", "The inventors report that the chosen combination of excipients confers a high degree of flocculation.[@b14-dddt-11-059],[@b15-dddt-11-059]\n\nSuspending agents are used in riluzole oral suspension to impart viscosity to the vehicle and facilitate a stable dispersion by minimizing sedimentation and agglomeration of drug particles. ", "Importantly, the viscosity of the suspension must be optimized to minimize sedimentation of drug particles under the static conditions of storage, while maintaining acceptable flow properties upon the application of shear in the form of gentle agitation. ", "Such properties facilitate ease of resuspendability and dispensing of product by the patient and the absence of caking upon prolonged storage. ", "However, controlled flocculation must be maintained, and the combination of suspending agents to enhance and preserve physical stability and flocculation is crucial.[@b14-dddt-11-059],[@b15-dddt-11-059] Riluzole oral suspension also contains sweetener (saccharin), antifoam and a further suspending agent aluminum magnesium silicate, which may also contribute advantages by associating with dissolved drug, thereby reducing the anesthetic effect.", "\n\nRiluzole oral suspension is an innovative, physically and chemically stable aqueous liquid formulation, which has been developed utilizing expertise in pharmaceutical oral suspension technology and the design of optimized delivery systems for human administration, which is exemplified by a product shelf-life of 3 years, or 15 days after first opening.[@b17-dddt-11-059]\n\nClinical performance and bioequivalence of oral suspension\n----------------------------------------------------------\n\nRiluzole oral suspension has been reported to exhibit a surprisingly favorable palatability profile compared with riluzole solution, indicating a significant advance especially in providing an effective treatment for ALS with potential for a high degree of patient acceptability and compliance.[@b14-dddt-11-059],[@b15-dddt-11-059]\n\nAlthough comprising dissolved drug and solid drug particles suspended in an aqueous vehicle, there results minimal or no anesthetic effect in the mouth upon oral administration.[@b14-dddt-11-059],[@b15-dddt-11-059] The unique and patented liquid suspension formulation of riluzole appears to minimize the local anesthetic effect of drug particles in the mouth by providing a protective barrier between the drug particles and the oral mucosa and tongue. ", "Furthermore, this is in contrast to the presentation of crushed riluzole tablets that are noted to have a significant anesthetic effect on the tongue.[@b6-dddt-11-059],[@b10-dddt-11-059]\n\nAlthough the pathogenesis of ALS is not completely elucidated, it is suggested that glutamate (the primary excitatory neurotransmitter in the central nervous system) plays a role for cell death in the disease. ", "Riluzole is proposed to act by inhibiting glutamate processes, although the mode of action is unclear. ", "Riluzole oral suspension is indicated to extend life or the time to mechanical ventilation for patients with ALS. ", "Clinical trials have demonstrated that riluzole extends survival for patients with ALS, with survival being defined as patients who were alive, not intubated for mechanical ventilation and tracheotomy-free.[@b17-dddt-11-059]\n\nRiluzole tablets are rapidly absorbed after oral administration with maximal plasma concentrations occurring within 60--90 minutes (*C*~max~ =173±72 \\[standard deviation\\] ng/mL). ", "About 90% of the dose is absorbed, and the absolute bioavailability is 60%±18%. ", "The rate and extent of absorption is reduced when riluzole is administered with high-fat meals (decrease in *C*~max~ of 44%, decrease in AUC of 17%).[@b17-dddt-11-059]\n\nIn a bioequivalence study between novel riluzole oral suspension and Rilutek^®^ (tablets), the total exposure of riluzole 50 mg tablets and riluzole 5 mg/mL oral suspension was equivalent. (", "ratio: 106.84%; 90% confidence interval \\[CI\\]: 96.98%--117.71%). ", "Understandably, riluzole is more rapidly absorbed after the administration of oral suspension (*T*~max~ approximately 30 minutes), with a *C*~max~ approximately 20% higher than after the administration of riluzole tablets. (", "ratio: 122.32%; 90% CI: 103.28%--144.88%).[@b17-dddt-11-059]\n\nThe innovative technology employed in riluzole oral suspension clearly provides an opportunity for more accurate dosing and enhanced patient compliance compared with crushed tablets administered with food puree, yoghurt or via an enteral feeding tube. ", "The suspension is readily administered by means of a graduated oral dosing syringe, and the concentration is consistent with a volume that is easy to measure and not burdensome. ", "No product manipulation or premixing is required, and dilution with liquids is not necessary. ", "Crucially, it is reported that there is minimal or no anesthetic effect in the mouth upon administration of the oral suspension.[@b14-dddt-11-059],[@b15-dddt-11-059]\n\nConclusion\n==========\n\nRiluzole is a disease-modifying treatment that has been shown to slow the course of ALS. ", "Based on the recognized efficacy of riluzole, the demonstrated bioequivalence of riluzole oral suspension, reduced anesthesia in the mouth and the significant advantages of a liquid presentation exhibiting shear-thinning (pseudoplastic) behavior over tablets for ALS patients, riluzole 5 mg/mL oral suspension is the drug of choice. ", "Teglutik^®^ is a patented formulation that represents a significant therapeutic advancement for patients with ALS.", "\n\n**Disclosure**\n\nAnn Margaret Dyer is the sole director of PharmaSci Consulting Limited, and Alan Smith is an independent expert and a member of the Scientific Advisory Board of PharmaSci Consulting Limited. ", "The authors are solely responsible for the article content. ", "PharmaSci Consulting was invited by Martindale Pharma to conduct an independent evaluation of available riluzole formulations. ", "The authors report no other conflicts of interest in this work.", "\n\n###### \n\nQualitative composition of riluzole 5 mg/mL oral suspension\n\n Component Function\n ----------------------------- ------------------------------------\n Riluzole Drug substance\n Liquid sorbitol Vehicle, sweetening agent\n Aluminum magnesium silicate Suspending agent\n Xanthan gum Suspending agent\n Saccharin sodium Sweetening agent\n Simethicone emulsion 30% Antifoaming agent\n Sodium lauryl sulfate Anionic surfactant, wetting agent\n Macrogol cetostearyl ether Nonionic surfactant, wetting agent\n Purified water Aqueous vehicle\n\n###### \n\nClassification of sodium lauryl sulfate and macrogol cetostearyl ether\n\n Hydrophobic group Hydrophilic group\n ------------------------------------------------------------------------ ------------------- ----------------------\n **Anionic surfactant** \n Sodium lauryl sulfate C~12~H~25~− SO~4~^−^Na^+^\n **Nonionic surfactant** \n Macrogol cetostearyl ether[\\*](#tfn1-dddt-11-059){ref-type=\"table-fn\"} C~16~H~33~− (OCH~2~CH~2~)*~n~*OH\n\n**Note:**\n\nSeveral grades are available depending on the degree of ethoxylation, with repeat units (*n*) of polyethylene glycol varying between 2 and 20.", "\n" ]
{ "pile_set_name": "PubMed Central" }
[ 0, 0.011363636363636364, 0, 0.003616636528028933, 0.006644518272425249, 0.009966777408637873, 0.01098901098901099, 0.008583690987124463, 0, 0, 0.0017921146953405018, 0, 0.0026954177897574125, 0, 0, 0, 0.00796812749003984, 0.005714285714285714, 0, 0, 0.006578947368421052, 0, 0.005442176870748299, 0, 0, 0.0055248618784530384, 0.00558659217877095, 0, 0.0038022813688212928, 0, 0.002105263157894737, 0, 0, 0, 0.0029069767441860465, 0, 0, 0, 0.0035714285714285713, 0, 0, 0, 0, 0.010101010101010102, 0, 0.0078125, 0, 0.005376344086021506, 0.004739336492890996, 0, 0, 0, 0.008547008547008548, 0, 0.008968609865470852, 0, 0.004901960784313725, 0, 0.006060606060606061, 0, 0, 0, 0, 0, 0, 0, 0, 0.006060606060606061, 0, 0, 0.002242152466367713, 0.003125, 0.007537688442211055, 0.009708737864077669, 0.017543859649122806, 0.007389162561576354, 0, 0.005571030640668524, 0, 0, 0.006369426751592357, 0, 0, 0.010752688172043012, 0.003003003003003003, 0.008771929824561403, 0.019138755980861243, 0, 0.015748031496062992, 0, 0.0025823111684958036, 0 ]
0.003119
5
[ "BOSTON – While the Bruins have lost at least one member of their front office with assistant general manager Jim Benning making it official on Friday that he’s the new GM of the Vancouver Canucks, Bruins principal owner and NHL Chairman of the Board of Governors Jeremy Jacobs made it clear he loves the direction his team is headed.", "\n\nJacobs was asked specifically about the management team of president Cam Neely, general manager Peter Chiarelli and Jacobs’ son, Charlie, earlier this week at his end-of-the season press conference, and the praise was flowing inside TD Garden like watered-down Budweiser.", "\n\n“It’s a leadership team, and Cam [Neely] leads and is part of it. ", "It’s starting with Charlie [Jacobs], through Peter [Chiarelli], through Claude [Julien] and Cam [Neely],” said Jacobs. “", "I think they have met and exceeded our expectations, my expectations at least. ", "I think we’ve done a terrific job to create a winning institution. ", "I think we have delivered on what we promised.", "\n\n“We didn’t win this year, and got to the Finals the year before and all. ", "These are enviable positions to be in. ", "I love being here after a season like we just had [where there is] disappointment in the playoffs,\" he said. \"", "Our objective is the Cup. ", "It isn’t necessarily to have the best team during the regular season as it is to win the Stanley Cup. ", "We will continue that objective, and I think we will continue to grow from here.”", "\n\nThe Bruins have qualified for the playoffs seven consecutive seasons and are riding a consecutive sellout streak of 206 home at TD Garden with the last non-sellout back on Dec. 2, 2009. ", "Television ratings are way, way up for Bruins games and the profitability of the Original Six team has never been better.", "\n\nWhile the on-ice product royally disappointed their fan base in the postseason for the first time in four years – the first-round loss to the Washington Capitals was glossed over in the honeymoon period of the Cup – the health and well-being of the Bruins business is also a nod to what Charlie Jacobs, Neely, Chiarelli and Julien have brought to the table the past seven years.", "\n\nClearly, in words and actions, that’s made the elder Jacobs a very happy man when the subject turns to his hockey team." ]
{ "pile_set_name": "Pile-CC" }
[ 0.015015015015015015, 0.014652014652014652, 0.014705882352941176, 0.025, 0.012658227848101266, 0, 0, 0.013333333333333334, 0, 0, 0, 0, 0, 0, 0, 0.010526315789473684, 0.008264462809917356 ]
0.006715
5
[ "It appeared protesters and gas mask-wearing police officers would clash again Saturday night on Oakland streets, four days after a violent police crackdown shoved the Occupy Oakland movement into the international spotlight, but, just as emotions peaked, organizers held up peace signs.", "\n\nMarchers turned around, patted themselves on the back and returned to their adopted home, Frank H. Ogawa Plaza.", "\n\nSaturday’s near-confrontation happened on another roller-coaster day for Occupy Oakland: Someone created and circulated a fake letter and website in the name of Jean Quan, the embattled mayor; protesters planned for a citywide work stoppage Wednesday; and an injured war veteran and protester changed hospitals.", "\n\nThe night appeared set to spiral out of control shortly after 8 p.m. as an anti-police violence rally wrapped up and protesters picked up signs and headed toward Oakland Police Department headquarters.", "\n\nOfficers had not been seen all day, but lines of riot police wearing gas masks began flanking the marchers until Seventh and Clay streets, less than two blocks from police headquarters. ", "Marchers stopped and began taunting the police, but just as emotions reached a fevered pitch, organizers announced: “Peace people to the front!” ", "Numerous march organizers walked in between the police and protesters, faced the marchers and held up peace signs with both hands.", "\n\nAfter a few minutes, the march retraced its steps and snaked its way back in the direction of the tent city.", "\n\nSporadically, on the return to the plaza, protesters tagged anti-cop graffiti with spray paint and smashed parking meters despite organizers’ pleas, and as they returned to City Hall someone busted a window of the Oakland police recruitment center. ", "Police appeared to escort the group from afar and did not intervene.", "\n\nThe rest of Saturday was not nearly as tense, but Quan’s difficult week continued as someone created a fake website and circulated a bogus letter on city letterhead with her forged signature, saying the mayor apologized for “ordering” last week’s “violent repression” of the first Occupy Oakland camp.", "\n\nUsing public health and safety to justify the raid “reached the height of absurdity,” and Quan had “a change of heart,” the hoax letter said. ", "The document — posted on a fake-but-professional-looking website (http://oaklandmayor.com) — also says she heartily endorses the general strike.", "\n\nQuan’s spokeswoman, Sue Piper, quickly sent out a statement disavowing the letter.", "\n\n“It is completely fabricated,” the statement read. “", "Any official announcement from the Mayor or city of Oakland is posted on the city’s website at www.oaklandnet.com and sent directly to the media.”", "\n\nNo one has taken responsibility for creating the hoax site, but protesters and others around the country have criticized and questioned the mayor since Tuesday’s police crackdown on the camp. ", "In response, Oakland City Council members scheduled a special meeting for 5:30 p.m. Thursday asking the public to weigh in on the Occupy Oakland movement and demonstrations to develop a strategy to keep them peaceful.", "\n\nIt was also announced that former Marine and Iraq War veteran Scott Olsen, 24, who was seriously injured by what witnesses said was a tear-gas canister fired by police Tuesday, has been moved from Highland Hospital in Oakland to an undisclosed medical facility, where he is still in fair condition, a hospital spokesman said.", "\n\nOn Saturday, filmmaker Michael Moore, fresh off his Friday visit to Oakland, drove across the Bay Bridge and visited the San Francisco encampment in Justin Herman Plaza, where he rallied protesters.", "\n\nDuring the day in Oakland, protesters prepared for Wednesday’s proposed general strike, canvassing business districts from Chinatown to the Fruitvale district to involve merchants and residents, urging them to close their doors, stay home from work and gather in downtown Oakland in a massive show of solidarity. ", "The general assembly voted to picket any business that punishes an employee for walking out.", "\n\nCity Administrator Deanna Santana said she released a memo to workers saying that if they would like to honor the general strike they would have to go through the formal process and get supervisor approval but could use a sick day or furlough day.", "\n\nStaff writer Cecily Burt and the Associated Press contributed to this report." ]
{ "pile_set_name": "OpenWebText2" }
[ 0, 0.008849557522123894, 0.003194888178913738, 0.0049261083743842365, 0, 0, 0, 0, 0, 0, 0.0033003300330033004, 0.006944444444444444, 0.006944444444444444, 0.011904761904761904, 0, 0.00684931506849315, 0, 0.004608294930875576, 0.0030581039755351682, 0.01, 0, 0, 0.004016064257028112, 0.02531645569620253 ]
0.004163
5
[ "Transport in Saint Vincent and the Grenadines\n\nThere are no railways in Saint Vincent and the Grenadines.", "\n\nAs of 1996, there were 829 km of highways, of which 580 km are paved.", "\n\nPorts and harbours:\nKingstown\n\nMerchant marine:\ntotal:\n825 ships (1,000 GT or over) totaling 7,253,092 GT/\nships by type:\nbarge carrier 1, bulk 142, cargo 400, chemical tanker 31, combination bulk 10, combination ore/oil 5, container 47, liquified gas 5, livestock carrier 5, multi-functional large load carrier 3, passenger 3, petroleum tanker 60, refrigerated cargo 41, roll-on/roll-off 51, short-sea passenger 12, specialized tanker 8, vehicle carrier 1 (1999 est.)", "\nnote:\na flag of convenience registry; includes ships from 20 countries among which are Croatia 17, Slovenia 7, People's Republic of China 5, Greece 5, United Arab Emirates 3, Norway 2, Japan 2, and Ukraine 2 (1998 est.)", "\n\nAirports:\n6 (2005)\n\nAirports - with paved runways:\ntotal:\n5\n914 to 1,523 m:\n4\nunder 914 m:\n1 (2005)\n\nThere is one airport with an unpaved runway, under 914 m (2005 est.)", "\n\nSee also\n Saint Vincent and the Grenadines\n List of airports in Saint Vincent and the Grenadines\n List of airlines of Saint Vincent and the Grenadines\n\nCategory:Transport in Saint Vincent and the Grenadines" ]
{ "pile_set_name": "Wikipedia (en)" }
[ 0.01904761904761905, 0, 0, 0.004545454545454545, 0, 0.009615384615384616 ]
0.005535
5
[ "Mid Cornwall GP surgeries\n\nPlease click on the interactive map above for details of each surgery, or see the list below.", "\n\nGet involved in commissioning and with a Patient Participation Group (PPG) by talking to the Practice Manager at your GP surgery.", "\n\nYou can also contact the Practice Manager to find out who the mandated GP is (it’s not necessarily one of the senior partners). ", "A mandated GP is the member of the practice mandated to work on commissioning new services with NHS Kernow. ", "As you can see on this page, each surgery has their own website, where you can can find contact details for the Practice Manager." ]
{ "pile_set_name": "Pile-CC" }
[ 0, 0.022900763358778626, 0.015384615384615385, 0.018518518518518517, 0.007751937984496124 ]
0.012911
5
[ "Proper way to reset computer?", "\n\nI just bought a 13\" a couple days ago. ", "I'm coming from an older 15\" MBP. ", "After using it for these couple of days I'm starting to think I miss the 15\" screen, so I'm thinking about exchanging the 13\" for the 15\".", "\n\nI did migration assistant with this new 13\" so what is the best way to wipe the computer clean if I decide to exchange it?", "\n\nI just bought a 13\" a couple days ago. ", "I'm coming from an older 15\" MBP. ", "After using it for these couple of days I'm starting to think I miss the 15\" screen, so I'm thinking about exchanging the 13\" for the 15\".", "\n\nI did migration assistant with this new 13\" so what is the best way to wipe the computer clean if I decide to exchange it?", "\n\nClick to expand...\n\nMake a bootable Mavericks installer, boot from it and reformat the drive.", "\n\nHold the Option key while powering on your Mac, this should bring up the Startup Manager.", "\n\nSelect the Recovery disk (10.9 Recovery?) ", "and press return.", "\n\nSelect Disk Utility\n\nSelect Macintosh HD (or your volume if you renamed it) from the list on the left\n\nClick the Erase tab in the right portion of the window\n\nClick the Security Options button towards the bottom of the window\n\nUse the slider to determine how secure of an erase you want. ", "The \"Fastest\" option is obviously the fastest however it does not securely erase data. ", "The next fastest option uses one pass of writing zeros to remove all of your data. ", "The next fastest option uses two passes of writing random data followed by a pass of writing known data to destroy your actual data. ", "The slowest, but most secure option is the same as before but makes seven passes. ", "Depending on the size of your disk the time difference between these could be huge. ", "The most secure option could take anywhere from 3 - 18 hours in my experience. ", "Either of the previous two will take a good bit less time while still providing reasonable enough security if the Mac is returned to a reputable retailer.", "\n\nOnce you've chosen your security option click OK.", "\n\nClick Erase.", "\n\nWhen the process is done the Macintosh HD volume will be blank. ", "You can either install Mountain Lion / Mavericks back over it using the Install Mac OS X option on your recovery disk or you can leave it blank.", "\n\nStaff Member\n\nI just bought a 13\" a couple days ago. ", "I'm coming from an older 15\" MBP. ", "After using it for these couple of days I'm starting to think I miss the 15\" screen, so I'm thinking about exchanging the 13\" for the 15\".", "\n\nI did migration assistant with this new 13\" so what is the best way to wipe the computer clean if I decide to exchange it?", "\n\nMacRumors attracts a broad audience\nof both consumers and professionals interested in\nthe latest technologies and products. ", "We also boast an active community focused on\npurchasing decisions and technical aspects of the iPhone, iPod, iPad, and Mac platforms." ]
{ "pile_set_name": "Pile-CC" }
[ 0, 0, 0.029411764705882353, 0, 0, 0, 0.029411764705882353, 0, 0, 0.010526315789473684, 0, 0, 0, 0, 0, 0, 0, 0, 0, 0, 0, 0, 0, 0.015151515151515152, 0.013888888888888888, 0, 0.029411764705882353, 0, 0, 0.007936507936507936, 0.022556390977443608 ]
0.005106
5
[ "Behavior of interchromatin granules during the cell cycle.", "\nWe investigated at the ultrastructural level, by different cytochemical and immunocytological approaches, the behavior of interchromatin granules (IGs) during interphase and mitosis in two cell lines (HEp-2 and Ehrlich tumor cells). ", "Identical results were found in all two cell types. ", "In interphase cells, IGs group into irregular clusters of varying size. ", "They are frequently associated with coiled bodies and homogeneous fibrillar bodies. ", "Analysis of serial sections reveals that IG clusters occupy distinct regions within the nucleoplasm. ", "During prophase, the aggregation of granules in these clusters gives rise to compact, spherical, granular structures. ", "These disperse in the mitotic cytoplasm at the breakdown of the nuclear envelope. ", "At early telophase, some of them come into close contact with the periphery of reforming nuclei. ", "IG clusters reappear in the daughter nuclei only after the chromosomes have decondensed during late telophase. ", "Concomitantly, the cytoplasmic granular structures disappear. ", "During the cell cycle, IG are silver-stainable and EDTA-positive. ", "They are also constantly labeled by the polyadenylate nucleotidyl transferase-immunogold technique for detecting RNA. ", "These results support the view that IGs persist throughout the whole cell cycle." ]
{ "pile_set_name": "PubMed Abstracts" }
[ 0, 0.004273504273504274, 0, 0, 0, 0.009900990099009901, 0, 0, 0, 0.009009009009009009, 0, 0.015151515151515152, 0.00847457627118644, 0 ]
0.003344
5
[ "\n\nTeam Fortress 2 getting micro-transactions via Steam Wallet - trafficlight\nhttp://www.teamfortress.com/mannconomy/FAQ/\n\n======\ntvon\nTeam Fortress 2 has offered absolutely stunning value to the buyer over the\npast two years. ", "Paying $15+ for DLC for the Call of Duty, GTA and Battlefield\ngames is an absolute joke compared to the massive improvements that have come\nto Team Fortress 2 from the publisher _for free_.", "\n\nWhat I'd like to see, however, is some hard numbers that demonstrate this has\nbeen worth it to them from a financial perspective.", "\n\n~~~\narohner\nI can't find the link right now, but I'm pretty sure I've seen them say:\n\n1) TF2 is profitable, even including the close to ten years that it spent in\ndevelopment(!)", "\n\n2) They make money off the updates (because of new sales), but not always\nenough to break even for the update\n\n3) Valve views TF2 as their sandbox to see how players respond to certain\nideas, so they find it useful to keep TF2 around even if it weren't carrying\nits weight anymore.", "\n\n(edit this backs up some of my claims:\n[http://www.pcgamer.com/2010/08/19/interview-valve-on-the-\nfut...](http://www.pcgamer.com/2010/08/19/interview-valve-on-the-future-of-\nteam-fortress-2/2/))\n\n------\ncjeane\n_However, each time you fund your wallet, the minimum amount of funds that you\ncan put in your Steam Wallet is $5 / £4 / 5€ to keep transactions and payment\nservice provider fees to a minimum._", "\n\nIs $5 really a micro-transaction?", "\n\n~~~\njws\nI think the individual transactions can be quite small. ", "You just have to\ncommit to making $5 of them because of the non-micro-transaction handlers.", "\n\nApparently you can buy gear. ", "I haven't played TF2, but from my \"too many\" TF\ndays, there were many times I'd have gladly payed $0.10 to see a sniper lagged\nfor just 200ms.", "\n\n------\nnoonespecial\nWe've been talking about this for years, since the bad old days of TF _before\nthe turn of the century_ (1).", "\n\nWouldn't it be awesome for online games like TF to have a small \"buy in\" and\nthen a prize purse for the winners? ", "That might be enough to bring this old\nfogey back to Team Fortress again.", "\n\n(1) Yes, kids, they had Team Fortress, on the quake engine, playable online in\nthe 1900's. ", "It was blocky, it was slow, it was guaranteed to be lagged to\nhell, _and we liked it that way_. ", "Now get off my lawn.", "\n\n------\nfrou_dh\nWhile Valve's continued development is impressive, as someone who hasn't\nplayed since what must be 2008, the stream of new systems and facets that have\nbeen laid over the game make it intimidating to return (harshly: game bloat).", "\n\n~~~\nmquander\nHave you actually tried it and been discouraged? ", "Honestly, 100% of the core\ngameplay is identical to what it was in 2008. ", "The bonuses are just new maps,\nslightly different alternate weapons, a new game mode or two (payload, king of\nthe hill), and hats. ", "It's not as if they added a dozen new classes or\nsomething; they release exactly enough content to keep it fresh for people who\nare playing all the time.", "\n\nI'm surprised that anyone would find it intimidating -- frankly, if you took\n2008 me, put me in a hole for two years, and brought me out to play TF2 CTF or\nCP, I would probably barely even notice that there were new things.", "\n\n~~~\nfrou_dh\nGood to know. ", "I haven't played since, I just always seemed to see changes\nmentioned on news sites.", "\n\nSomeone also told me there were 360-like achievements (with perks) and there\nended up being servers with custom maps to rack them up with minimal effort.", "\n\n~~~\nmquander\nYeah, there's a few hundred achievements (!) ", "which exist for a combination of\nfun and to unlock the alternate weapons. (", "If you don't get the achievements,\nyou'll gradually get the weapons anyway via random \"drops\" when you die and/or\ncrafting, trading, or (now) RMT.)", "\n\n" ]
{ "pile_set_name": "HackerNews" }
[ 0.004424778761061947, 0.015873015873015872, 0, 0, 0, 0.0049382716049382715, 0, 0, 0, 0, 0, 0, 0, 0.0136986301369863, 0.010752688172043012, 0, 0, 0.0040650406504065045, 0, 0, 0, 0, 0, 0, 0, 0, 0, 0, 0, 0 ]
0.001792
5
[ "The present invention relates to data communications between computer based stations in a network and more particularly to data communications between computer-based stations in a network wherein data is selectively compressed and decompressed to improve network utilization and efficiency.", "\n1. ", "Prior Art\nIn some network systems, network data bandwidth requirements may increase during busy periods of the day. ", "In many respects, the requirement for data bandwidth varies over the period of a day in a manner similar to the requirement for electric power which varies from peak hours during the daytime to relatively low demand during the early morning hours.", "\nIf all the data transmitted across a network are compressed by a predetermined ratio such as two to one, the effective bandwidth of the network is approximately doubled. ", "Using data compression is a much more cost effective and efficient solution to the bandwidth requirement problem than upgrading the network to a faster data rate which would require replacement of very expensive hardware in the system.", "\nAlso, employing a compression technique which is hardware based requires new network adapter card designs. ", "This also results in a higher cost.", "\nA software-based compression could use existing adapter cards, but software-based compression requires a large number of system processor cycles which is wasteful when compression is not required, for example, when network utilization is low.", "\nAn article in the IBM Technical Disclosure Bulletin, Volume 35, Number 6, published November 1992, at page 394 entitled \"Time-based Compression Selection Line Detector Methodology\" describes a technique for periodically polling network activity and comparing data transfer time to the time required to compress the same block of data. ", "The technique described in the article is intrusive in that it sends a frame into the network to discover buffer transfer times. ", "This limits the responsiveness of the system and contributes to network and target station load. ", "Contributing to network load during times of high network activity is not a good way to improve network responsiveness. ", "Further, the technique described in the article does not measure network utilization. ", "There are various other reasons than high network utilization that may cause a timed poll to take a \"long time.\" ", "For example, if the target station is busy, the timed poll may indicate a busy network, and the implementation of data compression may actually degrade system performance. ", "Also, the technique measures samples which may yield an inaccurate representation of the average utilization over the sampling period since the sampling period is so long, the recommendation being ten minutes.", "\nThe article does not teach nor suggest a dynamic system for efficient data transfer in which data is automatically compressed when network activity exceeds a predetermined threshold." ]
{ "pile_set_name": "USPTO Backgrounds" }
[ 0, 0, 0, 0, 0, 0, 0, 0, 0, 0.005952380952380952, 0, 0, 0, 0, 0, 0, 0, 0 ]
0.000331
5
[ "Q:\n\nHighlight text from jdt java text editor using line numbers in eclipse plugin\n\nI am trying to write an eclipse plugin which highlights some text in java editor after user save the text (ResourceChangeListener). ", "I am implementing ILightweightLabelDecorator and extending BaseLabelProvider, The method \npublic void decorate(Object arg0, IDecoration arg1)\n\ngetting called but I am getting Objects of type org.eclipse.jdt.internal.core.*", "\ne.g org.eclipse.jdt.internal.core.", "PackageDeclaration.", "\nI need line numbers from that object so I can highlight that text.", "\n ASTNode object has a property to get the position (line numbers) but I am not getting that one. ", "How can I get ASTNode from org.eclipse.jdt.internal.core.* ", "\n objects?", "\nThanks in advance.", "\n\nA:\n\nPackageDeclaration is part of the JDT Java Model which is a lighter weight version of the AST used by a lot of the Java code. ", "As such it isn't related to ASTNode.", "\nMany Java Model objects (including PackageDeclaration) implement ISourceReference which tells you about the source code. ", "This includes getSource and getSourceRange methods.", "\n\n" ]
{ "pile_set_name": "StackExchange" }
[ 0.004651162790697674, 0, 0.02857142857142857, 0, 0, 0, 0, 0, 0, 0.007575757575757576, 0, 0.00819672131147541, 0.0196078431372549, 0 ]
0.0049
5
[ "and for the truth to which all of these voices point:\nThat you are constantly aware of your creation – of its needs and\nwants; of its faith and lack of faith.", "\n\nYou constantly care about us,\n\nabout our lives as they are\n\nabout what we, in your love, have the potential to be.", "\n\nAll the voices you send to us,\n\nremind us of your eternal love for us -\n\nof your constant faithfulness to your promise to be our\nGod\n\nand your invitation to us to be your people.", "\n\nWe confess that we have turned away from you -\n\nand day by day, we continue to turn away from you -\n\nWe turn away from you\n\nin the things we think\n\nin the things we say,\n\nin the things we do.", "\n\nWe turn away from you\n\nin the things we ought to think, but don't\n\nin the things we ought to say but don't\n\nin the things we ought to do but don't.", "\n\nWe act as if, when Jesus said “follow me”\n\nhe meant someone else, not us.", "\n\nTurn us around to face you again -\n\ngive us the will and the ability\n\nto hear Jesus' call again,\n\nand to follow.", "\n\nIn his name we pray, Amen.", "\n\nKids'\ntime\n\nRemember what our candy cane reminded us about last\nweek? (", "Shepherds)\n\nThere's another candy cane this week. ", "I'm thinking if\nyou hold it up like this it looks like a letter. ", "Does some big prep\nkid want to tell me what letter this looks like? (", "J)\n\nWhat name starts with “J” that we're thinking about\nat Christmas? (", "Jesus) You're right. ", "Christmas is Jesus' birthday!", "\n\nGuess what? ", "This candy cane's got something written on\nit! ", "Isn't that a surprise? ", "Who can read it out?", "\n\nAn angel of the Lord appeared to him in a dream and\nsaid, “Joseph, son of David, do not be afraid to take Mary as your\nwife, for the child conceived in her is from the Holy Spirit. ", "She\nwill bear a son, and you are to name him Jesus, for he will save his\npeople from their sins.” ", "Matthew 1:20-21\n\n(Does Bec have a present under the tree today? ", "How does\nthat help remind us about Jesus at Christmas?)", "\n\nAdvent\nCandle lighting liturgy (Young Adults)\n\n1st\nreader: We have grown up in a world that's known continual wars.", "\n\n2nd\nreader: From time to time, the place has changed, and the reason has\nchanged, but fighting continues.", "\n\n1st\nreader: Even within communities and homes, people don't seem to be\nable to live in peace.", "\n\n2nd\nreader: Yet Jesus comes as Prince of Peace – as the one who can\nhelp us do what we just can't on our own.", "\n\nAll young adults: We look for Jesus to bring peace this\nChristmas.", "\n\n(Relight first purple candle from last week, and pink\ncandle for repentance this week.)", "\n\nLast Sunday we heard words of hope – hope based in\nGod's action of coming to us in Jesus.", "\n\nToday as we progress through to the second week of Lent,\nwe hear words of comfort, or peace, – but comfort which comes with\na call for us to respond.", "\n\nIt is comfort mixed with something else. ", "Let's start\nwith Isaiah, very briefly. ", "In Isaiah, God declares “comfort” for\nthe nation of Israel because they've served their sentence –\nthey've been punished enough. ", "Israel may have been suffering, but not\nsuffering innocently. ", "The word for “comfort” can mean “be\ncomforted or consoled” or it can mean “repent or be sorry”.", "\nBoth meanings are very closely linked. ", "We cannot be comfortable in\nour relationship with God, if we are not living in a right\nrelationship with God. ", "Repentance, which literally is “turning\nback” to God, is the way for us to approach God to come into that\nrelationship.", "\n\nNow let's look ahead to the gospel reading:\n\nIt's interesting to note here, that Mark introduces his\ngospel, not with a story of Jesus, but with the story of the\nappearance of John.", "\n\nWhen we look at John's message, we need to see something\nof his character. ", "John lived the message he brought – which is a\ngood way to know if you can trust any leader.", "\n\nJohn's birth was a miracle – his parents were already\nin old age and unable to have children. ", "His birth had also been\nannounced by the angel Gabrie, who gave instructions as to how he was\nto be raised – from birth he was dedicated to God in terms of the\nNazirite vow – which included all sorts of rules of holiness.", "\n(Think of Samson – also bound by a Nazirite vow for life so not\nallowed to cut his hair.) ", "Nazirite vows were usually taken on for a\nperiod of time, such as we might take on an extra spiritual\ndiscipline during lent. ", "For John, it was his life, right from birth.", "\nThat's why we hear about his clothes and diet – he's not taking on\nany luxuries of life, because he is totally dedicated to God.", "\n\nJohn appeared out of the wilderness as a prophet –\njust as many prophets before him spent time in the wilderness. ", "This\nis important, because in Jesus' day, people were expecting the\nprophet Elijah to return, and come ahead of the Messiah. ", "John's\nappearance from the wilderness helped to put him in line with that\nheritage of prophets.", "\n\nMany came out, expecting John to be the Messiah. ", "At this\ntime of history, with the Roman army occupying Israel, there were\nlots of “messiahs” wanting to bring a military or political\nsalvation for the nation.", "\n\nJohn was quick to dispel that belief. ", "He made it very\nclear that his role was one of preparation – to get people ready\nfor the coming of the Messiah they had been expecting for\ngenerations.", "\n\nHe called out to people to prepare through repentance.", "\nThat's what John's baptism was about. ", "It was a cleansing ceremony.", "\nThat was something well known in his time – wealthy Jewish houses\nhad baptisteries and cisterns for just such a purification ritual,\nwhich was done over and over again. ", "It was not the same as Christian\nbaptism which is a once-and-for-all-time incorporation into the death\nand resurrection of Jesus.", "\n\nJohn's baptism was a declaration of an intent to turn\nback to God, to wash away the things that were wrong in life and\nstart again fresh.", "\n\nLet's have a quick look at the 2 Peter reading: in which\npeople are also called to repentance, as a preparation for God to\nact. ", "The reason Christ hasn't returned yet is because God has given\nus time to prepare, time to repent.", "\n\nSo in each of these three time periods: Old Testament,\nin Jesus' day, and in the post-resurrection life of the very early\nchurch, our hope that God will act is accompanied by a call to\nprepare through repentance.", "\n\nThe Biblical concept of Sin is about turning away from\nGod. ", "Conversely, the Biblical concept of repentance is turning around\n– to turn back to God.", "\n\nWe have a hope that God will act. ", "We can rely on that,\nas our readings last week assured us. ", "But in the meantime, we are\ncalled on to act in response to that hope and the assurance that\ncomes with it. ", "We must act by turning our lives to face toward God –\nto strive to be at peace with God. ", "Again we see a link between peace\nor comfort and repentance.", "\n\nThe reason that the world has not ended before now –\nthat Jesus has not yet returned, is not that God has forgotten and we\nshould give up hope. ", "Instead, it is because God is patient with us,\nand wants to give us all the opportunity to repent, to turn back to\nalign our lives with God's will.", "\n\nWe are all given the opportunity to remember our origins\n– that we are creatures, made by God's hand – to turn back and\nlive the lives we were created to live: at peace with our maker and\nwith each other.", "\n\nHymn\nTogether in Song 270 On Jordan's Bank the Baptist's cry\n\nNotices\n\nOffering\n\nPrayers\nof the People\n\nMerciful God,\n\nTwo millennia after John came out of the wilderness -\n\nwe hear his call to us today-\n\nRepent, and prepare the way of the Lord.", "\n\nYou know the things we need to repent of:\n\nthings in the life of our world\n\nthings in the life of our nation\n\nthings in the life of our city and community\n\nthings in the lives of our families\n\nthings in our own lives.", "\n\nHelp us to face the things which are wrong, we pray\n\nto accept that sin has been a part of our lives\n\nand to let go of it.", "\n\nHelp us to turn around\n\nto live lives in accord with your will\n\nso that we may truly\n\nprepare the way of the Lord.", "\n\nGod of peace – we pray for peace\n\nwe pray not just for the absence of war -\n\nbut for the peace of your Kingdom at work in the world.", "\n\nWe pray for wisdom among world leaders that leads to\nrespect and understanding.", "\n\nWe pray for gentleness and respect and compassion among\nindividuals.", "\n\nIn defiance of the way things are – we trust in your\npower and pray for what could be.", "\n\nWe pray for a world which takes seriously the Christmas\npromise of peace on earth and goodwill to all -\n\nAnd we pray that you use us as you will, to help to\nbring this about wherever we are.", "\n\nMonday, 28 November 2011\n\n2nd\nreader: From time to time, the place has changed, and the reason has\nchanged, but fighting continues.", "\n\n1st\nreader: Even within communities and homes, people don't seem to be\nable to live in peace.", "\n\n2nd\nreader: Yet Jesus comes as Prince of Peace – as the one who can\nhelp us do what we just can't on our own.", "\n\nAll\nyoung adults: We look for Jesus to bring peace this Christmas.", "\n\nGood morning\n\nAs we progress through\nour Advent readings, today Isaiah speaks to us of comfort and John\ncalls us to repent.", "\n\nWe live in a world of\nturmoil, and often it seems our individual lives are in constant\nturmoil as well. ", "What would it be like to be comforted, to actually\nbe at peace? ", "What would it take to achieve this – John's answer\nwould be to repent – to turn back to God.", "\n\nSunday School children\nknow the “right answer” to just about every question is “Jesus”.", "\nThe same is true in the adult world – although nothing seems quite\nso simple from our point of view. ", "What we want when we seek comfort\nin the turmoil of our lives is to be taken out of the turmoil – we\nwant to be rescued. ", "What God offers is usually something different.", "\n\nWhen we repent –\nwhen we turn to face God – what we are doing is focussing on a\ndifferent point. ", "The turmoil is background, it's placed in\nperspective. ", "We're not rescued from the trauma – we're shown that\nit is not as important as we thought it was. ", "If we focus on the\ngoodness of God, how important can any evil or pain in this world\nactually be? ", "There is no comparison.", "\n\nThat's not to say we\nshould not act, when we are able, to change things that are wrong\nwith this world. ", "In fact the prophets call down to us throughout the\nages, that we must act for justice wherever God gives us the ability\nto do so – we must care for the widow and orphan, the homeless, the\npoor. ", "Turning to face God will always inspire us to do better, to\nseek good not just for ourselves, but for all people and especially\nthose least able to help themselves.", "\n\nToday, as we journey\nthrough Advent, let's take the time to repent, to turn to focus our\nlives once again on God, and to get everything into perspective.", "\n\nThursday, 24 November 2011\n\nAs Thom said last week – last Sunday was the final\nSunday of the church year. ", "Today, we begin the year again – with\nthe first Sunday of Advent – our time of looking forward to the\ncoming of Jesus – as the baby of Bethlehem, and as ruler and judge\nof the world.", "\n\nWhat do these tinselly things on the pulpit today look\nlike? ", "Sort of like walking sticks? (", "Candy canes!)", "\n\nYum. ", "Who likes candy canes?", "\n\nLet's have a look at it.... You guys have been talking\nin Sunday School about when Jesus was born, right?", "\n\nWell, this candy can sort of reminds me of someone who\nwas there. ", "There were some people who would have used sticks with\nbendy ends like this in their work. ", "They would have used them to help\npull sheep out of trouble, and keep them together. ", "Who were they?", "\n(Shepherds)\n\nHow did they fit into the story of Jesus being born, can\nanyone tell me? (", "Let kids tell the story.)", "\n\nDid I see one of you put a present under the Christmas\ntree just now? ", "Can I open it? (", "It's a sheep!) ", "Wow. ", "Let's keep the\nsheep up here under the Christmas tree right up until Christmas to\nremind us about the story.", "\n\nNow – I have some candy canes here with something\nwritten on them. ", "Who can read it? (", "When the angels had left them and\ngone into heaven, the shepherds said to one another, “Let us go now\nto Bethlehem and see this thing which the Lord has made known to us.”", "\nSo they went with haste and found Mary and Joseph, and the Child\nlying in the manger. ", "Luke 2:16-17)\n\nWho'd like one of these candy canes? ", "Do you think we\nshould share them with the adults? ", "I've got a job for you to do\nright now – you guys are doing the advent candle this week – and\nthen we're singing a hymn. ", "While we're singing, you can give the\ncandy canes out.", "\n\nAdvent\nCandle Week 1 – Hope – Kids\n\n1st\nreader: Christmas is coming there's lots to hope for.", "\n\n2nd\nreader: Presents, and lollies, and cakes, and a visit from Santa.", "\n\n1st\nreader: Hope is young, it looks forward to what is coming...\n\n2nd\nreader: Hope waits with excitement.", "\n\nAll\nkids: We hope for Jesus this Christmas.", "\n\nLight first candle.", "\n\nHymn:\nTogether in Song 276 Light One Candle – verse 1.", "\n\nScripture:\n\nIsaiah 64:1-9\n\nMark 13:24-37\n\nThis is the Word of the Lord\n\nThanks be to God.", "\n\nSermon:\n\nToday is the beginning of Advent. ", "It's time we all\nstarted thinking about getting ready for Christmas. ", "For a couple of\nweeks, maybe longer, the shops have had their Christmas decorations\nup and urged you to buy lots of things to prepare for Christmas.", "\n\nToday my family will do things we have a long-standing\ntradition of doing on the first day of Advent – we'll put up our\nChristmas tree and nativity scene and decorate the house. ", "Presents\nwill start appearing under the tree over the next week or so.", "\n\nIn one sense that's all very appropriate – we are\npreparing for Christmas. ", "In another we're a bit premature. ", "We put\nout all of our nativity scene. ", "Some families, and some churches, are\na little more patient, and a little more in line with the events of\nAdvent and Christmas. ", "They begin with Mary and Joseph, adding baby\nJesus and the shepherds on Christmas eve night, and then bringing in\nthe wise men at Epiphany (the twelfth day of Christmas – the 6th\nof January.)", "\n\nPutting the pieces in place as we remember them in the\nchurch calendar is perhaps more symbolic of what the season of Advent\nis about. ", "It is a time of waiting, and it's a time of waiting for\nmore than our celebrations of Christmas.", "\n\nAdvent is about waiting for Christ to come as the\nmessiah – the baby of Bethlehem, which has already happened; but\nmore than that, it is about waiting for him to return as judge and\nruler of the world. ", "So our readings for the four Sundays of Advent\nwill take us through the ancient prophecies about the coming\nMessiah, the New Testament promise of the birth of Jesus; and also\nstrange prophecies about the end of the world. ", "Advent ties all of\nthis together.", "\n\nThe Church is really an Advent people. ", "We live in the\ntime of already-but-not-yet. ", "Christ has come, yet we wait for his\ncoming, just as the people of Isaiah's time waited.", "\n\nIn the meantime, we live with the problems of the world\naround us. ", "In Isaiah's time, the people of Israel were exiled,\nwaiting for a leader who would take them home. ", "They cried out to God\nfor help, and the prophet brought them God's words of hope, using\nGod's faithfulness in the past as evidence that could be relied on\nfor trusting God in the future.", "\n\nThe earliest information we have about the Gospel writer\nMark is that he was a disciple of Peter, and wrote down Peter's\nstories of Jesus. ", "When Mark was writing, Christians had seen the fall\nof Jerusalem, the temple torn down. ", "They had escaped the Holy City in\ntime, but had become scattered. ", "They had been persecuted firstly by\nJews, and were beginning to be persecuted by Romans. ", "Like Israel in\nexile, the church had become a people isolated from their spiritual\nhome and not free to worship openly. ", "Everything seemed to have been\ntaken away from them.", "\n\nAnd Mark found a message for hope for the Church in\nPeter's recollection that Jesus had said lots of bad stuff was going\nto happen, but that there was hope in the midst of the bad stuff. ", "At\nsome time, a time that no-one could predict (even though people will\npersist in trying), the son of Man would return, coming in the\nclouds. ", "It was a reflection of what the prophet Daniel had said in\nOld Testament times.", "\n\nIn some of the passages we have read in the past few\nweeks, “keep awake” or “be prepared” has been a warning. ", "Now\nit's a sign of hope. ", "No matter how tough things get, don't lose hope;\nhold on, keep waiting and don't give up. ", "Christ will come. ", "No matter\nwhat happens, his promise can be relied upon – even if heaven and\nearth cease to exist – Christ's word will still hold true.", "\n\nHer we are, in the 21st century since Jesus,\nand he still hasn't returned. ", "We have been through the period of\nChristendom, when the church held sway over temporal as well as\nspiritual powers; when the church, in effect ruled much of the world.", "\nWe've been through the Reformation, when we discovered that the\nchurch could become just as corrupt as any political power and does\nat times need to be challenged, and reformed. ", "We've been through a\ntime when Western nations considered themselves to be Christian\nnations, and the only division was between Christian denominations.", "\n\nNow, we've reached a time when people are free to choose\nwhat faith they will have, if any. ", "There's no longer any social\nexpectation that people will be Christian. ", "We've also reached a time\nwhere the popular perception of Christianity has been watered down\nenough that people can call themselves “Christian” without really\nknowing what that means, or practising the Christian faith in any\nway.", "\n\nI was amazed a few years back, when census results\nshowed that the small town of Kin Kin in the Sunshine Coast\nHinterland had 100 Uniting Church members – I was amazed because\nKin Kin was one of the congregations I was minister for at the time,\nand we had four people and one horse attending worship. (", "The horse\nwas named Crystal, and she used to stick her head in the door and pay\nrapt attention to the service.) ", "Our membership roll included four\nnames (the horse hadn't been baptised or confirmed.) ", "Attending\nworship is pretty much the most basic of the Christian disciplines.", "\nYet people who clearly weren't doing that still felt free to call\nthemselves Christian.", "\n\nWe're also seeing a rise of other faiths in our society:\nworld religions, astrology, witchcraft, cults of all sorts, paganism,\nspiritualism, ancient gnosticism in its latest guise as New Age\nreligion.", "\n\nThe world we live in is resembling more and more the\nworld faced by the early church. ", "Our problem is that we have been\nthrough the time of Christendom, when Christianity was seen as\n“normal” - and we've lost that understanding of what it is to be\na community of faith, in a world that doesn't necessarily share that\nfaith. ", "And so people in our day and age throw up their hands in\nhorror and exclaim it is hopeless, the church is declining, it's\ndying.", "\n\nThe truth is, time is moving, and we can't turn it back\nto some ideal past. (", "And if we could, we would probably discover that\npast wasn't so ideal, after all.) ", "The church is not dying. ", "It may\nhave to change, but it's changed endless times before, and probably\nwill endless times again. ", "It may need to find different ways of doing\nthings; but it is Christ's body in this world – and it will not\ndie.", "\n\nThe world we live in is in many ways like the world of\nMark's day. ", "Christianity is one choice among many for people to\nbelieve in. ", "Any sense of privilege that attached to being Christian\nhas long gone. ", "In fact, for many people, being Christian is seen with\nnegative connotations. (", "And it was in Mark's day too.)", "\n\nIn Australia, we don't live under any particular\npersecution, although Christians in some countries really do. ", "But\nthere are times when sticking to our faith may make us seem different\nor strange.", "\n\nThe Mark reading is directed at a church which is in a\nlot of ways like us. ", "They're not the centre of society (for them they\nnever have been.) ", "Sometimes they're considered with the suspicion (to\nsay the very least) by their neighbours.", "\n\nAnd Mark reminds this church – this is all OK. ", "It's\nall OK because Jesus said all this kind of stuff will happen – but\nthat doesn't mean it's the end. ", "The problems we see (in their case\nthe fall of Jerusalem, in our case a slow decline in the number of\npractising Christians in our country) are only a part of a much\nbigger picture. ", "In the bigger picture – these things don't matter\nthat much compared to the promise that Jesus will come back, and make\neverything right.", "\n\nThis is the future orientation of the advent season,\nindeed of the whole life of the church. ", "At some time, Jesus will come\nback. ", "This isn't the kind of hope for the future that involves doing\nas some groups which anticipate the end of the world do and go into\nbunkers or arm themselves to the teeth for some great battle. ", "It's a\nhope – and it's based in God – not a fear based in some\nexpectation of the evil human beings can do to each other. ", "There is\nno point in going into hiding, or storing up food or doing anything\nto change the pattern of our days. ", "There is no point, because we\ndon't know when Jesus will return. (", "The church has been waiting more\nthan 2000 years, so obviously a bit of patience is required with\nthis.)", "\n\nSo if we don't prepare the way the radical groups and\ncults do, how do we prepare for Jesus' coming?", "\n\nOne clue is that the angels are sent out throughout the\nworld to gather in the elect. ", "Obviously, God's people are meant to be\nout in the world. ", "We need to be in worship – but that's meant to be\nthe fuel we use for life in the world. ", "Our being an active part of\nthe world is not supposed to change as long as this world exists. ", "We\nare warned to keep awake, to be prepared at all times.", "\n\nIf we are to be out in the world doing what it is God\nwants at any time, we need to go back to the basics and ask so what\nis it God wants of us? ", "Jesus summarised it in two simple-sounding\ninstructions: Love God, and love your neighbour as yourself.", "\n\nThey may sound simple – but they are difficult to\nsustain in the long-term. ", "To always act in a way which shows infinite\nrespect for God, others, and ourselves, is a tough call. ", "It sounds\neasy – and it can be easy for a short time. ", "But in human beings\nthere is a tendency to want the best for ourselves – even if it\ncosts others. (", "Or for some people to go to the other extreme and take\nso little care of themselves they are of no use to anyone else.)", "\nThat's what makes it difficult long term, to put God first, and to\nmeasure our actions by what is loving towards God, and our neighbours\nas ourselves.", "\n\nSo in Advent, we are reminded to focus again on a\nquestion that has been with the Christian Church right from its birth\n– if Jesus were to return today, how would he feel about what he\nfound us all doing?", "\n\nAnd we look forward in hope, that despite all the things\nhuman beings have managed to do wrong in the world – Jesus will\nreturn – and all things will be put right for eternity.", "\n\nNobody knows\nhow to hope like a kid in Advent. ", "There's just so many exciting\nthings to look forward to, school holidays, presents, celebrations.", "\n\nBut our hope\nof advent is more than just a hope for all the good things we can\nhave and do in our celebrations. ", "Advent is about looking two\ndifferent ways in history – we look back and anticipate Jesus\ncoming to us as the baby of Bethlehem – but we also look forward\nand anticipate Jesus coming again as ruler and judge of the world.", "\n\nWe don't\ntalk about judgement much any more. ", "It's not seen as good publicity\nfor the church. ", "But it truly is a part of our advent hope. ", "Jesus will\ncome as judge – and the basis on which he will judge is the quality\nof our love. ", "Each person we encounter in our daily life is Jesus\ncoming to us in another form. ", "How will we love them?", "\n\nThe world\nthat Jesus judges – the one he puts right according to the way he\nwants things done – is a world of love. ", "And so we have much to hope\nfor through advent – as we hope for Jesus' return, and a future\nwhere his love is the most important thing there is.", "\n\nWednesday, 16 November 2011\n\nAnother option to give a gift that will make a real difference this Christmas - the Uniting World Catalogue.", "\n\nAfter worship, once or twice during Advent, Lindy will organise a stall after church where you may buy Uniting World gifts. (", "ie you buy a card that tells the person you give it to that you gave a chicken or whatever on their behalf.)", "\n\nTuesday, 15 November 2011\n\nAs most\nof you know I do a blog about lupus. ", "At the moment, I'm taking part\nin “Health Blogging Month” - a challenge where I'm given a topic\nto write on every day for a month.", "\n\nOne of\nthe topics that I've been asked to think about is a “little red\nengine” blog – to write ten to fifteen things starting with “I\nthink I can....”, and then write five that begin with “I know I\ncan.....”\n\nOf\ncourse, in the blogging challenge, this relates to living with lupus.", "\nBut I wonder what a list like that would look like if it was done for\nour church? ", "When we look around on Sunday morning and see the people\nsitting beside us. ", "When we are together as Christ's body – and when\nwe go out separately to do our daily activities – what do we think\nwe can do? ", "What do we know we can do? ", "When we realise that we are\nnever alone, but that the Spirit of God works along with us, what do\nwe know we can do?", "\n\nToday is\nChrist the King, or Reign of Christ Sunday in the church calendar –\nthe last day of the church year. ( ", "We begin again next Sunday with\nthe first day of Advent.) ", "Today we recall that Jesus is king, and\nthat he will come again to judge the living and the dead. ", "And we\nrecall those things he has told us about judgment: that “whatever\nyou do to the least of these, you do to me.” ", "What can we do? ", "When\neven the things that seem small to us, can turn out to be a lot –\nthen even a small congregation like ours can surely do incredible\nthings!", "\n\nThursday, 10 November 2011\n\nI've been\nthinking a lot these past couple of weeks about that old saying “you\ndon't judge a book by its cover.” ", "I've been thinking about it,\nbecause I've bought a new pain control device that's actually really\nhelpful, and has even allowed me to reduce the amount of pain\ncontrolling drugs I take.", "\n\nWhat's\nremarkable is that I've seen ads for this device for years, and\ndismissed it as “junk” because the ads looked like something\nincredibly dodgy. ", "Despite the ads, however, the machine turns out to\nbe a TENS machine, the same as my physiotherapist uses. ", "It uses\nelectrical currents to disrupt the pain messages from nerves to the\nbrain. ", "For me it works. ", "I can have physiotherapy while sitting at my\ndesk.", "\n\nIt reminds\nme of the story of Samuel going off to anoint a new king. ", "He goes to\nJesse's family and each of the sons come out in turn. ", "Some look big\nand strong, and Samuel thinks they'd make impressive-looking kings,\nand God says no. ", "Eventually, there's only one son left – the\nyoungest, the kid who looks after the sheep. ", "He's not big and strong.", "\nHe's not expecting to inherit the family fortune. ", "No-one's grooming\nhim to be a success at anything more than watching the sheep. ", "And God\nsays – this is the one. ", "Samuel can't see why, but he anoints the\nkid anyway. ", "King David would be a legend, an example, and a reminder\nof glory, throughout the history of Israel.", "\n\nNobody but\nGod saw what was inside the kid who looked after the sheep. ", "Nobody\nbut God saw who he could be.", "\n\nLet's recap anyway, and\ntry to take into account actual values as we would understand them –\nbecause we're dealing with huge amounts here.", "\n\nA businessman decided to\ngo establish a new business in another country. ", "He'd be away a fair\nwhile, because he was developing a totally new market. ", "He had his\nbusiness-class plane ticket booked, and his suits and laptop computer\npacked. ", "He just needed to leave someone responsible for looking after\nhis local financial interests while he was out of the country.", "\n\nHe got his lawyer to draw\nup the papers, then called in three of his employees to tell them\nwhat they were going to be doing.", "\n\nTo the first one he gave\na power of attorney over five million dollars. (", "Yes, that's about the\nmodern equivalent of the amount of money Jesus talked about.) ", "The\nemployee gulped, shuffled his feet, and said, “Sure boss, I'll do\nmy best.”", "\n\nThe second employee, he\ngave power of attorney over two million dollars, and said take care\nof it. ", "The second employee said, “OK Boss, I'll do my best.”", "\n\nThe third employee, he\ngave power of attorney over one million dollars. (", "This must have been\nthe new guy.) ", "He said, “Don't worry, Boss, it will be safe and\nsound when you get back.”", "\n\nThe three workers were in\na strange situation. ", "They apparently weren't accountants, or anyone\nat all used to dealing with this kind of money.", "\n\nThe first employee, with\nhis five million decided that he was going to invest in a new\nbusiness on the boss' behalf. ", "He established a high-end interior\ndecorating business and won contracts to refurbish Parliament House\nand the Sydney Opera House. ", "Business was booming, in fact, he doubled\nthe money.", "\n\nThe employee with the two\nmillion decided to try investing – he put some money in term\ndeposits, some in some risky high-return investments, and some in\nsome less risky lower-return investments. ", "He was very careful with\nhis investments, and eventually, he too, doubled his money.", "\n\nThe third employee, the\nnew guy, had a bright idea. ", "He saw the ventures the other two had\ngone into and realised that both had some risks involved. ", "So he did\nsomething that he believed was perfectly safe. ", "He put the money in a\nsafe. ", "It didn't increase. ", "It didn't decrease. ", "It didn't do anything.", "\nBut it was safe, in the safe. ", "He added a fence with razor wire at the\ntop, forgot security code for his ultra-sensitive burglar alarm and\nhad to stay very still inside until someone rescued him.", "\n\nAfter a couple of years\naway, the businessman returned to see how things were going at home.", "\n\nThe first employee was a\nbit apologetic. ", "A politician had reneged on a major contract when the\nmedia discovered that she was putting a $50 toilet roll holder in her\noffice as a paper-weight. ", "But he had ten million to hand over, having\nstarted with five. ", "The boss was impressed, and told employee number\none to hang on to the ten million and keep up the good work.", "\n\nThe second employee was\nalso a bit apologetic. ", "He'd lost a bit on the global financial\ncrisis, but things had started coming good again. ", "He had four million\nto hand over after having started with two. ", "The boss was impressed,\nand told employee number two to hang on to the four million and keep\nup the good work.", "\n\nThen came the third\nemployee. ", "He handed over the original million dollars and explained\nthat he really hadn't wanted to take a risk with the money. ", "The boss\nwasn't happy. ", "Even bank interest would have been some profit! ", "The\nboss sacked employee three on the spot – and told employee one to\ntake charge of that million as well and do something worthwhile with\nit.", "\n\nNowadays, employee one is\nwinning design awards left, right and centre. ", "There's a tv series and\na magazine on how to copy his style in your own home.", "\n\nEmployee two's got a\nreputation for buying up failing businesses and making them\nprofitable. ", "Newspaper columnists watch his every move, because any\ncompany he's interested in will turn to gold.", "\n\nEmployee three's still\nlooking for work that will suit his talents.", "\n\nWell, that's a fairly\namusing way to look at the parable. ", "Now let's get serious.", "\n\nLet's put ourselves in\nthe story. ", "Jesus is the businessman, who has gone away, leaving us\nthe Holy Spirit, and the gifts of the Spirit, for the building up of\nGod's reign. ", "We all have sets of resources entrusted to our care.", "\nWill we use the resources, stretch them, take risks with them? ", "Or\nwill we safely hide them away? ", "At some time Jesus will return and say\n“What did you do with the things I gave you?”", "\n\nSo what resources have\nwe?", "\n\nWe all have bodies. ", "Some\nhave bodies that are more able than others. ", "What do we do with our\nbodies, our hands, our feet, our eyes, our ears, and mouths, to\nfurther the reign of God where we are? ", "What places do we physically\ngo? ", "What things do we physically do? ", "How could those be used to carry\nout God's work?", "\n\nWe all have things we're\ngood at. ", "Some people might be good at sewing. ", "Some people might be\ngood at cooking. ", "Some people might be good at arts and crafts. ", "Some\npeople might be good at gardening. ", "Some people might be good at\norganising things. ", "Some people might be good at typing. ", "Some might be\ngood at book-keeping. ", "Some people are good at praying. ", "Some are good\nat visiting others. ", "Some people in this congregation are good at\nleading worship. ", "Each of us is good at at least one thing. ", "Many of us\nwould be good at more than one thing.", "\n\nLike the workers in the\nparable, we all have different gifts to work with. ", "And like the\nworkers, we all have different capacities to put those gifts to use.", "\nThe difference between employees one and two and employee three is\nthat one and two took the risk of actually using what they were\ngiven. ", "They had a go. ", "They could have failed abysmally, but they took\nthe risk any way.", "\n\nEmployee Three, however,\nkept what was good to himself. ", "He made sure his gifts were never in\ndanger. ", "He made sure, in fact, that no-one ever even saw that he had\ngifts. ", "He failed, not because he didn't look after his gifts well,\nbut because he missed the point of having them. ", "The Bible tells us\nthe reason the Holy Spirit gives us gifts is for the building up of\nthe community of faith. ", "It's not about personal development, or\npersonal spirituality. ", "Whatever we have is meant to be shared. ", "We\neach have different gifts to benefit all of us.", "\n\nHow can we use the things\nwe're good at to further the reign of God. ", "Usually, when I ask\nquestions like that I leave it to you to think about quietly. ", "Today,\nI'm going to ask you to tell someone near you, one gift you have, and\nhow you intend to use it. ", "Tell someone near you, and hear what gift\nthey have and how they plan to use it. ", "But don't leave it at that.", "\nSome time in the future, when you see that person, ask them how\nthey're going at what they planned to do.", "\n\n(Discussion time)\n\nNotices\n\nPresent\nCertificates of Eldership\n\nOffering\n\nPrayers\nof the People\n\nGod of all things\n\nwe bring you the needs of\nyour world\n\nneeds which you know\nintimately,\n\nbut which we are barely\naware of.", "\n\nWe bring you the needs of\npeople in faraway places\n\nThe Horn of Africa\n\nThe people living in\nJapan, in the aftermath of the Fukushima disaster\n\nBooks by Iris, available in Paperback and eBook versions\n\nPatchwork\n\nPatchwork is an anthology of short stories and poems by author and blogger Iris Carden. ", "In this volume, you will deal with the aftermath of a dog bite in Bad Moon Rising, spend a sleepless night with The Possum in the Roof, and investigate a weird religious cult in The Time of Blood and Death. ", "The print version of the book has a bonus story not in the eBook version.", "\n\nBeside Still Waters\n\nA book of sermons and brief reflections on Christian Scripture, by Rev Iris Carden. ", "There is no specific order to the items in the book, they are intended to each be a \"surprise\" in that they are not related to the items around them. ", "It is hoped that in each, the reader will find something new or special, or unexpected, a message from God. ", "Rev Iris Carden has a Master's Degree in Theology and more than 10 years of experience as a Christian minister.", "\n\nCat-it-orial\n\nMr Bumpy is such a talented cat, he even runs his own website: mrbumpycat.com. ", "He is a blogger, and a very bad cat. ", "His favourite hobby is world domination. ", "His next hobby is harassing the humans and other animals he shares a home with. ", "In this book, you can see the world through the eyes of a megalomanicat, and some of the other animals who share his home.", "\n\nGroup Meeting\n\n(Novella) In a facility for people recovering from mental illness: a group of people with sinister pasts starts to be visited by a girl who doesn't exist.", "\n\nKarlee\n\nFailing author Terry Dixon is made an offer that seems unbelievable. ", "He can have all of his problems solved, have everything he has ever wanted, for a price. ", "The price is something that \"will not be too difficult\" for him to pay - but it is not specified what it actually will be. ", "With bills mounting up and a deadline looming, Terry agrees to a deal with something he knows nothing about.", "<\n\nAbout the Author\n\nIris Carden is a retired Uniting Church minister and former journalist. ", "Lupus forced her to stop working. ", "On good days she writes." ]
{ "pile_set_name": "Pile-CC" }
[ 0, 0, 0, 0, 0, 0, 0, 0, 0, 0, 0, 0, 0, 0, 0, 0, 0, 0, 0, 0.01092896174863388, 0, 0, 0, 0, 0, 0, 0, 0, 0, 0, 0, 0, 0.02564102564102564, 0, 0, 0, 0, 0, 0.008403361344537815, 0, 0, 0.010869565217391304, 0, 0.004524886877828055, 0.01098901098901099, 0, 0, 0, 0, 0, 0.010526315789473684, 0, 0, 0.025, 0, 0, 0.02564102564102564, 0, 0, 0, 0, 0, 0, 0, 0.016129032258064516, 0.011494252873563218, 0, 0, 0, 0, 0, 0, 0, 0, 0.008097165991902834, 0, 0, 0, 0, 0, 0, 0, 0, 0, 0, 0, 0, 0.008, 0, 0, 0, 0, 0, 0, 0, 0, 0, 0, 0, 0, 0, 0, 0, 0.0064516129032258064, 0, 0.005494505494505495, 0, 0, 0.07692307692307693, 0, 0, 0.009345794392523364, 0, 0, 0, 0, 0, 0, 0, 0, 0, 0, 0, 0, 0, 0.005847953216374269, 0.034482758620689655, 0, 0, 0, 0, 0, 0, 0, 0, 0, 0, 0, 0, 0, 0, 0, 0, 0, 0, 0, 0, 0.010471204188481676, 0.0072992700729927005, 0, 0.004901960784313725, 0.0045045045045045045, 0, 0.024390243902439025, 0, 0.011363636363636364, 0, 0.010101010101010102, 0, 0.014184397163120567, 0.011363636363636364, 0, 0, 0, 0, 0.015873015873015872, 0, 0, 0, 0, 0, 0, 0, 0, 0, 0, 0, 0, 0, 0, 0.009868421052631578, 0.008928571428571428, 0, 0, 0, 0, 0, 0, 0, 0, 0, 0, 0, 0, 0, 0.015625, 0, 0, 0, 0, 0, 0, 0, 0, 0.02040816326530612, 0, 0, 0, 0, 0, 0, 0, 0, 0, 0, 0, 0, 0, 0, 0, 0, 0, 0, 0, 0, 0, 0, 0, 0, 0.0048543689320388345, 0, 0, 0, 0, 0.004524886877828055, 0, 0, 0, 0, 0, 0, 0, 0, 0.007194244604316547, 0.023622047244094488, 0, 0, 0, 0, 0, 0, 0, 0, 0, 0, 0, 0, 0, 0, 0, 0, 0, 0, 0.009345794392523364, 0, 0, 0, 0, 0.015384615384615385, 0, 0, 0, 0, 0, 0, 0, 0, 0, 0, 0, 0, 0, 0, 0, 0, 0, 0, 0, 0, 0, 0, 0, 0.013513513513513514, 0, 0, 0, 0.015267175572519083, 0, 0, 0, 0, 0, 0, 0, 0, 0, 0, 0, 0, 0.010638297872340425, 0, 0, 0, 0, 0, 0, 0, 0, 0, 0, 0, 0, 0, 0, 0, 0, 0, 0, 0, 0, 0, 0, 0, 0, 0, 0, 0, 0, 0, 0, 0, 0, 0, 0, 0.02702702702702703, 0.02631578947368421, 0, 0.025, 0, 0.02702702702702703, 0.027777777777777776, 0.030303030303030304, 0, 0, 0, 0, 0, 0, 0, 0, 0, 0, 0, 0, 0, 0, 0, 0, 0, 0, 0, 0, 0, 0.037037037037037035, 0, 0.0045045045045045045, 0.0165016501650165, 0, 0.0136986301369863, 0.009345794392523364, 0, 0, 0.009009009009009009, 0.010526315789473684, 0, 0, 0, 0, 0, 0.012658227848101266, 0, 0, 0.009259259259259259, 0.03225806451612903, 0, 0 ]
0.00229
5
[ "Anoctamin 6 mediates effects essential for innate immunity downstream of P2X7 receptors in macrophages.", "\nPurinergic P2X7 receptors (P2X7R) are fundamental to innate immune response. ", "In macrophages, transient stimulation of P2X7R activates several transport mechanisms and induces the scrambling of phospholipids with subsequent membrane blebbing and apoptosis. ", "These processes support phagocytosis and subsequent killing of phagocytosed bacteria. ", "Here we demonstrate that the stimulation of P2X7 receptors activates anoctamin 6 (ANO6, TMEM16F), a protein that functions as Ca(2+) dependent phospholipid scramblase and Ca(2+)-activated Cl(-) channel. ", "Inhibition or knockdown of ANO6 attenuates ATP-induced cell shrinkage, cell migration and phospholipid scrambling. ", "In mouse macrophages, Ano6 produces large ion currents by stimulation of P2X7 receptors and contributes to ATP-induced membrane blebbing and apoptosis, which is largely reduced in macrophages from Ano6-/- mice. ", "ANO6 supports bacterial phagocytosis and killing by mouse and human THP-1 macrophages. ", "Our data demonstrate that anoctamin 6 is an essential component of the immune defense by macrophages." ]
{ "pile_set_name": "PubMed Abstracts" }
[ 0, 0, 0, 0, 0.0049261083743842365, 0.008695652173913044, 0.004739336492890996, 0.011494252873563218, 0 ]
0.003317
5
[ "Girl group TWICE will cut down malicious false rumors and sexual harassment comments.", "\n\nRecently, there were sexual harassment comments regarding TWICE members’ stage outfits and malicious false rumors posted and spread on online communities and SNS.", "\n\nJYP Entertainment made a statement about this on 5th, “This company, starting today, will strongly confront malicious false rumors, personal attack comments, and all posts and sexual harassment related comments. ", "We will take all legal actions against production and distribution of posts that harms TWICE members and the group.”", "\n\nThey continued, “This company has been checking malicious posts about TWICE that have been reported by fans, and have been measuring the level of maliciousness in these posts. ", "Recently, the number and maliciousness of these posts have increased to the point we can no longer tolerate. ", "We will protect our artists for their mental health and for their promotional activities.”", "\n\nThe company also stated, “As more and more of these cases have shown up in the industry, the verdicts for such acts have been increasing to the point of actual time serving in jail. ", "This company is currently checking details of the targets of these legal actions as well as the level of actions we will take. ", "We will, also, apply these same legal measures to future cases of related acts.”", "\n\nMust Read : Photo )) 161223 TWICE Heading to KBS Music Bank" ]
{ "pile_set_name": "OpenWebText2" }
[ 0, 0.012195121951219513, 0.004672897196261682, 0, 0, 0, 0, 0, 0, 0, 0.01639344262295082 ]
0.003024
5
[ "package cn.yang.common.generator;\n\nimport io.netty.channel.", "ChannelHandlerContext;\n\n/**\n * @author Cool-Coding\n * 2018/8/3\n */\npublic interface PuppetNameGenerate {\n String getPuppetName(ChannelHandlerContext ctx);\n}\n" ]
{ "pile_set_name": "Github" }
[ 0, 0.005952380952380952 ]
0.002976
5
[ "Q:\n\nHow to return the value of a CSS color element using Javascript?", "\n\nI have a PDF that I ran through PDFtoHTML to create an HTML page I can manipulate. ", " There are multiple headlines that I'd like to select based off of a single color, rgb(0, 129, 162), because that seems to be the only discernible way to differentiate the headings from the rest of the text. ", " There is a style element applying color to all span and div elements in the head element that applies the color.", "\nspan.cls_022{font-family:Arial,serif;font-size:11.1px;color:rgb(0, 129, 162);font-weight:normal;font-style:normal:text-decoration: none}\n\nThe HTML looks something like this below:\n<div style=\"left: 72.02px; top: 204.98px; position: absolute\">\n <div class=\"cls_022\" style=\"left: 72.02px; top: 204.98px; position:absolute;\">\n <span class=\"cls_022\">Text I'd like to select</span>\n </div>\n</div>\n\nNow, I can select and return the style element of the that contains the span with\ndocument.getElementsByClassName(\"cls_022\")[0].style.cssText\n\nAnd that will return the style within the tag.", "\nWithin the dev tools I can see that it has a color property of rgb(0, 129, 162) and that is what I want to use to SELECT AND RETURN THE VALUE OF THE CSS COLOR PROPERTY.", "\nAny thoughts?", "\n\nA:\n\nThis could achieve what you want:\nvar elem = document.getElementsByClassName(\"cls_022\")[1];\nvar cssColor = window.getComputedStyle(elem, null).getPropertyValue(\"color\");\n\nvar targetElems = document.querySelectorAll(\"span.cls_022\");\r\n//targetElems.forEach(el => console.log(el));\r\n//console.log(targetElems); //<--- If there are no spans with other color, and this is what you want, querySelectorAll return a NodeList.", "\r\n\r\nlet blueTitles = [];\r\ntargetElems.forEach(el => {\r\n if(window.getComputedStyle(el, null).getPropertyValue(\"color\") === 'rgb(0, 129, 162)') {\r\n blueTitles.push(el);\r\n }\r\n});\r\n\r\n//console.log(blueTitles); // <---- blueTitles is an array only contains spans with class \"cls_022\" and color rgb(0, 129, 162)\nspan.cls_022 {\r\n font-family: Arial, serif;\r\n font-size: 11.1px;\r\n color: rgb(0, 129, 162);\r\n font-weight: normal;\r\n font-style: normal:text-decoration: none\r\n}\r\n\r\nspan.red {\r\n color: red;\r\n}\n<div style=\"left: 0px; top: 0px; position: absolute\">\r\n <div class=\"cls_022\" style=\"left: 10px; top: 10px; position:absolute;\">\r\n <span class=\"cls_022\">Text I'd like to select</span>\r\n </div>\r\n</div>\r\n<div style=\"left: 0px; top: 100px; position: absolute\">\r\n <div class=\"cls_022\" style=\"left: 10px; top: 10px; position:absolute;\">\r\n <span class=\"cls_022\">Multiple headlines 1</span>\r\n </div>\r\n</div>\r\n<div style=\"left: 0px; top: 200px; position: absolute\">\r\n <div class=\"cls_022\" style=\"left: 10px; top: 10px; position:absolute;\">\r\n <span class=\"cls_022\">Multiple headlines 2</span>\r\n </div>\r\n</div>\r\n<div style=\"left: 0px; top: 300px; position: absolute\">\r\n <div class=\"cls_022\" style=\"left: 10px; top: 10px; position:absolute; \">\r\n <span class=\"cls_022 red\">Multiple headlines 3</span>\r\n </div>\r\n</div>\n\n" ]
{ "pile_set_name": "StackExchange" }
[ 0.029411764705882353, 0.011764705882352941, 0, 0, 0, 0, 0, 0.0047169811320754715, 0.0022304832713754648 ]
0.005347
5
[ "using System;\nusing System.", "Web.", "Http;\nusing System.", "Web.", "Mvc;\nusing Czar.", "AuthPlatform.", "WebApi.", "Areas.", "HelpPage.", "ModelDescriptions;\nusing Czar.", "AuthPlatform.", "WebApi.", "Areas.", "HelpPage.", "Models;\n\nnamespace Czar.", "AuthPlatform.", "WebApi.", "Areas.", "HelpPage.", "Controllers\n{\n /// <summary>\n /// The controller that will handle requests for the help page.", "\n /// </summary>\n public class HelpController : Controller\n {\n private const string ErrorViewName = \"Error\";\n\n public HelpController()\n : this(GlobalConfiguration.", "Configuration)\n {\n }\n\n public HelpController(HttpConfiguration config)\n {\n Configuration = config;\n }\n\n public HttpConfiguration Configuration { get; private set; }\n\n public ActionResult Index()\n {\n ViewBag.", "DocumentationProvider = Configuration.", "Services.", "GetDocumentationProvider();\n return View(Configuration.", "Services.", "GetApiExplorer().ApiDescriptions);\n }\n\n public ActionResult Api(string apiId)\n {\n if (!", "String.", "IsNullOrEmpty(apiId))\n {\n HelpPageApiModel apiModel = Configuration.", "GetHelpPageApiModel(apiId);\n if (apiModel !", "= null)\n {\n return View(apiModel);\n }\n }\n\n return View(ErrorViewName);\n }\n\n public ActionResult ResourceModel(string modelName)\n {\n if (!", "String.", "IsNullOrEmpty(modelName))\n {\n ModelDescriptionGenerator modelDescriptionGenerator = Configuration.", "GetModelDescriptionGenerator();\n ModelDescription modelDescription;\n if (modelDescriptionGenerator.", "GeneratedModels.", "TryGetValue(modelName, out modelDescription))\n {\n return View(modelDescription);\n }\n }\n\n return View(ErrorViewName);\n }\n }\n}" ]
{ "pile_set_name": "Github" }
[ 0.07407407407407407, 0, 0.05263157894736842, 0, 0.125, 0.07692307692307693, 0.14285714285714285, 0, 0, 0.06666666666666667, 0.07692307692307693, 0.14285714285714285, 0, 0, 0.041666666666666664, 0.07692307692307693, 0.14285714285714285, 0, 0, 0, 0.01015228426395939, 0.0035087719298245615, 0, 0, 0, 0, 0.008403361344537815, 0, 0.010638297872340425, 0, 0.004166666666666667, 0, 0, 0, 0.0625, 0 ]
0.031076
5
[ "Muitos identificadores de Via Verde estão a accionar a luz amarela à passagem nas portagens, em alguns casos, como na Ponte 25 de Abril, accionando o alarme sonoro. ", "O problema afecta identificadores que foram activados por cartões da Caixa Geral de Depósitos.", "\n\n\n\nA Renascença constatou esta quarta-feira na Loja da Via Verde de Palmela, situada em plena A2, que há clientes que não foram informados da situação e tiveram de pagar a portagem para pedirem explicações para uma questão que afinal lhes é alheia.", "\n\nA Via Verde não paga a deslocação, as portagens nem o tempo perdido. ", "Franco Caruso, do gabinete de comunicação da empresa, considera que a comunicação social relatou atempadamente o problema, que persiste desde o final da semana passada.", "\n\nFranco Caruso disse à Renascença que, em primeira instância, os clientes da Via Verde deviam ter contactado a empresa. ", "Se optaram por decidir pessoalmente resolver o problema junto das lojas, os custos de deslocação não podem ser imputados à empresa.", "\n\n“A Via Verde tornou visível este problema, os clientes Via Verde têm conhecimento de que podem pedir esclarecimentos através de uma linha de atendimento, através do site da Via Verdes e nas redes sociais através da página de Facebook. ", "Portanto, a opção de se deslocarem a um centro físico nosso, sem recorrer a esses canais anteriormente, é uma opção livre dos nossos clientes e não vejo como possa justificar nós estarmos a ser onerados por essas decisões livres dos clientes.”", "\n\nO gabinete de comunicação da Via Verde explica que a culpa do problema é da Caixa Geral de Depósitos. ", "Deverá estar resolvido no início da próxima semana.", "\n\n“O que provocou esta anomalia foi um erro no sistema do lado da Caixa Geral de Depósitos no processo de validação do ficheiro de cobrança da Via Verde. ", "Portanto, este erro impediu o correcto processamento das transacções. ", "A nossa expectativa é que esta situação, identificada no final da semana passada, esteja resolvida até dia 16”, adianta Franco Caruso.", "\n\nSe até lá a Via Verde accionar a luz amarela à passagem nas portagens, os utilizadores devem verificar primeiro se o identificador foi activado por um cartão da Caixa Geral de Depósitos. ", "Se a dúvida persistir, os automobilistas devem telefonar para a empresa antes de se deslocarem a alguma das lojas, porque ninguém lhe vai pagar nem o tempo perdido, nem os custos relativos à viagem.", "\n\n\"Problema técnico pontual\"\n\nEm resposta enviada à Renascença, a CGD confirma um “problema técnico pontual”, que foi “corrigido desde a semana passada na madrugada de quarta para quinta-feira, e que não afectou todos os cartões”.", "\n\n“Após essa correcção, a CGD reprocessou todas as transacções à medida que nos eram enviadas pela Via Verde, e, neste momento, a CGD crê não existirem mais reprocessamentos a serem efectuados, algo que só a Via Verde pode confirmar”, adianta o banco do Estado.", "\n\n“O atraso na resolução deve-se ao processo que existe entre a Via Verde, a SIBS e nós, que introduz um atraso entre o nosso reprocessamento e a sua repercussão nos sistemas da Via Verde, não permitindo uma recuperação mais célere, apesar dos esforços por nós desenvolvidos junto daquelas duas entidades”, conclui a Caixa." ]
{ "pile_set_name": "OpenWebText2" }
[ 0, 0, 0.004016064257028112, 0.014084507042253521, 0.005952380952380952, 0.01652892561983471, 0.007633587786259542, 0.008438818565400843, 0, 0, 0, 0.006493506493506494, 0, 0.014925373134328358, 0.010582010582010581, 0.010101010101010102, 0, 0.011494252873563218, 0.009287925696594427 ]
0.006291
5
[ "Abo Kopfhörer im Test Sonys neuster Lärmunterdrücker ist sehr gut – und sehr teuer\n\nWer im Homoffice oder unterwegs seine Ruhe möchte, kommt an Noise-Cancelling-Kopfhörern nicht vorbei. ", "Wir haben das neue Topmodell von Sony getestet und mit dem halb so teuren Vorgänger verglichen." ]
{ "pile_set_name": "OpenWebText2" }
[ 0.021505376344086023, 0.021052631578947368 ]
0.021279
5
[ "Lynx\n\nA new line of fragrances wants to help gamers nab high scores when it comes to body odor.", "\n\nLynx, known in the US as Axe, has partnered with Microsoft for a line of hygiene products called Lynx Xbox, according to GameSpot. ", "The line includes shower gel, deodorant and body spray.", "\n\nMarketing copy describes the aroma of these offerings as \"a fresh scent of pulsing green citrus, featuring top notes of kaffir lime and winter lemon, aromatic herbal middle notes of mint and sage, and woody bottom notes of patchouli and clearwood.\" ", "Game on.", "\n\nNow playing: Watch this: Xbox One S: All-Digital Edition drops discs\n\n\"In games, even the most insurmountable challenge can be overcome by lifting your game,\" Tania Chee, business group lead for Xbox ANZ, said in a press release Wednesday. \"", "Now, powering up can be as simple as a quick spray before you head out the door.\"", "\n\nLynx Xbox will be available only in Australia and New Zealand, starting in July, with no word on whether there'll be a global release. ", "Microsoft does have its E3 press conference happening Sunday at 1 p.m. PT (4 p.m. ET), and anything could be announced then.", "\n\nLynx didn't respond to a request for comment.", "\n\nWho knows? ", "These side-scrolling sundries could help gamers avoid smelling like they haven't showered in a fortnite.", "\n\n(Sorry.)", "\n\nOriginally published on June 4.", "\n\nJune 5: Adds statement by Xbox ANZ." ]
{ "pile_set_name": "OpenWebText2" }
[ 0, 0.03007518796992481, 0, 0, 0, 0.00411522633744856, 0, 0.0072992700729927005, 0.008064516129032258, 0, 0, 0, 0, 0, 0.02702702702702703 ]
0.005105
5
[ "Q:\n\nWhy x[i][:]=x[:][i] where x is a list of lists?", "\n\nI am working on a list of lists and accessing columns has been very confusing. ", "\nLet's assume x is defined as following:\n x = [[int(np.random.rand()*100) for i in xrange(5)] for x in xrange(10)]\n pprint.pprint(x)\n\n[[86, 92, 95, 78, 68],\n [76, 80, 44, 30, 73],\n [48, 85, 99, 35, 14],\n [3, 84, 50, 39, 47],\n [3, 7, 67, 28, 65],\n [19, 13, 98, 53, 33],\n [9, 97, 35, 25, 89],\n [48, 3, 48, 5, 1],\n [21, 40, 72, 61, 62],\n [58, 43, 84, 69, 26]]\n\nNow, both x[1][:] and x[:][1] yield the same result:\n[76, 80, 44, 30, 73]\n\nCan someone explain why?", "\nThank you\n\nA:\n\nThe behavior is pretty simple to understand if you break up the two indexing operations of each of your expressions into separate pieces.", "\n\nx[1] will be the second value from your list of lists (the list [76, 80, 44, 30, 73]).", "\nx[1][:] is a copy of x[1] (a slice that spans the whole list).", "\nx[:] is a (shallow) copy of x (the list of lists).", "\nx[:][1] is the second value from the copied list of lists, which is the same object as x[1].", "\n\nSo, the two expressions work out to be equal. ", "Note that because the first expression copies the list (with the [:] slice at the end), they're not both the same object (x[1][:] is x[:][1] will be False).", "\nIf you were using a 2D numpy array, you'd get different behavior, since you can slice in arbitrary dimensions (using slightly different syntax):\nimport numpy as np\n\nx = np.array([[86, 92, 95, 78, 68],\n [76, 80, 44, 30, 73],\n [48, 85, 99, 35, 14],\n [3, 84, 50, 39, 47],\n [3, 7, 67, 28, 65],\n [19, 13, 98, 53, 33],\n [9, 97, 35, 25, 89],\n [48, 3, 48, 5, 1],\n [21, 40, 72, 61, 62],\n [58, 43, 84, 69, 26]])\n\nprint(x[1,:]) # prints the values of the second row: [76 80 44 30 73]\nprint(x[:,1]) # prints the values of the second column: [92 80 85 84 7 13 97 3 40 43]\n\nThis may be what you were looking for.", "\n\nA:\n\nx[:][1] is the whole array row 1 and x[1][:] is all row 1 of x so they both point to the same row.", "\nIn [2]: x[:]\nOut[2]: \n[[86, 92, 95, 78, 68],\n [76, 80, 44, 30, 73],\n [48, 85, 99, 35, 14],\n [3, 84, 50, 39, 47],\n [3, 7, 67, 28, 65],\n [19, 13, 98, 53, 33],\n [9, 97, 35, 25, 89],\n [48, 3, 48, 5, 1],\n [21, 40, 72, 61, 62],\n [58, 43, 84, 69, 26]]\nIn [3]: x[1]\nOut[3]: [76, 80, 44, 30, 73]\n\nIn [4]: x[1][:]\nOut[4]: [76, 80, 44, 30, 73]\n\nUsing x[:] is often used if you wanted to store a copy of x in a variable and not just a reference to it:\nIn [6]: x1 = x[:]\n\nIn [7]: x1\nOut[7]: \n[[86, 92, 95, 78, 68],\n [76, 80, 44, 30, 73],\n [48, 85, 99, 35, 14],\n [3, 84, 50, 39, 47],\n [3, 7, 67, 28, 65],\n [19, 13, 98, 53, 33],\n [9, 97, 35, 25, 89],\n [48, 3, 48, 5, 1],\n [21, 40, 72, 61, 62],\n [58, 43, 84, 69, 26]]\n\nCreating a copy:\nIn [11]: x1 = x[:]\nIn [12]: id(x1) # different objects so different id's\nOut[12]: 140130415845104 \nIn [13]: id(x)\nOut[13]: 140130400510176\n\nCreating a reference:\nIn [14]: x1 = x\n\nIn [15]: id(x) # same object so same id's\nOut[15]: 140130400510176\n\nIn [16]: id(x1)\nOut[16]: 140130400510176\n\n" ]
{ "pile_set_name": "StackExchange" }
[ 0, 0, 0, 0, 0, 0, 0, 0, 0, 0.00641025641025641, 0.002785515320334262, 0, 0 ]
0.000707
5
[ "Tags\n\nHalf Asleep\n\nSleep, though one of my greatest joys in life, is frustratingly unproductive. ", "As much as I love to curl up in bed, close my eyes and flip the off-switch on my consciousness, the realization that I spend 24-33% of my day just staring at the backs of my eyelids is exasperating. ", "Imagine the possibilities if I could instead spend that time reading books, or exercising, or writing a paper for class, or any number of other things! ", "While sleep is certainly important for rejuvenating our bodies and solidifying newly formed memories [1], couldn’t it be possible to do those things while still being able to read, or move, or write? ", "I do have two hemispheres of my brain, after all…do they really BOTH need to be asleep?", "\n\nWhile the answer for me and all other humans is “yes”, the same is not true for all members of the animal kingdom. ", "Animals such as dolphins and birds demonstrate a phenomenon known as unihemispheric slow wave sleep, which means that one half of their brain can be asleep while the other is awake! ", "But in order to appreciate how these animals’ sleep differs from our own, let’s first get up to speed on what a typical, human brain looks like when it’s asleep.", "\n\nOh, sweet sleep…\n\nSleeping may feel like a transition from an “on” state to an “off” state, but this is far from the case. ", "Have you ever started a time-consuming process on your computer – say uploading photos or running some lengthy code – and then let it sit for long enough that a screensaver appeared? ", "This is like your brain when you’re asleep; while your body looks like it’s been shut down, your brain is still working really hard in the background – its activity just shifts into a different “mode”.", "\n\nFigure 1 – Electrical brain activity recorded during wakefulness (top) and the different stages of sleep\n\nIn fact, there are multiple different activity “modes” that your brain shifts into over the course of a regular night’s sleep. ", "These modes can be distinguished by the characteristic shapes of the waves in electrical brain activity (Figure 1). ", "For instance, look at the difference between “Awake” activity and Stage 3 and 4 activity; while brain activity during wakefulness exhibits frequent but small fluctuations, activity during stages 3 and 4 is appropriately called slow wave sleep for the slow (about 1-2 per second) but large changes in voltage that occur. ", "These big waves are the classic neural signature of deep sleep (but to learn about REM sleep as well as why sleep is important, check out a previous NeuWrite article).", "\n\nIn humans, when one half of the brain is in slow wave sleep, the other half has no choice but to be as well. ", "But in animals that exhibit unihemispheric slow wave sleep, only one half of their brain (uni-hemispheric) is in deep (slow wave) sleep while the other is awake!", "\n\nJust keep swimming!", "\n\nPerhaps the best known examples of animals that can sleep with half of their brains are cetaceans – aquatic mammals like dolphins and whales. ", "Electrical brain activity measured by EEG from each hemisphere of a sleeping dolphin shows an obvious difference (Figure 2); while one hemisphere exhibits the big, slow changes in electrical activity characteristic of slow wave sleep (like in Fig 1), activity in the other hemisphere is indistinguishable from activity during full wakefulness. ", "This unihemispheric sleep state enables dolphins to keep one eye open, to come up to the surface for air and to “just keep swimming” – even while they’re asleep! [", "2]\n\nFig. ", "2 – brain activity during sleep in dolphins. ", "Rows 1 and 2 indicate right and left brain hemispheres, respectively. ", "Notice that left hemisphere activity during right hemisphere sleep and right hemisphere activity during left hemisphere sleep are both indistinguishable from awake activity!", "\n\nUnihemispheric sleep in cetaceans is thought to serve multiple other functions as well. ", "For instance, having one eye open may help the dolphin or whale perform sentinel functions – in other words, literally keeping an eye out for danger. ", "When researchers presented a visual stimulus so that it was processed by the dolphin’s “asleep” hemisphere, the dolphin did not respond at all. ", "But if the stimulus was shown to the other eye so that it was processed by the “awake” hemisphere, the dolphin immediately reacted with a startle response or sudden, fast swimming [3]. ", "Meanwhile, it has also been observed that a sleeping dolphin’s open eye is almost always pointed towards its schoolmates or, if it’s a newborn calf, towards its mother [4], suggesting that unihemispheric sleep may also be advantageous by helping sleeping cetaceans stick with their groups.", "\n\nGet your ducks in a row!", "\n\nHow do those sleeping flamingos not fall down in their sleep?!", "\n\nHave you ever been to the zoo and noticed a sleeping flamingo standing on one leg and maintaining perfect posture? ", "This is because birds, like cetaceans, also possess the ability to sleep with half of their brains. ", "In the flamingo’s case, this is thought to allow them to keep standing on the leg that is controlled by the awake side of the brain, while having even just one brain hemisphere go to sleep allows them to lower their metabolism and reap sleep’s other benefits [5].", "\n\nUnihemispheric slow wave sleep in birds is believed to help the animals be alert to potential predators. ", "In a study of mallard ducks sleeping in a row, the ducks in the middle of the row had one eye open only about 12% of the time asleep, while ducks on the ends were more than twice as likely to have one eye open, and the open eye was almost always pointed away from the group. ", "Meanwhile, EEG recordings of the ducks’ electrical brain activity confirmed that the ducks on the ends of the row demonstrated the neural signatures of slow wave sleep more strongly in the hemisphere corresponding to the eye that pointed away from the other ducks (Figure 3). ", "Overall, the ducks that were flanked by other ducks spent significantly more time fully (bihemispherically) asleep, while the “sentinel” ducks on the ends of the row spent more time in unihemispheric slow wave sleep so that their eyes on the exposed ends were alert to possible predators. ", "In fact, these sentinel ducks in unihemispheric sleep responded in a fraction of a second to simulated predatory attacks [6]. ", "Put more simply, a row of sleeping ducks collectively takes on the neural characteristics of a single, alert duck!", "\n\nFigure 3 – unihemispheric slow wave sleep in a row of sleeping ducks. ", "While the sleeping duck in the middle of the row demonstrates the neural signatures of sleep in both hemispheres, the duck on the left (viewer’s right) keeps its left eye open because its right hemisphere (which processes visual information from the left eye) is awake while its left hemisphere exhibits slow wave sleep. ", "The opposite is true of the duck on the right. ", "The depicted neural data is not real data but purely schematic.", "\n\nWhy can’t my brain do that?!", "\n\nAlthough the neural basis of unihemispheric slow wave sleep is still largely unknown, neuroscientists have been able to rule out some of the usual suspects. ", "For instance, the corpus callosum – the bundle of fibers that links the brain’s two hemispheres and allows neural signals to flow between them – is a logical candidate for making sleep in humans and most other mammals exclusively bihemispheric. ", "However, split-brain patients and people born without a corpus callosum still experience bihemispheric sleep; the electrical signals in each hemisphere are less correlated with each other, but they still both exhibit the neural signatures of slow wave sleep [7]. ", "Thus, while the corpus callosum plays a role in synchronizing the two hemispheres during sleep, it is not solely responsible for ensuring that they are both in the same “mode” of sleep or wakefulness at the same time.", "\n\nSo let’s turn to an alternative suspect: the brainstem. ", "The brainstem is an evolutionarily ancient brain structure connecting the brain to the spinal cord, and it regulates many unconscious functions that are necessary for our survival, such as breathing, regulating heart rate, and – you guessed it – sleep. ", "The brainstem has emerged as a top contender for controlling whether sleep is uni- or bi-hemispheric because cats – normally bihemispheric sleepers, like us – have been shown to exhibit unihemispheric slow wave sleep after their brainstem was cut down the middle, even when their corpus callosum was completely intact! [", "8] Still, exactly how the brainstem regulates sleep and how this differs between animals that experience bihemispheric or unihemispheric sleep is a fascinating question that has yet to be answered.", "\n\nWhile neuroscientists ponder the neural mechanisms underlying these different forms of sleep, it is also fascinating to consider why, and how, we may have evolved without the ability to sleep with only one half of our brain. ", "Some researchers have speculated that unihemispheric sleep may have been selected against over the course of mammalian evolution because it is a less efficient means of sleep. ", "Since most mammals’ sleeping habitats do not necessitate maintained alertness, the costs outweighed the benefits. ", "In cetaceans, on the other hand, the benefits of unihemispheric sleep may have outweighed the costs because of these animals’ need to come up for air and continuously swim in their underwater environments [7].", "\n\nHowever, what if this unique ability wasn’t completely lost in humans after all? ", "A very recent study reported that on the first night that people spent in a new place, their right hemispheres demonstrated more slow wave activity in a particular brain network than their left hemispheres. ", "Moreover, only their left hemispheres showed a neural response to a surprising auditory stimulus, suggesting that their left hemisphere was more alert or “awake” than their right! [", "9] This could explain why many people report having disrupted sleep on their first night of vacation, but more relevant to this article, it raises the intriguing possibility that the human brain is at least partially capable of decoupling its two hemispheres during sleep when it is in an unfamiliar and potentially dangerous or unpredictable environment. ", "Nevertheless, the degree of decoupling found in humans is nowhere near that of cetaceans or birds during unihemispheric sleep.", "\n\nAs much as I often think that my time could be better spent if my brain, or even just half of my brain, didn’t spend so much time asleep, I have to believe that there’s probably a good reason for it. ", "Still, I can’t help but wonder, what would it feel like for one half of the brain to be asleep while the other is awake? ", "Would it feel like a fitful night’s sleep, or something completely different and unimaginable? ", "If we ever figure out a way to communicate with dolphins, that’s the first thing I’ll ask." ]
{ "pile_set_name": "Pile-CC" }
[ 0, 0, 0, 0, 0, 0, 0, 0, 0, 0, 0, 0, 0, 0, 0.011976047904191617, 0, 0, 0, 0, 0.0029069767441860465, 0, 0, 0, 0, 0, 0, 0, 0.006944444444444444, 0, 0, 0, 0, 0, 0, 0, 0, 0, 0.0036231884057971015, 0, 0, 0, 0, 0, 0, 0, 0, 0, 0, 0, 0, 0, 0, 0, 0, 0, 0, 0, 0, 0, 0, 0, 0, 0, 0, 0, 0, 0 ]
0.00038
5
[ "Pages\n\nWednesday, October 2, 2013\n\nASCP French Linen Side Table\n\nThe other day I was at an auction and one of the pieces I was interested in was this little reproduction gramophone cabinet below. ", "Then I bumped into a blog reader who had also bought a couple of pieces of furniture from me and I was distracted chatting to her and missed the bidding! ", "Luckily I found out it had been passed in so later I approached the auctioneer and he let me buy it for a great price. ", "I think no one else knew what to do with it. ", "I loved its shape and knew that paint could transform it.", "\n\nI have yet to test out to see if it's in working condition, but I didn't buy it to use it. ", "When I went to pick it up the other day I realised that it was two separate pieces. ", "I will work out what to do with the top part some other day - gut it probably, I'm sure a plan will come to me.", "\n\nThe other day the kids and I had a day at home which has been rare this school holidays. ", "We laid out a drop sheet in the carport and got busy with some craft projects.", "\n\nWhile the kids made a birdhouse and wooden jet, I got busy with ASCP French Linen on the base table. ", "It is such a gorgeous shape. ", "I also gave the table a wash with Pure White to bring out the grain.", "\n\nI love French Linen with a wash over white. ", "The resulting colour is a soft subtle grey. ", "I had previously used a white wash over Paris Grey and a Paris Grey wash over French Linen but hadn't tried white over French Linen. ", "I recently saw Karen from Restyled Vintage use these colours of a hall stand and really liked the result so thought I'd use it on a few recently projects. ", "It really brings out the oak grain on this table.", "\n\nthe book is a lovely illustrated book of sonnets that was mum's. ", "I love the paintings.", "\n\nOnce the whitewash wash was very dry I waxed it using MMS Wax. ", "I have found that when using washes that if your paint hasn't had long enough to harden the wax will remove the wash, so if you are doing this technique don't be impatient. ", "I also gave the table a very slight distress to bright out some of the details.", "\n\nThis woven chair is still one of my all time favourite pieces. ", "So much work to get this look though. ", "I won't ever sell despite getting asked a lot to part with it.", "\n\nIf the table doesn't sell by the time I get around to sorting out the top part I might join them together to make a little cabinet. ", "Do you like the washed look? ", "Any other ideas on what to do with the top part of the gramophone?", "\n\nFrench linen is my favorite ASCP color. ", "I like to use French linen as the base, then dry brush with Duck Egg and Old White followed by clear wax and dark wax in the crevices and finally German's Silver on the high points. ", "A lot of work, but so worth it for a special piece. ", "The side table looks great. ", "Can't wait to see what you do with the top.", "\n\nSo loving the \"washed\" look you've done! ", "With regards the top bit, it will be interesting to see what it looks like \"gutted\" - a coffee table with shelf in the middle? ", "Would a small modern stereo fit inside the middle bit?" ]
{ "pile_set_name": "Pile-CC" }
[ 0.00510204081632653, 0, 0, 0, 0, 0, 0, 0, 0, 0, 0, 0, 0, 0.021739130434782608, 0, 0.022556390977443608, 0.0064516129032258064, 0, 0, 0, 0.015384615384615385, 0, 0, 0, 0, 0, 0, 0, 0, 0, 0, 0, 0, 0, 0, 0, 0 ]
0.001925
5
[ "Q:\n\nCall different function based on a boolean\n\nI have a boolean in my function that decide what function to call. ", "Both of the function that are beying called giving back an Array.", "\nNow in my eyes Hex[] areaHexes does exists but the compiler does not compile because it thinks its not set ( does not exists ).", "\nHow do I call one of both function properly based on what value bool semiRandom has? ", "\nvoid ElevateArea(int q, int r, int range, bool semiRandom = false, float centerHeight = 1f)\n{\n Hex centerHex = GetHexAt(q, r);\n\n if (semiRandom) \n { \n Hex[] areaHexes = GetSemiRandomHexesWithinRangeOf(centerHex, range);\n } \n else\n {\n Hex[] areaHexes = GetHexesWithinRangeOf(centerHex, range);\n }\n\n foreach (Hex h in areaHexes)\n {\n //if (h.Elevation < 0)\n // h.Elevation = 0;\n\n h.Elevation += 0.5f * Mathf.", "Lerp(1f, 0.25f, Hex.", "Distance(centerHex, h ) / range);\n }\n}\n\nA:\n\nThe reason it's not working is that you are currently declaring two local variables called areaHexes, each of which has a scope which is just the block they're declared in - so neither of them is in scope when you try to use them.", "\nBrandon's answer (declaring the variable outside the if statement and then assigning to it from different places) will work fine - areaHexes is now in scope when you use it later. ", "However, you can do it more simply with a conditional ?: ", "operator:\nHex[] areaHexes = semiRandom\n ? ", "GetSemiRandomHexesWithinRangeOf(centerHex, range)\n : GetHexesWithinRangeOf(centerHex, range);\n\nA:\n\nYour areaHexes is being declared locally within your if-else blocks, it's not visible outside of the scope of those blocks. ", "You have two different local areaHexes variables:\nif (semiRandom) \n{\n // This definition of areaHexes is visible only within these { }\n // and is not the same as the one in the else block below\n Hex[] areaHexes = GetSemiRandomHexesWithinRangeOf(centerHex, range);\n} \nelse\n{\n // This definition of areaHexes is visible only within these { }\n // and is not the same one as the one above\n Hex[] areaHexes = GetHexesWithinRangeOf(centerHex, range);\n}\n\nDeclare it outside:\nHex[] areaHexes;\n\nif (semiRandom) { \n areaHexes = GetSemiRandomHexesWithinRangeOf(centerHex, range);\n}\nelse {\n areaHexes = GetHexesWithinRangeOf(centerHex, range);\n}\n\nOr, use the tertiary ?: ", "operator Jon showed.", "\nYou should look up variable scope rules for C#.", "\n\nA:\n\nDeclare your array before your condition, like so:\nHex[] areaHexes;\n\nif (semiRandom) \n{ \n areaHexes = GetSemiRandomHexesWithinRangeOf(centerHex, range);\n} \nelse\n{\n areaHexes = GetHexesWithinRangeOf(centerHex, range);\n}\n\n" ]
{ "pile_set_name": "StackExchange" }
[ 0, 0.015384615384615385, 0, 0, 0.0021141649048625794, 0, 0, 0.0055248618784530384, 0, 0, 0.004424778761061947, 0.002911208151382824, 0.05, 0, 0.004310344827586207 ]
0.005645
5
[ "{\n \"@timestamp\": \"2017-10-12T08:05:34.853Z\",\n \"azure\": {\n \"metrics\": {\n \"data_usage\": {\n \"total\": 131072\n },\n \"document_count\": {\n \"total\": 2\n },\n \"document_quota\": {\n \"total\": 107374182400\n }\n },\n \"namespace\": \"Microsoft.", "DocumentDb/databaseAccounts\",\n \"resource\": {\n \"group\": \"obs-infrastructure\",\n \"id\": \"/subscriptions/70bd6e64-4b1e-4835-8896-db77b8eef364/resourceGroups/obs-infrastructure/providers/Microsoft.", "DocumentDb/databaseAccounts/obsaccount\",\n \"name\": \"obsaccount\",\n \"tags\": {\n \"defaultExperience\": \"Core (SQL)\"\n },\n \"type\": \"Microsoft.", "DocumentDb/databaseAccounts\"\n },\n \"subscription_id\": \"70bd6e64-4b1e-4835-8896-db77b8eef364\",\n \"timegrain\": \"PT5M\"\n },\n \"cloud\": {\n \"provider\": \"azure\",\n \"region\": \"westeurope\"\n },\n \"event\": {\n \"dataset\": \"azure.monitor\",\n \"duration\": 115000,\n \"module\": \"azure\"\n },\n \"metricset\": {\n \"name\": \"monitor\",\n \"period\": 10000\n },\n \"service\": {\n \"type\": \"azure\"\n }\n}\n" ]
{ "pile_set_name": "Github" }
[ 0.005555555555555556, 0.013636363636363636, 0.010362694300518135, 0.004329004329004329 ]
0.008471
5
[ "24 4 3 24x24/left_ptr_watch_0001.png 16\n24 4 3 24x24/left_ptr_watch_0002.png 16\n24 4 3 24x24/left_ptr_watch_0003.png 16\n24 4 3 24x24/left_ptr_watch_0004.png 16\n24 4 3 24x24/left_ptr_watch_0005.png 16\n24 4 3 24x24/left_ptr_watch_0006.png 16\n24 4 3 24x24/left_ptr_watch_0007.png 16\n24 4 3 24x24/left_ptr_watch_0008.png 16\n24 4 3 24x24/left_ptr_watch_0009.png 16\n24 4 3 24x24/left_ptr_watch_0010.png 16\n24 4 3 24x24/left_ptr_watch_0011.png 16\n24 4 3 24x24/left_ptr_watch_0012.png 16\n24 4 3 24x24/left_ptr_watch_0013.png 16\n24 4 3 24x24/left_ptr_watch_0014.png 16\n24 4 3 24x24/left_ptr_watch_0015.png 16\n24 4 3 24x24/left_ptr_watch_0016.png 16\n24 4 3 24x24/left_ptr_watch_0017.png 16\n24 4 3 24x24/left_ptr_watch_0018.png 16\n24 4 3 24x24/left_ptr_watch_0019.png 16\n24 4 3 24x24/left_ptr_watch_0020.png 16\n24 4 3 24x24/left_ptr_watch_0021.png 16\n24 4 3 24x24/left_ptr_watch_0022.png 16\n24 4 3 24x24/left_ptr_watch_0023.png 16\n24 4 3 24x24/left_ptr_watch_0024.png 16\n24 4 3 24x24/left_ptr_watch_0025.png 16\n24 4 3 24x24/left_ptr_watch_0026.png 16\n24 4 3 24x24/left_ptr_watch_0027.png 16\n24 4 3 24x24/left_ptr_watch_0028.png 16\n24 4 3 24x24/left_ptr_watch_0029.png 16\n24 4 3 24x24/left_ptr_watch_0030.png 16\n24 4 3 24x24/left_ptr_watch_0031.png 16\n24 4 3 24x24/left_ptr_watch_0032.png 16\n24 4 3 24x24/left_ptr_watch_0033.png 16\n24 4 3 24x24/left_ptr_watch_0034.png 16\n24 4 3 24x24/left_ptr_watch_0035.png 16\n24 4 3 24x24/left_ptr_watch_0036.png 16\n24 4 3 24x24/left_ptr_watch_0037.png 16\n24 4 3 24x24/left_ptr_watch_0038.png 16\n24 4 3 24x24/left_ptr_watch_0039.png 16\n24 4 3 24x24/left_ptr_watch_0040.png 16\n24 4 3 24x24/left_ptr_watch_0041.png 16\n24 4 3 24x24/left_ptr_watch_0042.png 16\n24 4 3 24x24/left_ptr_watch_0043.png 16\n24 4 3 24x24/left_ptr_watch_0044.png 16\n24 4 3 24x24/left_ptr_watch_0045.png 16\n24 4 3 24x24/left_ptr_watch_0046.png 16\n24 4 3 24x24/left_ptr_watch_0047.png 16\n24 4 3 24x24/left_ptr_watch_0048.png 16\n24 4 3 24x24/left_ptr_watch_0049.png 16\n24 4 3 24x24/left_ptr_watch_0050.png 16\n24 4 3 24x24/left_ptr_watch_0051.png 16\n24 4 3 24x24/left_ptr_watch_0052.png 16\n24 4 3 24x24/left_ptr_watch_0053.png 16\n24 4 3 24x24/left_ptr_watch_0054.png 16\n24 4 3 24x24/left_ptr_watch_0055.png 16\n24 4 3 24x24/left_ptr_watch_0056.png 16\n24 4 3 24x24/left_ptr_watch_0057.png 16\n24 4 3 24x24/left_ptr_watch_0058.png 16\n24 4 3 24x24/left_ptr_watch_0059.png 16\n24 4 3 24x24/left_ptr_watch_0060.png 16\n32 5 4 32x32/left_ptr_watch_0001.png 16\n32 5 4 32x32/left_ptr_watch_0002.png 16\n32 5 4 32x32/left_ptr_watch_0003.png 16\n32 5 4 32x32/left_ptr_watch_0004.png 16\n32 5 4 32x32/left_ptr_watch_0005.png 16\n32 5 4 32x32/left_ptr_watch_0006.png 16\n32 5 4 32x32/left_ptr_watch_0007.png 16\n32 5 4 32x32/left_ptr_watch_0008.png 16\n32 5 4 32x32/left_ptr_watch_0009.png 16\n32 5 4 32x32/left_ptr_watch_0010.png 16\n32 5 4 32x32/left_ptr_watch_0011.png 16\n32 5 4 32x32/left_ptr_watch_0012.png 16\n32 5 4 32x32/left_ptr_watch_0013.png 16\n32 5 4 32x32/left_ptr_watch_0014.png 16\n32 5 4 32x32/left_ptr_watch_0015.png 16\n32 5 4 32x32/left_ptr_watch_0016.png 16\n32 5 4 32x32/left_ptr_watch_0017.png 16\n32 5 4 32x32/left_ptr_watch_0018.png 16\n32 5 4 32x32/left_ptr_watch_0019.png 16\n32 5 4 32x32/left_ptr_watch_0020.png 16\n32 5 4 32x32/left_ptr_watch_0021.png 16\n32 5 4 32x32/left_ptr_watch_0022.png 16\n32 5 4 32x32/left_ptr_watch_0023.png 16\n32 5 4 32x32/left_ptr_watch_0024.png 16\n32 5 4 32x32/left_ptr_watch_0025.png 16\n32 5 4 32x32/left_ptr_watch_0026.png 16\n32 5 4 32x32/left_ptr_watch_0027.png 16\n32 5 4 32x32/left_ptr_watch_0028.png 16\n32 5 4 32x32/left_ptr_watch_0029.png 16\n32 5 4 32x32/left_ptr_watch_0030.png 16\n32 5 4 32x32/left_ptr_watch_0031.png 16\n32 5 4 32x32/left_ptr_watch_0032.png 16\n32 5 4 32x32/left_ptr_watch_0033.png 16\n32 5 4 32x32/left_ptr_watch_0034.png 16\n32 5 4 32x32/left_ptr_watch_0035.png 16\n32 5 4 32x32/left_ptr_watch_0036.png 16\n32 5 4 32x32/left_ptr_watch_0037.png 16\n32 5 4 32x32/left_ptr_watch_0038.png 16\n32 5 4 32x32/left_ptr_watch_0039.png 16\n32 5 4 32x32/left_ptr_watch_0040.png 16\n32 5 4 32x32/left_ptr_watch_0041.png 16\n32 5 4 32x32/left_ptr_watch_0042.png 16\n32 5 4 32x32/left_ptr_watch_0043.png 16\n32 5 4 32x32/left_ptr_watch_0044.png 16\n32 5 4 32x32/left_ptr_watch_0045.png 16\n32 5 4 32x32/left_ptr_watch_0046.png 16\n32 5 4 32x32/left_ptr_watch_0047.png 16\n32 5 4 32x32/left_ptr_watch_0048.png 16\n32 5 4 32x32/left_ptr_watch_0049.png 16\n32 5 4 32x32/left_ptr_watch_0050.png 16\n32 5 4 32x32/left_ptr_watch_0051.png 16\n32 5 4 32x32/left_ptr_watch_0052.png 16\n32 5 4 32x32/left_ptr_watch_0053.png 16\n32 5 4 32x32/left_ptr_watch_0054.png 16\n32 5 4 32x32/left_ptr_watch_0055.png 16\n32 5 4 32x32/left_ptr_watch_0056.png 16\n32 5 4 32x32/left_ptr_watch_0057.png 16\n32 5 4 32x32/left_ptr_watch_0058.png 16\n32 5 4 32x32/left_ptr_watch_0059.png 16\n32 5 4 32x32/left_ptr_watch_0060.png 16\n48 7 5 48x48/left_ptr_watch_0001.png 16\n48 7 5 48x48/left_ptr_watch_0002.png 16\n48 7 5 48x48/left_ptr_watch_0003.png 16\n48 7 5 48x48/left_ptr_watch_0004.png 16\n48 7 5 48x48/left_ptr_watch_0005.png 16\n48 7 5 48x48/left_ptr_watch_0006.png 16\n48 7 5 48x48/left_ptr_watch_0007.png 16\n48 7 5 48x48/left_ptr_watch_0008.png 16\n48 7 5 48x48/left_ptr_watch_0009.png 16\n48 7 5 48x48/left_ptr_watch_0010.png 16\n48 7 5 48x48/left_ptr_watch_0011.png 16\n48 7 5 48x48/left_ptr_watch_0012.png 16\n48 7 5 48x48/left_ptr_watch_0013.png 16\n48 7 5 48x48/left_ptr_watch_0014.png 16\n48 7 5 48x48/left_ptr_watch_0015.png 16\n48 7 5 48x48/left_ptr_watch_0016.png 16\n48 7 5 48x48/left_ptr_watch_0017.png 16\n48 7 5 48x48/left_ptr_watch_0018.png 16\n48 7 5 48x48/left_ptr_watch_0019.png 16\n48 7 5 48x48/left_ptr_watch_0020.png 16\n48 7 5 48x48/left_ptr_watch_0021.png 16\n48 7 5 48x48/left_ptr_watch_0022.png 16\n48 7 5 48x48/left_ptr_watch_0023.png 16\n48 7 5 48x48/left_ptr_watch_0024.png 16\n48 7 5 48x48/left_ptr_watch_0025.png 16\n48 7 5 48x48/left_ptr_watch_0026.png 16\n48 7 5 48x48/left_ptr_watch_0027.png 16\n48 7 5 48x48/left_ptr_watch_0028.png 16\n48 7 5 48x48/left_ptr_watch_0029.png 16\n48 7 5 48x48/left_ptr_watch_0030.png 16\n48 7 5 48x48/left_ptr_watch_0031.png 16\n48 7 5 48x48/left_ptr_watch_0032.png 16\n48 7 5 48x48/left_ptr_watch_0033.png 16\n48 7 5 48x48/left_ptr_watch_0034.png 16\n48 7 5 48x48/left_ptr_watch_0035.png 16\n48 7 5 48x48/left_ptr_watch_0036.png 16\n48 7 5 48x48/left_ptr_watch_0037.png 16\n48 7 5 48x48/left_ptr_watch_0038.png 16\n48 7 5 48x48/left_ptr_watch_0039.png 16\n48 7 5 48x48/left_ptr_watch_0040.png 16\n48 7 5 48x48/left_ptr_watch_0041.png 16\n48 7 5 48x48/left_ptr_watch_0042.png 16\n48 7 5 48x48/left_ptr_watch_0043.png 16\n48 7 5 48x48/left_ptr_watch_0044.png 16\n48 7 5 48x48/left_ptr_watch_0045.png 16\n48 7 5 48x48/left_ptr_watch_0046.png 16\n48 7 5 48x48/left_ptr_watch_0047.png 16\n48 7 5 48x48/left_ptr_watch_0048.png 16\n48 7 5 48x48/left_ptr_watch_0049.png 16\n48 7 5 48x48/left_ptr_watch_0050.png 16\n48 7 5 48x48/left_ptr_watch_0051.png 16\n48 7 5 48x48/left_ptr_watch_0052.png 16\n48 7 5 48x48/left_ptr_watch_0053.png 16\n48 7 5 48x48/left_ptr_watch_0054.png 16\n48 7 5 48x48/left_ptr_watch_0055.png 16\n48 7 5 48x48/left_ptr_watch_0056.png 16\n48 7 5 48x48/left_ptr_watch_0057.png 16\n48 7 5 48x48/left_ptr_watch_0058.png 16\n48 7 5 48x48/left_ptr_watch_0059.png 16\n48 7 5 48x48/left_ptr_watch_0060.png 16\n64 9 7 64x64/left_ptr_watch_0001.png 16\n64 9 7 64x64/left_ptr_watch_0002.png 16\n64 9 7 64x64/left_ptr_watch_0003.png 16\n64 9 7 64x64/left_ptr_watch_0004.png 16\n64 9 7 64x64/left_ptr_watch_0005.png 16\n64 9 7 64x64/left_ptr_watch_0006.png 16\n64 9 7 64x64/left_ptr_watch_0007.png 16\n64 9 7 64x64/left_ptr_watch_0008.png 16\n64 9 7 64x64/left_ptr_watch_0009.png 16\n64 9 7 64x64/left_ptr_watch_0010.png 16\n64 9 7 64x64/left_ptr_watch_0011.png 16\n64 9 7 64x64/left_ptr_watch_0012.png 16\n64 9 7 64x64/left_ptr_watch_0013.png 16\n64 9 7 64x64/left_ptr_watch_0014.png 16\n64 9 7 64x64/left_ptr_watch_0015.png 16\n64 9 7 64x64/left_ptr_watch_0016.png 16\n64 9 7 64x64/left_ptr_watch_0017.png 16\n64 9 7 64x64/left_ptr_watch_0018.png 16\n64 9 7 64x64/left_ptr_watch_0019.png 16\n64 9 7 64x64/left_ptr_watch_0020.png 16\n64 9 7 64x64/left_ptr_watch_0021.png 16\n64 9 7 64x64/left_ptr_watch_0022.png 16\n64 9 7 64x64/left_ptr_watch_0023.png 16\n64 9 7 64x64/left_ptr_watch_0024.png 16\n64 9 7 64x64/left_ptr_watch_0025.png 16\n64 9 7 64x64/left_ptr_watch_0026.png 16\n64 9 7 64x64/left_ptr_watch_0027.png 16\n64 9 7 64x64/left_ptr_watch_0028.png 16\n64 9 7 64x64/left_ptr_watch_0029.png 16\n64 9 7 64x64/left_ptr_watch_0030.png 16\n64 9 7 64x64/left_ptr_watch_0031.png 16\n64 9 7 64x64/left_ptr_watch_0032.png 16\n64 9 7 64x64/left_ptr_watch_0033.png 16\n64 9 7 64x64/left_ptr_watch_0034.png 16\n64 9 7 64x64/left_ptr_watch_0035.png 16\n64 9 7 64x64/left_ptr_watch_0036.png 16\n64 9 7 64x64/left_ptr_watch_0037.png 16\n64 9 7 64x64/left_ptr_watch_0038.png 16\n64 9 7 64x64/left_ptr_watch_0039.png 16\n64 9 7 64x64/left_ptr_watch_0040.png 16\n64 9 7 64x64/left_ptr_watch_0041.png 16\n64 9 7 64x64/left_ptr_watch_0042.png 16\n64 9 7 64x64/left_ptr_watch_0043.png 16\n64 9 7 64x64/left_ptr_watch_0044.png 16\n64 9 7 64x64/left_ptr_watch_0045.png 16\n64 9 7 64x64/left_ptr_watch_0046.png 16\n64 9 7 64x64/left_ptr_watch_0047.png 16\n64 9 7 64x64/left_ptr_watch_0048.png 16\n64 9 7 64x64/left_ptr_watch_0049.png 16\n64 9 7 64x64/left_ptr_watch_0050.png 16\n64 9 7 64x64/left_ptr_watch_0051.png 16\n64 9 7 64x64/left_ptr_watch_0052.png 16\n64 9 7 64x64/left_ptr_watch_0053.png 16\n64 9 7 64x64/left_ptr_watch_0054.png 16\n64 9 7 64x64/left_ptr_watch_0055.png 16\n64 9 7 64x64/left_ptr_watch_0056.png 16\n64 9 7 64x64/left_ptr_watch_0057.png 16\n64 9 7 64x64/left_ptr_watch_0058.png 16\n64 9 7 64x64/left_ptr_watch_0059.png 16\n64 9 7 64x64/left_ptr_watch_0060.png 16\n96 13 11 96x96/left_ptr_watch_0001.png 16\n96 13 11 96x96/left_ptr_watch_0002.png 16\n96 13 11 96x96/left_ptr_watch_0003.png 16\n96 13 11 96x96/left_ptr_watch_0004.png 16\n96 13 11 96x96/left_ptr_watch_0005.png 16\n96 13 11 96x96/left_ptr_watch_0006.png 16\n96 13 11 96x96/left_ptr_watch_0007.png 16\n96 13 11 96x96/left_ptr_watch_0008.png 16\n96 13 11 96x96/left_ptr_watch_0009.png 16\n96 13 11 96x96/left_ptr_watch_0010.png 16\n96 13 11 96x96/left_ptr_watch_0011.png 16\n96 13 11 96x96/left_ptr_watch_0012.png 16\n96 13 11 96x96/left_ptr_watch_0013.png 16\n96 13 11 96x96/left_ptr_watch_0014.png 16\n96 13 11 96x96/left_ptr_watch_0015.png 16\n96 13 11 96x96/left_ptr_watch_0016.png 16\n96 13 11 96x96/left_ptr_watch_0017.png 16\n96 13 11 96x96/left_ptr_watch_0018.png 16\n96 13 11 96x96/left_ptr_watch_0019.png 16\n96 13 11 96x96/left_ptr_watch_0020.png 16\n96 13 11 96x96/left_ptr_watch_0021.png 16\n96 13 11 96x96/left_ptr_watch_0022.png 16\n96 13 11 96x96/left_ptr_watch_0023.png 16\n96 13 11 96x96/left_ptr_watch_0024.png 16\n96 13 11 96x96/left_ptr_watch_0025.png 16\n96 13 11 96x96/left_ptr_watch_0026.png 16\n96 13 11 96x96/left_ptr_watch_0027.png 16\n96 13 11 96x96/left_ptr_watch_0028.png 16\n96 13 11 96x96/left_ptr_watch_0029.png 16\n96 13 11 96x96/left_ptr_watch_0030.png 16\n96 13 11 96x96/left_ptr_watch_0031.png 16\n96 13 11 96x96/left_ptr_watch_0032.png 16\n96 13 11 96x96/left_ptr_watch_0033.png 16\n96 13 11 96x96/left_ptr_watch_0034.png 16\n96 13 11 96x96/left_ptr_watch_0035.png 16\n96 13 11 96x96/left_ptr_watch_0036.png 16\n96 13 11 96x96/left_ptr_watch_0037.png 16\n96 13 11 96x96/left_ptr_watch_0038.png 16\n96 13 11 96x96/left_ptr_watch_0039.png 16\n96 13 11 96x96/left_ptr_watch_0040.png 16\n96 13 11 96x96/left_ptr_watch_0041.png 16\n96 13 11 96x96/left_ptr_watch_0042.png 16\n96 13 11 96x96/left_ptr_watch_0043.png 16\n96 13 11 96x96/left_ptr_watch_0044.png 16\n96 13 11 96x96/left_ptr_watch_0045.png 16\n96 13 11 96x96/left_ptr_watch_0046.png 16\n96 13 11 96x96/left_ptr_watch_0047.png 16\n96 13 11 96x96/left_ptr_watch_0048.png 16\n96 13 11 96x96/left_ptr_watch_0049.png 16\n96 13 11 96x96/left_ptr_watch_0050.png 16\n96 13 11 96x96/left_ptr_watch_0051.png 16\n96 13 11 96x96/left_ptr_watch_0052.png 16\n96 13 11 96x96/left_ptr_watch_0053.png 16\n96 13 11 96x96/left_ptr_watch_0054.png 16\n96 13 11 96x96/left_ptr_watch_0055.png 16\n96 13 11 96x96/left_ptr_watch_0056.png 16\n96 13 11 96x96/left_ptr_watch_0057.png 16\n96 13 11 96x96/left_ptr_watch_0058.png 16\n96 13 11 96x96/left_ptr_watch_0059.png 16\n96 13 11 96x96/left_ptr_watch_0060.png 16\n" ]
{ "pile_set_name": "Github" }
[ 0 ]
0
5
[ "A prospective clinical trial for assessing the efficacy of a minimally invasive protocol in patients with bisphosphonate-associated osteonecrosis of the jaws.", "\nThe objective of this study was to assess whether a minimally invasive protocol can be effective in the long-term control of necrotic areas and pain in patients suffering osteonecrosis of the jaw associated with the use of bisphosphonate drugs (BRONJ). ", "Thirty-four consecutive patients (14 male, 20 female) with BRONJ under treatment with zoledronate, pamidronate, or alendronate were enrolled. ", "All of the patients received professional oral hygiene treatment and antiseptic oral rinses, and if in pain they assumed an antibiotic therapy with amoxicillin/clavulanate potassium and metronidazole for ten days. ", "At the baseline visit, as well as at each 3-month recall, the size of the osteonecrotic lesions were measured and the pain level assessed with a visual analog scale. ", "The results from the general linear model showed a statistically significant (F = 16.1; P < .01; r(2) = 0.95) time-related decrease in the size of exposed bone areas during the nonsurgical therapy (from 12.5 ± 12.0 mm to 8.8 ± 10.3 mm). ", "This conservative protocol seems to provide successful treatment in the vast majority of patients." ]
{ "pile_set_name": "PubMed Abstracts" }
[ 0, 0.003937007874015748, 0.007042253521126761, 0, 0, 0, 0 ]
0.001568
5
[ "If robots are going work around humans, they will have to be softer and safer. ", "A Harvard team has designed a new actuator with that in mind. ", "Its movements are similar to those of a human bicep muscle, using vacuum power to automate soft rubber beams. ", "Like real muscles, the actuators are soft, shock-absorbing, and pose no danger, according to the researchers.", "\n\nThe work is led by George Whitesides, Ph.D., a Core Faculty member at Harvard’s Wyss Institute for Biologically Inspired Engineering, the Woodford L. and Ann A. Flowers University Professor of Chemistry and Chemical Biology in Harvard University’s Faculty of Arts and Sciences (FAS), and a Director of the Kavli Institute for Bionano Science and Technology at Harvard University.", "\n\nWhitesides’ team took an unconventional approach to its design, relying on vacuum to decrease the actuator’s volume and cause it to buckle. ", "While conventional engineering would consider bucking to be a mechanical instability and a point of failure, in this case the team leveraged this instability to develop VAMPs (vacuum-actuated muscle-inspired pneumatic structures). ", "Previous soft actuators rely on pressurized systems that expand in volume, but VAMPs mimic true muscle because they contract, which makes them useful in confined spaces and for a variety of purposes.", "\n\nThe actuator has soft elastomeric rubber beams filled with small, hollow chambers of air like a honeycomb. ", "By applying vacuum, the chambers collapse and the entire actuator contracts, generating movement. ", "The internal honeycomb structure can be custom tailored to enable linear, twisting, bending, or combinatorial motions.", "\n\nThe team envisions that robots built with VAMPs could be used to assist the disabled or elderly, to serve food, deliver goods, and perform other tasks related to the service industry. ", "Soft robots could also make industrial production lines safer and faster, and quality control easier to manage by enabling human operators to work in the same space.", "\n\nFail-safe design\n\nVAMPs are designed to prevent failure — even when damaged with a 2mm hole, the team showed that VAMPs will still function. ", "In the event that major damage is caused to the system, it fails safely. “", "It can’t explode, so it’s intrinsically safe,” said Whitesides. ", "Whereas other actuators powered by electricity or combustion could cause damage to humans or their surroundings, loss of vacuum pressure in VAMPs would simply render the actuator motionless.", "\n\n“These self-healing, bioinspired actuators bring us another step closer to being able to build entirely soft-bodied robots, which may help to bridge the gap between humans and robots and open entirely new application areas in medicine and beyond,” said Wyss Founding Director Donald Ingber, M.D., Ph.D., who is also the Judah Folkman Professor of Vascular Biology at Harvard Medical School and the Boston Children’s Hospital Vascular Biology Program, as well as Professor of Bioengineering at Harvard’s John A. Paulson School of Engineering and Applied Sciences (SEAS).", "\n\nThe work was reported June 1 in the journal Advanced Materials Technologies.", "\n\nHarvard’s Office of Technology Development has filed patents on this and related inventions, and the soft actuator technology has been licensed to Soft Robotics, Inc., a startup launched in 2013 and cofounded by Whitesides. ", "The company is developing robotic grasping systems toward initial applications including picking and packing in unstructured environments — for example, handling fruits and vegetables in produce distribution warehouses. ", "Longer term, this technology can be leveraged to develop products for biomedical applications.", "\n\nAbstract of Buckling Pneumatic Linear Actuators Inspired by Muscle\n\nThe mechanical features of biological muscles are difficult to reproduce completely in synthetic systems. ", "A new class of soft pneumatic structures (vacuum-actuated muscle-inspired pneumatic structures) is described that combines actuation by negative pressure (vacuum), with cooperative buckling of beams fabricated in a slab of elastomer, to achieve motion and demonstrate many features that are similar to that of mammalian muscle." ]
{ "pile_set_name": "OpenWebText2" }
[ 0, 0.016129032258064516, 0, 0, 0.023622047244094488, 0, 0, 0, 0, 0, 0, 0, 0, 0.006993006993006993, 0, 0, 0.005263157894736842, 0.014010507880910683, 0.01282051282051282, 0.008849557522123894, 0, 0, 0, 0 ]
0.003654
5
[ "#!", "/usr/bin/env bash\n\necho \"/* /index.html 200\" >> ./build/_redirects\n" ]
{ "pile_set_name": "Github" }
[ 0, 0.013888888888888888 ]
0.006944
5
[ "Anorexia in space and possible etiologies: an overview.", "\nSpace travelers experience a flight duration-dependent loss in weight and body mass while in a microgravity environment, despite the absence of increased energy expenditure. ", "Anorexia in space can lead to in-flight caloric deficits of 1330 kcal per 70 kg astronaut per day in the presence of abundant food and has a critical effect on endurance and performance. ", "Microgravity, alterations in the light-and-dark cycle, and exposure to radiation energy are the environmental stresses believed to influence appetite, food intake, and gastrointestinal function during space flight. ", "Review of data and recent studies in rodents during microgravity showed a release of stress hormones and complex neuroendocrine and physiologic changes involving the modulation of hypothalamic activity, food intake-related hormones, and cytokines. ", "The shift of dietary preference to carbohydrates, which occurs in astronauts, denotes a stress physiologic response and augments free-plasma tryptophan concentration in the brain, the precursor of the potent anorexic agent, serotonin (5-HT). ", "Alterations of other neuroendocrine mediators, including corticotropin-releasing factor (CRF), coordinate the stress response, leading to a decrease in appetite and gastrointestinal function. ", "Our laboratories used the antiorthostatic tail-suspension technique to successfully mimic some of these anorexia-related stress responses and to directly demonstrate the role of 5-HT in microgravity-related decreased food intake and delayed gastric emptying. ", "Further rodent studies from our laboratories demonstrated the adverse effect of altered dark-and-light cycles on food intake and body weight. ", "Radiation energy, through its documented effects on appetite, probably contributes to the decreased caloric intake by astronauts. ", "Modulation of hypothalamic activity, 5-HT, and CRF play a critical role in anorexia related to microgravity and circadian rhythm alterations. ", "Specific gene knockout mice (e.g., 5-HT or CRF and their respective receptors) may prove fruitful in defining the pathways by which anorexia in space occurs. ", "An understanding of these pathophysiologic problems as they relate to appetite, food intake, gastric emptying and gastrointestinal function, sufficiently to derive successful practical solutions, may lead to a quantitative enhancement of physiologic well-being and performance status, serving as a productive countermeasure in space." ]
{ "pile_set_name": "PubMed Abstracts" }
[ 0, 0, 0, 0, 0, 0, 0.005208333333333333, 0, 0, 0, 0.007042253521126761, 0.006329113924050633, 0 ]
0.001429
5
[ "Our laboratory is developing practical synthetic routes to families of chemically complex natural products that have been shown to inhibit the growth of human cancer cells. ", "In this way, we are able to prepare large numbers of related structures (analogs) for evaluation as new chemotherapeutic agents, compounds with potentially improved properties that would be otherwise unavailable for study. ", "The synthetic routes we are developing also allow us to prepare chemical probes that are useful for the identification of the cellular targets of the natural products families we study, which in many cases are not known. ", "The dual impacts of this work, providing molecules for target binding and target identification, are of potentially great potential utility in the extraordinarily challenging problem of developing new cancer therapies. ", "Indeed, many of our current front-line small-molecule therapies for cancer are natural products or were derived from natural products lead structures, and many important targets of cancer have been identified through the use of probes prepared by chemical modification of natural products. ", "Among the classes of natural products we are studying are the cortistatins, a family of steroidal alkaloids isolated from a marine sponge that potently inhibit the proliferation of human umbilical vein endothelial cells;the daphniglaucins, unique plant-derived substances that have been shown to inhibit the growth of a lymphoma-derived cell line;the trioxacarcins, bacterial fermentation products with extremely potent inhibitory properties toward growing cancer cells and known to alkylate duplex DNA;salinosporamides, isolates from a marine bacterium that target the proteasome (also targeted by Velcade(R), an approved chemotherapeutic agent for multiple myeloma), and avrainvillamides, fungal natural products shown in our laboratory to target nucleophosmin, a protein whose mutation has been implicated in ~35% of all acute myelogenous lukemias (AMLs) and a novel target for chemotherapeutic intervention." ]
{ "pile_set_name": "NIH ExPorter" }
[ 0, 0, 0, 0, 0, 0.0010976948408342481 ]
0.000183
5
[ "// Copyright 2019 The Chromium Authors. ", "All rights reserved.", "\n// Use of this source code is governed by a BSD-style license that can be\n// found in the LICENSE file.", "\n\n#ifndef THIRD_PARTY_BLINK_RENDERER_MODULES_WEBGPU_GPU_VALIDATION_ERROR_H_\n#define THIRD_PARTY_BLINK_RENDERER_MODULES_WEBGPU_GPU_VALIDATION_ERROR_H_\n\n#include \"third_party/blink/renderer/platform/bindings/script_wrappable.h\"\n#include \"third_party/blink/renderer/platform/wtf/text/wtf_string.h\"\n\nnamespace blink {\n\nclass GPUValidationError : public ScriptWrappable {\n DEFINE_WRAPPERTYPEINFO();\n\n public:\n static GPUValidationError* Create(const AtomicString& message);\n GPUValidationError(const AtomicString& message);\n\n // gpu_validation_error.idl\n const String& message() const;\n\n private:\n String message_;\n\n DISALLOW_COPY_AND_ASSIGN(GPUValidationError);\n};\n\n} // namespace blink\n\n#endif // THIRD_PARTY_BLINK_RENDERER_MODULES_WEBGPU_GPU_VALIDATION_ERROR_H_\n" ]
{ "pile_set_name": "Github" }
[ 0.025, 0, 0.019230769230769232, 0.006501950585175552 ]
0.012683
5
[ "Top 5 Apple Black Friday Deals: Save On iPhone 5c, 5s, iPad Air and More\n\nIt’s official, Winter is Here! ", "Black Friday is right around the corner, and this year its starting earlier than ever. ", "Some major retailers are stating their sales at 6pm on Thanksgiving, November 28th. ", "There’s no doubt that Apple products are the number one item on many a shoppers gift list, and major retailers are offering some pretty exciting deals for everything from iPads, iPhones, iPods to Apple TV’s.", "\n\nWe ran through Wal-Mart, Best Buy, Target and Radio Shack’s Black Friday flyers and found some pretty amazing deals on Apple products, ranging from iPads, iPhones, iPods and even Apple TV’s. ", "While every retailer offers pretty much all of Apple’s products, some have better deals than others. ", "To maximize your family time on America’s hungriest holiday, we’ve put together a list of the top deals for Apple products so that you can hurry home after the shopping frenzy and hopefully enjoy a hot cup of cider with your family.", "\n\nCheck out the list below to see where you should pick up your Apple products this Black Friday.", "\n\n1. ", "iPhone 5c 16GB\n\nSale Dates: November 28th, 6PM to December 1st\n\nPrice: $45 with new two-year contract, $75 Gift Card\n\nWal-Mart wins when it comes to a cheap iPhone 5c, offering the device for $45 with a new two-year contract through Verizon or AT&T. Wal-Mart is throwing in a $75 Gift Card with every iPhone 5c purchase, so maybe you could even grab an iPhone 5c as well, just for the heck of it. ", "The iPhone 5c runs on Apple’s iOS 7, comes in a variety of colors and touts Apple’s new A7 processor.", "\n\n2. ", "iPhone 5s 16GB\n\nSale Dates: November 28th, 6PM to December 1st\n\nPrice: $189.99 with new two-year contract, $75 Gift Card\n\nWal-Mart wins again when it comes to the iPhone 5s, offering the 16GB version of the smartphone for $189.99 with a new two-year contract. ", "A $75 Gift Card is included, which will help you load up on apps for your shiny device. ", "The iPhone 5s is the more powerful of the two recently released iPhones, and comes in three different color options.", "\n\n3. ", "iPad Air 16GB Wi-Fi\n\nSale Dates: November 28th 8PM to November 30th\n\nPrice: $479, $100 Gift Card\n\nTarget’s got the best deal on the iPad Air this Black Friday, with a price of $479 plus a $100 Gift Card. ", "Target is offering a $100 Gift Card on any iPad purchase, so you’re not limited to the iPad Air.", "\n\n4. ", "Apple TV\n\nSale Dates: November 29th 8AM through November 30th\n\nPrice: $99, $20 Gift Card\n\nWhen it comes to the Apple TV, Radio Shack’s offering is the best around. ", "Great for streaming video on your TV, the Apple TV retails for $99 and includes a $20 Gift Card. ", "At $99, the Apple TV is an affordable option to Roku and other streaming video boxes.", "\n\n5. ", "iPad Mini 16GB Wi-Fi\n\nSale Dates: November 28th, 6PM to December 1st\n\nPrice: $299 with $100 Gift Card\n\nAgain, Wal-Mart? ", "If you’re looking for an iPad mini, Wal-Mart’s your best bet. ", "The iPad mini 16GB Wi-Fi version retails for $299, and Wal-Mart is throwing in a $100 Gift Card with every purchase. ", "While it’s not the newest iPad around, it is still a bargain at $299, and the $100 Gift Card can be used to buy other gifts on your list.", "\n\nIt is also worth noting that a few retailers have smartphone trade-in programs running which could save you some money if you’re looking for an iPhone 5c or 5s. ", "Wal-Mart and Radio Shack are offering up to $300 in credit when you trade in an older smartphone, and sign a new two-year contract. ", "Keep in mind this means up to $300, with pricing depending on model and condition. ", "Radio Shack is only offering trade-ins for iPhone models, while Wal-Mart offers trad-ins on devices of all stripes. ", "To see how much your old device may be worth, check out Wal-Mart’s exchange website here.", "\n\nSo there you have it, the best deals on Apple products this Black Friday. ", "These aren’t all of the Apple deals this Black Friday, so if you’re looking for an iPod, Mac Book Pro or even a new Apple keyboard, be sure to check out our Black Friday posts listed below, where you can find each retailer’s Apple deals. ", "Happy hunting!" ]
{ "pile_set_name": "Pile-CC" }
[ 0.009523809523809525, 0, 0, 0.024154589371980676, 0.04145077720207254, 0.009900990099009901, 0.004310344827586207, 0.010309278350515464, 0, 0.005037783375314861, 0.019801980198019802, 0, 0.007692307692307693, 0, 0.008620689655172414, 0, 0.014705882352941176, 0.03125, 0, 0.018292682926829267, 0.010309278350515464, 0.023529411764705882, 0, 0.025, 0.03225806451612903, 0.017094017094017096, 0.0072992700729927005, 0, 0.015151515151515152, 0, 0.02586206896551724, 0.011235955056179775, 0.013157894736842105, 0.02100840336134454, 0 ]
0.011627
5
[ "Shri M Venkaiah Naidu, Hon’ble Vice-President of India, today laid the foundation stone of Kaushal Bhawan, the new office building of the Ministry of Skill Development and Entrepreneurship (MSDE) at Chanakyapuri, in the presence of Shri Dharmendra Pradhan, Honorable Minister of Petroleum and Natural Gas and Skill Development and Entrepreneurship and Smt Meenakshi Lekhi, Member of Parliament from New Delhi.", "\n\nImagined as a state-of-the-art structure\nwith Building Management Systems (BMS) for automatic fault detection and will\nbe constructed using a composite structure of steel in the super structure and\nRCC frame structure with diaphragm walls for earth retention around trees.", "\n\nThe building will comprise three\nbasements for parking and services and eight floors of modern offices, an\nauditorium with capacity to accommodate 250 people, a mini convention centre\nwith four meeting rooms cum training rooms of capacity for 30 to 60 people along\nwith pre-function spaces in the ground floor. ", "It will also have a cafeteria,\nfood court, yoga hall, sports facilities and gymnasiums. ", "The total built up\narea proposed is 22,000 sq meters.", "\n\nSpeaking at the foundation stone laying ceremony, Shri Venkaiah Naidu, Hon’ble Vice-President of India, said “Ram Rajya is incomplete without Gram Rajya. ", "We must focus on rural empowerment and financial inclusion. ", "Different ministries must work together, especially for skilling in rural areas. ", "Reform, perform and transform should be the mantra everyone should strive for in this journey to make India an even more prosperous economy. ", "I congratulate Shri Dharmendra Pradhan and the Ministry of Skill Development and Entrepreneurship on the shilanyas of the new building. ", "It is an important milestone in the journey to building a skilled India. ", "Today, we are not merely laying the foundation stone of a building called Kaushal Bhawan, we are laying the foundation of a strong, secure and prosperous India. ", "I am confident that the enthusiasm of the youth fuelled with skills and training will take the nation to the zenith of success.”", "\n\nShri Dharmendra Pradhan, Minister of Petroleum & Natural Gas and Skill Development and Entrepreneurship, said, “Laying of foundation stone of Kaushal Bhawan marks another landmark in the journey our Government undertook four years ago, when the Skill Development and Entrepreneurship Ministry was formed. ", "This journey would not have been possible without the support and faith of the youth who participated in the initiatives designed to take them to newer heights”.", "\n\nIt is only fitting that the Hon’ble\nVice-President Shri Venkaiah Naidu is laying the foundation stone for Kaushal\nBhawan, the land for which was allotted when he was Minister of Housing, Urban\nDevelopment and Urban Poverty Alleviation. ", "I thank him for his guidance and his\nsupport in laying foundation for a building that will inspire people to work in\na positive environment and with a positive mindset.", "\n\nTo create human resources for the world\nthrough Skill India, we also need to provide right environment for the human\nresources in the Ministry of Skill Development and Entrepreneurship. ", "I am sure\nSkill Bhawan will provide a more positive work environment to our team members\nand from here skill related initiatives will be conceived and implemented in\nsync with the growing aspirations of a New India”.", "\n\nThe\nnew permanent building of the MSDE has been designed on five founding\nprinciples:-\n\nSustainable Architecture: The\n4-star griha ratings shall be energy\nefficient and use solar energy for common area and exterior lighting. ", "Its water\nmanagement strategy shall be based on zero discharge for which a tertiary STP\nshall be employed to ensure recycling of water for flushing, air-conditioning\nand horticulture.", "\n\nContextual Architecture: The building\ncampus has large trees like banyan and neem, the building design envelopes the\ntrees and integrates them into one. ", "The big banyan tree will be the defining\nelement in the entrance experience and shall provide the much-needed cool\nmicroclimate for Delhi’s hot summers. ", "The exterior material palette will use\nnatural materials like exposed bricks, Delhi quartzite and Dholpur stone to\ncreate an architecture that belongs to Delhi.", "\n\nDemocratic Architecture: Principles\nof universal accessibility have been applied and ramps, lifts, tactile flooring,\netc, has been provisioned. ", "To fulfil contemporary aspirations, facilities like\nfood court, separate gymnasiums, yoga hall, business centre have been\nprovisioned to create a convenient work place for all. ", "Transparency in the\ninterior spaces shall be of a higher degree using glass lifts, glazed lift\nlobbies and staircases, glazed meeting rooms and glazed official’s cabins to\nensure higher safety and oversight thereby promoting democracy by design.", "\n\nCost Effective: The\ndesign shall be cost effective and space efficient to optimize the use of the\nbuilt environment.", "\n\nSafety and Security: In\nterms of fire safety, structural safety and digital safety have been inbuilt\ninto the design by providing a security hold area in the double height entrance\nfoyer that will have the security bollards and biometrics for access control." ]
{ "pile_set_name": "Pile-CC" }
[ 0.014669926650366748, 0.0036496350364963502, 0, 0, 0, 0.01282051282051282, 0, 0, 0, 0.014705882352941176, 0, 0.006211180124223602, 0, 0.013029315960912053, 0, 0.008403361344537815, 0, 0.010638297872340425, 0.004629629629629629, 0, 0.00546448087431694, 0, 0, 0.00625, 0, 0, 0, 0, 0 ]
0.003465
5
[ "/* ****************************************************************************\n * Start OHLC series code\t\t\t\t\t\t\t\t\t\t\t\t\t *\n *****************************************************************************/\n\n// 1 - Set default options\ndefaultPlotOptions.ohlc = merge(defaultPlotOptions.column, {\n\tlineWidth: 1,\n\ttooltip: {\n\t\tpointFormat: '<span style=\"color:{point.color}\">\\u25CF</span> <b> {series.name}</b><br/>' + // docs\n\t\t\t'Open: {point.open}<br/>' +\n\t\t\t'High: {point.high}<br/>' +\n\t\t\t'Low: {point.low}<br/>' +\n\t\t\t'Close: {point.close}<br/>'\n\t},\n\tstates: {\n\t\thover: {\n\t\t\tlineWidth: 3\n\t\t}\n\t},\n\tthreshold: null\n\t//upColor: undefined\n});\n\n// 2 - Create the OHLCSeries object\nvar OHLCSeries = extendClass(seriesTypes.column, {\n\ttype: 'ohlc',\n\tpointArrayMap: ['open', 'high', 'low', 'close'], // array point configs are mapped to this\n\ttoYData: function (point) { // return a plain array for speedy calculation\n\t\treturn [point.open, point.high, point.low, point.close];\n\t},\n\tpointValKey: 'high',\n\n\tpointAttrToOptions: { // mapping between SVG attributes and the corresponding options\n\t\tstroke: 'color',\n\t\t'stroke-width': 'lineWidth'\n\t},\n\tupColorProp: 'stroke',\n\n\t/**\n\t * Postprocess mapping between options and SVG attributes\n\t */\n\tgetAttribs: function () {\n\t\tseriesTypes.column.prototype.getAttribs.apply(this, arguments);\n\t\tvar series = this,\n\t\t\toptions = series.options,\n\t\t\tstateOptions = options.states,\n\t\t\tupColor = options.upColor || series.color,\n\t\t\tseriesDownPointAttr = merge(series.pointAttr),\n\t\t\tupColorProp = series.upColorProp;\n\n\t\tseriesDownPointAttr[''][upColorProp] = upColor;\n\t\tseriesDownPointAttr.hover[upColorProp] = stateOptions.hover.upColor || upColor;\n\t\tseriesDownPointAttr.select[upColorProp] = stateOptions.select.upColor || upColor;\n\n\t\teach(series.points, function (point) {\n\t\t\tif (point.open < point.close && !", "point.options.color) {\n\t\t\t\tpoint.pointAttr = seriesDownPointAttr;\n\t\t\t}\n\t\t});\n\t},\n\n\t/**\n\t * Translate data points from raw values x and y to plotX and plotY\n\t */\n\ttranslate: function () {\n\t\tvar series = this,\n\t\t\tyAxis = series.yAxis;\n\n\t\tseriesTypes.column.prototype.translate.apply(series);\n\n\t\t// do the translation\n\t\teach(series.points, function (point) {\n\t\t\t// the graphics\n\t\t\tif (point.open !", "== null) {\n\t\t\t\tpoint.plotOpen = yAxis.translate(point.open, 0, 1, 0, 1);\n\t\t\t}\n\t\t\tif (point.close !", "== null) {\n\t\t\t\tpoint.plotClose = yAxis.translate(point.close, 0, 1, 0, 1);\n\t\t\t}\n\n\t\t});\n\t},\n\n\t/**\n\t * Draw the data points\n\t */\n\tdrawPoints: function () {\n\t\tvar series = this,\n\t\t\tpoints = series.points,\n\t\t\tchart = series.chart,\n\t\t\tpointAttr,\n\t\t\tplotOpen,\n\t\t\tplotClose,\n\t\t\tcrispCorr,\n\t\t\thalfWidth,\n\t\t\tpath,\n\t\t\tgraphic,\n\t\t\tcrispX;\n\n\n\t\teach(points, function (point) {\n\t\t\tif (point.plotY !", "== UNDEFINED) {\n\n\t\t\t\tgraphic = point.graphic;\n\t\t\t\tpointAttr = point.pointAttr[point.selected ? '", "selected' : ''] || series.pointAttr[NORMAL_STATE];\n\n\t\t\t\t// crisp vector coordinates\n\t\t\t\tcrispCorr = (pointAttr['stroke-width'] % 2) / 2;\n\t\t\t\tcrispX = mathRound(point.plotX) - crispCorr; // #2596\n\t\t\t\thalfWidth = mathRound(point.shapeArgs.width / 2);\n\n\t\t\t\t// the vertical stem\n\t\t\t\tpath = [\n\t\t\t\t\t'M',\n\t\t\t\t\tcrispX, mathRound(point.yBottom),\n\t\t\t\t\t'L',\n\t\t\t\t\tcrispX, mathRound(point.plotY)\n\t\t\t\t];\n\n\t\t\t\t// open\n\t\t\t\tif (point.open !", "== null) {\n\t\t\t\t\tplotOpen = mathRound(point.plotOpen) + crispCorr;\n\t\t\t\t\tpath.push(\n\t\t\t\t\t\t'M',\n\t\t\t\t\t\tcrispX,\n\t\t\t\t\t\tplotOpen,\n\t\t\t\t\t\t'L',\n\t\t\t\t\t\tcrispX - halfWidth,\n\t\t\t\t\t\tplotOpen\n\t\t\t\t\t);\n\t\t\t\t}\n\n\t\t\t\t// close\n\t\t\t\tif (point.close !", "== null) {\n\t\t\t\t\tplotClose = mathRound(point.plotClose) + crispCorr;\n\t\t\t\t\tpath.push(\n\t\t\t\t\t\t'M',\n\t\t\t\t\t\tcrispX,\n\t\t\t\t\t\tplotClose,\n\t\t\t\t\t\t'L',\n\t\t\t\t\t\tcrispX + halfWidth,\n\t\t\t\t\t\tplotClose\n\t\t\t\t\t);\n\t\t\t\t}\n\n\t\t\t\t// create and/or update the graphic\n\t\t\t\tif (graphic) {\n\t\t\t\t\tgraphic\n\t\t\t\t\t\t.attr(pointAttr) // #3897\n\t\t\t\t\t\t.animate({ d: path });\n\t\t\t\t} else {\n\t\t\t\t\tpoint.graphic = chart.renderer.path(path)\n\t\t\t\t\t\t.attr(pointAttr)\n\t\t\t\t\t\t.add(series.group);\n\t\t\t\t}\n\n\t\t\t}\n\n\n\t\t});\n\n\t},\n\n\t/**\n\t * Disable animation\n\t */\n\tanimate: null\n\n\n});\nseriesTypes.ohlc = OHLCSeries;\n/* ****************************************************************************\n * End OHLC series code\t\t\t\t\t\t\t\t\t\t\t\t\t *\n *****************************************************************************/\n" ]
{ "pile_set_name": "Github" }
[ 0.001092896174863388, 0.0025380710659898475, 0, 0.0026041666666666665, 0.010416666666666666, 0.0047169811320754715, 0, 0.0013386880856760374 ]
0.002838
5
[ "Q:\n\nadding object to nsmutablearray with custom delegate function giving error\n\ni'm declaring an array in firstviewcontroller like \n@interface ClassesList : UIViewController\n{\n NSMutableArray *className;\n}\n@property (retain,nonatomic) NSMutableArray *className;\n\nand defining an delegate method to add an object to array in next view controller\n@protocol Createclass <NSObject>\n\n-(void)addnewObject:(NSString*)string;;\n\n@end\n\nand sending string with textfiled string\n[self.delegate addnewClass:_className.text];\n\nwhile calling this metod in firstviewcontroller it is showing error \nNSInvalidArgumentException', reason: '-[__NSArrayI addObject:]: unrecognized selector sent to instance \n\nhere is my edit\n-(void)addnewClass:(NSString *)string\n{\n [className addObject:string];\n [self.table reloadData];\n}\n\ni'm getting an dictionary from plist file which stores a list of classes\nNSArray *paths = NSSearchPathForDirectoriesInDomains(NSDocumentDirectory, NSUserDomainMask, YES);\n\nNSString *documentsDirectory = [paths objectAtIndex:0];\n\n NSString *path = [documentsDirectory stringByAppendingPathComponent:@\"classesArray.plist\"];\n\nNSDictionary *StudentDict=[[NSDictionary alloc]initWithContentsOfFile:path];\n\nNSArray *See = StudentDict[@\"alldata\"];\n\nclassName = [[NSMutableArray alloc]init];\n\nclassName= [See valueForKey:@\"class_name\"];\n\nA:\n\nIf [See valueForKey:@\"class_name\"] contains array then that is your error cause:\n[See valueForKey:@\"class_name\"];\n\nreturns immutable array.", "\nReplace:\nclassName = [[NSMutableArray alloc]init];\nclassName= [See valueForKey:@\"class_name\"];\n\nwith:\nclassName= [NSMutableArray arrayWithArray:[See valueForKey:@\"class_name\"]];\n\nand you should be fine.", "\n\n" ]
{ "pile_set_name": "StackExchange" }
[ 0.0026917900403768506, 0, 0 ]
0.000897
5
[ "Zaqatala City Stadium\n\nZaqatala \"Simurq\" is a multi-use stadium in Zaqatala, Azerbaijan. ", "It is currently used mostly for football matches and it considered as home ground of FK Simurq Zaqatala, since October 2008.", "\n\nThe stadium was built in the area Galadyuzyu in place of old arena. ", "The total area of 2.3 hectares. ", "The draft was prepared by the Bulgarian \"Dynamic Resource\", commissioned by the company \"General Constructions\". ", "The work took place from 2006 to August 2008. ", "At its construction has been spent around US$12 million. ", "The stadium holds 3500 spectators, one-sided platform for VIP-guests, calculated at 181. ", "The main field, the size of which 105x68 meters of artificial surface, several small training lawns, artificial lighting, drainage and modern electronic street system, running track, tickets for ticketing, special places for the disabled, a large heating system, water reservoir at 300 cubic meters - this is the basic indicators of the stadium «Simurq».", "\n\nThe first floor of building is only for teams and judges. ", "There are rooms for each team, doctors and massage rooms, a laboratory for doping tests, the press center. ", "On the second floor there are Internet cafes, bars, VIP-bar, but on the third floor there seats for the press.", "\n\nReferences\n\nExternal links\n1.http://dynamicsport.eu/\n\nCategory:Football venues in Azerbaijan\nCategory:Multi-purpose stadiums in Azerbaijan\nCategory:Sports venues completed in 2008\nCategory:Simurq PIK\nCategory:2008 establishments in Azerbaijan" ]
{ "pile_set_name": "Wikipedia (en)" }
[ 0, 0.008064516129032258, 0.014285714285714285, 0, 0, 0, 0, 0.011235955056179775, 0, 0, 0, 0, 0.004098360655737705 ]
0.002899
5
[ "Tag: sweaters\n\nThe only thing that gets me through winter is my love of sweaters. ", "Especially as an Angeleno, I get to layer them like crazy, and can totally get away without a coat. ", "But not all sweaters are knitted equally! ", "Here are six tips to ensure a super great fit! (", "psst, please feel free to share!).", "\n\nI’m 50 Shades of Obsessed with my Rag & Bone marled yarn black and white boxy sweater so of course I would break it out as soon as I possibly could! ", "Rag & Bone has sold out of this exact sweater, but their fall collection features some similarly cut knits you might need: charcoal metallic knit raglan ($350) and Fairisle contrast crew sweater ($450)." ]
{ "pile_set_name": "Pile-CC" }
[ 0, 0.01, 0, 0, 0, 0.006622516556291391, 0.009900990099009901 ]
0.003789
5
[ "Transumbilical single-incision laparoscopic distal pancreatectomy: primary experience and review of the English literature.", "\nSingle-incision laparoscopic surgery (SILS) may represent an improvement over conventional laparoscopic surgery, and has been applied in many surgical procedures. ", "However, for pancreatic surgery, experience is rather limited. ", "The clinical records of 11 cases in which transumbilical single-incision laparoscopic distal pancreatectomy (TUSI-LDP) was performed at our institution since June 2009 were retrospectively analyzed, and all the literatures concerning TUSI-LDP were retrospectively reviewed. ", "All the 11 patients were female. ", "The ages ranged from 20 to 73 years, with an average age of 38.0 years. ", "The average body mass index (BMI) was 22.67 (18.6-26.2). ", "Most TUSI-LDPs were successfully performed, with only one conversion to multi-incision surgery. ", "Splenic preservation was performed in six cases. ", "The mean operation time was 163.18 ± 63.18 minutes (range 95-300), and the mean intraoperative blood loss was 159.09 ± 181.02 ml (range 10-500 ml). ", "The surgical wounds healed well, with good cosmetic wound healing, and the patients were discharged from hospital in a mean of 7.45 ± 1.44 days (range 5-10). ", "Only one patient developed pancreatic leakage, which ceased spontaneously with only a drain for 61 days. ", "The parameters were comparable with those found in the English literature. ", "These recent experiences suggest that SILS in pancreatic surgery is feasible for a select group of patients with relatively small lesions and low BMI, and that, with the gradual accumulation of surgeons' experience with SILS and improvement of laparoscopic instruments, it might become a safe option for some patients." ]
{ "pile_set_name": "PubMed Abstracts" }
[ 0, 0, 0, 0.0036496350364963502, 0, 0, 0.017543859649122806, 0, 0, 0, 0, 0, 0, 0.0031446540880503146 ]
0.001738
5
[ "Cite this article as: Dehghan-nayeri N, Shali M, Vaezi A, Navabi N, Ghaffari F. Cardiac patients' beliefs about their illness and treatment: A sequential exploratory mixed methods design. *", "Med J Islam Repub Iran*. ", "2019 (18 Sep);33:98. ", "https://doi.org/10.34171/mjiri.33.98\n\nIntroduction {#s1}\n============\n\nHeart-related illnesses are amongst the most important ones worldwide. ", "Such illnesses include a wide variety of different heart-related problems such as cardiac ischemia, angina pectoris and acute coronary syndrome which all can increase the chance of getting acute myocardial infarction (AMI) ([@R1], [@R2]). ", "Coronary diseases and specially the AMI are among the main reasons of death in Iran as the 2008 reports announce 33-38% of impact rate ([@R3]). ", "Coronary diseases do also affect health, social, and economic domains of society ([@R4]).", "\n\nReducing the risk factors by reforming the patients' beliefs and concepts is known as an appropriate preventing solution. ", "Patients have less treatment regimen adherence and do not easily match with their illness, treatments, and preventional programs in case they find false beliefs about their disease ([@R5]). ", "Such beliefs are founded based on experience and other sources of information which the patient receives from the society or his relatives ([@R6]) as the disease-related information can be received from advanced health care providers (the official diagnosis and treatment systems) or the referential systems (family members, friends, and media) or even the computer-based sources ([@R7]).", "\n\nPatients' beliefs are effective in their disability or lifestyle. ", "Those who have more false beliefs about heart diseases have slower recovery pace and therefore, experience high levels of anxiety; as their mental and functional state may be affected too ([@R8], [@R9]).", "\n\nThe beliefs regarding the illness and treatment influence over the daily activities and can disturb the treatment regimen adherence. ", "Therefore, it is needed to examine such phenomenon in full details while in case of the lack of any accurate tool, this purpose is inaccessible. ", "There are numerous papers researching cardiac patients' beliefs about illness and treatment ([@R10]) but fewer ones presenting an appropriate exploratory tool. ", "Appropriate, valid, and accurate tools are needed to evaluate patients' beliefs about illness and treatment.", "\n\nQualitative studies lead to receiving deep data, and by creating a reliable and valid tool from such data, the needs of the clients, and the needed interventions can be recognized more accurately. ", "The researchers of this study started their research based on this fact to provide high-quality care, increase the life quality, and survival probability as the qualitative studies result in ([@R11]).", "\n\nMethods {#s2}\n=======\n\nThe Inductive-Sequential Mixed Methods Design was adopted to develop the questionnaire in this study which is a combination of a qualitative and a quantitative method. ", "The two methods got mixed for data mixing ([@R12]). ", "The data were transferred into SPSS software program, version 18.0 for Windows (α\\<0.05).", "\n\nStep one: qualitative study {#s2-0-1}\n---------------------------\n\nThe patients' conception of illness and treatment was probed in two parts, including the literature and related tools review and field work (semi-structured interviews with cardiac patients) for the qualitative step. ", "Many searches were done for clarifying the concept of beliefs about illness and treatment in literature review using keywords, such as beliefs, cardiac patients, tool design and psychometric tests were searched in google scholar, google, Science Direct and Pubmed databases. ", "Finally, 17 papers were selected for the study. ", "Textual content analysis was used for analyzing the papers. ", "The data analysis process was started with careful papers studying. ", "Afterward, a list of items for the final data check was prepared.", "\n\nFieldwork {#s2-0-2}\n---------\n\nThe fieldwork step consisted of inviting the cardiac patients hospitalized in CCU or post-CCU wards or the ones referring to the academic heart clinics and hospitals of Zanjan University of Medical Sciences located in Zanjan province, Iran. ", "Participants were selected on the basis of having stable medical conditions, gender, willingness for participation, and having enough communicative skills. ", "The maximum variability for age, gender, educational level, living place, and illness duration was considered. ", "The sampling was purposive. ", "Detailed semi-structured interviews were used for the data gathering.", "\n\nThe interviews were recorded. ", "The interview-guide questions like \"would you please tell us how do you feel about the cardiac disease?\", \"", "what causes cardiac diseases in your opinion?\", \"", "what can make you better?\" ", "and \"what can make your disease condition worse?\" ", "were asked. ", "The next questions were asked by considering the early responses and interview guide. ", "Probing questions like \"what do you mean?\" ", "or \"could you give more details\" were designed, if needed. ", "In the end, the participants were asked to state any missed detail or point, and subsequently, they were informed about the possibility of further interviews.", "\n\nFace-to-face interviews in a quite room in CCU wards, post-CCU wards, or heart clinics lasting for 35-45 minutes were done. ", "The data saturation appeared when the codes became repetitive, and no new category or subcategory was created after 20 interviews with the patients who had inclusion criteria. ", "The data gathering was ended with two further interviews. ", "Qualitative content analysis was used then for analyzing the data conventionally applying the Granheim and Lundman pattern ([@R13]).", "\n\nThe interviews were transcribed word by word right after the end of the talks at this stage. ", "Units were formed initially during the data analysis. ", "The whole transcribed text of each interview has been supposed to be an analyzing unit in this manuscript. ", "The conceptual units were then created which all included the participants' statements regarding different aspects of the main concept. ", "Coding was the next step in which, codes referring to a common concept were categorized in one group while the differences, similarities, and coordination made the subcategories in a steady comparison process. ", "The concealed content and concepts of the data were selected as the main categories then.", "\n\nTrustworthiness {#s2-0-3}\n---------------\n\nThe data gathered in the qualitative step were carefully checked in order to fit the Guba & Lincoln criteria of credibility, dependability, confirmability, and transferability ([@R14]). ", "Peer check and member check were used at this step. ", "To validate the combination study investigator to deal with the threats of data collection in both qualitative and quantitative study we used different participants. ", "Enough sample size capable of getting the data saturation was used in the qualitative step while a bigger size was used in the quantitative step for better validation.", "\n\nStep two: quantitative studies {#s2-0-4}\n------------------------------\n\nInductive (items extracted from participants' experiences) and deductive (codes extracted from the literature review) methods were used in the designing of the tool. ", "Interview Codes include: Simplicity of treatment, lack of community due to illness and the impossibility of returning to life before illness. ", "All the related and in-access tools were checked in the literature review in order not to lose any possible item. ", "Literature review Codes include: Permanent consumption of medication, outcome of illness, physical deterioration of the underlying cause of the disease, and cause of the disease. ", "The researcher prepared a draft of items according to the categories and sub-categories of \"beliefs about illness and treatment\" concept ([Table 1](#T1){ref-type=\"table\"}).", "\n\n###### A sample format for defining the items of the questionnaire.", "\n\n ----------------------------------------------------------------------------------------------------------------------------------------------- ------------------------------------------ ---------------------- ---------------------------------------------------------------\n Comments and experiences of the participants Code Construct Item\n I shouldn't work anymore. ", "No shopping and no commute. ", "Being in society is dangerous for me. ", "I feel like staying home. ", " Feeling danger in society lifestyle Cardiac patients should avoid being in society/social places.", "\n I know that nothing can be done for such a damaged heart. ", "Medications help me a little. ", "I have to go on to see what God will do. ", " The ineffectiveness of treatment methods Prognosis It is impossible to renew a damaged heart.", "\n My father also had a heart problem. ", "I believe when it comes to inheritance, no matter what I could have done, I would develop such a disease. ", " No effect of preventative behaviors Treatment efficiency Heart diseases are hereditary and have no way to prevent.", "\n ----------------------------------------------------------------------------------------------------------------------------------------------- ------------------------------------------ ---------------------- ---------------------------------------------------------------\n\nAll the possible items were checked, and an item-pool was formed including 58 items. ", "Each item was reviewed by the research team according to the research subject, and the final ones could be used in calculating the constructs. ", "The formal, content, and construct validity was done at this stage.", "\n\nFace validity {#s2-0-5}\n-------------\n\nThe face validity was checked in both qualitative and quantitative methods. ", "It is considered as a degree in which the questionnaire appears effective in terms of its stated aims qualitatively. ", "Ten cardiac patients hospitalized in CCU or post-CCU wards or the ones referring to the heart clinics were asked to comment on difficulty, relevancy, and ambiguous levels in face-to-face interviews to check the face validity qualitatively. ", "Afterward, the item impact method was applied to decrease or eliminate the inappropriate items and determine each item's importance as a quantitative method ([@R15]). ", "Participants were asked to score a 5-point scale, from \"with no importance\" to \"very important\" in this study. ", "The least point item impact was set to be 1.5.", "\n\nContent validity {#s2-0-6}\n----------------\n\nBoth qualitative and quantitative methods were used to check the content validity. ", "Five cardiologists with sufficient working experience in the clinical and educational settings and four expert nurses with the experience of working at CCU and post-CCU wards were asked to comment on the grammar, wording, allocation, and scaling of the items. ", "Two indices of CVI and CVR were used to check the content validity quantitatively.", "\n\nContent Validity Ratio (CVR) {#s2-0-7}\n----------------------------\n\nLawshe model was used to determine the CVR in this study ([@R16]). ", "Ten subject experts (nursing faculty members with the working experience in clinical settings, tool making, and teaching cardiac nursing subjects) were asked to check the necessity of items according to a 3-point scale ( necessary to unnecessary). ", "The results were compared with the scale of Lawshe table.", "\n\nThe minimum acceptable CVR is found to be 0.62 based on the Lawshe's table and 10 master experts' ideas ([@R16]). ", "CVR strict was used in this study meaning that only the necessary options were kept in the CVR formula.", "\n\nContent Validity Index (CVI) {#s2-0-8}\n----------------------------\n\nThe CVI was promoted initially by Waltz and Bausell ([@R17]). ", "Ten experts (the same people helped in CVR determination) were asked to check if each item fits in the Likert 4-point scale (from absolutely relevant to irrelevant).", "\n\nIn this study, the S-CVI was calculated. ", "It was done to find which item needs to be reformed or eliminated, to know whether any further item is needed to completely cover the related aspect or not and to check that all the aspects of a construct are presented in a correct number of items or not. ", "Then, it was decided to reform or to eliminate the item based on its CVI level. ", "The items with a score of 0.7 or less were candidates for elimination.", "\n\nConstruct validity {#s2-0-9}\n------------------\n\nThe construct validity is considered as a degree in determining the questionnaire's efficiency in the created constructs ([@R18], [@R19], [@R20]). ", "KMO and Bartlett sphericity tests were used before extracting the components. ", "The KMO value of 0.7 or higher was considered as the condition of sample size sufficiency in the content analysis ([@R21]). ", "550 samples were used in this study for the exploratory content analysis (EFA). ", "The minimum factor loading for keeping each item has been set as 0.5 in this study.", "\n\nReliability {#s2-0-10}\n-----------\n\nThe reliability of this study was checked using internal consistency and stability methods. ", "The minimum value of 0.6 was considered for Cronbach's alpha ([@R22]). ", "The Test-retest and Interclass Correlation Coefficient (ICC) were used to check the stability. ", "The acceptable minimum for ICC was set at 0.4 ([@R23]).", "\n\nResults {#s3}\n=======\n\nIn this study 319 codes were extracted from the analyzing phase which formed 6 categories including prognosis, prevention, contexts, treatment efficiency, mentality and lifestyle as well as 9 sub-categories including understanding the danger, attitude toward disease, attitude toward treatment, society's culture, feeling hopeless, treatment regimen ignorance, self-curing, trying to survive and physical outcomes.", "\n\nThe early items' pool was formed with 58 items after the literature review. ", "Initial codes were examined by the research team, and similar items were deleted. ", "Eventually, the pool included 58 items. ", "These 58 items were evaluated for formal (quantitative and qualitative) validity. ", "Four items were reviewed in terms of qualitative and quantitative face validity, and 5 items that received the score of 1.5 or less in the face validity were eliminated. ", "A 53-item questionnaire remained at the end. ", "Then 2 items were reviewed in the content validity check and all the possible changes were applied. ", "The quantitative survey of content validity was based on the CVR and CVI methods. ", "Fourteen items were eliminated using CVR method. ", "Three other items were also ignored after receiving a score of 0.71, 0.71 and 0.73, respectively, in the CVI method. ", "The mean content validity score (S-CVI) for all the remaining items was equal to 0.92 in the next step.", "\n\nA 36-item questionnaire based on 5-point Likert scale (4 for strongly agree and 0 for strongly disagree) was prepared from the exploratory content analysis phase. ", "The total internal consistency was found to be 0.89 in the early reliability check. ", "3 items were removed in the internal consistency check of each item. ", "EFA was used in the evaluation of the factor structure of the scale. ", "Analyzing the main components was done by varimax rotation on the 33-item questionnaire. ", "The KMO value of 0.7 or higher was considered as the condition of sample size sufficiency in the content analysis. ", "The Bartlett test with the final value of 2560.2 was also considered as significant (p\\<0.001), meaning that there was enough consistency among the items for factor analysis.", "\n\nThe KMO was obtained at 0.89. ", "Meanwhile, Bartlett\\'s test of sphericity at 7290.841 was significant at 0.0001 level and 525 degree of freedom (p=0.001).", "\n\nEigenvalues of above one and the scree plot were used to determine the number factors in the instrument. ", "Answers with factor loadings equal to or above 0.4 were considered acceptable. ", "In this stage, 3 items were removed. ", "In this regard, six factors and 30 items with eigenvalues above one and loading level equal to or above 0.4 were extracted ([Table 2](#T2){ref-type=\"table\"}). ", "The scree plot also confirmed the existence of six factors ([Fig. ", "1](#F1){ref-type=\"fig\"}). ", "After that, the analysis was performed with these six factors as the foundation. ", "The six-factor solution explained 51.7% of the total variance.", "\n\n###### Results of EFA Using Rotated Component Matrix\n\n ---- ---------------------------------------------------------------------------------------- ----------- ------- ------- ------- ------- -------\n NO Items Component \n 1 Heart-related problems at any age can shorten your life for sure. ", " 1 2 3 4 5 6\n 2 It is important to avoid any factor resulting in angina or chest pain. ", " 0.694 \n 3 It is impossible to renew a damaged heart. ", " 0.502 \n 4 It is important to be ready for a heart attack. ", " 0.525 \n 5 Heart attack is accompanied by chest pain. ", " 0.502 \n 6 Changes in lifestyle can decrease the rate of further heart attacks. ", " 0.504 \n 7 Blood cholesterol control is not needed before aging. ", " 0.569 \n 8 Diabetes treatment decreases the heart-related problems. ", " 0.592 \n 9 Stress is of the main reasons for heart diseases. ", " 0.750 \n 10 Heart diseases are hereditary and have no way to prevent. ", " 0.548 \n 11 This disease is prevalent among men mostly. ", " 0.602 \n 12 Continues use of drugs is worrying. ", " 0.511 \n 13 There is no need for drugs in case the blood pressure is normal. ", " 0.622 \n 14 Resting is the best treatment for heart diseases. ", " 0.553 \n 15 Death by heart disease is Divine destiny. ", " 0.514 \n 16 Heart attack weakens the heart wall and tears the heart rapidly. ", " 0.556 \n 17 Heart is like a battery; more work, more exhaustion. ", " 0.594 \n 18 Heart diseases are mostly coming from the patients' lifestyle. ", " 0.654 \n 19 Aging makes the heart weaker and farther from doing the right job. ", " 0.563 \n 20 Heart disease has some limitations alongside but not much important problems. ", " 0.562 \n 21 Experiencing post-heart disease life is impossible. ", " 0.600 \n 22 Heart disease is part of body deterioration. ", " 0.567 \n 23 Continues use of drugs is the only way for staying alive after getting heart problems. ", " 0.513 \n 24 Health literacy is important in heart-related problems prevention. ", " 0.522 \n 25 The sudden incidence of heart problems increases the damages. ", " 0.517 \n 26 Getting heart disease is like catching a cold. ", "It's easy to deal with it. ", " 0.592 \n 27 It is necessary to avoid any stress or mental pressure for a cardiac patient. ", " 0.509\n 28 Unhealthy and abnormal food intake is harmful to a cardiac patient. ", " 0.537\n 29 Any kind of physical activity, even walking, is dangerous for a cardiac patient. ", " 0.581\n 30 Cardiac patients should avoid being in society/social places. ", " 0.608\n ---- ---------------------------------------------------------------------------------------- ----------- ------- ------- ------- ------- -------\n\n![](", "mjiri-33-98-g001){#F1}\n\nTo evaluate the internal consistency, the Cronbach's alpha was calculated for each item and totally, which was 0.95. ", "The test-retest method was used for probing the stability. ", "The results showed that the scores of first and second tests were statistically significant (p\\<0.001 and ICC=0.83), approving the repeatability of the subscales and the questionnaire besides showing high stability ([Table 3](#T3){ref-type=\"table\"}).", "\n\n###### Intraclass Correlation Coefficient and Cronbach's Alpha Values of six Factors\n\n -------- ------------------------- ------------------ ------------------ ------------------------------------\n factor Subscale N Cronbach's Alpha Intraclass Correlation Coefficient\n 1 prognosis 5 items (1-5) 0.76 0.77\n 2 prevention 3 items (6-8) 0.83 0.85\n 3 contexts 3 items (9-11) 0.78 0.79\n 4 treatment efficiency 3 items (12-14) 0.81 0.83\n 5 mentality 12 items (15-26) 0.89 0.89\n 6 lifestyle 4 items (27-30) 0.82 0.84\n Total The whole questionnaire 30 items 0.83 0.85\n -------- ------------------------- ------------------ ------------------ ------------------------------------\n\nThe questionnaire included six subscales including prognosis, prevention, contexts, treatment efficiency, mentality, and lifestyle. ", "The scoring scale in this questionnaire was based on the five-point Likert scale denoting strongly agree score with 4 to strongly disagree score with 1. ", "The questions 1, 3, 10, 11, 12, 13, 15, 16, 21, 26, 29 and 30 are scored negatively. ", "where the line cutting score is considered to be 48 meaning that reciving the score of 48 or less shows false beliefs and the higher scores indicate true beliefs about the disease and the treatment process.", "\n\nDiscussion {#s4}\n==========\n\nEach item showed a fair relationship with one of the subscales, and the structural validity degree of mean correlation seemed to be reasonable, as indicated by the load factor of each item. ", "The statistically significant relationship between the subscales and total score of the questionnaire was observed ([@R25]).", "\n\nThe reliability of the scale was checked both by the internal consistency and retest methods, thus, strengthening the reliability test results. ", "The results showed that the questionnaire has good internal consistency. ", "However, further analysis on bigger scales may show stronger relationships. ", "This questionnaire can assess the cardiac patients' beliefs about illness and treatment in all the six domains of prognosis, prevention, contexts, treatment efficiency, mentality, and lifestyle. ", "Further surveys for using the questionnaire in bigger cultures and populations is needed.", "\n\nRecognizing and treating the disease both have very important roles in promoting health-related behaviors ([@R26]). ", "Patients' beliefs about cardiac diseases are crucial to appreciating the disease, following the treatment process and satisfaction from the process ([@R27]).", "\n\nQuestionnaires to assess specific beliefs and understanding in heart disease have also been developed and may be used in clinical settings to investigate patients' illness beliefs---for example, the York angina questionnaire ([@R28]) and the cardiac misconceptions scale ([@R29]).", "\n\nRecognizing the patients' beliefs is an important aspect of caring permanence of cardiac disease sufferers. ", "Reforming the beliefs and behaviors of mentioned patients by improving the skills, role behaving, and behavioral interventions for accommodating the patient and his family members with the cardiac disease ([@R30]). ", "Recognizing the patients' beliefs gives the chance of exploring the situation from the patients' viewpoint. ", "This psychological approach can be achieved by patients' own guidance throw their beliefs.", "\n\nConclusion {#s5}\n==========\n\nThis questionnaire, which we recommend as a mini scale, can be used both in statistical and practical studies. ", "It can be used in health systems as a scale for probing and examining the cardiac patients' beliefs about illness and treatment, and it can help for assessing each patient's educational needs. ", "On the other hand, this scale can assess the true or false beliefs; therefore, it can be used for providing the appropriate intervention at the first, second and the third level of prevention based on client needs.", "\n\nWithin the clinical context, a consideration of both types of belief in patients will lead to the more patient-centered approach and are likely to lead to more effective management and patient care.", "\n\nScreening the beliefs by the use of this tool leads the healthcare providers to select an appropriate intervention to change the patients' behaviors. ", "In other words, recognizing the beliefs can result in designing more effective interventions by the policymakers and experiencing healthier lifestyle among the patients.", "\n\nEthical considerations {#s6}\n======================\n\nThe ethical considerations mentioned by the researchers include receiving Tehran University of Medical Sciences ethics committee approval, receiving participation consent after stating the research aims from the participants, giving the right of withdrawal from the study at any stage to the participants, keeping participants' secrecy and providing the results to the participants in case of requesting for.", "\n\nAcknowledgments {#s7}\n===============\n\nThis project has been registered at Nursing and Midwifery Care Research Center of Tehran University of Medical Sciences with the registration number of 26139. ", "We do appreciate all the cardiac patients and faculty-members helped us for doing this research.", "\n\nConflict of Interests {#s8}\n=====================\n\nThe authors declare that they have no competing interests.", "\n" ]
{ "pile_set_name": "PubMed Central" }
[ 0.015873015873015872, 0, 0, 0.007042253521126761, 0.02092050209205021, 0.013888888888888888, 0.011235955056179775, 0, 0.005263157894736842, 0.005154639175257732, 0, 0.009852216748768473, 0, 0, 0.00625, 0, 0, 0.005, 0, 0.019230769230769232, 0.02247191011235955, 0, 0.007272727272727273, 0, 0, 0, 0, 0.0072992700729927005, 0, 0, 0, 0, 0, 0, 0, 0, 0, 0, 0, 0, 0, 0, 0.007936507936507936, 0, 0, 0.015151515151515152, 0, 0, 0, 0, 0, 0, 0.008658008658008658, 0, 0, 0, 0, 0.007042253521126761, 0, 0, 0, 0, 0, 0, 0, 0, 0.006535947712418301, 0, 0, 0, 0, 0, 0, 0, 0, 0, 0, 0, 0, 0.004166666666666667, 0.005988023952095809, 0, 0, 0, 0.0038461538461538464, 0, 0.014492753623188406, 0, 0, 0.02586206896551724, 0.019417475728155338, 0.03007518796992481, 0, 0, 0, 0, 0, 0.015151515151515152, 0.02564102564102564, 0.016129032258064516, 0.0125, 0, 0, 0.028169014084507043, 0.010526315789473684, 0.03636363636363636, 0, 0, 0, 0, 0, 0, 0, 0, 0.012195121951219513, 0.02040816326530612, 0.008547008547008548, 0, 0, 0, 0, 0.014492753623188406, 0, 0.008695652173913044, 0.005747126436781609, 0.0625, 0.01639344262295082, 0, 0, 0, 0, 0.015151515151515152, 0, 0, 0, 0, 0, 0, 0, 0, 0, 0, 0, 0.007462686567164179, 0, 0, 0, 0, 0, 0, 0, 0, 0, 0, 0, 0, 0, 0, 0, 0, 0, 0, 0, 0, 0, 0.007874015748031496, 0, 0.0070921985815602835, 0, 0.004, 0.0008968609865470852, 0, 0, 0, 0, 0.008064516129032258, 0, 0, 0, 0, 0, 0.00847457627118644, 0.006369426751592357, 0.0070921985815602835, 0, 0.004651162790697674, 0, 0, 0, 0, 0, 0, 0, 0, 0.0021598272138228943, 0.01, 0, 0, 0 ]
0.003461
5
[ "/* -*- Mode: Java; tab-width: 2; indent-tabs-mode: nil; c-basic-offset: 2 -*- */\n/* vim: set shiftwidth=2 tabstop=2 autoindent cindent expandtab: */\n/* Copyright 2012 Mozilla Foundation\n *\n * Licensed under the Apache License, Version 2.0 (the \"License\");\n * you may not use this file except in compliance with the License.", "\n * You may obtain a copy of the License at\n *\n * http://www.apache.org/licenses/LICENSE-2.0\n *\n * Unless required by applicable law or agreed to in writing, software\n * distributed under the License is distributed on an \"AS IS\" BASIS,\n * WITHOUT WARRANTIES OR CONDITIONS OF ANY KIND, either express or implied.", "\n * See the License for the specific language governing permissions and\n * limitations under the License.", "\n */\n/* globals VBArray */\n\n'use strict';\n\n// Checking if the typed arrays are supported\n(function checkTypedArrayCompatibility() {\n if (typeof Uint8Array !", "== 'undefined') {\n // some mobile versions do not support subarray (e.g. safari 5 / iOS)\n if (typeof Uint8Array.prototype.subarray === 'undefined') {\n Uint8Array.prototype.subarray = function subarray(start, end) {\n return new Uint8Array(this.slice(start, end));\n };\n Float32Array.prototype.subarray = function subarray(start, end) {\n return new Float32Array(this.slice(start, end));\n };\n }\n\n // some mobile version might not support Float64Array\n if (typeof Float64Array === 'undefined')\n window.", "Float64Array = Float32Array;\n\n return;\n }\n\n function subarray(start, end) {\n return new TypedArray(this.slice(start, end));\n }\n\n function setArrayOffset(array, offset) {\n if (arguments.length < 2)\n offset = 0;\n for (var i = 0, n = array.length; i < n; ++i, ++offset)\n this[offset] = array[i] & 0xFF;\n }\n\n function TypedArray(arg1) {\n var result;\n if (typeof arg1 === 'number') {\n result = [];\n for (var i = 0; i < arg1; ++i)\n result[i] = 0;\n } else if ('slice' in arg1) {\n result = arg1.slice(0);\n } else {\n result = [];\n for (var i = 0, n = arg1.length; i < n; ++i) {\n result[i] = arg1[i];\n }\n }\n\n result.subarray = subarray;\n result.buffer = result;\n result.byteLength = result.length;\n result.set = setArrayOffset;\n\n if (typeof arg1 === 'object' && arg1.buffer)\n result.buffer = arg1.buffer;\n\n return result;\n }\n\n window.", "Uint8Array = TypedArray;\n\n // we don't need support for set, byteLength for 32-bit array\n // so we can use the TypedArray as well\n window.", "Uint32Array = TypedArray;\n window.", "Int32Array = TypedArray;\n window.", "Uint16Array = TypedArray;\n window.", "Float32Array = TypedArray;\n window.", "Float64Array = TypedArray;\n})();\n\n// Object.create() ?", "\n(function checkObjectCreateCompatibility() {\n if (typeof Object.create !", "== 'undefined')\n return;\n\n Object.create = function objectCreate(proto) {\n function Constructor() {}\n Constructor.prototype = proto;\n return new Constructor();\n };\n})();\n\n// Object.defineProperty() ?", "\n(function checkObjectDefinePropertyCompatibility() {\n if (typeof Object.defineProperty !", "== 'undefined') {\n var definePropertyPossible = true;\n try {\n // some browsers (e.g. safari) cannot use defineProperty() on DOM objects\n // and thus the native version is not sufficient\n Object.defineProperty(new Image(), 'id', { value: 'test' });\n // ... another test for android gb browser for non-DOM objects\n var Test = function Test() {};\n Test.prototype = { get id() { } };\n Object.defineProperty(new Test(), 'id',\n { value: '', configurable: true, enumerable: true, writable: false });\n } catch (e) {\n definePropertyPossible = false;\n }\n if (definePropertyPossible) return;\n }\n\n Object.defineProperty = function objectDefineProperty(obj, name, def) {\n delete obj[name];\n if ('get' in def)\n obj.__defineGetter__(name, def['get']);\n if ('set' in def)\n obj.__defineSetter__(name, def['set']);\n if ('value' in def) {\n obj.__defineSetter__(name, function objectDefinePropertySetter(value) {\n this.__defineGetter__(name, function objectDefinePropertyGetter() {\n return value;\n });\n return value;\n });\n obj[name] = def.value;\n }\n };\n})();\n\n// Object.keys() ?", "\n(function checkObjectKeysCompatibility() {\n if (typeof Object.keys !", "== 'undefined')\n return;\n\n Object.keys = function objectKeys(obj) {\n var result = [];\n for (var i in obj) {\n if (obj.hasOwnProperty(i))\n result.push(i);\n }\n return result;\n };\n})();\n\n// No readAsArrayBuffer ?", "\n(function checkFileReaderReadAsArrayBuffer() {\n if (typeof FileReader === 'undefined')\n return; // FileReader is not implemented\n var frPrototype = FileReader.prototype;\n // Older versions of Firefox might not have readAsArrayBuffer\n if ('readAsArrayBuffer' in frPrototype)\n return; // readAsArrayBuffer is implemented\n Object.defineProperty(frPrototype, 'readAsArrayBuffer', {\n value: function fileReaderReadAsArrayBuffer(blob) {\n var fileReader = new FileReader();\n var originalReader = this;\n fileReader.onload = function fileReaderOnload(evt) {\n var data = evt.target.result;\n var buffer = new ArrayBuffer(data.length);\n var uint8Array = new Uint8Array(buffer);\n\n for (var i = 0, ii = data.length; i < ii; i++)\n uint8Array[i] = data.charCodeAt(i);\n\n Object.defineProperty(originalReader, 'result', {\n value: buffer,\n enumerable: true,\n writable: false,\n configurable: true\n });\n\n var event = document.createEvent('HTMLEvents');\n event.initEvent('load', false, false);\n originalReader.dispatchEvent(event);\n };\n fileReader.readAsBinaryString(blob);\n }\n });\n})();\n\n// No XMLHttpRequest.response ?", "\n(function checkXMLHttpRequestResponseCompatibility() {\n var xhrPrototype = XMLHttpRequest.prototype;\n if (!('", "overrideMimeType' in xhrPrototype)) {\n // IE10 might have response, but not overrideMimeType\n Object.defineProperty(xhrPrototype, 'overrideMimeType', {\n value: function xmlHttpRequestOverrideMimeType(mimeType) {}\n });\n }\n if ('response' in xhrPrototype ||\n 'mozResponseArrayBuffer' in xhrPrototype ||\n 'mozResponse' in xhrPrototype ||\n 'responseArrayBuffer' in xhrPrototype)\n return;\n // IE9 ?", "\n if (typeof VBArray !", "== 'undefined') {\n Object.defineProperty(xhrPrototype, 'response', {\n get: function xmlHttpRequestResponseGet() {\n return new Uint8Array(new VBArray(this.responseBody).toArray());\n }\n });\n return;\n }\n\n // other browsers\n function responseTypeSetter() {\n // will be only called to set \"arraybuffer\"\n this.overrideMimeType('text/plain; charset=x-user-defined');\n }\n if (typeof xhrPrototype.overrideMimeType === 'function') {\n Object.defineProperty(xhrPrototype, 'responseType',\n { set: responseTypeSetter });\n }\n function responseGetter() {\n var text = this.responseText;\n var i, n = text.length;\n var result = new Uint8Array(n);\n for (i = 0; i < n; ++i)\n result[i] = text.charCodeAt(i) & 0xFF;\n return result;\n }\n Object.defineProperty(xhrPrototype, 'response', { get: responseGetter });\n})();\n\n// window.btoa (base64 encode function) ?", "\n(function checkWindowBtoaCompatibility() {\n if ('btoa' in window)\n return;\n\n var digits =\n 'ABCDEFGHIJKLMNOPQRSTUVWXYZabcdefghijklmnopqrstuvwxyz0123456789+/=';\n\n window.btoa = function windowBtoa(chars) {\n var buffer = '';\n var i, n;\n for (i = 0, n = chars.length; i < n; i += 3) {\n var b1 = chars.charCodeAt(i) & 0xFF;\n var b2 = chars.charCodeAt(i + 1) & 0xFF;\n var b3 = chars.charCodeAt(i + 2) & 0xFF;\n var d1 = b1 >> 2, d2 = ((b1 & 3) << 4) | (b2 >> 4);\n var d3 = i + 1 < n ? ((", "b2 & 0xF) << 2) | (b3 >> 6) : 64;\n var d4 = i + 2 < n ? (", "b3 & 0x3F) : 64;\n buffer += (digits.charAt(d1) + digits.charAt(d2) +\n digits.charAt(d3) + digits.charAt(d4));\n }\n return buffer;\n };\n})();\n\n// Function.prototype.bind ?", "\n(function checkFunctionPrototypeBindCompatibility() {\n if (typeof Function.prototype.bind !", "== 'undefined')\n return;\n\n Function.prototype.bind = function functionPrototypeBind(obj) {\n var fn = this, headArgs = Array.prototype.slice.call(arguments, 1);\n var bound = function functionPrototypeBindBound() {\n var args = Array.prototype.concat.apply(headArgs, arguments);\n return fn.apply(obj, args);\n };\n return bound;\n };\n})();\n\n// IE9/10 text/html data URI\n(function checkDataURICompatibility() {\n if (!('", "documentMode' in document) ||\n document.documentMode !", "== 9 && document.documentMode !", "== 10)\n return;\n // overriding the src property\n var originalSrcDescriptor = Object.getOwnPropertyDescriptor(\n HTMLIFrameElement.prototype, 'src');\n Object.defineProperty(HTMLIFrameElement.prototype, 'src', {\n get: function htmlIFrameElementPrototypeSrcGet() { return this.$src; },\n set: function htmlIFrameElementPrototypeSrcSet(src) {\n this.$src = src;\n if (src.substr(0, 14) !", "= 'data:text/html') {\n originalSrcDescriptor.set.call(this, src);\n return;\n }\n // for text/html, using blank document and then\n // document's open, write, and close operations\n originalSrcDescriptor.set.call(this, 'about:blank');\n setTimeout((function htmlIFrameElementPrototypeSrcOpenWriteClose() {\n var doc = this.contentDocument;\n doc.open('text/html');\n doc.write(src.substr(src.indexOf(',') + 1));\n doc.close();\n }).bind(this), 0);\n },\n enumerable: true\n });\n})();\n\n// HTMLElement dataset property\n(function checkDatasetProperty() {\n var div = document.createElement('div');\n if ('dataset' in div)\n return; // dataset property exists\n\n Object.defineProperty(HTMLElement.prototype, 'dataset', {\n get: function() {\n if (this._dataset)\n return this._dataset;\n\n var dataset = {};\n for (var j = 0, jj = this.attributes.length; j < jj; j++) {\n var attribute = this.attributes[j];\n if (attribute.name.substring(0, 5) !", "= 'data-')\n continue;\n var key = attribute.name.substring(5).replace(/\\-([a-z])/g,\n function(all, ch) { return ch.toUpperCase(); });\n dataset[key] = attribute.value;\n }\n\n Object.defineProperty(this, '_dataset', {\n value: dataset,\n writable: false,\n enumerable: false\n });\n return dataset;\n },\n enumerable: true\n });\n})();\n\n// HTMLElement classList property\n(function checkClassListProperty() {\n var div = document.createElement('div');\n if ('classList' in div)\n return; // classList property exists\n\n function changeList(element, itemName, add, remove) {\n var s = element.className || '';\n var list = s.split(/\\s+/g);\n if (list[0] === '') list.shift();\n var index = list.indexOf(itemName);\n if (index < 0 && add)\n list.push(itemName);\n if (index >= 0 && remove)\n list.splice(index, 1);\n element.className = list.join(' ');\n }\n\n var classListPrototype = {\n add: function(name) {\n changeList(this.element, name, true, false);\n },\n remove: function(name) {\n changeList(this.element, name, false, true);\n },\n toggle: function(name) {\n changeList(this.element, name, true, true);\n }\n };\n\n Object.defineProperty(HTMLElement.prototype, 'classList', {\n get: function() {\n if (this._classList)\n return this._classList;\n\n var classList = Object.create(classListPrototype, {\n element: {\n value: this,\n writable: false,\n enumerable: true\n }\n });\n Object.defineProperty(this, '_classList', {\n value: classList,\n writable: false,\n enumerable: false\n });\n return classList;\n },\n enumerable: true\n });\n})();\n\n// Check console compatability\n(function checkConsoleCompatibility() {\n if (!('", "console' in window)) {\n window.console = {\n log: function() {},\n error: function() {},\n warn: function() {}\n };\n } else if (!('", "bind' in console.log)) {\n // native functions in IE9 might not have bind\n console.log = (function(fn) {\n return function(msg) { return fn(msg); };\n })(console.log);\n console.error = (function(fn) {\n return function(msg) { return fn(msg); };\n })(console.error);\n console.warn = (function(fn) {\n return function(msg) { return fn(msg); };\n })(console.warn);\n }\n})();\n\n// Check onclick compatibility in Opera\n(function checkOnClickCompatibility() {\n // workaround for reported Opera bug DSK-354448:\n // onclick fires on disabled buttons with opaque content\n function ignoreIfTargetDisabled(event) {\n if (isDisabled(event.target)) {\n event.stopPropagation();\n }\n }\n function isDisabled(node) {\n return node.disabled || (node.parentNode && isDisabled(node.parentNode));\n }\n if (navigator.userAgent.indexOf('Opera') !", "= -1) {\n // use browser detection since we cannot feature-check this bug\n document.addEventListener('click', ignoreIfTargetDisabled, true);\n }\n})();\n\n// Checks if navigator.language is supported\n(function checkNavigatorLanguage() {\n if ('language' in navigator)\n return;\n Object.defineProperty(navigator, 'language', {\n get: function navigatorLanguage() {\n var language = navigator.userLanguage || 'en-US';\n return language.substring(0, 2).toLowerCase() +\n language.substring(2).toUpperCase();\n },\n enumerable: true\n });\n})();\n" ]
{ "pile_set_name": "Github" }
[ 0.01238390092879257, 0.009523809523809525, 0.009523809523809525, 0, 0.0017761989342806395, 0.002136752136752137, 0.014184397163120567, 0.02857142857142857, 0.029411764705882353, 0.02857142857142857, 0.027777777777777776, 0.018518518518518517, 0, 0.004694835680751174, 0.011111111111111112, 0.004191114836546521, 0, 0, 0.0016, 0, 0.004672897196261682, 0, 0.003246753246753247, 0, 0.016129032258064516, 0.00510204081632653, 0, 0, 0, 0.03225806451612903, 0, 0.0019230769230769232, 0.0032679738562091504, 0, 0.002304147465437788, 0.0017667844522968198 ]
0.007629
5
[ "It’s no secret that Google is working on a third operating system that might someday replace both Android and Chrome. ", "Called Fuchsia, the new OS would solve the problem of Android fragmentation while simultaneously allowing Google to run it on a bunch of different types of devices, from smartphones to tablets and computers, but also smart home gadgets, smartwatches, and other gadgets. ", "Fuchsia is expected to run Android apps so that the transition from Android would happen almost seamlessly.", "\n\nBut Google isn’t just developing a new OS for the future of computing. ", "It’s also working on a new coding engine that would let developers easily deploy apps on all platforms, including Apple’s iOS. ", "If all this sounds familiar, that’s because Google released a Flutter beta in late February at MWC 2018, which developers can use to code apps for Android and iOS. ", "On Tuesday, Google announced that the first stable version of Flutter is available for everyone to try, complete with many improvements as well as new features.", "\n\nFlutter’s main advantage over other coding alternatives, at least as far as Google is concerned, is that it allows developers to create apps for both Android and iPhone platforms a lot faster than before. ", "Even though we’re not in the early days of app development for iPhone and Android, developers still favor iOS over Android in many cases, and some titles hit iPhones and iPads before they’re available in the Play store. ", "With Flutter, companies won’t have to invest in two development teams, one for each major mobile OS.", "\n\nImage source: Google\n\nA comment from Capital One’s senior director of engineering featured in Google’s press release perfectly explains why Flutter is great for the future of Android, emphasis ours:\n\nWe are excited by Flutter’s unique take on high-performing cross-platform development. ", "Our engineers have appreciated the rapid development promise, and hot reload capabilities, and over the past year we have seen tremendous progress in the framework and especially the native integration story. ", "Flutter can allow Capital One to think of features not in an ‘iOS or Android-first’ fashion, but rather in a true mobile-first model. ", "We are excited to see Flutter 1.0 and continue to be impressed with the pace of advancement and the excitement in the engineering community.", "\n\nWith Flutter, Google hopes that companies will code all their new apps at the same time and then deploy them simultaneously on iPhone and Android. ", "When the time comes, Flutter will likely also allow developers to create iOS and Fuchsia apps simultaneously. ", "Flutter should make porting Android apps to Fuchsia much easier. ", "It’s an open source project too, which means that it’ll support other coding tools out there like Kotlin and Java for Android, and Swift and Objective-C for iOS.", "\n\nAnnounced at Flutter Live, Flutter 1.0 doesn’t just allow cross-platform app development on a single codebase. ", "It also brings over several features that should make it appealing to use. ", "Flutter should allow developers to build beautiful apps that will adhere to both Google’s Material Design guidelines and Apple’s design guidelines. ", "In fact, the iOS Settings screenshot below was created in Flutter.", "\n\nImage source: Google\n\nFlutter should also be faster to use, supporting “glitch-free, jank-free graphics” at the native speeds. ", "Flutter supports a “stateful hot reload” feature, which lets developers make changes to their apps while they’re testing them without restarting or losing the state of the app.", "\n\nFlutter 1.0 brings a couple of notable new features, including “Add to App,” which will let developers update existing apps with the help of Flutter, or convert existing apps in stages. “", "Platform Views” is a new Flutter trick that will let users add UI features to existing apps with the help of Flutter, like the transparent “Go to London” button in the following screenshots:\n\nImage source: Google\n\nFlutter is already in use internally at Google on apps including Google Maps and Google Ads. ", "But various other developers have already created apps in Flutter, including Capital One, Alibaba, Groupon, Hamilton, JD.com, Philips Hue, Reflectly, and Tencent. ", "Square also announced two Flutter SDKs to bring payment support to apps via Flutter.", "\n\nFlutter 1.0 is available for download at this link, where you’ll also get more information about Google’s newest coding toolkit for mobile apps." ]
{ "pile_set_name": "OpenWebText2" }
[ 0.025423728813559324, 0.007407407407407408, 0.018691588785046728, 0.0136986301369863, 0.007874015748031496, 0.012195121951219513, 0.00625, 0.014492753623188406, 0.022727272727272728, 0, 0.010380622837370242, 0, 0.014925373134328358, 0, 0.013422818791946308, 0, 0.015384615384615385, 0.018633540372670808, 0, 0, 0.006756756756756757, 0.015151515151515152, 0, 0, 0, 0.006514657980456026, 0.03680981595092025, 0, 0.00684931506849315 ]
0.009434
5
[ "Q:\n\nC++ use lambda function as template function specialization\n\nIs it possible to use a lambda function to create an alias to a template class function? ", "Something like this:\n#include <iostream>\nusing namespace std;\n\nint calcDouble(int a) { return a * 2; }\nint calcMultiply_10(int a) { return a * 10; }\n\nstruct foo\n{\n template<void (*func)(int)>\n int generic(int value)\n {\n return func(value);\n }\n\n static auto double_10 = [this] { generic<calcDouble>(10); };\n static auto double_20 = [this] { generic<calcDouble>(20); };\n static auto multiply_10_20 = [this] { generic<calcMultiply_10>(20); };\n}\n\nint main() {\n foo f;\n\n cout << \"double_10: \" <<f.double_10() << endl;\n cout << \"double_20: \" <<f.double_20() << endl;\n return 0;\n}\n\nA:\n\nYour particular example doesn't compile, and would be dangerous if it did - capturing this by value means that as soon as the class is copied/moved the captured this will point to a wrong or invalid memory location.", "\nJust use member functions:\nauto double_10() { return generic<calcDouble>(10); }\nauto double_20() { return generic<calcDouble>(20); }\nauto multiply_10_20() { return generic<calcMultiply_10>(20); }\n\nlive example on wandbox\n\n" ]
{ "pile_set_name": "StackExchange" }
[ 0, 0, 0.004291845493562232 ]
0.001431
5
[ "---\nabstract: 'We introduce and study the combinatorial optimization problem with interaction costs (COPIC). ", "COPIC is the problem of finding two combinatorial structures, one from each of two given families, such that the sum of their independent linear costs and the interaction costs between elements of the two selected structures is minimized. ", "COPIC generalizes the quadratic assignment problem and many other well studied combinatorial optimization problems, and hence covers many real world applications. ", "We show how various topics from different areas in the literature can be formulated as special cases of COPIC. ", "The main contributions of this paper are results on the computational complexity and approximability of COPIC for different families of combinatorial structures (e.g. spanning trees, paths, matroids), and special structures of the interaction costs. ", "More specifically, we analyze the complexity if the interaction cost matrix is parameterized by its rank and if it is a diagonal matrix. ", "Also, we determine the structure of the intersection cost matrix, such that COPIC is equivalent to independently solving linear optimization problems for the two given families of combinatorial structures.'", "\naddress:\n- 'Department of Mathematics, Simon Fraser University Surrey, Central City, 250-13450 102nd AV, Surrey, British Columbia,V3T 0A3, Canada'\n- 'Institute of Discrete Mathematics, Graz University of Technology, Steyrergasse 30, 8010 Graz, Austria'\nauthor:\n- Stefan Lendl\n- Ante Ćustić\n- 'Abraham P. Punnen'\nbibliography:\n- 'bipartite-comb.bib'\ntitle: 'Combinatorial Optimization problems with interaction costs: complexity and solvable cases'\n---\n\n[^1] [^2]\n\nIntroduction\n============\n\nLet a family $\\f_1$ of subset of $[m]=\\{1,2,\\ldots ,m\\}$, and a family $\\f_2$ of subsets of $[n]=\\{1,2,\\ldots ,n\\}$ represent feasible solutions. ", "We assume that $\\f_1$ and $\\f_2$ have a compact representation of size polynomial in $m$ and $n$, respectively, although the number of feasible solutions in each family could be of size exponential in $m$ or $n$. For each element $i\\in [m]$ a linear cost $c_i$ is given. ", "Also, for each element $j\\in [n]$ a linear cost $d_j$ is given. ", "In addition, for any $(i,j)\\in [m]\\times [n]$ their *interaction cost* $q_{ij}$ is given. ", "Then the *combinatorial optimization problem with interaction costs* (COPIC) is the problem of finding $S_1\\in \\f_1$ and $S_2\\in \\f_2$ such that $$f(S_1,S_2) = \\sum_{i\\in S_1}\\sum_{j\\in S_2}q_{ij} + \\sum_{i\\in S_1}c_i + \\sum_{j\\in S_2}d_j$$ is minimized. ", "We denote an instance of this problem by ${\\textsc{COPIC}}(\\f_{1},\\f_{2},Q,c,d)$, where $Q=(q_{ij})$ is the interaction cost matrix and $c=(c_i)$, $d=(d_j)$ are linear cost vectors of the instance. ", "This generalizes the classical *linear cost combinatorial optimization problem*, where for a given family $\\f$ of subsets of $[n]$, and cost vector $w \\in \\r^{n}$ one tries to find a set $S \\in \\f$ minimizing $$\\sum_{i \\in S} w_{i}.$$ We denote an instance of this problem by ${\\textsc{LCOP}}(\\f, w)$.\n\nCOPIC generalizes many well studied combinatorial optimization problems. ", "For example, when $\\f_1$ and $\\f_2$ are respectively the family of perfect matchings in bipartite graphs $G_1$ and $G_2$ with respective edge sets $[m]$ and $[n]$, then COPIC reduces to the *bilinear assignment problem* (BAP) [@CSPB16]. ", "BAP is a generalization of the well studied *quadratic assignment problem* [@C98] and the *three-dimensional assignment problem* [@S00] and hence COPIC generalizes these problems as well. ", "When $\\f_1$ and $\\f_2$ contain all subsets of $[m]$ and $[n]$ respectively, COPIC reduces to the *bipartite unconstrained quadratic programming problem* [@Duarte2014123; @punnen2015bipartite; @Glover2015; @Karapetyan2017] studied in the literature by various authors and under different names. ", "Also, when $\\f_1$ and $\\f_2$ are feasible solutions of generalized upper bound constraints on $m$ and $n$ variables, respectively, COPIC reduces to the *bipartite quadratic assignment problem* and its variations [@CusticPunnenORL17; @Punnen2016715]. ", "Most quadratic combinatorial optimization problems can also be viewed as special cases of COPIC, including the *quadratic minimum spanning tree problem* [@AssadXu1992], *quadratic set covering problem* [@BazaraaGoode1975], *quadratic travelling salesman problem* [@JagerMolitor2008], etc. ", "Thus all the applications studied in the context of these special cases are applications of COPIC as well. ", "COPIC is a special case of bilinear integer programs [@Konno81; @Adams93; @Freire12] when $\\f_1$ and $\\f_2$ can be represented by polyhedral sets. ", "To further motivate the study of COPIC, let us consider the following illustration.", "\n\nA spanning tree of a graph needs to be constructed as a backbone network. ", "To construct a link of the tree, many different tasks need to be completed, such as digging, building conduits, laying fiber cables, lighting dark fiber etc. ", "Each of the tasks needs to be assigned to different contractors and for each link in a graph the costs vary by quotes from different contractors. ", "We want to assign the tasks to contractors and choose an appropriate tree topology so that the overall construction cost is minimized. ", "This optimization problem can be formulated as a COPIC where feasible solution sets $\\f_1$ and $\\f_2$ correspond to spanning trees and assignments of tasks to contractors, respectively.", "\n\nIn this paper we investigate various theoretical properties of COPIC. ", "To understand the impact of interaction costs in combinatorial optimization we will analyze special cases of the interaction cost matrix $Q$ for representative well-studied sets of feasible solutions. ", "Among others, the classes of interaction cost matrices $Q$ that we will be focused on in this paper include matrices of fixed rank, and diagonal matrices. ", "In the literature many quadratic-like optimization problems have been investigated in the context of fixed rank or low rank cost matrices, for example see [@Allemand2001; @Bouras96; @punnen2015bipartite; @YK95]. ", "Further, the importance of investigating COPIC with diagonal matrices is illustrated by its direct connections to problems of disjointness of combinatorial structures [@roskind1985note; @gabow1992forests; @vygen1994disjoint; @frank1988packing], packing, covering and partitioning problems [@bernath2015tractability], as well as to problems of congestion games [@ackermann2008impact; @werneck2000finding]. ", "In this paper we also pose the problem of identifying cost structures of COPIC instances that can be reduced to an instance with no interaction costs. ", "These instances are called *linearizable instances* [@CDW16; @KP11; @PK13; @CP15; @CSPB16]. ", "We suggest an approach of identifying such instances for COPIC with specific feasible solution structures along with a characterization of linearizable instances.", "\n\nThe aforementioned topics are investigated on COPIC’s with representative well-studied sets of feasible solutions $\\f_1$, $\\f_2$. To make easy future references to different sets of feasible solutions we introduce shorthand notations. ", "We denote by $2^{[n]} = \\{S \\colon S \\subseteq [n]\\}$ the unconstrained solution set. ", "Given a matroid $\\M$ we denote by $\\base(\\M)$ the set of bases of $\\M$. We denote by ${\\mathcal{U}_{n}^{k}}$ the uniform matroid, whose base set $\\base({\\mathcal{U}_{n}^{k}})$ is the set of all $k$-sets of $[n]$. Given a graph $G$, $\\M(G)$ is the graphic matroid of $G$, whose base set $\\base(\\M(G))$ is the set of all spanning trees of $G$ (or spanning forests if $G$ is not connected). ", "The set of all maximum matchings of $G$ is denoted by ${\\mathcal{PM}}(G)$. Given two terminals $s,t \\in V(G)$ the set of all $s$-$t$-paths in $G$ is denoted by ${\\mathcal{P}}_{s,t}(G)$. If $G$ is a directed graph ${\\mathcal{P}}_{s,t}(G)$ is the set of all directed $s$-$t$-paths in $G$. The set of all cuts in $G$ is denoted by ${\\mathcal{CUT}}(G)$ and ${\\mathcal{CUT}}_{s,t}(G)$ is the set of all $s$-$t$-cuts in $G$.\n\nUsing these definitions, for example, the bipartite unconstrained quadratic programming problem [@punnen2015bipartite] is denoted by ${\\textsc{COPIC}}(2^{[m]}, 2^{[n]}, Q, c, d)$.\n\nThe structure of this paper is as follows. ", "We begin by discussing the complexity of COPIC with no significant constraints on the cost structure in Section \\[sec:complexity\\]. ", "Section \\[sec:rank\\] investigates the case when the interaction cost matrix $Q$ is of fixed rank. ", "Using the methods from parametric optimization we show that in the case when one of the solution sets is unconstrained, i.e. $\\f_1=2^{[n]}$ or $\\f_2=2^{[m]}$, and linear cost optimization over the other solution set can be done in polynomial time, the problem becomes polynomially solvable. ", "Further, we show that approximability may be achieved in the case of $Q$ with fixed rank. ", "We also show that if the number of breakpoints of multi-parametric linear optimization over both sets of feasible solutions is polynomially bounded and if $Q$ has fixed rank, then COPIC can be solved in polynomial time. ", "Section \\[sec:diagonal\\] investigates COPIC’s where interaction cost matrix $Q$ is diagonal. ", "That is, there is a one-to-one relation between ground elements of $\\f_1$ and $\\f_2$ and the interaction costs appear only between the pairs of the relation. ", "The complexity of COPIC with various well-knows feasible structures (matroids, paths, matchings, cuts, etc.) ", "in the context of diagonal matrix $Q$ are considered, and their relationship to some existing results in the literature is presented. ", "Characterization of linearizable instances is investigated in Section \\[sec:lin\\]. ", "The paper is concluded with Section \\[sec:conclusion\\], where we summarize the results and suggest some problems for future work.", "\n\nGeneral complexity {#sec:complexity}\n==================\n\nBeing a generalization of many hard combinatorial optimization problems, the general COPIC is NP-hard. ", "Moreover, even for the “simple\" case with no constraints on the feasible solutions it results in the bipartite unconstrained quadratic programming problem which is NP-hard [@punnen2015bipartite]. ", "${\\textsc{COPIC}}(2^{[m]}, 2^{[n]}, Q, c, d)$ can easily be embedded into a COPIC for most sets of feasible solutions $\\f_1$ and $\\f_2$, which implies again NP-hardness. ", "However, ${\\textsc{COPIC}}(2^{[m]}, 2^{[n]}, Q, c, d)$ is known to be solvable in polynomial time if $Q \\leq 0$ and if $Q,c,d \\geq 0$ (see Punnen et al.", " [@punnen2015bipartite]). ", "This is not true anymore if $\\f_{1}, \\f_{2}$ are bases of a uniform matroid, for which we obtain the following hardness result.", "\n\n\\[compumat\\] ${\\textsc{COPIC}}(\\base({\\mathcal{U}_{m}^{k_1}}), \\base({\\mathcal{U}_{n}^{k_2}}), Q, 0, 0)$ is strongly NP-hard even if $Q \\geq 0$.\n\nWe give a reduction from a strongly NP-hard version of the cardinality constrained directed minimum cut problem.", "\n\nLet $\\vec{K}_{m,n}$ be a digraph with vertex sets $[m]$ and $[n]$ and arcs $(i,j)$ for each $i \\in [m]$ and $j \\in [n]$. The *k-card min directed cut problem* asks for a minimum cost directed cut $\\delta^{+}(S) = \\{ (i,j) \\colon i \\in S, j\n\\notin S \\}$ such that $|\\delta^{+}(S)| = k$. Using similar arguments as in [@bruglieri2004cardinality] one can show that this directed version of the minimum cut problem is strongly NP-hard. ", "Now we show how this problem can be solved in polynomial time, assuming a polynomial time algorithm for ${\\textsc{COPIC}}(\\base({\\mathcal{U}_{m}^{k_1}}),\n\\base({\\mathcal{U}_{n}^{k_2}}), Q, 0, 0)$ exists.", "\n\nFor each $k_{1} = 1,2, \\dots, m$ check if $\\frac{k}{k_{1}}$ is an integer. ", "If so set $k_{2} = \\frac{k}{k_{1}}$ and solve the instance ${\\textsc{COPIC}}(\\base({\\mathcal{U}_{m}^{k_{1}}}),\n\\base({\\mathcal{U}_{n}^{k_{2}}}), Q, 0, 0)$, obtaining solution sets $S_{1}, S_{2}$. Note that $|S_{1}| |S_{2}| = k$, i.e. it corresponds to exactly $k$ edges. ", "We can define an equivalent directed cut $\\delta^{+}(S)$ by setting $$S = S_{1} \\cup ([n] \\setminus S_{2}).$$ This way the directed cuts $\\delta^{+}(S)$ are in one to one correspondence with solutions of COPIC. ", "Doing this for all possible pairs $(k_1, k_2)$, we can obtain all possible $k$-cuts as feasible solutions of instances of ${\\textsc{COPIC}}(\\base({\\mathcal{U}_{m}^{k_{1}}}), \\base({\\mathcal{U}_{n}^{k_{2}}}), Q, 0, 0)$. Taking the minimum found via all such COPIC problems solves the $k$-card directed min cut problem in the given bipartite digraph.", "\n\nTheorem \\[compumat\\] can be used to show that ${\\textsc{COPIC}}(\\f_{1}, \\f_{2}, Q, 0,\n0)$ is NP-hard already for $Q \\geq 0$ for most sets of feasible solutions $\\f_{1}, \\f_{2}$, since in many cases cardinality constraints can be easily encoded in more complicated sets of feasible solutions.", "\n\nOn the positive side, if we fix one of the two solutions, e.g. $S_{1} \\in \\f_{1}$, then finding the corresponding optimal solution $S_{2} \\in \\f_{2}$ reduces to solving ${\\textsc{LCOP}}(\\f_{2}, h)$, where $$\\label{lcop_costs}\n h_{j}:= \\sum_{i \\in S_{1}} q_{ij} + d_{j} \\ \\ \\ \\text{ for } j \\in [n].$$ This implies that if the cardinality of one set of feasible solutions, say $\\f_1$, is polynomially bounded in the size of the input, then we can solve COPIC by solving linear instances ${\\textsc{LCOP}}(\\f_2,h)$ (where $h$ is defined by ) for all $S_1\\in\\f_1$.\n\nIf $m = O(\\log n)$ and ${\\textsc{LCOP}}(\\f_{2}, h)$ can be solved in polynomial time for any cost vector $h \\in \\r^{n}$, then ${\\textsc{COPIC}}(\\f_{1}, \\f_{2}, Q, c, d)$ can be solved in polynomial time.", "\n\nThe interaction matrix with fixed rank {#sec:rank}\n======================================\n\nIn this section we investigate the behavior of COPIC in terms of complexity and approximability when the rank of the interaction costs matrix $Q$ is fixed. ", "In the literature, many optimization problems have been investigated in the context of fixed rank or low rank cost matrices. ", "This also includes problems with quadratic-like objective functions. ", "For example, the Koopmans-Beckmann QAP [@Bouras96], the unconstrained zero-one quadratic maximization problem [@Allemand2001], bilinear programming problems [@YK95], the bipartite unconstrained quadratic programming problem [@punnen2015bipartite], among others.", "\n\nLet ${\\operatorname{rk}}(Q)$ denotes the rank of a matrix $Q$. Then ${\\operatorname{rk}}(Q)$ is at most $r$, if and only if there exist vectors $a_{p}\n= (a^{(p)}_{1}, a^{(p)}_{2}, \\dots, a^{(p)}_{m}) \\in \\r^{m}$ and $b_{p} = (b^{(p)}_{1}, b^{(p)}_{2}, \\dots, b^{(p)}_{n}) \\in \\r^{n}$ for $p=1,2,\\ldots,r$, such that $$\\label{fact}\n Q = \\sum_{p=1}^r a_{p} {b_{p}^{T}}.$$ We say that is a *factored form* of $Q$. Then ${\\textsc{COPIC}}(\\f_{1}, \\f_{2}, Q,c,d)$, where $Q$ is of fixed rank $r$, becomes minimizing $$\\label{COPICfixed}\n f(S_1,S_2)=\\sum_{p=1}^r \\left(\\sum_{i\\in S_1} a_{i}^{(p)} \\sum_{j\\in S_2}\n b_{j}^{(p)} \\right) + \\sum_{i\\in S_1} c_{i} + \\sum_{j\\in S_2} d_{j},$$ such that $S_1\\in\\f_1$, $S_2\\in\\f_2$.\n\nIn the following, we show that if $\\f_{1} (= 2^{[m]})$ is unrestricted, i.e. the set of all subsets of $[m]$, then we can generalize the results of Punnen et al.", " [@punnen2015bipartite] to solve the problem. ", "Using methods of multi-parametric optimization we also demonstrate how to tackle more-general problems where both sets of feasible solutions are constrained, if their parametric complexity is bounded.", "\n\nThese results are obtained using methods from binary and linear optimization. ", "To apply these techniques we will formulate our problem in terms of binary variables. ", "We achieve this in a straightforward way, by introducing variables $x \\in\n\\{0,1\\}^{m}, y \\in \\{0,1\\}^{n}$ in one to one correspondence with a solution $S_{1}, S_{2}$, such that $x_{i} = 1$ iff $i \\in S_{1}$, and $y_{j} = 1$ iff $j \\in S_{2}$. The vector $x$ and $y$ are respectively called the incidence vectors of $S_1$ and $S_2$. Thus the family of feasible solutions can be represented in terms of the incidence vectors, i.e. $\\f'_{1} = \\{ x \\in \\{0,1\\}^{m} \\colon S_{1} \\in \\f_{1}\\text{ and } (x_{j} = 1 \\Leftrightarrow j \\in\nS_{1}) \\}$ and $\\f'_{2} = \\{ y \\in \\{0,1\\}^{n} \\colon S_{2} \\in \\f_{2} \\text{\nand } (y_{j} = 1 \\Leftrightarrow j \\in S_{2}) \\} $. ", "Now, rank $r$ COPIC can be formulated as the binary optimization problem: $$\\begin{aligned}\n \\min\\ & \\sum_{p=1}^{r} ({a_{p}^{T}} x) ({b_{p}^{T}} y) + {c^{T}} x +\n {d^{T}} y\\\\\n \\text{s.t.\\ \\ } & x \\in \\f'_{1}\\\\\n & y \\in \\f'_{2}\\end{aligned}$$\n\nOne-sided unconstrained fixed rank COPIC\n----------------------------------------\n\nIn this section we consider the case where $\\f'_{1} = \\{0,1\\}^{m}$. Observe that COPIC is equivalent to the following linear relaxation of the constraint $x \\in \\{0,1\\}^{m}$. $$\\begin{aligned}\n \\min\\ & \\sum_{p=1}^{r} ({a_{p}^{T}} x) ({b_{p}^{T}} y) + {c^{T}} x +\n {d^{T}} y\\\\\n \\text{s.t.\\ \\ } & x \\in [0,1]^{m}\\\\\n & y \\in \\f'_{2}\\end{aligned}$$ To solve this problem, consider the multi-parametric linear program (MLP) $$\\begin{aligned}\n h_{1}(\\lambda) := \\min\\ & c^T x\\\\\n \\text{s.t.\\ } & {a_{p}^{T}} x = \\lambda_{p} \\quad \\text{for }p = 1,2,\\dots,r\\\\\n & x \\in [0,1]^{m},\\end{aligned}$$ where $\\lambda = (\\lambda_{1}, \\lambda_{2}, \\dots, \\lambda_{r}) \\in \\r^{r}$. Then $h_{1}(\\lambda)$ is a piecewise linear convex function [@gal1995param]. ", "A basis structure for MLP is a partition $(\\base, \\mathcal{L}, \\mathcal{U})$ of $[m]$, such that $|\\base| = r$. With each basic feasible solution of MLP we associate a basis structure $(\\base, \\mathcal{L}, \\mathcal{U})$, where $\\mathcal{L}$ is the index set of nonbasic variables at the lower bound $0$, $\\mathcal{U}$ is the index set of nonbasic variables at the upper bound $1$ and $\\base$ is the index set of basic variables. ", "Given a dual feasible basis structure $(\\base, \\mathcal{L}, \\mathcal{U})$, the set of values $\\lambda \\in \\r^{r}$ for which the corresponding basic solution is optimal is called the characteristic region of $(\\base, \\mathcal{L},\n\\mathcal{U})$. Since $h_{1}(\\lambda)$ is piecewise linear convex, $h_{1}(\\lambda)$ is linear if $\\lambda$ is restricted to a characteristic region associated with a dual feasible basic structure $(\\base, \\mathcal{L}, \\mathcal{U})$. We call the extreme points of the characteristic regions of $(\\base, \\mathcal{L}, \\mathcal{U})$ as breakpoints and denote the set of these breakpoints by $B_{1}$ and define $x(\\lambda)$ as the optimal basic feasible solution of $h_{1}(\\lambda)$ at each $\\lambda \\in\nB_{1}$. By the results of Punnen et al.", " [@punnen2015bipartite Theorem 3] we know that $x(\\lambda) \\in \\{0,1\\}^{m}$. Let $y(\\lambda) \\in \\f_{2}$ be an optimal solution to our instance of COPIC when $x$ is fixed at $x(\\lambda)$. In this case COPIC reduces to $$\\begin{aligned}\n \\min\\ & \\left(\\sum_{p=1}^{r} ({a_{p}^{T}} x(\\lambda)) {b_{p}^{T}} +\n {d^{T}}\\right) y\\\\\n \\text{s.t.\\ \\ } & y \\in \\f_{2}\\end{aligned}$$ which is an instance of ${\\textsc{LCOP}}(\\f_{2}, f)$, with $f = \\sum_{p=1}^{r}\n({a_{p}^{T}} x(\\lambda)) b_{p} + d$. This allows us to calculate $y(\\lambda)$ in $O(\\max\\{r m n, T(\\f_{2}) \\})$ time, using an $T(\\f_{2})$-time algorithm for ${\\textsc{LCOP}}(\\f_{2}, f)$, for each $\\lambda \\in B_{1}$.\n\n\\[thm:fixrank-paramsol\\] There exists an optimal solution to ${\\textsc{COPIC}}(2^{[m]}, \\f_{2}, Q, c, d)$ with ${\\operatorname{rk}}(Q) = r$ amongst the solutions $\\{(x(\\lambda), y(\\lambda)) \\colon\n \\lambda \\in B_{1}\n\\}$.\n\nRank $r$ COPIC is equivalent to solving the bilinear program $$\\begin{aligned}\n \\min\\ & \\sum_{p=1}^{r} \\lambda_{p} ({b_{p}^{T}} y) + {c^{T}} x +\n {d^{T}} y\\\\\n \\text{s.t.\\ \\ } & {a_{p}^{T}} x = \\lambda_{p} \\quad p = 1, 2, \\dots, r\\\\\n & x \\in [0,1]^{m}, y \\in \\f_{2}, \\lambda \\in \\r^{r}.", "\n \\end{aligned}$$ Let $h(\\lambda)$ be the optimal value if $\\lambda$ is fixed, then we can decompose $h(\\lambda)$ into $h(\\lambda) = h_{1}(\\lambda) + h_{2}(\\lambda)$, where $$\\begin{aligned}\n h_{2}(\\lambda) = \\min\\ & \\sum_{p=1}^{r} \\lambda_{p} ({b_{p}^{T}} y) +\n {d^{T}} y \\\\\n \\text{s.t.\\ \\ } & y \\in \\f_{2}.", "\n \\end{aligned}$$ So rank $r$ COPIC can be reduced to solving $$\\min_{\\lambda \\in \\r^{r}} h(\\lambda).$$ We already argued above that $h_{1}(\\lambda)$ is a piecewise linear convex function in $\\lambda$. Using the fact that $h_{2}(\\lambda)$ is the pointwise minimum of linear functions, we obtain that $h_{2}(\\lambda)$ is a piecewise linear concave function in $\\lambda$ [@boyd2004convex]. ", "This implies that $h_{1}(\\lambda)$ is linear, if $\\lambda$ is restricted to any characteristic region of $h_{1}(\\lambda)$ and thus $h(\\lambda)$ is concave on each of these regions. ", "This implies that the minimum of $h(\\lambda)$ is attained at a breakpoint of $h_{1}(\\lambda)$, which implies the result since $B_{1}$ is defined as the set of these breakpoints.", "\n\nAnalogously to Punnen et al.", " [@punnen2015bipartite], we can use Theorem \\[thm:fixrank-paramsol\\] to solve rank $r$ COPIC using the following approach.", "\n\n1. ", " Compute the set $\\bar{S}$ of all optimal basic feasible solutions corresponding to the extreme points of the characteristic region of a dual feasible basis structure $(\\base, \\mathcal{L},\n \\mathcal{U})$ of $h_{1}(\\lambda)$.\n\n2. ", " For each $x \\in \\bar{S}$ compute the best $y \\in \\f_{2}$ by solving ${\\textsc{LCOP}}(\\f_{2}, f)$, with $f = \\sum_{p=1}^{r} ({a_{p}^{T}} x) b_{p} + d$.\n\n3. ", " Output the best pair $(x,y)$ with minimum total cost found in the last step.", "\n\nBy the arguments above it follows that this algorithm finds an optimal solution. ", "There are $\\binom{m}{r}$ choices for $\\base$ and each of them gives a unique allocation of non-basic variables to $\\mathcal{L}$ and $\\mathcal{U}$ (uniqueness following from non-degeneracy which can be achieved by appropriate perturbation of the cost vector). ", "The basis inverse can be obtained in $O(r^{3})$ time and given this inverse $\\mathcal{L}$ and $\\mathcal{U}$ can be identified in $O(m r^{3})$ time, such that $(\\base, \\mathcal{L}, \\mathcal{U})$ is dual feasible. ", "This implies that the set of dual feasible basis structures is bounded by $\\binom{m}{r}$ and can be calculated in $O(\\binom{m}{r} (r^{3} + m\nr^{2}))$ time. ", "By [@punnen2015bipartite Theorem 3], we know that the number of extreme points associated with $(\\base, \\mathcal{L}, \\mathcal{U})$ is bounded by $2^{r}$ and how to calculate the optimal solution of $h_{1}(\\lambda)$ for $\\lambda$ fixed at these extreme points without explicitly calculating $\\lambda$. This allows us to compute $\\bar{S}$ in $O(\\binom{m}{r} 2^{r} m)$ time. ", "Fixing $x \\in \\bar{S}$, the best corresponding solution $y$ can be computed in $O(\\max\\{m r n, {\\operatorname{T}}(\\f_{2})\\})$ time. ", "Summarizing this gives the following result.", "\n\nIf ${\\operatorname{rk}}(Q) = r$ and there is a ${\\operatorname{T}}(\\f_{2})$-time algorithm for ${\\textsc{LCOP}}(\\f_{2}, f)$ for every $f \\in \\r^{n}$, then ${\\textsc{COPIC}}(2^{[m]}, \\f_{2}, Q, c,\n d)$ can be solved in $O(\\binom{m}{r} 2^{r} \\max\\{m r n, {\\operatorname{T}}(\\f_{2})\\})$ time.", "\n\nAn identical approach works for sets of feasible solutions $\\f_{1}$, for which we can solve the linear cost minimization problem, extended by a constant number of side constraints of the form ${a_{p}^{T}} x = \\lambda_{p}$ and the number of breakpoints (in $\\lambda$) is polynomially bounded. ", "But this does not help for most non-continuous problems, because already for the bases of a uniform matroid this corresponds to a partition problem.", "\n\nWe can now use Theorem \\[thm:fixrank-paramsol\\] to obtain approximation algorithms for rank $r$ COPIC based on approximation algorithms for the linear problem with feasible solutions in $\\f_{2}$.\n\n\\[thm:fixrank-approx\\] ${\\textsc{COPIC}}(2^{[m]}, \\f_{2}, Q, c, d)$ such that ${\\textsc{LCOP}}(\\f_{2}, f)$ admits a ${\\operatorname{T}}(\\f_{2})$ time $\\alpha$-approximation algorithm for arbitrary $f \\in \\r^{n}$, has a $O(\\binom{m}{r} 2^{r} \\max\\{m r n, {\\operatorname{T}}(\\f_{2})\\})$ time $\\alpha$-approximation algorithm.", "\n\nBy Theorem \\[thm:fixrank-paramsol\\] there exists an optimal solution $$(x^{*}, y^{*}) = (x(\\lambda^{*}), y(\\lambda^{*})) \\in \\{(x(\\lambda),\n y(\\lambda)) \\colon \\lambda \\in B_{1}\\}.$$ By the method above we will in some iteration find $x^{*}$ as one of the extreme points of a characteristic region of $h_{1}(\\lambda)$. Then calculating $y^{*}$ is equivalent to solving ${\\textsc{LCOP}}(\\f_{2}, f)$ with $f = \\sum_{p=1}^{r}\n ({a_{p}^{T}} x^{*}) b_{p} + d$. Instead of solving this problem to optimality we can use our $\\alpha$-approximation algorithm and obtain a solution $\\tilde{y} \\in\n \\f_{2}$ such that $$\\tilde{h}_{2} := \\sum_{p=1}^{r} ({a_{p}^{T}} x^{*}) ({b_{p}^{T}}\n \\tilde{y}) + {d^{T}} \\tilde{y}\n \\leq \\alpha h_{2}(\\lambda^{*}).$$ Altogether for our found solution $(x^{*}, \\tilde{y})$ we obtain a bound on the objective value given by $$h_{1}(\\lambda^{*}) + \\tilde{h}_{2} \\leq h_{1}(\\lambda^{*}) + \\alpha\n h_{2}(\\lambda^{*}) \\leq \\alpha h(\\lambda^{*}).$$\n\nFor the more general case of rank $r$ COPIC, where both $\\f_{1}$ and $\\f_{2}$ are constrained, we can still obtain a FPTAS based on the results of Mittal and Schulz [@mittal2013fptas], for a restricted class of objective functions.", "\n\nConsider the separable bi-linear programming problem $$\\begin{aligned}\n \\min\\ & \\sum_{p=1}^{r} ({a_{p}^{T}} x) ({b_{p}^{T}} y) + {c^{T}} x + {d^{T}} y\\\\\n \\text{s.t.\\ \\ } & x \\in P_{1}\\\\\n & y \\in P_{2}\n \\end{aligned}$$ where $P_{1}, P_{2}$ are polytopes, completely given in terms of linear inequalities or by a polynomial time separation oracle, for fixed $r$. Then the problem admits a FPTAS giving a solution that is an extreme point of $P_{1}, P_{2}$, if ${c^{T}} x > 0, {d^{T}} y > 0$ and ${a_{p}^{T}} x > 0, {b_{p}^{T}} y > 0$ for $p=1,2,\\dots,r$ over the polytopes $P_{1}, P_{2}$.\n\nThis result directly implies a FPTAS for ${\\textsc{COPIC}}(\\f_{1}, \\f_{2}, Q, c, d)$, if ${\\operatorname{rk}}(Q) = r$ and the sets $\\f_{1},\n\\f_{2}$ can be represented as polytopes of polynomial size or polytopes with a polynomial time separation oracle. ", "This is for instance the case for matroid constraints. ", "See [@mittal2013fptas] for a detailed description of the FPTAS.", "\n\nGeneral fixed rank COPIC via multi-parametric optimization\n----------------------------------------------------------\n\nTo solve fixed rank COPIC when both sets of feasible solutions $\\f_{1}$ and $\\f_{2}$ are constrained, we again apply methods from parametric optimization. ", "Since in many cases additional linear constraints of the form ${a_{p}^{T}} x\n= \\lambda_{p}$ imply NP-hardness, we cannot follow an identical approach as above. ", "Instead, we analyze and solve multi-parametric objective versions for both sets of feasible solutions directly. ", "Given linear cost vectors $a_{1}, a_{2}, \\dots\na_{r} \\in \\r^{n}$ and $c \\in \\r^{n}$ in addition to a set of feasible solutions $\\f \\subseteq \\{0,1\\}^{n}$, the problem of finding optimal solutions to $$\\begin{aligned}\n \\min\\ & \\sum_{p=1}^{r} \\mu_p ( {a_{p}^{T}} x) +\n {c^{T}} x \\\\\n \\text{s.t. } & ", "x \\in \\f \\end{aligned}$$ for all possible values of $\\mu \\in \\r^{r}$ is called multi-parametric linear optimization over $\\f$. In this section the number of vectors $a$ will always be fixed to $r$. For every fixed $\\mu \\in \\r^{r}$ this is equivalent to solving an instance of ${\\textsc{LCOP}}(\\f, h)$ for $h = \\sum_{p=1}^{r} \\mu_{p} a_{p} +\nc$. We denote this problem by ${\\textsc{MPLCOP}}(\\f, a, c)(\\mu)$.\n\nIt is well known that ${\\textsc{MPLCOP}}(\\f, a, c)(\\mu)$ is a piecewise-linear concave function in $\\mu$ on $\\r^{r}$. For such a function the parameter space $\\r^{r}$ can be partitioned into regions $M_{1}, M_{2}, \\dots, M_{l}$, such that in each of these regions the optimal objective value is linear in $\\mu$ and for each $i =\n1,2, \\dots, l$ there exists a solution $x_{i} \\in \\f$ that achieves this value on the whole region $M_{i}$. The smallest needed number $l$ of such regions is called the parametric complexity of ${\\textsc{MPLCOP}}(\\f, a, c)$. Bökler and Mutzel [@bokler2015output] showed that there is an output-sensitive algorithm for ${\\textsc{MPLCOP}}(\\f, a, c)$ to obtain all the solutions $x_{1}, x_{2}, \\dots, x_{l}$ with running time $O(\\operatorname{poly}(n, m, l^{r}))$, if ${\\textsc{LCOP}}(\\f, h)$ can be solved in polynomial time.", "\n\nGiven an instance of fixed rank COPIC $$\\begin{aligned}\n \\min\\ & \\sum_{p=1}^{r} ({a_{p}^{T}} x) ({b_{p}^{T}} y) + {c^{T}} x +\n {d^{T}} y\\\\\n \\text{s.t.\\ \\ } & x \\in \\f'_{1}\\\\\n & y \\in \\f'_{2}\\end{aligned}$$ and its optimal solution $(x^{*}, y^{*}) \\in \\f'_{1} \\times \\f'_{2}$, we observe that $x^{*}$ is an optimal solution to ${\\textsc{MPLCOP}}(\\f'_{1}, a, c)(\\mu^{*})$ for $\\mu_{p}^{*} = {b_{p}^{T}} y^{*}$ and $y^{*}$ is an optimal solution to ${\\textsc{MPLCOP}}(\\f'_{2}, b, d)(\\lambda^{*})$ for $\\lambda_{p}^{*} = {a_{p}^{T}}\nx^{*}$. This yields the following approach for solving such instances of COPIC:\n\n1. ", " Obtain optimal solutions $x_{1}, x_{2}, \\dots, x_{l_{1}}$ for all possible parameter values $\\mu$ of\n\n ${\\textsc{MPLCOP}}(\\f'_{1}, a, c)(\\mu)$ and $y_{1},\n y_{2}, \\dots, y_{l_{2}}$ for all possible parameter values $\\lambda$ of\n\n ${\\textsc{MPLCOP}}(\\f'_{2}, b, d)$.\n\n2. ", " Calculate their corresponding parameter values $\\lambda^{(1)}, \\lambda^{(2)}, \\dots, \\lambda^{(l_{1})}$ and $\\mu^{(1)},\n \\mu^{(2)}, \\dots, \\mu^{(l_{1})}$ as $\\lambda^{(i)}_{p} = {a_{p}^{T}} x_{i}$ and $\\mu^{(j)}_{p} = {b_{p}^{T}} y_{j}$.\n\n3. ", " For each pair $(x_{i}, y_{j})$ check if $x_{i}$ is optimal for ${\\textsc{LCOP}}(\\f'_{1}, a, c)(\\mu^{(j)})$ and $y_{j}$ is optimal for ${\\textsc{LCOP}}(\\f'_{2}, b, d)(\\lambda^{(i)})$.\n\n4. ", " Among all the pairs that fulfill conditions in (3), take the one with minimum objective value for our instance of COPIC.", "\n\nTo guarantee that this method finds the optimal solution $(x^{*}, y^{*})$ the two given instances of ${\\textsc{MPLCOP}}$ must be non-degenerate. ", "This can be guaranteed by appropriate perturbations of the cost vectors. ", "Based on the algorithm of Bökler and Mutzel [@bokler2015output] we obtain the following result.", "\n\nLet $l_{1}, l_{2}$ be the parametric complexity of ${\\textsc{MPLCOP}}(\\f'_{1}, a, c),\n {\\textsc{MPLCOP}}(\\f'_{2}, b, d)$ respectively, and ${\\operatorname{rk}}(Q) = r$ is a constant. ", "If both ${\\textsc{LCOP}}(\\f_{1}, h)$ and ${\\textsc{LCOP}}(\\f_{2}, h)$ can be solved in polynomial time for arbitrary linear cost vectors $h$, then ${\\textsc{COPIC}}(\\f_{1}, \\f_{2}, Q, c, d)$ can be solved in $O(\\operatorname{poly}(n, m, l_{1}^{r}, l_{2}^{r}))$ time.", "\n\nIf $\\f$ is the set of bases of a matroid, Ganley et al.", " [@ganley1995multi] showed that the parametric complexity of ${\\textsc{MPLCOP}}(\\f, a,\nc)$ for arbitrary $a, c$ over the whole parameter region $\\r^{r}$ is polynomially bounded, if $r$ is fixed.", "\n\n\\[thmganley\\] If $\\f = \\base(\\M)$ is the set of bases of a matroid $\\M$ with $n$ elements, the parametric complexity of ${\\textsc{MPLCOP}}(\\f, a, c)$ for arbitrary $a$ and $c$ is bounded by $O(n^{2r-2})$.\n\nThis implies a polynomial time algorithm for ${\\textsc{COPIC}}(\\base(\\M_{1}),\n\\base(\\M_{2}), Q, c, d)$ for arbitrary matroids $\\M_{1}, \\M_{2}$ and fixed rank matrix $Q$. For rank 1 problems Eppstein [@eppstein1998geometric] gives stronger bounds for the parametric complexity. ", "It is conjectured that for higher rank matrices and bases of matroids as feasible solution, stronger bounds than the one given in Theorem \\[thmganley\\] can be achieved. ", "For other types of feasible solutions, like paths or bipartite matchings, this approach does not yield polynomial time algorithms.", "\n\nAnother set of feasible solutions for which this approach yields a polynomial time algorithm are global cuts in a graph. ", "Karger [@karger2016enumerating] currently gives the best bound for the parametric complexity of cuts and obtains several other related sets of feasible solutions with similar polynomial bounds. ", "In this case we are even able to bound the number of distinct cuts that can become optimal, instead of just the parametric complexity, so degeneracy is not even an issue here.", "\n\nIf $\\f = {\\mathcal{CUT}}(G)$ the number of cuts that can become optimal in ${\\textsc{MPLCOP}}(\\f, a,\n c)$ for arbitrary $a$ and $c$ over all choices of $\\mu$ is bounded by $O(n^{r+1})$.\n\nAlready for the case $r=1$ subexponential lower bounds for the parametric complexity of ${\\textsc{MPLCOP}}(\\f, a, c)$ for paths ($\\f =\n{\\mathcal{P}}_{s,t}(G)$) and matchings ($\\f = {\\mathcal{PM}}(G)$) in a graph $G$ are known (Gusfield [@gusfield1980sensitivity], Carstensen [@carstensen1984parametric]). ", "However, in the setting of smoothed analysis Brunsch and Röglin [@brunsch2015improved] showed that the parametric complexity of ${\\textsc{MPLCOP}}(\\f, a, c)$ is bounded by $O(n^{2r} \\phi^{r})$ for every perturbation parameter $\\phi \\geq 1$, all costs $a,c$ and arbitrary sets of feasible solutions $\\f$.\n\nDiagonal interaction matrix {#sec:diagonal}\n===========================\n\nIn this section we analyze the special case of COPIC, refereed to as *diagonal COPIC*, where for a given vector $a \\in \\r^{n}$ the matrix $Q = (q_{ij})$ is given as the diagonal $n\\times n$ matrix $$q_{ij} = \\begin{cases}\n a_{i} & \\text{if } i=j\\\\\n 0 & \\text{otherwise}.", "\n\\end{cases}$$ This results in finding solutions $S_{1} \\in \\f_{1} \\subseteq \\{0,1\\}^{n}$ and $S_{2} \\in \\f_{2} \\subseteq \\{0,1\\}^{n}$ that minimize the objective function $$f(S_1,S_2) = \\sum_{i\\in S_1 \\cap S_{2}} a_{i} + \\sum_{i\\in S_1}c_i +\n\\sum_{j\\in S_2}d_j.$$ Such instances are denoted by ${\\textsc{COPIC}}(\\f_{1}, \\f_{2}, {\\operatorname{diag}}(a), c, d)$.\n\nAlready this very restricted version of COPIC includes many well-studied problems of combinatorial optimization. ", "For example, problems that ask for two disjoint combinatorial structures among an element set can all be handled by solving COPIC with identity interaction matrix $Q = I$ and $c=d=0$. This includes the disjoint spanning tree problem [@roskind1985note], disjoint matroid base problem [@gabow1992forests], disjoint path problems [@vygen1994disjoint; @frank1988packing], disjoint matchings problem [@Frieze83] and many others. ", "Bernáth and Király [@bernath2015tractability] analyzed the computational complexity of many combinations of different packing, covering and partitioning problems on graphs and matroids. ", "It is easy to model all of these problems as instances of diagonal COPIC. ", "The hardness results for packing problems in this paper directly imply NP-hardness results for diagonal COPIC with $Q=I$ and $c=d=0$ for several classes of problems. ", "In this section we further investigate complexity of diagonal COPIC. ", "Some results investigated in this section are summarized in Table \\[tab:diagonal-summary\\].", "\n\n[l | X X X X X ]{} $\\f_{1}\\ \\setminus\\ \\f_{2}$ & $2^{[n]}$ & $\\base({\\mathcal{U}_{n}^{k_{2}}})$ & $\\base(\\M_{2})$ & ${\\mathcal{PM}}(G)$ & ${\\mathcal{P}}_{s_{2}, t_{2}}(G)$\\\n$2^{[n]}$ & $O(n)$ & P & P & P & P\\\n$\\base({\\mathcal{U}_{n}^{k_{1}}})$ & & P & P & open & open\\\n$\\base(\\M_{1})$ & & & P & open & NP-hard\\\n${\\mathcal{PM}}(G)$ & & & & NP-hard [@Frieze83] & open\\\n${\\mathcal{P}}_{s_{1}, t_{1}}(G)$ & & & & & NP-hard\\\n\nUnconstrained feasible sets {#sec:diagonal-unconstrained}\n---------------------------\n\nWe start by considering diagonal COPIC with unconstrained feasible sets.", "\n\n\\[uncondiag\\] ${\\textsc{COPIC}}(2^{[n]}, 2^{[n]}, {\\operatorname{diag}}(a), c, d)$ can be solved in linear time.", "\n\nFor each $e \\in [n]$ independently we have four different choices:\n\n- $e \\notin S_{1} \\cup S_{2}$: this contributes $0$ to $f(S_{1}, S_{2})$\n\n- $e \\in S_{1}, i \\notin S_{2}$: this contributes $c_{e}$ to $f(S_{1}, S_{2})$\n\n- $e \\notin S_{1}, i \\in S_{2}$: this contributes $d_{e}$ to $f(S_{1}, S_{2})$\n\n- $e \\in S_{1} \\cap S_{2}$: this contributes $a_{e} + c_{e} + d_{e}$ to $f(S_{1}, S_{2})$\n\nSo for each $e \\in [n]$ we can independently find $\\min\\{ 0, c_{e}, d_{e},\n a_{e} + c_{e} + d_{e} \\}$ and select the corresponding solution accordingly. ", "This can be done in constant time for each $e \\in [n]$, so the overall running time is $O(n)$.\n\nThe result of Theorem \\[uncondiag\\] can be generalized. ", "Using a straightforward dynamic programming approach, ${\\textsc{COPIC}}(\\f_1,\\f_2,Q,c,d)$ with matrix $Q$ of bandwidth $O(\\log n)$ can be solved in polynomial time. ", "This result is presented as Theorem 2.11 in the PhD thesis of Sripratak [@Sripratak].", "\n\n${\\textsc{COPIC}}(\\f, 2^{[n]}, {\\operatorname{diag}}(a), c, d)$ can be solved by solving ${\\textsc{LCOP}}(\\f, f)$, where $f_{i} = \\min\\{c_{i} + d_{i} + a_{i}, c_{i}\\} -\n \\min\\{d_{i}, 0\\}$ for each $i \\in [n]$.\n\nFor each $i \\in [n]$ we can determine independently if it should be included in $S_{2}$, given that it is included in $S_{1}$ or not. ", "The cost for an element $i\\in [n]$ is therefore uniquely determined as $f_{1}(i) =\n \\min\\{ c_{i} + d_{i} + a_{i}, c_{i} \\}$, if $i \\in S_{1}$ and as $f_{2}(i) = \n \\min\\{ d_{i}, 0 \\}$ if $i \\notin S_{1}$. So we can determine an optimal solution $S_{1}$ by solving the minimization problem over $\\f$ for the linear cost function $f_{1} -\n f_{2}$. The corresponding corresponding optimal $S_{2}$ can be easily obtained in $O(n)$ time.", "\n\nUniform and Partition Matroids {#sec:diagonal-uniformmat}\n------------------------------\n\nIn the following two subsections we investigate diagonal COPIC where $\\f_1$ and $\\f_2$ correspond to bases of different types of matroids. ", "For bases of uniform and partition matroids, which are defined by standard cardinality constraints, the main insight is that we can solve our problem in polynomial time using matching algorithms.", "\n\nGiven a graph $G=(V,E)$ and a function $b \\colon V {\\rightarrow}2^{\\NN}$ an edge set $M\n\\subseteq E$ is a $b$-factor, if $|M \\cap \\delta(v)| \\in b(v)$ for each $v \\in\nV$. If $b(v) = \\{k\\}$ for some integer $k \\in \\NN$ we simply write $b(v) =\nk$. Given an additional cost function $c \\colon E {\\rightarrow}\\r$ a minimum cost $b$-factor can be found in polynomial time, if all the $b$-values $b(v)$ are sequences of consecutive integers $[b_{1}; b_{2}] = \\{b_{1}, b_{1}+1, \\dots,\nb_{2}\\}$ (see [@lovasz2009matching Section 10.2]).", "\n\n\\[thm:uniformmatroid-matching\\] ${\\textsc{COPIC}}(\\base({\\mathcal{U}_{n}^{k_{1}}}), \\base({\\mathcal{U}_{n}^{k_{2}}}), {\\operatorname{diag}}(a), c, d)$ can be solved in polynomial time.", "\n\nWe create an equivalent instance of the minimum cost $b$-factor problem on a graph $G$ (see Figure \\[fig:uniformmatroid-matching\\]). ", "To achieve this, we introduce two special vertices $x$ and $y$ with $b(x) = k_{1}$ and $b(y) = k_{2}$ and another $3n$ vertices $i_{x}$, $i_{y}$ and $i_{m}$ for $i=1,2,\\dots,n$, i.e., for each element of the ground set of the two matroids. ", "We set $b(i_{x}) = b(i_{y}) = 1$ and $b(i_{m}) = \\{0, 1 \\}$. The $k_{1}$ vertices matched with $x$ and $k_2$ vertices matched with $y$ correspond to the sets $S_{1}$ and $S_{2}$, respectively.", "\n\nWe introduce edges $\\{x, i_{x} \\}$ with cost $c_{i} + \\frac{a_{i}}{2}$ and $\\{y, i_{y} \\}$ with cost $d_{i} + \\frac{a_{i}}{2}$. We also connect $\\{\n i_{x}, i_{y} \\}$ with edges of cost $0$ and $\\{ i_{x},\n i_{m} \\}$ and $\\{ i_{x}, i_{m} \\}$ both with cost $-\\frac{a_{i}}{2}$.\n\n\\(x) at (0,-4) ; (y) at (12,-4) ;\n\n(x1) at (4,-1) ; (y1) at (8,-1) ; (z1) at (6,-2) ;\n\n(xn) at (4,-6) ; (yn) at (8,-6) ; (zn) at (6,-7) ;\n\n(middle) at (6, -3.5) ;\n\n(x1) – node\\[weight\\][$c_{1} + \\frac{a_{1}}{2}$]{} (x); (x) – node\\[weight\\][$c_{n} + \\frac{a_{n}}{2}$]{} (xn);\n\n\\(y) – node\\[weight\\][$d_{1} + \\frac{a_{1}}{2}$]{} (y1); (yn) – node\\[weight\\][$d_{n} + \\frac{a_{n}}{2}$]{} (y);\n\n(x1) – (y1); (x1) – node\\[weight\\][$-\\frac{a_{1}}{2}$]{} (z1); (z1) – node\\[weight\\][$-\\frac{a_{1}}{2}$]{} (y1);\n\n(xn) – (yn); (xn) – node\\[weight\\][$-\\frac{a_{n}}{2}$]{} (zn); (zn) – node\\[weight\\][$-\\frac{a_{n}}{2}$]{} (yn);\n\nIt is easy to see that there is a one-to-one mapping between feasible solutions of the given diagonal COPIC and this instance of the $b$-factor problem, and moreover, the corresponding costs are the same. ", "Any feasible $b$-factor $M$ must contain exactly $k_{1}$ edges of the form $\\{x,\n i_{x}\\}$ and $k_{2}$ edges of the form $\\{y, i_{y}\\}$. These can be identified with the solution sets $S_{1}$ and $S_{2}$ for our diagonal COPIC. ", "Given any such partial $b$-factor there exists exactly one completion to a feasible $b$-factor, using additional edges inside the triangles $i_{x},\n i_{y}, i_{m}$ for each $i \\in [n]$, according to the following four cases. ", "We can also directly observe that the cost of the enforced $b$-factor and the solution $S_{1}, S_{2}$ is the same.", "\n\n1. ", " $i \\notin S_{1}, i \\notin S_{2}$: Both $i_{x}$ and $i_{y}$ are unmatched. ", "The only way to match both is by using the single edge $\\{i_{x}, i_{y}\\}$ and leaving $i_{m}$ unmatched, which is feasible since $0 \\in b(i_{m})$. The contribution to the total cost is $0$.\n\n2. ", " $i \\in S_{1}, i \\notin S_{2}$: In this case $i_{x}$ is already matched but $i_{y}$ is still unmatched. ", "The only feasible way to match $i_{y}$ is using the edge $\\{i_{y}, i_{m}\\}$, which contributes $c_{i}$ to the cost.", "\n\n3. ", " $i \\notin S_{1}, i \\in S_{2}$: This case is symmetric to case (2). ", "The cost contribution is $d_{i}$.\n\n4. ", " $i \\in S_{1}, i \\in S_{2}$: In this case $i_{x}$ and $i_{y}$ are both already matched and $i_{m}$ cannot be matched anymore. ", "We get a cost contribution of $a_{i} +\n c_{i} + d_{i}$.\n\nGiven a partition $S_{1}, S_{2}, \\dots, S_{t}$ of the ground set $E$ and integers $g_{1}, g_{2}, \\dots, g_{t}$, such that $0 \\leq g_{i} \\leq |B_{i}|$ for all $i=1,2,\\dots,t$, the set $\\{ X \\subseteq E \\colon |X \\cap S_{i}| = g_{i}\n\\text{ for all }i=1,2,\\dots,t \\}$ forms the collection of all bases of a partition matroid.", "\n\nIf $\\M_1,\\M_2$ are partition matroids, ${\\textsc{COPIC}}(\\base(\\M_{1}),\n \\base(\\M_{2}), {\\operatorname{diag}}(a), c, d)$ can be solved in polynomial time.", "\n\nTo prove this theorem, we use the approach based on matchings as in the proof of Theorem \\[thm:uniformmatroid-matching\\], with minor modifications. ", "Instead of special vertices $x$ and $y$, we introduce $x$- and $y$-vertices for each set in the partition of the ground set and connect these vertices only to the $i_{x}, i_{y}$-vertices that are in the corresponding set of the partition. ", "The equivalence of this construction can be shown analogously.", "\n\nOne can even further generalize the concept of partition matroids. ", "Given a partition $S_{1}, S_{2}, \\dots ,S_{t}$ of the element set $E$ and integers $f_{1}, f_{2}, \\dots, f_{t}$,$ g_{1}, g_{2}, \\dots,\ng_{t}$, $k$, such that $0 \\leq f_{i} \\leq g_{i} \\leq |S_{i}|$ for all $i=1,2,\\dots,t$ and $\\sum_{i=1}^{t}\nf_{i} \\leq k \\leq \\sum_{i=1}^{t} g_{i}$. The set $\\{ X \\subseteq E \\colon |X| = k \\text{ and } f_{i} \\leq |X \\cap S_{i}|\n\\leq g_{i}\\, \\forall i=1,2,\\dots,t \\}$ is the set of bases of a generalized partition matroid [@frank2011connections]. ", "A similar approach based on matchings still applies, if $\\f_{1}, \\f_{2}$ are sets of bases of a generalized partition matroids.", "\n\nThe combination of bases of a uniform matroid with other sets of feasible solutions in diagonal COPIC is similar to different versions of linear problems with capacity side constraints. ", "The following result is an example that can be derived using methods for the well-studied constrained shortest path problem with uniform edge weights, for which dynamic programming can be used to solve the problem in polynomial time (see Dumitrescu and Boland [@dumitrescu2001algorithms] for a review).", "\n\n${\\textsc{COPIC}}(\\base({\\mathcal{U}_{m}^{k}}), {\\mathcal{P}}_{s,t}(G), {\\operatorname{diag}}(a), 0, d)$ can be solved in polynomial time, if $a \\geq 0$ and $d \\geq 0$.\n\nMatroid bases as feasible sets {#sec:diagonal-mat}\n------------------------------\n\nAnother problem of great interest is the case when $\\f_{1}, \\f_{2}$ are sets of spanning trees of a graph, especially if the underlying graphs are isomorphic. ", "A generalization of this problem is the case when $\\f_{i} = \\base(\\M_{i})$ are the sets of bases of (not necessarily isomorphic) matroids $\\M_{1}, \\M_{2}$. In this section we assume familiarity with matroids and refer the reader to Oxley [@oxley2006matroid] for further definitions, results and notations.", "\n\nWe will first focus on the case without linear costs, i.e. $c \\equiv d \\equiv 0$. So the problem we are interested in is, given a ground set $E = [n]$ and a cost vector $a \\in \\r^{n}$, to minimize the objective function $$f(B_{1}, B_{2}) = \\sum_{i \\in B_{1} \\cap B_{2}} a_{i}$$ under the restrictions that $B_{1} \\in \\base(\\M_{1}), B_{2} \\in \\base(\\M_{2})$ for two given matroids $\\M_{1}, \\M_{2}$ over the ground set $E$.\n\n### Minimum cardinality base intersection\n\nIf the cost vector $a \\equiv 1$ (together with $c\\equiv d\\equiv 0$) this gives the problem of minimizing the size of the intersection of the two matroid bases $B_{1}$ and $B_{2}$. It contains as a special case the disjoint matroid base problem for two given matroids, since there exist two disjoint bases if and only if the optimal solution has objective value $0$.\n\nGabow and Westermann [@gabow1992forests] showed that the disjoint matroid base problem can be efficiently solved under the assumption that there exist efficient oracles to solve the static-base circuit problem. ", "This means that for both matroids $\\M_{i}$, $i=1,2$, independent set $S$ and element $e \\notin S$, we can efficiently decide if $S\n\\cup \\{e\\}$ is independent in $\\M_{i}$, and if not, output all elements in $C(e, S)$, the unique cycle contained in $S \\cup \\{e\\}$ of the matroid $\\M_{i}$.\n\n### Minimum cost base intersection\n\nThe more general case, where for each element $i \\in B_{1} \\cap B_{2}$ we pay a non-negative cost $a_{i} \\geq 0$ was already studied in the algorithmic game theory literature. ", "It is equivalent to computing the socially optimal state of a two player matroid congestion game. ", "Ackermann et al.", " [@ackermann2008impact] show that this problem can be solved in polynomial time for an arbitrary number of players using the same approach that was used by Werneck et al.", " [@werneck2000finding] to calculate the socially optimal state in spanning tree congestion games.", "\n\nTo keep this work self contained we give a summary of their algorithm using the notation of diagonal COPIC. ", "We reduce the problem to an equivalent instance of the minimum cost disjoint base problem, for which we can guarantee the existence of two disjoint bases.", "\n\nThe idea of the construction is to double all elements of $E$. The new ground set of elements is denoted by $E' = E_{1} \\cup E_{2}$, where $E_{1}, E_{2}$ are two disjoint copies of the original ground set $E$. For $i \\in E$ we write $i_{1}$ for the copy of $i$ inside $E_{1}$ and $i_{2}$ for its copy in $E_{2}$. We set $a_{i_{1}}= a_{i}$ and $a_{i_{2}}:= 0$ for each $i \\in E$ and introduce two new matroids $\\M'_{1}, \\M'_{2}$, each with $E'$ as their ground set. ", "The independent sets of $\\M'_{j}$ are all sets $S' \\subseteq E'$ that do not contain both $i_{1}$ and $i_{2}$ for any $i \\in E$ and where $\\{ i \\in E \\colon i_{1} \\in S'\\text{ or }\ni_{2} \\in S' \\}$ is independent in $\\M_{j}$, for $j=1,2$.\n\nGiven two disjoint bases $B'_{1}$ of $\\M'_{1}$ and $B'_{2}$ of $\\M'_{2}$, they induce, not necessarily disjoint, bases $B_{1}, B_{2}$ of $\\M_{1},\\M_{2}$. For every element $i \\in B_{1} \\cap B_{2}$ we know that both $i_{1}$ and $i_{2}$ were used in $B'_{1}$ and $B'_{2}$. So for this element the cost $a_{i}$ is payed in the disjoint base problem. ", "For all other elements $i_{2}$ is used, since $0 =\na_{i_{2}} \\leq a_{i_{1}}$.\n\nEfficient methods for solving the minimum cost disjoint base problem for general matroids obtained by Gabow and Westermann [@gabow1992forests] can be used to solve our transformed minimum cost base intersection instance.", "\n\nThe case of arbitrary real costs $a_{e} \\in \\r$ can also be handled. ", "This is not included in the algorithmic game theory literature, since in that context a positive impact of congestion (i.e. $a_i<0$) does not make sense.", "\n\nFirst, we find a set $B \\in \\inds(\\M_{1}) \\cap \\inds(\\M_{2})$ of minimum cost and we contract this set. ", "For all edges $e \\in E \\setminus B$ with $a_{e} < 0$ it holds that $B+e \\notin \\inds(\\M_{1}) \\cap \\inds(\\M_{2})$, or we could improve the solution, so these elements can never be in the intersection of a feasible solution together with $B$. Hence we can run the algorithm from above on the remaining instance. ", "The optimality of this approach follows from the following lemma.", "\n\nLet $B$ be an element of $\\inds(\\M_{1}) \\cap \\inds(\\M_{2})$ with minimum cost $a(B):=\\sum_{i\\in B}a_i$, and $B_{1}, B_{2}$ be two bases. ", "Then $B_{1}, B_{2}$ can be transformed into two new bases $\\tilde{B}_{1}, \\tilde{B}_{2}$ such that $B \\subseteq \\tilde{B}_{1}\n \\cap \\tilde{B}_{2}$ and $a(\\tilde{B}_{1} \\cap \\tilde{B}_{2}) \\leq a(B_{1} \\cap B_{2})$.\n\nLet $e \\in B \\setminus (B_{1} \\cap B_{2})$. There are three different cases on how to add $e$ to the intersection.", "\n\n1. ", " $e \\notin B_{1} \\cup B_{2}$: In this case we have $f_{i} \\in\n C_{i}(e, B_{i}) \\setminus B$ for both $i=1,2$. By modifying the bases to $\\tilde{B}_{i} = B_{i} + e - f_{i}$ we get that $$a(\\tilde{B}_{1} \\cap \\tilde{B}_{2}) = a(B_{1} \\cap B_{2}) + a_{e}\n - \\begin{cases}a_{f} & f_{1} = f_{2}\\\\ 0 & f_{1} \\neq f_{2}\\end{cases}$$\n\n2. ", " $e \\in B_{1}, e \\notin B_{2}$: In this case we have $f \\in\n C_{2}(e, B_{2}) \\setminus B$ and we can modify $\\tilde{B}_{2} = B_{2} + e - f$. this gives a modified cost of $$a(\\tilde{B}_{1} \\cap \\tilde{B}_{2}) = a(B_{1} \\cap B_{2}) + a_{e}\n - \\begin{cases}a_{f} & f \\in B_{1}\\\\ 0 & f \\notin B_{1}\\end{cases}$$\n\n3. ", " $e \\notin B_{1}, e \\in B_{2}$: symmetric to case (2).", "\n\nWe apply these steps iteratively until $B$ is contained in the intersection. ", "We know that the sum of costs of the elements $e \\in \\tilde{B}_{1} \\cap\n \\tilde{B}_{2}$ with $a_{e} \\leq 0$ must now be smaller than before, since $B$ is minimum. ", "We never added any element $e$ to $\\tilde{B}_{1} \\cap\n \\tilde{B}_{2}$ with $a_{e} > 0$. This implies that $a(\\tilde{B}_{1} \\cap\n \\tilde{B}_{2}) \\leq a(B_{1} \\cap B_{2})$.\n\nThe approach above gives us the following result.", "\n\n${\\textsc{COPIC}}(\\base(\\M_{1}), \\base(\\M_{2}), {\\operatorname{diag}}(a), 0, 0)$ can be solved in polynomial time, for any two matroids $\\M_{1}, \\M_{2}$ and cost vector $a\n \\in \\r^{n}$.\n\n### The case $a \\geq 0, c\\equiv d$\n\nThis case can be solved analogously to the case without linear costs. ", "We create two identical helper matroids $\\M'_{1}, \\M'_{2}$, with the only difference that we set the costs of the elements to $a_{e} + c_{e}$ and $c_{e}$. Since $a_{e} \\geq 0$, it follows that the algorithm will prefer the copy of cost $c_{e}$ if it takes only one of the two elements into the solution. ", "This again implies that we obtain a one to one correspondence of solutions as in the discussion above.", "\n\n${\\textsc{COPIC}}(\\base(\\M_{1}), \\base(\\M_{2}), {\\operatorname{diag}}(a), c, c)$ can be solved in polynomial time, for any two matroids $\\M_{1}, \\M_{2}$ and cost vectors $a\n \\in \\r^{n}_{\\geq 0}, c \\in \\r^{n}$.\n\nIt remains an interesting open question whether we can also solve the case with arbitrary costs $a \\in \\r^{n}$ and the case with non-equal linear costs $c\n\\neq d$ in polynomial time, as it is possible for uniform and partition matroids.", "\n\nPairs of paths {#sec:diagonal-paths}\n--------------\n\nIn this section we analyze the special case when $\\f_{1}$ and $\\f_{2}$ correspond to the set of $s_{1}$-$t_{1}$- and $s_{2}$-$t_{2}$-paths in a graph. ", "We will again look at the case where the graphs corresponding to $\\f_{1}$ and $\\f_{2}$ are identical. ", "One must also make sure that there do not exist negative circles in the graph, else already optimizing over a linear cost function without interaction costs is NP-hard. ", "To simplify the exposition we will focus on $Q,c,d \\geq 0$. Table \\[tab:paths\\] is a summary of the results in this subsection. ", "It is important to differentiate between directed and undirected graphs, which is clear in the light of Proposition \\[paths-eq-copic\\] and the known complexity results of the edge-disjoint paths problem.", "\n\n\\[paths-eq-copic\\] Given a graph $G$, ${\\textsc{COPIC}}({\\mathcal{P}}_{s_{1}, t_{1}}(G), {\\mathcal{P}}_{s_{2}, t_{2}}(G), {\\operatorname{diag}}(a), 0, 0)$ with $a > 0$ has a solution with objective value $0$, if and only if there exist two edge-disjoint paths $s_{i}$-$t_{i}$-paths in $G$.\n\n **directedness** **terminals** **cost restrictions** **complexity**\n ------------------ --------------- ---------------------------------------- ----------------\n directed arbitrary $Q=I, c=d=0$ NP-hard\n directed common $Q={\\operatorname{diag}}(\\infty)$ NP-hard\n undirected arbitrary $Q={\\operatorname{diag}}(\\infty), d=0$ NP-hard\n undirected arbitrary $c=d=0$ open\n undirected common $Q={\\operatorname{diag}}(\\infty)$ NP-hard\n both common $c=d$ P\n\n : Summary of the results for diagonal COPIC with paths as feasible solutions[]{data-label=\"tab:paths\"}\n\nIt is well known that the edge-disjoint paths problem is polynomial time solvable for every constant number of paths in undirected graphs [@robertson1995graph], but NP-hard already for 2 paths in directed graphs [@fortune1980directed]. ", "This imediatly yields the following result.", "\n\nGiven a directed graph $G$, ${\\textsc{COPIC}}({\\mathcal{P}}_{s_{1}, t_{1}}(G), {\\mathcal{P}}_{s_{2}, t_{2}}(G), {\\operatorname{diag}}(a), 0, 0)$ is NP-hard, even for $a \\equiv 1$.\n\nWe use the following results obtained by Eilam-Tzoreff [@eilam1998disjoint] to further classify the complexity of our problem.", "\n\n\\[eilam-poly\\] The undirected edge-disjoint two shortest paths problem is polynomial time solvable, even in the weighted case. ", "On the other hand, the undirected two edge-disjoint one shortest paths problem is NP-hard.", "\n\nIt is important to note that in the results of Eilam-Tzoreff, a shortest path always means a shortest path in the original graph, not a shortest path after removing the edges of the other disjoint path. ", "This is the reason why using Theorem \\[eilam-poly\\], we cannot conclude that ${\\textsc{COPIC}}({\\mathcal{P}}_{s_{1}, t_{1}}(G),\n{\\mathcal{P}}_{s_{2}, t_{2}}(G), {\\operatorname{diag}}(\\infty), c, c)$ is polynomial time solvable, since in our model we cannot enforce two shortest paths of the original graph. ", "If $c=d=1$ and $Q = {\\operatorname{diag}}(\\infty)$ Björklund and Husfeldt [@bjorklund2014shortest] showed in 2014 how to solve the problem using a polynomial time Monte Carlo algorithm. ", "The existence of a deterministic polynomial time algorithm is still unknown and a long-standing open problem.", "\n\nNevertheless, it is possible to use the hardness results of Eilam-Tzoreff [@eilam1998disjoint] to show that for general costs $c, d \\geq\n0$ the problem is NP-hard.", "\n\nGiven an undirected graph $G$, ${\\textsc{COPIC}}({\\mathcal{P}}_{s_{1}, t_{1}}(G), {\\mathcal{P}}_{s_{2}, t_{2}}(G), {\\operatorname{diag}}(\\infty),\n c, 0)$ is NP-hard for $c \\geq 0$.\n\nUsing a polynomial time algorithm for ${\\textsc{COPIC}}$ we can determine, if the two edge-disjoint one shortest paths problem has a solution. ", "Just run the algorithm and check if the objective value equals the length of a shortest $s_{1}$-$t_{1}$-path in the given graph.", "\n\nThis covers the case if $s_{1} \\neq s_{2}$ and $t_{1} \\neq t_{2}$. From the edge-disjoint path literature we know that the problem becomes easier, if one assumes a common source $s$ and a common sink $t$ for all the paths. ", "We can classify the complexity of this case for our problem, using the following results.", "\n\nGiven a graph or digraph $G$, ${\\textsc{COPIC}}({\\mathcal{P}}_{s, t}(G), {\\mathcal{P}}_{s, t}(G),\n {\\operatorname{diag}}(a), c, c)$ is solvable in polynomial time, for cost vectors $a,c \\geq 0$.\n\nWe reduce to a minimum cost flow problem. ", "Set $b(s) = 2$ and $b(t) = -2$ and double each edge/arc $e \\in E$ to two versions $e_{1}, e_{2}$ with $\\tilde{c}_{e_{1}} = c_e$ and $\\tilde{c}_{e_{2}} = a_e + c_e$. Now a minimum cost flow in this network will be integral and can be decomposed into two path flows, each sending one unit from $s$ to $t$. The cost of the flow corresponds to the cost of these two paths in our problem.", "\n\nGiven a graph or digraph $G$, ${\\textsc{COPIC}}({\\mathcal{P}}_{s, t}(G), {\\mathcal{P}}_{s, t}(G),\n {\\operatorname{diag}}(\\infty), c, d)$ is NP-hard.", "\n\nFor digraphs the statement follows from a reduction from directed two disjoint paths. ", "Given such an instance we introduce the new terminals $s$ and $t$ and add arcs $(s,\n s_{1}),$ $(s, s_{2}),$ $(t_{1}, t),$ $(t_{2}, t)$. We use $Q = {\\operatorname{diag}}(\\infty)$ and as linear costs $c_{(s, s_{1})} = c_{(t_{1}, t)} = d_{(s, s_{2})} = d_{(t_{2}, t)} = 0$ and $c_{(s, s_{2})} = c_{(t_{2}, t)} = d_{(s, s_{1})} = d_{ (t_{1}, t)} = \\infty$ and $c_e = d_e = 0$ for all other edges. ", "This enforces that paths $S_{i}$ are $s_{i}$-$t_{i}$-paths and the diagonal matrix with infinite entries ensures disjointness.", "\n\nIn the undirected case we apply the same construction as above but using the undirected two edge-disjoint one shortest paths problem. ", "To solve the decision problem analyzed by Eilam-Tzoreff [@eilam1998disjoint], we create COPIC with $c_e = 1$ and $d_e = 0$ for all the edges in the original network to enforce that $S_{1}$ is a shortest path. ", "After finding a finite cost solution to this problem we check if the length of $S_{1}$ is equal to the length of a shortest $s_{1}$-$t_{1}$-path in $G$.\n\nLinearizable instances {#sec:lin}\n======================\n\nIn this section we explore for which cost matrices COPIC leads to an equivalent problem where there is essentially no interaction between two structures of COPIC.", "\n\nMore precisely, we say that an interaction cost matrix $Q$ of a COPIC is *linearizable*, if there exist vectors $a=(a_i)$ and $b=(b_i)$ such that for all $S_1\\in\\f_1$ and $S_2\\in\\f_2$ $$\\sum_{i\\in S_1}\\sum_{j\\in S_2}q_{ij}=\\sum_{i\\in S_1}a_i+\\sum_{j\\in S_2}b_j$$ holds. ", "In that case we say that the pair of vectors $a$ and $b$ together is a *linearization* of $Q$.\n\nNote that for an instance ${\\textsc{COPIC}}(\\f_1,\\f_2,Q,c,d)$, $f(S_1,S_2)=\\sum_{i\\in S_1}\\bar{a}_i+\\sum_{j\\in S_2}\\bar{b}_j$ for some $\\bar{a}=(\\bar{a}_i)$, $\\bar{b}=(\\bar{b}_i)$ and all $S_1\\in \\f_1$, $S_2\\in \\f_2$, if and only if $Q$ is linearizable. ", "Hence, we extend our notion of linearizability and say that an instance ${\\textsc{COPIC}}(\\f_1,\\f_2,Q,c,d)$ is linearizable if and only if $Q$ is linearizable. ", "Our aim is to characterize all linearizable instances of COPIC, with respect to given solution sets $\\f_1$ and $\\f_2$.\n\nLinearizable instances have been studied by various authors for the case of quadratic assignment problem [@CDW16; @KP11; @PK13], quadratic spanning tree problem [@CP15] and bilinear assignment problem [@CSPB16]. ", "Here we generalize the ideas from [@CSPB16] and suggest an approach for finding a characterization of linearizable instances of COPIC’s.", "\n\nAn interaction cost matrix $Q$ of a COPIC has *constant objective property with respect to $\\f_1$* if for every $j\\in [n]$ there exist a constant $K^{(1)}_j$, so that $$\\sum_{i\\in S_1}q_{ij}=K^{(1)}_j \\qquad \\text{ for all }\\ S_1\\in\\f_1.$$ Similarly, $Q$ has *constant objective property with respect to $\\f_2$* if for every $i\\in [m]$ there exist a constant $K^{(2)}_i$, so that $$\\sum_{j\\in S_2}q_{ij}=K^{(2)}_i \\qquad \\text{ for all }\\ S_2\\in\\f_2.$$ For $\\f_i$, $i=1,2$, let $\\text{CVP}_i(\\f_i$) be the vector space of all matrices with constant objective property with respect to $\\f_i$.\n\nCombinatorial optimization problems with constant objective property have been studied by various authors [@Berenguer79; @Burkard07; @CusticKlinz16; @Kaveh10].", "\n\nLet $\\text{CVP}_1(\\f_1)+\\text{CVP}_2(\\f_2)$ be the vector space of all interaction matrices $Q=(q_{ij})$ of COPIC, such that $q_{ij}=a_{ij}+b_{ij}$ $\\forall i,j$, for some $A=(a_{ij})\\in \\text{CVP}_1(\\f_1)$ and $B=(b_{ij})\\in \\text{CVP}_2(\\f_2)$.\n\n\\[lm:sufLin\\] If the interaction cost matrix $Q$ of ${\\textsc{COPIC}}(\\f_1,\\f_2,Q,c,d)$ is an element of $\\text{CVP}_1(\\f_1)+\\text{CVP}_2(\\f_2)$, then $Q$ is linearizable.", "\n\nLet $Q$ be of the form $Q=E+F$, where $E=(e_{ij})\\in \\text{CVP}_1(\\f_1)$ and $F=(f_{ij})\\in \\text{CVP}_2(\\f_2)$. Then $$\\begin{aligned}\n \\sum_{i\\in S_1}\\sum_{j\\in S_2}q_{ij} & = \\sum_{i\\in S_1}\\sum_{j\\in S_2} \\left(e_{ij}+f_{ij}\\right)\\\\\n & = \\sum_{j\\in S_2}\\left(\\sum_{i\\in S_1} e_{ij}\\right)+ \\sum_{i\\in S_1}\\left(\\sum_{j\\in S_2} f_{ij}\\right)\\\\\n & = \\sum_{j\\in S_2} K^{(1)}_j+ \\sum_{i\\in S_1}K^{(2)}_i.\\end{aligned}$$ Hence $Q$ is linearizable, and $a=(a_i)$, $b=(b_j)$ with $a_i=K^{(2)}_i$, $b_j=K^{(1)}_j$ is a linearization of $Q$.\n\nNow we show that the opposite direction is also true, provided some additional conditions are satisfied. ", "In fact, these additional conditions are satisfied for many well studied combinatorial optimization problems.", "\n\n\\[lm:nesLin\\] Let $\\f_1\\subseteq 2^{[m]}$ and $\\f_2\\subseteq 2^{[n]}$ be such that:\n\n1. ", " There exist an $m$ vector $a=(a_i)$, an $n$ vector $b=(b_j)$ and two non-zero constants $K_a,K_b$, such that $$\\sum_{i\\in S_1}a_i=K_a \\ \\ \\forall S_1\\in \\f_1\\ \\ \\text{ and } \\ \\ \\sum_{j\\in S_2}b_j=K_b \\ \\ \\forall S_2\\in \\f_2.$$\n\n2. ", " If an $m\\times n$ matrix $\\bar{Q}=(\\bar{q}_{ij})$ is such that $\\sum_{i\\in S_1}\\sum_{j\\in S_2} \\bar{q}_{ij}=0$ for all $S_1\\in\\f_1$, $S_2\\in\\f_2$, then $\\bar{Q}\\in \\text{CVP}_1(\\f_1)+\\text{CVP}_2(\\f_2)$.\n\nIf ${\\textsc{COPIC}}(\\f_1,\\f_2,Q,c,d)$ is linearizable, then $Q\\in \\text{CVP}_1(\\f_1)+\\text{CVP}_2(\\f_2)$.\n\nAssume that the conditions $(i)$ and $(ii)$ of Lemma \\[lm:nesLin\\] are satisfied, and that $Q$ is linearizable. ", "We will show that $Q\\in \\text{CVP}_1(\\f_1)+\\text{CVP}_2(\\f_2)$ by reconstructing the proof of Lemma \\[lm:sufLin\\] in reverse direction.", "\n\nSince $Q$ is linearizable, there exist $a=(a_i)$ and $b=(b_j)$ such that $$\\label{eq:nes11}\n\\sum_{i\\in S_1}\\sum_{j\\in S_2}q_{ij}=\\sum_{i\\in S_1} a_i + \\sum_{j\\in S_2}b_j \\quad \\forall S_1\\in \\f_i, S_2\\in \\f_2.$$ Note that from $(i)$ it follows that there exist matrices $\\hat{E}=(\\hat{e}_{ij})\\in \\text{CVP}_1(\\f_1)$ and $\\hat{F}=(\\hat{f}_{ij})\\in \\text{CVP}_2(\\f_2)$ such that $$\\label{eq:nes22}\n \\sum_{j\\in S_2}\\hat{f}_{ij}=a_i \\qquad \\forall S_2\\in\\f_2,\\ i\\in M,$$ $$\\label{eq:nes33}\n \\sum_{i\\in S_i}\\hat{e}_{ij}=b_j \\qquad \\forall S_1\\in\\f_1,\\ j\\in N.$$ Using and , we can rewrite as $$\\begin{aligned}\n \\sum_{i\\in S_1}\\sum_{j\\in S_2}q_{ij} & = \\sum_{i\\in S_1}\\left( \\sum_{j\\in S_2}\\hat{f}_{ij} \\right) + \\sum_{j\\in S_2}\\left( \\sum_{i\\in S_1}\\hat{e}_{ij} \\right)\\nonumber\\\\\n & = \\sum_{i\\in S_1}\\sum_{j\\in S_2} \\left(\\hat{e}_{ij}+\\hat{f}_{ij}\\right)\\end{aligned}$$ for all $S_1\\in\\f_1$, $S_2\\in\\f_2$. Hence it follows that $$\\sum_{i\\in S_1}\\sum_{j\\in S_2}\\left(q_{ij}-(\\hat{e}_{ij}+\\hat{f}_{ij})\\right)=0 \\quad \\forall S_1\\in \\f_i, S_2\\in \\f_2.$$ Now, from $(ii)$ it follows that $Q-(\\hat{E}+\\hat{F})=E+F$ for some $E\\in \\text{CVP}_1(\\f_1)$, $F\\in \\text{CVP}_2(\\f_2)$, and hence, $Q=(E+\\hat{E})+(F+\\hat{F})\\in \\text{CVP}_1(\\f_1) + \\text{CVP}_2(\\f_2)$.\n\nFrom Lemma \\[lm:sufLin\\] and Lemma \\[lm:nesLin\\] it follows that $\\text{CVP}_1(\\f_1) + \\text{CVP}_2(\\f_2)$ is the set of all linearizable matrices, provided that the corresponding COPIC satisfies properties $(i)$ and $(ii)$ of Lemma \\[lm:nesLin\\].", "\n\nIn most cases, property $(i)$ is straightforward to check. ", "For example, it is true for all COPIC’s for which elements of $\\f_1$ and $\\f_2$ are of fixed cardinality. ", "If $\\f_1$ and $\\f_2$ are $s$-$t$ paths in a graph, then again property $(i)$ is satisfied, although feasible solutions are of different cardinality. ", "Condition $(i)$ is not satisfied for unconstrained solution sets, i.e., when $\\f_1$ ($\\f_2$) is $2^{[m]}$ ($2^{[n]}$).", "\n\nNow we show how Lemma \\[lm:sufLin\\] and Lemma \\[lm:nesLin\\] can be used to characterize linearizable instances for some specific COPIC’s. ", "In particular, we consider unconstrained solution sets $2^{[m]}$, bases of the uniform matroids $\\base({\\mathcal{U}_{m}^{k}})$, spanning trees of a complete graph $\\base(\\mathcal{M}(K_m))$ and perfect matchings of a complete bipartite graph ${\\mathcal{PM}}(K_{m,m})$. For the case of ${\\mathcal{PM}}(K_{m,m})$ the set $[m]\\times [m]$ will be our set of edges of the perfect bipartite graph $K_{m,m}$. Hence, in the case of ${\\textsc{COPIC}}(\\f_1,\\f_2,Q,c,d)$ where $\\f_i={\\mathcal{PM}}(K_{m,m})$, the dimensions (number of indices) of the cost arrays $Q$ and $c$ or $d$ is increased by one, however our lemmas and $\\text{CVP}_i({\\mathcal{PM}}(K_{m,m}))$ remain to be well defined.", "\n\n1. ", " ${\\textsc{COPIC}}({\\mathcal{PM}}(K_{m,m}),{\\mathcal{PM}}(K_{n,n}),Q,c,d)$ is linearizable if and only if there are some arrays $A$, $B$, $C$, $D$ such that $q_{ijk\\ell}=a_{ijk}+b_{ij\\ell}+c_{ik\\ell}+d_{jk\\ell}$.\n\n2. ", " ${\\textsc{COPIC}}(\\base(\\mathcal{M}(K_m)),\\base(\\mathcal{M}(K_n)),Q,c,d)$ is linearizable if and only if there are some vectors $a$, $b$ such that $q_{ij}=a_{i}+b_{j}$.\n\n3. ", " ${\\textsc{COPIC}}(\\base({\\mathcal{U}_{m}^{k_1}}),\\base({\\mathcal{U}_{n}^{k_2}}),Q,c,d)$ is linearizable if and only if there are some vectors $a$, $b$ such that $q_{ij}=a_{i}+b_{j}$.\n\n4. ", " ${\\textsc{COPIC}}({\\mathcal{PM}}(K_{m,m}),\\base(\\mathcal{M}(K_n)),Q,c,d)$ is linearizable if and only if there are some arrays $A$, $B$, $C$ such that $q_{ijk}=a_{ij}+b_{ik}+c_{jk}$.\n\n5. ", " ${\\textsc{COPIC}}(\\base(\\mathcal{M}(K_m)) ,\\base({\\mathcal{U}_{n}^{k}}) ,Q,c,d)$ is linearizable if and only if there are some vectors $a$, $b$ such that $q_{ij}=a_{i}+b_{j}$.\n\n6. ", " ${\\textsc{COPIC}}({\\mathcal{PM}}(K_{m,m}) ,\\base({\\mathcal{U}_{n}^{s}}) ,Q,c,d)$ is linearizable if and only if there are some arrays $A$, $B$, $C$ such that $q_{ijk}=a_{ij}+b_{ik}+c_{jk}$.\n\nWe present a complete proof for $(iv)$, and indicate how other statements can be shown analogously.", "\n\nIn the case of ${\\textsc{COPIC}}({\\mathcal{PM}}(K_{m,m}),\\base(\\mathcal{M}(K_n)),Q,c,d)$, the interaction costs are represented in a three-dimensional array $Q$, since for convenience we represent the cost vector of $\\f_1={\\mathcal{PM}}(K_{m,m})$ in two indices. ", "It is well known that a linear assignment problem instance $R=(r_{ij})$ has the constant objective property if and only if $r_{ij}=s_i+t_j$, for some vectors $s$ and $t$. Hence $\\text{CVP}_1({\\mathcal{PM}}(K_{m,m}))=\\{A=(a_{ijk})\\colon\na_{ijk}=b_{ik}+c_{jk} \\text{ for some } B=(b_{ij}),C=(c_{ij})\\}$. A spanning tree problem on a complete graph has the constant objective property if and only if the cost vector is constant, therefore $\\text{CVP}_2(\\base(\\mathcal{M}(K_n)))=\\{A=(a_{ijk})\\colon a_{ijk}=b_{ij} \\text{ for some } B=(b_{ij})\\}$. Hence, $Q$ is an element of $\\text{CVP}_1({\\mathcal{PM}}(K_{m,m}))+\\text{CVP}_2(\\base(\\mathcal{M}(K_n)))$ if and only if there are some $A$, $B$ and $C$ such that $$\\label{STlin}\n q_{ijk}=a_{ij}+b_{ik}+c_{jk}.$$\n\nLemma \\[lm:sufLin\\] tells us that is a sufficient condition for $Q$ to be linearizable. ", "To show that it is also a necessary condition, we just need to show that properties $(i)$ and $(ii)$ of Lemma \\[lm:nesLin\\] are true for ${\\textsc{COPIC}}({\\mathcal{PM}}(K_{m,m}),\\base(\\mathcal{M}(K_n)),Q,c,d)$. $(i)$ is obviously true, hence it remains to show that if $Q$ is such that $$\\sum_{(i,j)\\in S_1}\\sum_{k\\in S_2} q_{ijk}=0\\qquad \\forall S_1\\in{\\mathcal{PM}}(K_{m,m}),\\ S_2\\in\\base(\\mathcal{M}(K_n)),$$ then $Q\\in \\text{CVP}_1({\\mathcal{PM}}(K_{m,m}))+\\text{CVP}_2(\\base(\\mathcal{M}(K_n)))$.\n\nLet $i,j\\in \\{2,3,\\ldots,m\\}$ be fixed, and let $S_{PM}',S_{PM}''\\in{\\mathcal{PM}}(K_{m,m})$ be such that $S_{PM}'\\setminus S_{PM}''=\\{(1,1),(i,j)\\}$ and $S_{PM}''\\setminus S_{PM}'=\\{(1,j),(i,1)\\}$. Further, let $k\\in \\{2,3,\\ldots,n\\}$ be fixed, and $S_{ST}',S_{ST}''\\in\\base(\\mathcal{M}(K_n))$ be such that $S_{ST}'\\setminus S_{ST}''=\\{1\\}$ and $S_{ST}''\\setminus S_{ST}'=\\{k\\}$. Note that such $S_{PM}',S_{PM}'',S_{ST}',S_{ST}''$ exist for all $i,j\\in \\{2,3,\\ldots,m\\}$, $k\\in \\{2,3,\\ldots,n\\}$.\n\nLet us assume that $Q$ satisfies property $(ii)$ of Lemma \\[lm:nesLin\\]. ", "Then, in particular, we have that $$\\label{eqcor0}\n \\sum_{(i,j)\\in S_{PM}'}\\sum_{k\\in S_{ST}'} q_{ijk}+\\sum_{(i,j)\\in S_{PM}''}\\sum_{k\\in S_{ST}''} q_{ijk}=\\sum_{(i,j)\\in S_{PM}'}\\sum_{k\\in S_{ST}''} q_{ijk}+\\sum_{(i,j)\\in S_{PM}''}\\sum_{k\\in S_{ST}'} q_{ijk},$$ which, after cancellations, gives us $$\\label{eqcor1}\n q_{111}+q_{ij1}+q_{1jk}+q_{i1k}=q_{11k}+q_{ijk}+q_{1j1}+q_{i11}$$ for all $i,j\\in \\{2,3,\\ldots,m\\}$, $k\\in \\{2,3,\\ldots,n\\}$. Note that holds true even if $i,j$ or $k$ is equal to $1$, since in that case everything chancels out. ", "Therefore, $q_{ijk}$ can be expressed as $$q_{ijk}=a_{ij}+b_{ik}+c_{jk} \\qquad \\forall i,j\\in [m],\\ \\forall k\\in [n],$$ where $$a_{ij}:=q_{ij1}-\\frac{1}{2}q_{1j1}-\\frac{1}{2}q_{i11}+\\frac{1}{3}q_{111},$$ $$b_{ik}:=q_{i1k}-\\frac{1}{2}q_{11k}-\\frac{1}{2}q_{i11}+\\frac{1}{3}q_{111},$$ $$c_{jk}:=q_{1jk}-\\frac{1}{2}q_{11k}-\\frac{1}{2}q_{1j1}+\\frac{1}{3}q_{111},$$ i.e., $Q\\in \\text{CVP}_1({\\mathcal{PM}}(K_{m,m}))+\\text{CVP}_2(\\base(\\mathcal{M}(K_n)))$. That proves statement $(iv)$ of the theorem.", "\n\nStatements $(i)$ and $(ii)$ of the theorem can be proved by considering equation with two pairs of $S_{PM}',S_{PM}''$ for the case of ${\\textsc{COPIC}}({\\mathcal{PM}}(K_{m,m}),{\\mathcal{PM}}(K_{n,n}),Q,c,d)$, and two pairs of $S_{ST}',S_{ST}''$ for the case of ${\\textsc{COPIC}}(\\base(\\mathcal{M}(K_m)),\\base(\\mathcal{M}(K_n)),Q,c,d)$. Using analogous approach, the remaining statements of the theorem can be shown.", "\n\nAs we mentioned before, property $(i)$ of Lemma \\[lm:nesLin\\] does not hold for unconstrained solution set $2^{[m]}$ ($2^{[n]}$), nevertheless, it is not hard to show that $\\text{CVP}_1(\\f_1) + \\text{CVP}_2(\\f_2)$ characterizes all linearizable matrices even if $\\f_1=2^{[m]}$ or $\\f_2=2^{[n]}$.\n\n${\\textsc{COPIC}}(\\f_1,\\f_2,Q,c,d)$ with $\\f_1=2^{[m]}$ ($\\f_2=2^{[n]}$) is linearizable if and only if $Q\\in\\text{CVP}_2(\\f_2)$ ($Q\\in\\text{CVP}_1(\\f_1)$).", "\n\nAssume that $\\f_1=2^{[m]}$. Note that $\\text{CVP}_1(2^{[m]})$ contains only the $m\\times n$ zero matrix, hence Lemma \\[lm:sufLin\\] implies that elements of $\\text{CVP}_2(\\f_2)$ are linearizable.", "\n\nNow let us assume that $Q$ is linearizable and not an element of $\\text{CVP}_2(\\f_2)$. Then there must exist some $i'\\in [m]$ and $S_2,S_2'\\in\\f_2$ such that $\\sum_{j\\in S_2}q_{i'j}\\neq \\sum_{j\\in S_2'}q_{i'j}$. Let $a=(a_i)$ and $b=(b_i)$ be a linearization of $Q$. Since $\\{i'\\}\\in 2^{[m]}$, we have that $$\\sum_{j\\in S_2}q_{i'j}=\\sum_{i\\in \\{i'\\}}\\sum_{j\\in S_2}q_{ij}=a_{i'}+\\sum_{j\\in S_2}b_j,$$ $$\\sum_{j\\in S_2'}q_{i'j}=\\sum_{i\\in \\{i'\\}}\\sum_{j\\in S_2'}q_{ij}=a_{i'}+\\sum_{j\\in S_2'}b_j.$$ Hence, $\\sum_{j\\in S_2}b_j\\neq \\sum_{j\\in S_2'}b_j$. However, since $\\emptyset \\in 2^{[m]}$ we have $$0=\\sum_{i\\in \\emptyset}\\sum_{j\\in S_2}q_{ij}=\\sum_{j\\in S_2}b_j \\quad \\text{ and }\\quad\n 0=\\sum_{i\\in \\emptyset}\\sum_{j\\in S_2'}q_{ij}=\\sum_{j\\in S_2'}b_j$$ which implies that $\\sum_{j\\in S_2}b_j= \\sum_{j\\in S_2'}b_j$, a contradiction.", "\n\nConclusion {#sec:conclusion}\n==========\n\nWe introduced a general model to study combinatorial optimization problems with interaction costs and showed that many classical hard combinatorial optimization problems are special cases. ", "In many cases, interaction costs can be identified as the origin of the hardness of these problems. ", "Therefore we considered special structures of interaction costs, and their impact on the computational complexity of the underlying combinatorial optimization problems. ", "We presented a general approach based on multi-parametric programming to solve instances parametrized with the rank of the interaction cost matrix $Q$. Complementary to that, we analyzed problems with diagonal interaction cost matrix $Q$, which can be used to enforce disjointness constraints. ", "Even for this special type of interaction costs, we can show that for many common sets of feasible solutions, that have no matroid structure, COPIC becomes NP-hard. ", "We also identified conditions on the interaction costs so that COPIC can be reduced to an equivalent instance with no interaction costs.", "\n\nTo further characterize how interaction costs impact the computational complexity of different combinatorial optimization problems, the following questions could be addressed.", "\n\n1. ", " Are the polynomially solvable cases of COPIC where matrix $Q$ has fixed rank $r$ W\\[1\\]-hard?", "\n\n2. ", " For cases of COPIC with diagonal matrix that can be efficiently solved, analyze the parameterized complexity with respect to the bandwith of $Q$.\n\n3. ", " Can ${\\textsc{COPIC}}(\\base({\\mathcal{U}_{m}^{k}}), {\\mathcal{P}}_{s,t}(G), {\\operatorname{diag}}(a), c, d)$ be solved in polynomial time, if $a \\geq 0, c \\geq 0$ and $d \\geq 0$?", "\n\n4. ", " Is ${\\textsc{COPIC}}(\\base(\\M_{1}), \\base(\\M_{2}), {\\operatorname{diag}}(a), c, d)$ solvable in polynomial time, without any restrictions on $\\M_{1}, \\M_{2}, a, c$ and $d$?", "\n\nFor the case of diagonal COPIC it would be interesting to study further types of sets of feasible solutions. ", "For example the matching-cut problem analyzed by Bonsma [@bonsma2009complexity] can be also formulated as a special case of diagonal COPIC, so analyzing graph cuts as feasible sets in diagonal COPIC is an interesting candidate for further research.", "\n\nAdditionally, understanding the influence of interaction costs with other special matrix structures, besides fixed rank and diagonal matrices, to the computational complexity of combinatorial optimization problems would be of interest.", "\n\n[^1]: This work was supported by NSERC discovery grant and an NSERC discovery accelerator supplement award awarded to Abraham P. Punnen\n\n[^2]: Stefan Lendl is supported by the Austrian Science Fund (FWF): W1230\n" ]
{ "pile_set_name": "ArXiv" }
[ 0.009174311926605505, 0.0041841004184100415, 0, 0.009009009009009009, 0, 0, 0.0048543689320388345, 0.01567398119122257, 0, 0, 0, 0.00392156862745098, 0, 0.0026595744680851063, 0.004219409282700422, 0.010638297872340425, 0.013605442176870748, 0.008, 0.006920415224913495, 0.009345794392523364, 0.027210884353741496, 0.012048192771084338, 0, 0, 0, 0, 0.005405405405405406, 0.013888888888888888, 0, 0, 0.014150943396226415, 0.0049382716049382715, 0.006622516556291391, 0.05434782608695652, 0.006172839506172839, 0.004219409282700422, 0, 0.002577319587628866, 0.004658385093167702, 0, 0, 0, 0, 0, 0.010752688172043012, 0, 0, 0, 0, 0, 0.018518518518518517, 0.00510204081632653, 0.0058823529411764705, 0.006578947368421052, 0, 0, 0.007692307692307693, 0.013824884792626729, 0, 0, 0, 0.004739336492890996, 0.0028735632183908046, 0.0034129692832764505, 0.003896103896103896, 0.008032128514056224, 0, 0, 0.011494252873563218, 0.0011248593925759281, 0, 0, 0, 0, 0.00909090909090909, 0.009954751131221719, 0.004662004662004662, 0.0013054830287206266, 0.008176614881439084, 0.006042296072507553, 0.0076726342710997444, 0, 0, 0, 0, 0, 0, 0, 0, 0, 0, 0, 0.00641025641025641, 0, 0, 0, 0, 0, 0, 0.0038314176245210726, 0.00819000819000819, 0.006896551724137931, 0.01818181818181818, 0.031746031746031744, 0.007246376811594203, 0.00625, 0, 0.006514657980456026, 0.006349206349206349, 0.00794912559618442, 0.006944444444444444, 0.011811023622047244, 0, 0.008264462809917356, 0, 0, 0, 0.005319148936170213, 0, 0.017543859649122806, 0.010309278350515464, 0.004123711340206186, 0, 0, 0, 0, 0, 0.006036217303822937, 0.012084592145015106, 0.008385744234800839, 0.0047169811320754715, 0, 0.013513513513513514, 0.006024096385542169, 0.014492753623188406, 0.01098901098901099, 0.01718213058419244, 0.008771929824561403, 0.0035778175313059034, 0.006578947368421052, 0, 0.023529411764705882, 0.005714285714285714, 0, 0.008658008658008658, 0, 0.005660377358490566, 0.010752688172043012, 0.007407407407407408, 0.008333333333333333, 0, 0.002688172043010753, 0.00425531914893617, 0, 0, 0, 0.02666666666666667, 0, 0.009615384615384616, 0.008695652173913044, 0, 0.014705882352941176, 0, 0, 0.005076142131979695, 0.006289308176100629, 0, 0, 0, 0, 0.008298755186721992, 0, 0.005319148936170213, 0.0033112582781456954, 0.007246376811594203, 0.003278688524590164, 0.0038240917782026767, 0.004, 0, 0, 0, 0, 0.00909090909090909, 0, 0.008565310492505354, 0.0068143100511073255, 0.010033444816053512, 0, 0, 0, 0.0032258064516129032, 0, 0.007194244604316547, 0.003003003003003003, 0, 0.008333333333333333, 0.008746355685131196, 0, 0, 0, 0.004405286343612335, 0.006711409395973154, 0.003289473684210526, 0, 0.00663716814159292, 0.0048543689320388345, 0, 0.005917159763313609, 0, 0.0049261083743842365, 0.003756574004507889, 0, 0.00967741935483871, 0.007751937984496124, 0.011111111111111112, 0.004878048780487805, 0.009771986970684038, 0.005376344086021506, 0, 0.012121212121212121, 0.0030303030303030303, 0, 0, 0, 0.00411522633744856, 0.0026109660574412533, 0.006535947712418301, 0, 0.0025188916876574307, 0, 0, 0.004784688995215311, 0.008021390374331552, 0.003676470588235294, 0.002857142857142857, 0, 0.018072289156626505, 0.014705882352941176, 0.006631299734748011, 0.0023752969121140144, 0.007451564828614009, 0, 0, 0.01276595744680851, 0.002347417840375587, 0.007407407407407408, 0.006596306068601583, 0, 0.009433962264150943, 0, 0, 0.007142857142857143, 0.004411764705882353, 0, 0, 0, 0, 0, 0, 0, 0.007547169811320755, 0.0011806375442739079, 0.004651162790697674, 0.0054249547920434, 0, 0, 0, 0, 0.0035714285714285713, 0.004310344827586207, 0, 0, 0, 0.012121212121212121, 0.007352941176470588, 0, 0, 0.010638297872340425, 0, 0, 0.00558659217877095, 0, 0.005780346820809248, 0, 0.012096774193548387, 0, 0.023474178403755867 ]
0.004482
5
[ "9 Benefits of Adding Carbohydrates to Your Diet\n\nCarbohydrates have gained a somewhat negative reputation from the media and so-called fitness gurus in the past couple of decades. ", "Carbs have been demonized as the cause of diseases such as diabetes and heart problems. ", "Many claims have also been made that consuming carbohydrates can make people overweight and unhealthy. ", "Modern diets do indeed consist of a lot of processed carbs and artificial fats that are definitely unhealthy. ", "But generalizing all types of carbohydrates as bad is absolutely wrong.", "\n\nAs a matter of fact, science backs the benefits of carbs, and it is quite a long list. ", "These benefits include improved energy, brain performance, heart health, mood, weight loss, metabolism, and even a reduced risk of cancer. ", "Eating carbohydrates (the natural ones) is not only good but actually necessary for the body’s optimal function and processes.", "\n\nSo, here is a breakdown of the benefits of carbohydrates and why you should add these to your diet.", "\n\nPasta is a great source of carbohydrates in your diet. (", "Image Source: Pexels)\n\nIt Provides Your Body with Energy\n\nCarbohydrates are the human body’s main source of fuel. ", "When food is broken down into starches and sugars and absorbed into the bloodstream, they turn into glucose (blood sugar). ", "Your body needs this glucose for energy to perform various activities. ", "The brain also needs it to work properly. ", "Not having the right level of glucose in the bloodstream can cause anyone to feel lethargic, weak, and lose focus.", "\n\nIt Can Boost Your Mood\n\nResearchers theorize that carbohydrates promote the production of serotonin (the feel-good chemical). ", "A study from the Archives of Internal Medicine shows that people who followed a low-carb diet for a year (20 to 40 grams of carbohydrates, or 1/2 cup of rice and a slice of bread) experienced anger, anxiety, and depression. ", "This statistic is higher than those with a low-fat and high-carb diet (whole grains, low-fat dairy, beans, and fruits).", "\n\nIt Promotes Weight Loss\n\nCarbohydrates have been blamed for weight gain, but they are actually important for healthy weight control. ", "The Center for Disease Control and Prevention (CDC) even recommends eating 14 grams of fiber for every 1000 calories per day.", "\n\nA research study from Brigham Young University followed middle-aged women’s eating habits for nearly two years. ", "They found out that those with higher amounts of fiber intake generally lost weight while women with lower levels of fiber in their diet gained a couple of pounds. ", "They explained that many forms of carbs contain dietary fiber, which is indigestible complex carbohydrates.", "\n\nComplex carbs make you feel full for a longer period of time as the body breaks it down slowly. ", "Eating complex carbohydrates can definitely help you eat less and control your caloric intake. ", "This is true if you eat a moderate amount of whole grains and a lot of fiber.", "\n\nMake sure you get your carbs from a variety of food sources. (", "Image Source: Pexels)\n\nAnother piece of research from the Journal of Nutrition shows that eating whole grains instead of refined grains can help the body shed off body and belly fat. ", "The study includes adults who ate three servings of whole grains every day and had around 2.4% less body fat and 3.6% less abdominal fat. ", "This is in comparison with participants who ate a quarter of a serving. ", "The intake of slow-release carbs (bran cereal, oatmeal, etc.) ", "three hours before exercise can help the body burn more fat.", "\n\nIt Promotes Better Digestion\n\nFiber-rich diet help reduce common digestive problems like constipation and indigestion. ", "Insoluble fiber (the fiber that does not break down in digestion) pushes other food along the digestive tract, which speeds up digestion. ", "This also adds bulk to the stool, which makes passing bowel movements a lot easier.", "\n\nIt Promotes Heart Health\n\nStudies show that increasing the soluble-fiber intake by 10 grams a day could lower the “bad cholesterol” LDL by as much as 5%. ", "On the other hand, those who eat more grains (bulgur, brown rice, quinoa, etc.) ", "do not only have lower LDL cholesterol but have higher levels of “good cholesterol” HDL.", "\n\nDietary fiber works by preventing the accumulation of bad cholesterol in the arteries. ", "You can, therefore, avoid having blockages that can lead to a stroke or heart attack. ", "Eating more whole-grains, fresh fruits and vegetables, oats, and whole wheat can lower your risk of heart attack and other related ailments.", "\n\nIn contrast, simple carbs from processed foods (cakes, cookies, etc.) ", "and products made from white flour tend to have a higher amount of sugar and fat.", "\n\nIt Keeps You Sharp\n\nA study from Tufts University showed that those who followed a low-carb diet for a week did worse on a working memory test and visuospatial memory. ", "This data was compared to other test subjects who had a low-calorie diet (based on American Dietetic Association guidelines).", "\n\nIt Improves Sleep Patterns\n\nFeeling sluggish? ", "You might want to add more food rich in slow-digesting carbs to your diet for a restful and rejuvenating sleep. ", "Slow-digesting carbohydrates promote the production of serotonin (feel-good chemicals), a neurotransmitter that helps ensure sound sleep. ", "Diets low in serotonin often result in insomnia.", "\n\nIt Improves Athletic Performance\n\nCarbohydrates are an essential part of any athlete’s diet. (", "Image Source: Pexels)\n\nAthletes often have diets rich in carbs to help them with their endurance and intermittent high-intensity performance. ", "It is also well documented that players need to replenish their carbohydrate storage, especially during periods of intense competition or training. ", "In fact, various forms of carbs are typically taken before, during, and after a physical event.", "\n\nIf you are always engaged in physical activities such as sports, or you’re active throughout the day, you must have good carbs in your diet to supply your body with energy.", "\n\nIt Can Reduce Risks of Cancer\n\nMost people think of potatoes and whole wheat as the only options for good carbs. ", "But there are literally hundreds of different food choices out there that are excellent sources of healthy carbohydrates.", "\n\nFood rich in antioxidants such as bell peppers, onions, tomatoes, and other vegetables can combat abnormal cellular growth. ", "The high-fiber nature of most vegetables promotes the removal of cholesterol and waste in the body. ", "With this, experts suggest there are benefits of wholesome carbohydrates in fighting cancer in its early stages, as the cells need glucose for fuel. ", "Eating food that is slow to convert into glucose reduces the supply of nutrients to cancer cells.", "\n\nParting Tips\n\nThe benefits of carbohydrates for general well-being cannot be denied. ", "They are an essential part of any healthy diet. ", "But do keep in mind that not all carbs are the same. ", "Different types contain a host of vitamins and minerals that the body needs. ", "Just keep in mind that eating any particular food group in excess tends to not be good for the body.", "\n\nIt is always best to consult with your physician or dietitian to help you find the best carbohydrates for your fitness goals and current health condition. ", "If you are on a vegan diet and want to find out how carbohydrates can fit into your diet plan, find out more here.", "\n\nWelcome to A Nation of Moms! ", "I'm Marysa, a busy mom of two girls, and our family lives in Upstate NY. ", "I juggle work, the kid's activities, family life, running a Girl Scout Troop, and blogging! ", "We love the great outdoors, vegetarian food, camping, travel, swimming, reading, and more.. plus sharing our experiences on the blog!" ]
{ "pile_set_name": "Pile-CC" }
[ 0.005555555555555556, 0, 0, 0, 0, 0, 0, 0, 0, 0, 0, 0, 0, 0, 0, 0, 0.004464285714285714, 0, 0, 0.016, 0.008771929824561403, 0, 0, 0, 0, 0, 0, 0.00546448087431694, 0, 0, 0, 0, 0, 0, 0, 0.00641025641025641, 0.0125, 0.011363636363636364, 0, 0, 0, 0, 0, 0.0058823529411764705, 0.008, 0, 0, 0, 0, 0, 0, 0, 0, 0, 0, 0, 0.007936507936507936, 0, 0, 0, 0, 0, 0, 0, 0, 0, 0, 0, 0.0136986301369863, 0.010869565217391304, 0 ]
0.001647
5
[ "Taking on Taiwan: from Taroko Gorge and Taipei\n\nI'M standing in Taiwan's jaw-dropping Taroko Gorge with a hard hat on, looking like the construction worker out of the Village People.", "\n\nJust as I am about to break into a bad karaoke version of Macho Man, a member of our tour group runs up to tell us that a rock the size of his fist has just landed right next to his head.", "\n\nSuddenly, the idea of having permanent helmet hair for the sake of safety doesn't seem like such a bad idea and I keep my hard hat firmly attached to my head for the rest of the trip to the gorge.", "\n\nDespite the risk of falling rocks, Taroko Gorge is a stunning destination to visit, even on a rainy day.", "\n\nThe road into the gorge is a modern engineering miracle, and small shrines to the 200 or more workers who died in the construction of the inland road stand near the entrance to the gorge, which is on Taiwan's stunning east coast.", "\n\nTaroko Gorge is north of Hualien, and home to one of the 12 recognised indigenous tribes that were Taiwan's original inhabitants.", "\n\nLong before the Dutch, Japanese and Chinese occupied the island, there were warring tribes living in Taiwan's mountainous interior and verdant coastal lowlands.", "\n\nThe Truku tribe certainly picked a stunning piece of real estate to occupy, as Taroko Gorge is now a national park centred on the Liwu River, which carved a 20km river valley through a solid mountain of marble.", "\n\nDotted along Taiwan's east coast are a number of other indigenous tribes and associated tourist activities, including traditional fishing and food preparation.", "\n\nThere's nothing like capturing your own nile perch in a hollow bamboo fish trap and then minutes later having it cooked in a herb-filled broth filled with hot rocks straight from the fire.", "\n\nThe east coast of Taiwan is less sparsely populated than the west side of the island. ", "A huge dividing range of mountains dominates every view, and there is something like 300 peaks over 3000m.\n\nThe highways and train tracks hug the coastline, and there are some decent beaches to be seen, even by Australian standards.", "\n\nWorld-class scuba diving and snorkelling is available off Green Island, which is easily accessible from the coastal city of Taitung.", "\n\nThirty-three kilometres ff the coast of Taiwan, Green Island has stunning coral reefs to dive upon and the country's only saltwater hot spring at Chaojih Hot Springs.", "\n\nThe approach of typhoon Noul prevents our particular tour group from visiting, and we hear later in the day that the island had to be evacuated.", "\n\nAlthough Taipei is Taiwan's capital and home to three million people, Kaohsiung, on the southern tip of the island, is not too far behind.", "\n\nKaohsiung, which was once Asia's busiest port, has some lovely night markets full of every food you could imagine.", "\n\nThere are massive oysters the size of a man's hands, and octopus on a stick resembling a fan of coral more than a seafood, not to mention \"stinky tofu\", which is fairly famous in Taiwan and looks, smells and tastes a bit like blue vein cheese. ", "One of the \"must-sees\" in nearby Tainan is a performance by the Ten Drum Art Percussion Group.", "\n\nKaohsiung and Taipei are linked by a very fast train capable of speeds of up to 300kmh. ", "A trip that used to take six hours by train now takes about one and a half hours.", "\n\nTaipei is like a cross between Hong Kong and Singapore, and has what used to be the tallest building in the world, Taipei 101, and what remains the world's fastest lift to the top floor.", "\n\nTaipei is home to some ritzy shops and colourful karaoke bars, and in the grungy Ximending district is the Modern Toilet Restaurant, where the seats are, literally, toilet seats, and ice sundaes are served out of toilet bowls.", "\n\nAs many countries tend to shun Taiwan in a diplomatic sense, it's easy to leave it off the Asian tourist trail, but it deserves a second look for its mountain trails, stunning coastline, food and culture.", "\n\n* The writer was a guest of Tourism Taiwan.", "\n\nTAIWAN FAST FACTS\n\nGetting there: Eva Air, Taiwan's national carrier, has two services a week, Brisbane to Taipei\n\nMust see: Chiang Kai-shek's memorial and hourly changing of the guard.", "\n\nMust do: Go for Peking Duck at Taipei's Celestial Restaurant\n\nMust stay: Regent Hotel, Taipei. ", "Worth it for the breakfast buffet alone" ]
{ "pile_set_name": "Pile-CC" }
[ 0.005494505494505495, 0.005291005291005291, 0, 0.009433962264150943, 0, 0, 0, 0, 0, 0, 0, 0, 0, 0.005952380952380952, 0, 0, 0, 0, 0, 0, 0, 0, 0.0043859649122807015, 0, 0.022222222222222223, 0.0106951871657754, 0.020618556701030927, 0 ]
0.003003
5
[ "Waiting for the bus: when base-rates refuse to be neglected.", "\nThe paper reports the results from 16 versions of a simple probability estimation task, where probability estimates derived from base-rate information have to be modified by case knowledge. ", "In the bus problem [adapted from Falk, R., Lipson, A., & Konold, C. (1994). ", "The ups and downs of the hope function in a fruitless search. ", "In G. Wright & P. Ayton (Eds.), ", "Subjective probability (pp. ", "353-377). ", "Chichester, UK: Wiley], a passenger waits for a bus that departs before schedule in 10% of the cases, and is more than 10 min delayed in another 10%. ", "What are Fred's chances of catching the bus on a day when he arrives on time and waits for 10 min? ", "Most respondents think his probability is 10%, or 90%, instead of 50%, which is the correct answer. ", "The experiments demonstrate the difficulties people have in replacing the original three-category 1/8/1 partitioning with a normalized, binary partitioning, where the middle category is discarded. ", "In contrast with typical studies of \"base-rate neglect\", or under-weighing of base-rates, this task demonstrates a reversed base-rate fallacy, where frequentistic information is overextended and case information ignored. ", "Possible explanations for this robust phenomenon are briefly discussed." ]
{ "pile_set_name": "PubMed Abstracts" }
[ 0, 0, 0.013157894736842105, 0, 0.0625, 0, 0, 0, 0.010101010101010102, 0, 0, 0, 0 ]
0.006597
5
[ "Introduction\n============\n\nAnkylosing spondylitis (AS) is a chronic, inflammatory, systemic, rheumatic disease that primarily affects the axial skeleton, peripheral joints, and entheses \\[[@B1]\\]. ", "Symptoms of AS include pain, joint stiffness, and the loss of spinal mobility. ", "These clinical symptoms and the subsequent disease progression result in substantial functional limitations and impairment of health-related quality of life (HRQOL) \\[[@B2]-[@B5]\\]. ", "Patient-reported outcome (PRO) measures have been used to provide information on the effectiveness of treatment on symptoms, functioning, and well-being outcomes. ", "PRO measures are necessary tools, given the impact of AS on HRQOL domains, especially pain, physical function, fatigue, and psychological well-being.", "\n\nCurrently, two PRO instruments have been employed in the evaluation of HRQOL in AS. ", "These are the Short Form 36 (SF-36) Health Survey, a generic measure of health status \\[[@B6]\\], and the AS Quality of Life Questionnaire (ASQOL) \\[[@B7]\\]. ", "Many AS studies have used the SF-36 \\[[@B8]-[@B19]\\], whereas use of the ASQOL has been somewhat limited \\[[@B9],[@B16],[@B20],[@B21]\\]. ", "These measures have demonstrated HRQOL impairment and loss of physical functioning for patients with AS, compared with the general population. ", "Using median SF-36 summary scores, van der Heijde and colleagues \\[[@B15]\\] demonstrated that baseline values of SF-36 Physical Component Summary (PCS) scores in patients with AS were less than the scores for the general populations of the US and Europe, while SF-36 Mental Component Summary (MCS) scores were comparable with those general populations. ", "At least one other study reported statistically lesser baseline SF-36 scores for all eight SF-36 domains, especially those pertaining to physical function, for patients with AS compared with the US general population \\[[@B22]\\].", "\n\nThe availability of new imaging techniques, therapies, and treatments over the past several years has changed the management of AS \\[[@B23]\\]. ", "Previously, treatment options for AS were limited to nonsteroidal anti-inflammatory drugs (NSAIDs) and physiotherapy. ", "With the availability of TNF antagonists (infliximab, adalimumab, and etanercept), AS patients have experienced improvements in the signs and symptoms of their disease. ", "In clinical trials of TNF antagonists, improvements in clinical symptoms of AS have been sustained, lasting up to three years \\[[@B11],[@B24]\\], although treatment discontinuation has been associated with relapse \\[[@B24],[@B25]\\]. ", "TNF antagonists in the treatment of AS have proven efficacy for patients experiencing treatment-resistant forms of AS \\[[@B26],[@B27]\\], for patients seeking to reduce the use of NSAIDs and analgesics \\[[@B28]\\], and for patients seeking effective low-dosage \\[[@B29],[@B30]\\] and less-frequent treatments \\[[@B12],[@B31]\\].", "\n\nAdalimumab, a fully human monoclonal antibody targeted against TNF, has demonstrated short-term improvement in clinical signs and symptoms, physical function, and HRQOL in patients with active AS \\[[@B16],[@B32]\\]. ", "Maintenance of improvement in clinical signs and symptoms (without new safety issues) has been demonstrated with adalimumab in patients who continued in an open-label extension trial \\[[@B33]\\]. ", "However, in light of the substantial impairment of physical function and HRQOL observed in patients with active AS, additional long-term data are needed to determine the maintenance of benefit on both physical function and quality of life. ", "Sustained improvements in HRQOL have been demonstrated through 72 weeks of etanercept therapy based on the SF-36 PCS and the EQ-5D \\[[@B19]\\] and through two years of infliximab therapy based on the SF-36 PCS \\[[@B18]\\], but neither study assessed HRQOL using the ASQOL, an AS-specific questionnaire. ", "The primary objective of our analyses was to demonstrate the effectiveness of adalimumab after three years of treatment in improving and sustaining patient-reported physical functioning and HRQOL.", "\n\nMaterials and methods\n=====================\n\nPatients and study design\n-------------------------\n\nThe Adalimumab Trial Evaluating Long-Term Efficacy and Safety in AS (ATLAS) was a multicenter, randomized, double-blind, placebo-controlled, phase III study designed to demonstrate the safety and efficacy of adalimumab in patients with active AS. ", "Complete study information for ATLAS has been previously published \\[[@B16],[@B17],[@B32]\\]. ", "Patients who were at least 18 years of age were recruited from 43 sites (21 in the US and 22 in Europe). ", "Eligibility criteria included a diagnosis of AS according to the modified New York criteria \\[[@B34]\\] and an inadequate response or intolerance to at least one NSAID. ", "Patients for whom one or more disease-modifying antirheumatic drug had failed were also allowed to participate. ", "Patients were randomized in a 2:1 ratio to receive either adalimumab 40 mg or matching placebo subcutaneously every other week for 24 weeks (Abbott Laboratories, Abbott Park, IL, USA). ", "Participants who did not achieve at least a 20% response according to the Assessment of SpondyloArthritis International Society criteria for improvement at weeks 12, 16, or 20 were eligible to receive open-label treatment with adalimumab 40 mg every other week (early escape therapy). ", "After week 24, patients were eligible to continue adalimumab treatment in an open-label extension study for up to five years.", "\n\nClinical and HRQOL data from both the 24-week double-blind period and the open-label extension of the ATLAS clinical trial were used for the data analyses described in this report. ", "Relevant institutional review boards at participating clinical centers approved the protocol, and all patients provided voluntary written informed consent.", "\n\nMeasures\n--------\n\nThree PRO instruments -- the Bath AS Functional Index (BASFI) \\[[@B35]\\], the SF-36 Health Survey \\[[@B6]\\], and the ASQOL \\[[@B7]\\] -- were used in the long-term follow-up of adalimumab treatment. ", "The Bath AS Disease Activity Index (BASDAI) was included as a measure of disease activity \\[[@B36]\\].", "\n\n### BASFI\n\nThe BASFI consists of 10 questions designed to determine the degree of functional limitation in patients with AS. ", "Each question is answered using a 10-cm visual analogue scale (VAS), with a recall period of the past week. ", "The mean of the 10 scales gives the BASFI score -- a value between 0 and 10, with a lower score indicating less functional limitation. ", "The BASFI has been found to be reliable and sensitive to changes in AS \\[[@B35],[@B37]\\]. ", "Pavy and colleagues \\[[@B38]\\] suggested a BASFI change of 17.5% is the minimum clinically important difference for AS patients. ", "For our study, however, we employed a 21% or higher BASFI change as clinically meaningful, \\[[@B39]\\] because it is more conservative and more consistent with Assessment in AS International Group Criteria for 20% improvement (ASAS20) response. ", "The ASAS20 response includes assessment of BASFI scores \\[[@B40]\\], and was the primary endpoint of ATLAS.", "\n\n### SF-36\n\nSF-36 is a generic health status instrument and consists of eight domains: Physical Function, Bodily Pain, Role Limitations--Physical (Role-Physical), General Health, Vitality, Social Function, Role Limitations--Emotional (Role-Emotional), and Mental Health. ", "The recall period is four weeks, with greater scores reflecting better health status. ", "A 5- to 10-point change in domain scores is considered clinically meaningful for patients with rheumatoid arthritis \\[[@B41]\\]. ", "SF-36 also contains two summary scores (PCS and MCS) for which a 2.5 to 3.0-point change in summary scores is considered clinically meaningful for patients with rheumatoid arthritis \\[[@B41]\\]. ", "Previous studies in patients with AS have used three or more points to determine a clinically meaningful change \\[[@B16]\\].", "\n\n### ASQOL\n\nThe ASQOL is a disease-specific instrument designed to measure HRQOL in patients with AS \\[[@B7]\\]. ", "This questionnaire was developed according to a needs-based model, which postulates that life gains its quality from the ability of individuals to satisfy their needs. ", "The final instrument contains 18 yes/no items on the impact of AS \\'at this moment\\'. ", "The total score ranges from 0 to 18, with lesser scores representing better AS-specific quality of life. ", "Recent research has suggested that the ASQOL is a psychometrically sound and responsive measure of disease-specific quality of life in patients with AS and differences of one to two points are clinically relevant \\[[@B7],[@B42],[@B43]\\].", "\n\n### BASDAI\n\nThe BASDAI is a patient-reported measure of AS disease activity. ", "This index uses six 10-cm horizontal VASs to measure the severity of fatigue, spinal and peripheral joint pain, localized tenderness, and morning stiffness in patients with AS. ", "The final BASDAI score has a range of 0 to 10; a lesser number represents less severe disease activity \\[[@B36]\\].", "\n\nStatistical analyses\n--------------------\n\nThe statistical analyses used data from the ATLAS clinical trial. ", "A two-tailed *P*value of 0.05 was used to judge statistical significance. ", "There was no adjustment for multiple statistical comparisons; however, interpretation of significant findings took the number of statistical tests into account. ", "The sample included all patients randomized in the ATLAS trial who had a baseline PRO assessment and at least one follow-up PRO assessment for the analyses of the 24-week double-blind study period. ", "Furthermore, we included only those patients who had a week-24 BASFI assessment and at least one more post-24-week assessment. ", "For the long-term extension period, the analyses are based on actual study visit data (ie, weeks from baseline of the ATLAS trial), not on duration of exposure to adalimumab.", "\n\nWe compared baseline demographic variables, selected clinical characteristics, and PRO measures between the randomized, double-blind clinical trial patient sample and the open-label extension patient sample to evaluate the extent of sample bias. ", "We used chi-square tests for categorical data and paired Student *t-*tests for independent groups for continuous variables.", "\n\nWe compared baseline to week-24 changes in mean BASDAI, BASFI, SF-36 summary scales and subscales, and ASQOL scores between placebo-treated and adalimumab-treated groups during the double-blind treatment period using analysis of covariance (ANCOVA) models. ", "ANCOVA models included baseline PRO score and treatment group. ", "Last-observation-carried-forward (LOCF) procedures were used to account for PRO scores after patients went into early escape therapy or who discontinued from the study. ", "If patients started on early escape treatment before the 24-week endpoint, their last complete pre-escape therapy PRO scores were used as the endpoint score for the LOCF.", "\n\nFor selected PRO measures (SF-36 PCS, BASFI, and ASQOL), chi-square tests were used to compare the percentage of responders, based on changes from baseline to week 24, between the placebo-treated and adalimumab-treated groups. ", "The minimum clinically important difference (MCID) from baseline for each PRO was defined as follows: SF-36 PCS responder, three or more point reduction; BASFI responder, 21% or more reduction; and ASQOL responder, 1.8 or more point reduction. ", "The percentages of responders for these PRO measures also were evaluated from baseline to each open-label extension follow-up visit, using these same MCID definitions.", "\n\nLong-term follow-up analyses for the open-label extension period were completed using observed data. ", "For the BASDAI, BASFI, SF-36 summary and subscale, and ASQOL scores, we calculated the change in scores from baseline to each follow-up visit and used paired Student *t-*tests to assess significance of the observed changes. ", "Changes from week 24 to each long-term follow-up visit also were calculated for the BASDAI, BASFI, and SF-36 summary and ASQOL scores. ", "For the BADAI, BASFI, and PCS scores, we also examined mean changes during the open-label extension (i.e., from week 24 on), based on LOCF analyses.", "\n\nEffect sizes were calculated for changes in PRO measures as (pretreatment mean -- posttreatment mean)/pretreatment standard deviation \\[[@B44]\\]. ", "Effect sizes were classified as small (0.20), moderate (0.50), or large (≥ 0.80) \\[[@B45]\\].", "\n\nResults\n=======\n\nPatients\n--------\n\nA total of 315 patients with active AS participated in the ATLAS study; 208 were randomized to receive adalimumab and 107 to receive placebo. ", "Most patients were white (95.6%) and male (74.9%). ", "The average age was 42.2 years, and mean disease duration was 10.9 years. ", "A total of 288 patients (91.4%) entered the open-label extension phase of the study. ", "Of these 288, 236 (81.9%) had data for three years beyond their baseline visits in the ATLAS study. ", "Of the 52 patients who enrolled in the open-label extension but did not complete three years, 17 discontinued because of adverse events.", "\n\nBaseline assessments\n--------------------\n\nBaseline demographic variables and mean PRO measures were comparable between the 24-week, randomized, double-blind clinical trial sample and those who participated in the open-label extension study (Table [1](#T1){ref-type=\"table\"}).", "\n\n###### \n\nBaseline demographic and clinical characteristics of patients with AS: comparison of patients entering double-blind and open-label extension study periods\n\n Week 24 of ATLAS (n = 315) Open-label extension (n = 288) *P value*^a^\n ------------------------- ---------------------------- -------------------------------- --------------\n Age, years 42.2 ± 11.57 42.4 ± 11.65 0.848\n Male, n (%) 236 (74.9) 219 (76.0) 0.777\n White, n (%) 301 (95.6) 276 (95.8) 1.000\n Disease duration, years 10.9 ± 9.47 10.9 ± 9.43 0.953\n BASDAI score, 0--10 cm 6.3 ± 1.69 6.3 ± 1.70 0.997\n BASFI score, 0--10 cm 5.4 ± 2.21 5.4 ± 2.19 0.900\n SF-36 PCS, 0--50 32.5 ± 7.98 32.4 ± 8.00 0.846\n SF-36 MCS, 0--50 43.7 ± 11.57 44.0 ± 11.48 0.804\n SF-36 domain scales \n  Physical Function 47.1 ± 22.08 46.8 ± 22.49 0.893\n  Role--Physical 20.3 ± 30.02 20.9 ± 30.25 0.808\n  Bodily Pain 55.9 ± 25.06 55.9 ± 25.08 0.979\n  General Health 42.6 ± 19.82 42.6 ± 20.06 0.997\n  Vitality 31.1 ± 16.12 30.9 ± 16.12 0.875\n  Social Function 33.1 ± 17.49 32.9 ± 17.30 0.895\n  Role--Emotional 54.4 ± 42.98 56.1 ± 42.90 0.641\n  Mental Health 61.7 ± 19.19 61.8 ± 19.17 0.952\n ASQOL, 0--18 10.3 ± 4.29 10.3 ± 4.29 0.945\n\nData are mean ± standard deviation unless otherwise noted.", "\n\n^a^*P*-values for comparing means based on two-sample paired Student *t*-tests; *P*-values for comparing percentages based on Fisher\\'s exact tests.", "\n\nAS = ankylosing spondylitis; ASQOL = AS Quality of Life Questionnaire; ATLAS = Adalimumab Trial Evaluating Long-Term Efficacy and Safety in AS; BASDAI = Bath AS Disease Activity Index; BASFI = Bath AS Functional Index; HRQOL = health-related quality-of-life; MCS = Mental Component Summary; PCS = Physical Component Summary; SF-36 = Short Form-36 Health Survey.", "\n\n### Baseline to week-24 double-blind results\n\nThere were statistically significant differences between the placebo-treated and adalimumab-treated groups for baseline to week-24 changes in the PRO measures (*P*= 0.015 to *P*\\< 0.001), except for the SF-36 MCS (*P*= 0.181) and Mental Health subscale scores (*P*= 0.551; Table [2](#T2){ref-type=\"table\"}). ", "For all PRO endpoints, the adalimumab group reported greater improvements.", "\n\n###### \n\nMean changes in PRO measures from baseline to week 24 of the ATLAS study\n\n PRO measure Placebo (n = 107) Adalimumab (n = 208) Overall *F*value^a^ *P*value^c^\n -------------------- ------------------- ---------------------- --------------------- -------------\n BASDAI, 0--10 cm -1.0 ± 0.23 -2.8 ± 0.17 30.30^b^ \\< 0.001\n BASFI, 0--10 cm -0.5 ± 0.19 -2.0 ± 0.14 23.14^b^ \\< 0.001\n SF-36 PCS, 0--50 2.1 ± 0.84 7.3 ± 0.59 24.56^b^ \\< 0.001\n SF-36 MCS, 0--50 2.1 ± 0.95 3.7 ± 0.67 42.05^b^ 0.181\n  Physical Function 4.1 ± 2.00 13.3 ± 1.42 19.32^b^ \\< 0.001\n  Role--Physical 10.0 ± 3.51 27.5 ± 2.51 23.00^b^ \\< 0.001\n  Bodily Pain 7.0 ± 2.03 20.7 ± 1.45 24.39^b^ \\< 0.001\n  General Health 1.5 ± 1.55 8.7 ± 1.11 21.89^b^ \\< 0.001\n  Vitality 5.8 ± 1.86 14.6 ± 1.33 21.33^b^ \\< 0.001\n  Social Function 5.0 ± 2.11 12.6 ± 1.51 34.86^b^ 0.003\n  Role--Emotional 5.0 ± 3.55 15.6 ± 2.51 77.42^b^ 0.015\n  Mental Health 4.8 ± 1.54 5.9 ± 1.11 30.43^b^ 0.551\n ASQOL, 0--18 -1.1 ± 0.40 -3.5 ± 0.28 17.28^b^ \\< 0.001\n\nData are least-square mean ± standard error.", "\n\n^a^ANCOVA model controlling for baseline score of patient was performed.", "\n\n^b^*P*\\< 0.001 based on ANCOVA.", "\n\n^c^*P-*values based on paired Student *t-*tests for comparison of means between placebo-treated and adalimumab-treated patients.", "\n\nANCOVA = analysis of covariance; ASQOL = AS Quality of Life Questionnaire; ATLAS = Adalimumab Trial Evaluating Long-Term Efficacy and Safety in AS; BASDAI = Bath AS Disease Activity Index; BASFI = Bath AS Functional Index; MCS = Mental Component Summary; PCS = Physical Component Summary; PRO = patient-reported outcome; SF-36 = Short Form-36 Health Survey.", "\n\nSimilar differences were observed between placebo-treated and adalimumab-treated patients for the BASFI and ASQOL measures. ", "For the SF-36 subscale and MCS scores, all differences were statistically significant (*P*\\< 0.05), except for the SF-36 MCS (*P*= 0.075), Social Function subscale (*P*= 0.243), and Mental Health subscale (*P*= 0.716) (data not shown). ", "In the responder analysis at week 24, a greater percentage of the adalimumab-treated group demonstrated changes exceeding the *a priori*-stated MCIDs compared with the placebo group for the SF-36 PCS (*P*\\< 0.001), BASFI (*P*\\< 0.001), and ASQOL (*P*\\< 0.001; Table [3](#T3){ref-type=\"table\"}). ", "Sixty-seven percent of the adalimumab-treated patients exceeded the three-point MCID for the SF-36 PCS, compared with 39.8% of the placebo-treated patients.", "\n\n###### \n\nResponder status for primary PRO measures from baseline to week 24 of the ATLAS study\n\n ---------------------------------------------------------------------------------------------------\n Placebo\\ Adalimumab 40 mg every other week\\ *P*value^a^\n (n = 107)\\ (n = 208)\\ \n n (%) n (%) \n ----------------------------------- ------------ ------------------------------------ -------------\n SF-36 PCS, 0--50 \n\n  Responder, ≥ 3-point reduction 41 (39.8) 138 (67.0) \\< 0.001\n\n BASFI, 0--10 \n\n  Responder, ≥ 21% reduction 38 (35.5) 142 (68.6) \\< 0.001\n\n ASQOL, 0--18 \n\n  Responder, ≥ 1.8-point reduction 46 (43.0) 135 (64.9) \\< 0.001\n ---------------------------------------------------------------------------------------------------\n\n^a^*P-*values based on chi-square test.", "\n\nASQOL = AS Quality of Life Questionnaire; ATLAS = Adalimumab Trial Evaluating Long-Term Efficacy and Safety in AS; BASFI = Bath AS Functional Index; PCS = Physical Component Summary; PRO = patient-reported outcome; SF-36 = Short Form-36 Health Survey.", "\n\nThree-year open-label results\n-----------------------------\n\nTable [4](#T4){ref-type=\"table\"} summarizes the changes from baseline to each follow-up assessment during the three-year treatment period for the BASDAI, BASFI, SF-36 PCS and MCS, SF-36 subscales, and ASQOL scores. ", "Statistically significant changes from baseline were observed for all of the BASDAI (all *P*\\< 0.001) and BASFI scores (all *P*\\< 0.001), at all time points. ", "The time course of mean scores for BASDAI and BASFI from baseline to the three-year endpoint is provided in Figures [1](#F1){ref-type=\"fig\"} and [2](#F2){ref-type=\"fig\"}, respectively. ", "Effect sizes for the BASFI scores ranged from 1.18 to 1.35. ", "Based on LOCF, mean changes in BASDAI scores were -3.44 (*P*\\< 0.001) at year 1, -3.65 (*P*\\< 0.001) at year 2, and -3.49 (*P*\\< 0.001) at year 3. ", "Mean change scores for BASFI via LOCF were -2.50 (*P*\\< 0.001) at year 1, -2.64 (*P*\\< 0.001) at year 2, and -2.63 (*P*\\< 0.001) at year 3.", "\n\n![", "Mean BASDAI scores during long-term adalimumab treatment. ", "Data are observed values for completers. ", "Refer to Tables 4 or 6 for number of patients at each time point. ", "The Bath Ankylosing Spondylitis Disease Activity Index (BASDAI) measures the severity of fatigue, spinal and peripheral joint pain, localized tenderness, and morning stiffness in patients with ankylosing spondylitis. ", "BASDAI scores range from 0 to 10, with lower scores indicating less severe disease activity.](ar2790-1){#F1}\n\n![", "Mean BASFI scores during long-term adalimumab treatment. ", "Data are observed values for completers. ", "Refer to Tables 4, 5, or 6 for number of patients at each time point. ", "The Bath Ankylosing Spondylitis Functional Index (BASFI) measures the degree of functional limitation in patients with ankylosing spondylitis. ", "BASFI scores range from 0 to 10, with a lower score indicating less functional limitation.](ar2790-2){#F2}\n\n###### \n\nChange in mean PRO measures from baseline to week 24 and follow-up visits^a^\n\n Time point from baseline of the ATLAS study \n ------------------------- --------------------------------------------- ------------- -------------- -------------- -------------- -------------- --------------\n BASDAI \n  n 288 278 274 270 262 242 236\n  Mean ± SD change -3.1 ± 2.42 -3.4 ± 2.41 -3.5 ± 2.55 -3.8 ± 2.33 -3.9 ± 2.44 -3.9 ± 2.39 -3.9 ± 3.39\n BASFI \n  n 288 ^b^ 274 270 261 242 236\n  Mean ± SD change -2.2 ± 1.91 ^b^ -2.6 ± 2.04 -2.8 ± 2.10 -2.9 ± 2.14 -2.9 ± 2.17 -3.0 ± 2.10\n SF-36 PCS \n  n 284 ^b^ 265 263 255 229 227\n  Mean ± SD change 8.2 ± 9.01 ^b^ 10.19 ± 9.50 10.8 ± 9.88 11.0 ± 9.88 11.3 ± 9.68 11.6 ± 9.65\n SF-36 MCS \n  n 284 ^b^ 265 263 255 229 227\n  Mean ± SD change 4.8 ± 10.27 ^b^ 5.6 ± 10.35 5.1 ± 11.06 5.7 ± 10.96 4.1 ± 10.84 5.6 ± 11.59\n SF-36 Physical Function \n  n 287 ^b^ 273 268 261 241 235\n  Mean ± SD change 15.6 ± 20.99 ^b^ 19.5 ± 21.38 21.8 ± 21.79 21.9 ± 22.26 22.2 ± 22.34 23.3 ± 21.94\n SF-36 Role--Physical \n  n 288 ^b^ 272 270 263 237 232\n  Mean ± SD change 30.3 ± 40.48 ^b^ 37.2 ± 40.81 35.2 ± 40.72 39.1 ± 41.75 36.1 ± 42.39 37.8 ± 43.90\n SF-36 Bodily Pain \n  n 288 ^b^ 274 270 263 237 233\n  Mean ± SD change 24.0 ± 21.78 ^b^ 29.0 ± 22.96 29.7 ± 23.24 29.9 ± 23.74 31.8 ± 24.42 31.7 ± 24.33\n SF-36 General Health \n  n 287 ^b^ 270 266 260 237 234\n  Mean ± SD change 8.7 ± 17.37 ^b^ 11.3 ± 18.68 12.7 ± 19.25 12.5 ± 19.92 12.0 ± 18.84 12.7 ± 18.71\n SF-36 Vitality \n  n 288 ^b^ 274 268 263 242 236\n  Mean ± SD change 16.3 ± 19.82 ^b^ 19.5 ± 19.96 20.1 ± 19.65 20.4 ± 20.05 18.4 ± 19.85 20.1 ± 19.70\n SF-36 Social Function \n  n 288 ^b^ 274 270 263 242 236\n  Mean ± SD change 16.1 ± 23.88 ^b^ 19.7 ± 24.50 18.5 ± 23.73 20.6 ± 25.36 17.9 ± 26.01 22.3 ± 25.16\n SF-36 Role--Emotional \n  n 286 ^b^ 271 268 260 235 230\n  Mean ± SD change 17.2 ± 41.24 ^b^ 21.0 ± 44.40 19.5 ± 46.74 21.5 ± 46.01 16.9 ± 44.80 20.0 ± 46.06\n SF-36 Mental Health \n  n 288 ^b^ 274 268 263 242 236\n  Mean ± SD change 8.8 ± 16.52 ^b^ 10.1 ± 17.15 10.2 ± 17.50 10.6 ± 16.63 8.7 ± 16.96 10.5 ± 18.32\n ASQOL \n  n 288 ^b^ 274 270 263 242 236\n  Mean ± SD change -4.1 ± 4.23 ^b^ -4.8 ± 4.41 -5.0 ± 4.32 -5.4 ± 4.28 -5.3 ± 4.35 -5.4 ± 4.36\n\n^a^All values *P*\\< 0.001 compared with baseline based on paired Student *t*-test.", "\n\n^b^Measure not assessed at this time point.", "\n\nASQOL = AS Quality of Life Questionnaire; ATLAS = Adalimumab Trial Evaluating Long-Term Efficacy and Safety in AS; BASDAI = Bath AS Disease Activity Index; BASFI = Bath AS Functional Index; MCS = Mental Component Summary; PCS = Physical Component Summary; PRO = patient-reported outcome; SD = standard deviation; SF-36 = Short Form-36 Health Survey.", "\n\nIn addition, statistically significant improvements were observed for both the SF-36 PCS and MCS scores (all *P*\\< 0.001). ", "Mean PCS scores improved by 8.2 points at week 24 and by 11.6 points at year 3 (Figure [3](#F3){ref-type=\"fig\"}). ", "The effect sizes for the PCS scores were 0.87, 0.97, and 1.04 at years 1, 2, and 3, respectively. ", "Mean changes in PCS scores by LOCF were 9.76 (*P*\\< 0.001) at year 1, 10.23 (*P*\\< 0.001) at year 2, and 10.03 (*P*\\< 0.001) at year 3. ", "The SF-36 Physical Function score also demonstrated statistically significant improvements of 15.6 (week-24 assessment) to 23.3 points (after three years of adalimumab exposure). ", "Comparable results were observed for the SF-36 MCS and other subscale scores (Table [4](#T4){ref-type=\"table\"}). ", "Mean changes from baseline in ASQOL scores improved at all time points from week 24 through to year 3 (all *P*\\< 0.001). ", "ASQOL effect sizes ranged from 1.11 to 1.26.", "\n\n![", "Mean SF-36 PCS scores during long-term adalimumab treatment. ", "Data are observed values for completers. ", "Refer to Tables 4, 5, or 6 for number of patients at each time point. ", "The Short Form-36 Health Survey (SF-36) Physical Component Summary (PCS) is a composite score of four physical functioning domains of the SF-36: Physical Functioning, Role--Physical, Bodily Pain, and General Health. ", "Greater scores indicate better health status.](ar2790-3){#F3}\n\nTable [5](#T5){ref-type=\"table\"} summarizes the responder analyses for the SF-36 PCS, BASFI, and ASQOL for up to three years of adalimumab treatment. ", "There were statistically significant percentages of responders for the SF-36 PCS scores from year 1 to year 3 (all *P*\\< 0.0001). ", "The percentage of responders was sustained from 75.8% (year 1) to 81.9% (year 3). ", "For the BASFI, we observed statistically significant percentages of responders (based on a 21% or greater improvement) across all assessments for up to three years of adalimumab treatment (all *P*\\< 0.001; Table [5](#T5){ref-type=\"table\"}). ", "The responder rate was sustained from year 1 to year 3, ranging from 73.0% to 81.8%. ", "Statistically significant percentages of ASQOL responders also were observed through up to three years of adalimumab treatment (all *P*\\< 0.001; Table [5](#T5){ref-type=\"table\"}). ", "When a responder was defined as an improvement of at least 1.8 points, the percentage of responders was consistent from year 1 (77.0%) to year 3 (79.7%).", "\n\n###### \n\nResponder status for primary PRO measures from baseline to long-term follow-up visits\n\n Primary PRO measure Time point from baseline of the ATLAS study \n ------------------------------------------------------- --------------------------------------------- --------------- --------------- --------------- --------------- ---------------\n SF-36 PCS responder, ≥ 3-point decrease from baseline \n  n ^b^ 265 263 255 229 227\n  Responders, n (%) ^b^ 201 (75.8)^a^ 197 (74.9)^a^ 204 (80.0)^a^ 182 (79.5)^a^ 186 (81.9)^a^\n BASFI responder, ≥ 21% reduction from baseline \n  n 278 274 270 261 242 236\n  Responders, n (%) 203 (73.0)^a^ 208 (75.9)^a^ 211 (78.1)^a^ 210 (80.5)^a^ 195 (80.6)^a^ 193 (81.8)^a^\n ASQOL responder, ≥ 1.8-point reduction from baseline \n  n ^b^ 274 270 263 242 236\n  Responders, n (%) ^b^ 211 (77.0)^a^ 212 (78.5)^a^ 213 (81.0)^a^ 193 (79.8)^a^ 188 (79.7)^a^\n\n^a^*P*\\< 0.001; chi-square test for equal proportions was performed.", "\n\n^b^Measure not assessed at this time point.", "\n\nASQOL = AS Quality of Life Questionnaire; ATLAS = Adalimumab Trial Evaluating Long-Term Efficacy and Safety in AS; BASFI = Bath AS Functional Index; PCS = Physical Component Summary; PRO = patient-reported outcome; SF-36 = Short Form-36 Health Survey.", "\n\nThe SF-36 Physical Function (all *P*\\< 0.001), Role--Physical (all *P*\\< 0.003), Bodily Pain (all *P*\\< 0.001), General Health (all *P*\\< 0.03), Vitality (*P*\\< 0.001), Social Function (all *P*\\< 0.001), and Role--Emotional (all *P*\\< 0.007) subscale scores also improved from baseline through year 3 (data not shown).", "\n\nTable [6](#T6){ref-type=\"table\"} summarizes the changes in selected PRO scores from the week-24 assessment to each open-label extension follow-up visit. ", "For the BASDAI, small but statistically significant changes were observed for all assessments up to year 3 (all *P*\\< 0.009), except for that observed at week 30 (*P*= 0.240). ", "Similarly, significant changes were observed from week 24 to each follow-up visit for BASFI scores (all *P*\\< 0.001), except for those observed at week 30 (*P*= 0.196) and week 36 (*P*= 0.100). ", "These change scores suggest maintenance of improvement in disease activity and functional outcomes during up to three years of adalimumab treatment. ", "SF-36 PCS scores also demonstrated statistically significant improvements from week 24 (*P*\\< 0.001; Table [6](#T6){ref-type=\"table\"}). ", "Changes in PCS scores ranged from 1.8 (year 1) to 2.6 (year 3). ", "No significant changes from week 24 were observed in the MCS (*P*\\> 0.05), with the exception of that observed at week 128 (*P*= 0.033). ", "Statistically significant improvements in ASQOL from week 24 were observed through year 3 (all *P*\\< 0.002).", "\n\n###### \n\nMean change in PRO measures from week 24 to each follow-up visit\n\n PRO measure Time point from baseline of the ATLAS study \n ------------------- --------------------------------------------- -------------- -------------- -------------- -------------- --------------\n BASDAI \n  n 278 274 270 262 242 236\n  Mean change ± SD -0.2 ± 1.41 -0.4 ± 1.59 -0.5 ± 1.65 -0.6 ± 1.87 -0.6 ± 1.80 -0.5 ± 2.88\n  *P*-value^a^ 0.004 \\< 0.001 \\< 0.001 \\< 0.001 \\< 0.001 0.009\n BASFI \n  n 278 274 270 261 242 236\n  Mean change ± SD -1.1 ± 10.68 -3.1 ± 13.07 -4.4 ± 14.22 -5.0 ± 15.90 -4.6 ± 15.87 -5.5 ± 14.27\n  *P-*value^a^ 0.100 \\< 0.001 \\< 0.001 \\< 0.001 \\< 0.001 \\< 0.001\n SF-36 PCS \n  n ^b^ 268 267 259 232 231\n  Mean change ± SD ^b^ 1.8 ± 6.71 2.2 ± 7.35 2.4 ± 7.60 2.5 ± 8.54 2.6 ± 7.94\n  *P-*value^a^ ^b^ \\< 0.001 \\< 0.001 \\< 0.001 \\< 0.001 \\< 0.001\n SF-36 MCS \n  n ^b^ 268 267 259 232 231\n  Mean change ± SD ^b^ 0.7 (7.55) 0.1 (7.50) 0.6 (7.37) -1.2 (8.60) 0.2 (8.29)\n  *P-*value^a^ ^b^ 0.150 0.892 0.176 0.033 0.718\n ASQOL \n  n ^b^ 274 270 263 242 236\n  Mean change ± SD ^b^ -0.5 (2.73) -0.7 (2.78) -1.0 (3.01) -0.8 (3.43) -1.0 (3.07)\n  *P-*value^a^ ^b^ 0.002 \\< 0.001 \\< 0.001 \\< 0.001 \\< 0.001\n\n^a^*P-*values based on paired Student *t-*test.", "\n\n^b^Measure not assessed at this time point.", "\n\nASQOL = AS Quality of Life Questionnaire; ATLAS = Adalimumab Trial Evaluating Long-Term Efficacy and Safety in AS; BASDAI = Bath AS Disease Activity Index; BASFI = Bath AS Functional Index; MCS = Mental Component Summary; PCS = Physical Component Summary; PRO = patient-reported outcome; SF-36 = Short Form-36 Health Survey.", "\n\nDiscussion\n==========\n\nThis long-term, open-label extension of the ATLAS study demonstrated maintenance of improvement in physical function and HRQOL scores in patients with AS treated with adalimumab for up to three years. ", "BASDAI scores demonstrated improvement in AS disease activity from baseline to the end of the 24-week double-blind treatment period and sustained improvement through to year 3. ", "The benefit of adalimumab on physical function was demonstrated with sustained improvements in BASFI and SF-36 PCS endpoints over the three-year course of treatment. ", "Improvements in the remaining SF-36 scores and the ASQOL provided further supportive evidence for more general and broad improvement in quality of life following therapy with adalimumab in patients with AS.", "\n\nDuring the 24-week double-blind treatment period, analyses of the ATLAS study data \\[[@B32]\\] demonstrated greater improvements in the adalimumab-treated patients compared with the placebo-treated patients for changes in the BASDAI, BASFI, SF-36 PCS, and ASQOL scores. ", "The SF-36 MCS scores did not differ between adalimumab and placebo groups. ", "The SF-36 subscale scores, except for the Mental Health domain, also demonstrated results favorable for adalimumab compared with placebo. ", "The changes seen in the SF-36 PCS scores exceeded the MCID and, therefore, are considered clinically meaningful. ", "Of adalimumab-treated patients, 67% reported a clinically meaningful improvement in SF-36 PCS scores compared with 40% in the placebo group. ", "The differences in the numbers of responders from baseline to week 24 for ASQOL scores also are considered clinically meaningful. ", "Sixty-five percent of adalimumab-treated patients exhibited a clinically significant response on the ASQOL compared with 43% in the placebo group.", "\n\nOver the course of the three-year open-label extension period, we observed significant improvements in all PRO scores, from baseline to each follow-up visit. ", "Changes in the BASFI over time exceed the MCID for the BASFI (ie, ≥ 21% improvement) and indicate that the observed improvement over the first 24 weeks of the ATLAS study was maintained for up to three years of adalimumab treatment. ", "Effect sizes for these changes in BASFI scores were large, ranging from 1.18 to 1.35. ", "The BASDAI scores indicated similar consistent improvements from baseline over the course of the three-year open-label extension study. ", "Seventy-six percent of study participants were classified as responders on the BASFI at year 1, and more than 80% were considered responders at years 2 and 3. ", "These findings are consistent with other long-term follow-up studies of other TNF antagonists in patients with AS \\[[@B8],[@B11],[@B25],[@B30]\\].", "\n\nThe changes observed in the SF-36 PCS scores over the three-year study demonstrated significant and clinically meaningful improvements in physical function and well-being with adalimumab treatment. ", "Previous research has indicated that the MCID for the SF-36 PCS is 2.5 to 3.0 points \\[[@B41]\\]; the changes of 10.1 points at year 1 to 11.6 points at year 3 consistently exceeded this value and, therefore, are clinically meaningful. ", "These improvements translate into effect sizes ranging from 1.26 to 1.45, which are considered very large for PRO endpoints \\[[@B46]\\]. ", "By year 1, 76% of the participating patients were classified as responders on the PCS (improvement ≥ 3 points), and 80% and greater were classified as responders at years 2 and 3. ", "These results are consistent with those of an earlier study of adalimumab in patients with AS, which demonstrated significant improvement in SF-36 PCS scores with 52-week open-label adalimumab treatment \\[[@B47]\\]. ", "Studies of other TNF antagonists in patients with AS also have detected similar improvements in SF-36 PCS and MCS scores during long-term follow-up studies \\[[@B8],[@B11],[@B22]\\].", "\n\nThe results from the physical well-being--related SF-36 subscale scores (ie, Physical Function, Bodily Pain, and Role--Physical) provide additional supportive evidence for the SF-36 PCS findings. ", "Changes in the Physical Function subscale score during long-term adalimumab treatment translate into effect sizes of 0.87, 0.97, and 1.04, for year 1, year 2, and year 3, respectively. ", "Bodily Pain and Role--Physical subscale scores indicated consistent and robust changes from baseline over the course of the open-label extension. ", "Improvements in Bodily Pain scores had large effect sizes (1.16 to 1.26), suggesting substantial improvements in self-reported pain during up to three years of adalimumab treatment (data not shown). ", "Comparable degrees of improvement were also observed for the remaining SF-36 subscale scores, although relatively less change and effect sizes were observed for the Mental Health related scores.", "\n\nAdalimumab treatment for up to three years improved AS-specific HRQOL, as measured by the ASQOL. ", "Clear and consistent improvements in mean ASQOL scores from baseline were observed over the course of the open-label extension study. ", "The observed effects are considered clinically significant as they exceed the 1.8-point MCID, and are associated with effect sizes ranging from 1.11 to 1.26. ", "More importantly, 77% of patients in the open-label extension study were classified as responders at year 1, and 80% were responders at years 2 and 3.", "\n\nThere are several potential limitations associated with this study. ", "First, the long-term results are based on an open-label extension study in which some placebo-treated patients switched to early escape adalimumab treatment from weeks 12 to 20, and all patients were switched to open-label adalimumab treatment after week 24. ", "The addition of these patients to the long-term follow-up may have attenuated the observed effect of adalimumab on PRO endpoints. ", "However, the data analyses found that despite the addition of these placebo-treated patients, the overall results suggest consistent and robust improvements in almost all PRO measures. ", "Second, the physical function and HRQOL measures are based on patients\\' self-reports, and responses to the questionnaires may have been affected by the patients\\' knowledge that they were receiving adalimumab treatment. ", "Although this knowledge may have had an initial impact on inflating PRO scores, it seems unlikely that this potential bias would have been sustained during the three-year follow-up period. ", "Third, although 82% of patients participating in the open-label extension study provided complete assessments at year 3, study dropouts may have slighted inflated the PRO endpoint scores. ", "And, finally, these data analyses were based on observed data. ", "However, when we employed LOCF, the results did not differ substantively from the observed-analysis results. ", "As expected, mean changes in PRO scores observed via LOCF were slightly lower than the mean changes seen with the observed analyses.", "\n\nConclusions\n===========\n\nIn conclusion, the results of this open-label extension study demonstrated the sustained benefit of the statistically significant physical function and HRQOL improvements observed with adalimumab during the initial 24 weeks of double-blind treatment \\[[@B16],[@B32]\\]. ", "Consistent and robust effects were seen on BASFI and SF-36 PCS scores, both indicating impact on physical functioning and well-being for up to three years. ", "These effects were also clinically meaningful. ", "The remaining HRQOL outcomes, including the remaining SF-36 subscale scores and the ASQOL, extend and further support these findings. ", "This study demonstrates that long-term treatment with adalimumab is associated with maintenance of effectiveness on physical functioning and well-being, as well as quality of life.", "\n\nAbbreviations\n=============\n\nANCOVA: analysis of covariance; AS: ankylosing spondylitis; ASAS20: Assessment in AS International Group Criteria for 20% improvement; ASQOL: AS Quality of Life Questionnaire; ATLAS: Adalimumab Trial Evaluating Long-Term Efficacy and Safety in AS; BASDAI: Bath AS Disease Activity Index; BASFI: Bath AS Functional Index; HRQOL: health-related quality-of-life; LOCF: last-observation-carried-forward; MCID: minimum clinically important difference; MCS: Mental Component Summary; NSAID: nonsteroidal anti-inflammatory drug; PCS: Physical Component Summary; PRO: patient-reported outcome; SF-36: Short Form-36 Health Survey; TNF: tumor necrosis factor; VAS: visual analog scale.", "\n\nCompeting interests\n===================\n\nDvdH has received consulting fees, research grants, and/or speaking fees from Abbott Laboratories, Amgen, Aventis, Bristol Meyers Squibb, Centocor, Pfizer, Roche, Schering-Plough, UCB, and Wyeth. ", "JS has received consulting fees, research grants, and/or speaking fees from Abbott Laboratories, Bristol-Meyers Squibb, Centocor, Pfizer, Roche, Schering-Plough, UCB, and Wyeth. ", "DR, NH, and CT are employees of United BioSource Corporation, which was contracted by Abbott Laboratories to complete the analyses reported here. ", "KLG, RLW, and HK are employees of Abbott Laboratories and own shares of Abbott stock.", "\n\nAuthors\\' contributions\n=======================\n\nDrs van der Heijde, Sieper, Wong, and Kupper (with other academic experts and members of Abbott Laboratories) designed the original clinical trial. ", "Drs van der Heijde and Sieper were members of the ATLAS Study Group who collected the clinical data. ", "Dr Revicki, Ms Thompson, and Ms Harnam completed the analyses. ", "Dr Revicki, Dr Gooch, and Ms Harnam drafted the manuscript. ", "All authors reviewed and approved the final content of the submitted manuscript.", "\n\nAcknowledgements\n================\n\nWe thank Dana L Randall, MS, PharmD, of Arbor Communications, Inc., Ann Arbor, Michigan, and Michael A. Nissen, ELS, of Abbott Laboratories, for assistance with manuscript editing, revision, and formatting. ", "This support was funded by Abbott Laboratories. ", "Abbott Laboratories funded the ATLAS study, provided study drug, and was responsible for collecting the clinical data in conjunction with the ATLAS Study Group. ", "Abbott Laboratories contracted with United BioSource Corporation to perform the current analyses.", "\n" ]
{ "pile_set_name": "PubMed Central" }
[ 0.005076142131979695, 0, 0.01098901098901099, 0, 0.006711409395973154, 0.011627906976744186, 0.025477707006369428, 0.051094890510948905, 0.006993006993006993, 0.0084985835694051, 0.0043859649122807015, 0.006896551724137931, 0, 0, 0.017241379310344827, 0.018518518518518517, 0.013824884792626729, 0.005128205128205128, 0.008333333333333333, 0.016611295681063124, 0, 0.005763688760806916, 0.043010752688172046, 0, 0.011904761904761904, 0, 0.005405405405405406, 0.0035087719298245615, 0, 0.01092896174863388, 0, 0.0273972602739726, 0.019801980198019802, 0.023622047244094488, 0, 0.007407407407407408, 0.03333333333333333, 0.015503875968992248, 0.01639344262295082, 0.02830188679245283, 0.022058823529411766, 0, 0.015625, 0.015463917525773196, 0.008130081300813009, 0.02654867256637168, 0, 0, 0, 0.016877637130801686, 0.02531645569620253, 0.005649717514124294, 0.017543859649122806, 0.009009009009009009, 0, 0, 0.005050505050505051, 0, 0.005747126436781609, 0, 0, 0.011583011583011582, 0, 0.005917159763313609, 0.0058823529411764705, 0.008733624454148471, 0.020491803278688523, 0.005988023952095809, 0, 0.013392857142857142, 0.022222222222222223, 0.02702702702702703, 0.006756756756756757, 0.021739130434782608, 0.011111111111111112, 0, 0, 0, 0.01, 0, 0, 0.0013908205841446453, 0, 0.01652892561983471, 0.019662921348314606, 0, 0.012017710309930424, 0, 0.030303030303030304, 0, 0.013927576601671309, 0.015873015873015872, 0.025423728813559324, 0.006779661016949152, 0.00641025641025641, 0.0062256809338521405, 0.011857707509881422, 0.014388489208633094, 0.006329113924050633, 0.010810810810810811, 0, 0, 0.02158273381294964, 0, 0, 0, 0, 0.009216589861751152, 0.008928571428571428, 0, 0, 0, 0.013986013986013986, 0.0006068881808526779, 0, 0.011396011396011397, 0, 0, 0.01020408163265306, 0.014705882352941176, 0.00558659217877095, 0, 0.008264462809917356, 0.022727272727272728, 0, 0.01639344262295082, 0, 0, 0.018518518518518517, 0.014084507042253521, 0, 0, 0.004149377593360996, 0, 0.005555555555555556, 0, 0.006223073240785065, 0, 0.011857707509881422, 0.009375, 0.0064516129032258064, 0.005681818181818182, 0.015463917525773196, 0, 0.007352941176470588, 0.015625, 0.0072992700729927005, 0.009259259259259259, 0.007792207792207792, 0, 0.012269938650306749, 0.008849557522123894, 0.005649717514124294, 0, 0.009708737864077669, 0.014760147601476014, 0, 0.007246376811594203, 0.008849557522123894, 0.0070921985815602835, 0.007692307692307693, 0.00684931506849315, 0, 0.008583690987124463, 0, 0.007352941176470588, 0, 0.027586206896551724, 0, 0.00425531914893617, 0.007352941176470588, 0.011111111111111112, 0.009302325581395349, 0.022222222222222223, 0.005050505050505051, 0.005405405405405406, 0.00684931506849315, 0, 0.005154639175257732, 0.020202020202020204, 0.007462686567164179, 0.006329113924050633, 0, 0, 0, 0, 0, 0.004524886877828055, 0, 0.005319148936170213, 0, 0, 0, 0.006756756756756757, 0, 0, 0.007462686567164179, 0, 0.014164305949008499, 0.03765690376569038, 0.0449438202247191, 0.0273972602739726, 0.03529411764705882, 0.020100502512562814, 0.019801980198019802, 0.047619047619047616, 0.05, 0, 0.020491803278688523, 0.020833333333333332, 0.018633540372670808, 0.020618556701030927, 0 ]
0.009129
5
[ "Amadeo Lladós\n\nAmadeo Lladós Sánchez-Toscano is a Grand Prix motorcycle racer from Spain.", "\n\nCareer statistics\n\nBy season\n\nRaces by year\n\nReferences\n\nExternal links\n Profile on motogp.com\n\nCategory:Spanish motorcycle racers\nCategory:Living people\nCategory:1991 births\nCategory:Moto2 World Championship riders" ]
{ "pile_set_name": "Wikipedia (en)" }
[ 0.011235955056179775, 0 ]
0.005618
5